Download as pdf or txt
Download as pdf or txt
You are on page 1of 109

RMM - Abstract Algebra Marathon 601 - 700

R M M
ROM A N IA N MAT HEMAT IC AL MAG AZINE

Founding Editor
DANIEL SITARU
Available online ISSN-L 2501-0099
www.ssmrmh.ro
www.ssmrmh.ro
Proposed by
Daniel Sitaru – Romania, Neculai Stanciu-Romania
Marin Chirciu-Romania, Togrul Ehmedov-Azerbaijan
Amir Sofi-Kosovo, Elsen Kerimov-Azerbaijan, Nguyen Van Canh-Vietnam
Carlos Paiva-Brazil, Samed Ahmedov-Azerbaijan
Hikmat Mammadov-Azerbaijan, SK Sabiruddin-India
Sakthi Vel-India, Mehmet Șahin-Turkiye, Mihaly Bencze-Romania
Samir Cabiyev-Azerbaijan, D.M.Bătinețu-Giurgiu-Romania
Kenan Rustemov-Azerbaijan, Sidi Abdallah Lemrabott-Mauritania
Qudrat Muhammadi-Afghanistan, Toubal Fethi-Algeria
Tran Quoc Thinh-Vietnam, Radu Diaconu-Romania
Pavlos Trifon-Greece, Rovsen Pirguliyev-Azerbaijan
Mohamed Amine Ben Ajiba-Morocco

1 RMM-ABSTRACT ALGEBRA MARATHON 601-700


www.ssmrmh.ro
Solutions by
Daniel Sitaru – Romania, Ravi Prakash-India, Pham Duc Nam-Vietnam
Tapas Das-India, Hikmat Mammadov-Azerbaijan, Le Thu-Vietnam
Soumava Chakraborty-India, Imamverdi Badalov-Azebaijan
Mohamed Amine Ben Ajiba-Morocco, Bedri Hajrizi-Kosovo
Jose Ferreira Queiroz-Brazil, Christos Tsifakis-Greece, Michael Sterghiou-
Greece, Bui Hong Suc-Vietnam, Fayssal Abdelli-Algeria, Amin Hajiyev-
Azerbaijan, Adrian Popa-Romania, Samir Zaakouni-Morocco, Ahmad
Ishnawahyudi-Indonesia, Ali Babazadeh-Azerbaijan, Myagmarsuren
Yadamsuren-Mongolia, Toubal Fethi-Algeria, Khanh Hai-Vietnam, George
Florin Șerban-Romania, Dragan Santrac-Bosnia and Herzegovina, Murad
Israfilov-Azerbaijan, Sakthi Vel-India, Qudrat Muhammadi-Afghanistan
Florentin Vișescu-Romania, Said Cerbah-Algeria, Eric Cismaru-Romania
Rajarshi Chakraborty-India, Ertan Yildirim-Turkiye

2 RMM-ABSTRACT ALGEBRA MARATHON 601-700


www.ssmrmh.ro
601. Solve for real numbers:
𝐬𝐢𝐧𝟑 𝒙 𝐬𝐢𝐧𝟐 𝒙 𝐜𝐨𝐬 𝒙 𝐬𝐢𝐧 𝒙 𝐜𝐨𝐬𝟐 𝒙 𝐜𝐨𝐬𝟑 𝒙
𝟑
|𝐬𝐢𝐧𝟑 𝟐𝒙 𝐬𝐢𝐧𝟐 𝟐𝒙 𝐜𝐨𝐬 𝟐𝒙 𝐬𝐢𝐧 𝟐𝒙 𝐜𝐨𝐬𝟐 𝟐𝒙 𝐜𝐨𝐬𝟑 𝟐𝒙| = 𝟎
𝐬𝐢𝐧 𝟑𝒙 𝐬𝐢𝐧𝟐 𝟑𝒙 𝐜𝐨𝐬 𝟑𝒙 𝐬𝐢𝐧 𝟑𝒙 𝐜𝐨𝐬𝟐 𝟑𝒙 𝐜𝐨𝐬𝟑 𝟑𝒙
𝐬𝐢𝐧𝟑 𝟒𝒙 𝐬𝐢𝐧𝟐 𝟒𝒙 𝐜𝐨𝐬 𝟒𝒙 𝐬𝐢𝐧 𝟒𝒙 𝐜𝐨𝐬𝟐 𝟒𝒙 𝐜𝐨𝐬𝟑 𝟒𝒙
Proposed by Daniel Sitaru-Romania
Solution 1 by Ravi Prakash-New Delhi-India
𝐬𝐢𝐧𝟑 𝒙 𝐬𝐢𝐧𝟐 𝒙 𝐜𝐨𝐬 𝒙 𝐬𝐢𝐧 𝒙 𝐜𝐨𝐬 𝟐 𝒙 𝐜𝐨𝐬 𝟑 𝒙
𝟑
𝑳𝒆𝒕 𝛀 = |𝐬𝐢𝐧𝟑 𝟐𝒙 𝐬𝐢𝐧𝟐 𝟐𝒙 𝐜𝐨𝐬 𝟐𝒙 𝐬𝐢𝐧 𝟐𝒙 𝐜𝐨𝐬 𝟐 𝟐𝒙 𝐜𝐨𝐬 𝟑 𝟐𝒙| =
𝐬𝐢𝐧 𝟑𝒙 𝐬𝐢𝐧𝟐 𝟑𝒙 𝐜𝐨𝐬 𝟑𝒙 𝐬𝐢𝐧 𝟑𝒙 𝐜𝐨𝐬 𝟐 𝟑𝒙 𝐜𝐨𝐬 𝟑 𝟑𝒙
𝐬𝐢𝐧𝟑 𝟒𝒙 𝐬𝐢𝐧𝟐 𝟒𝒙 𝐜𝐨𝐬 𝟒𝒙 𝐬𝐢𝐧 𝟒𝒙 𝐜𝐨𝐬 𝟐 𝟒𝒙 𝐜𝐨𝐬 𝟑 𝟒𝒙
= 𝐬𝐢𝐧𝟑 𝒙 𝐬𝐢𝐧𝟑 𝟐𝒙 𝐬𝐢𝐧𝟑 𝟑𝒙 𝐬𝐢𝐧𝟑 𝟒𝒙 𝚫, 𝒘𝒉𝒆𝒓𝒆
𝟏 𝒂 𝒂𝟐 𝒂𝟑
𝟏 𝒃 𝒃𝟐 𝒃𝟑 | 𝒂𝒏𝒅 𝒂 = 𝐜𝐨𝐭 𝒙 , 𝒃 = 𝐜𝐨𝐭 𝟐𝒙 , 𝒄 = 𝐜𝐨𝐭 𝟑𝒙 , 𝒅 = 𝐜𝐨𝐭 𝟒𝒙
𝚫=|
𝟏 𝒄 𝒄𝟐 𝒄𝟑
𝟏 𝒅 𝒅𝟐 𝒅𝟑
𝑼𝒔𝒊𝒏𝒈 𝑹𝟒 → 𝑹𝟒 − 𝑹𝟑 , 𝑹𝟑 → 𝑹𝟑 − 𝑹𝟐 𝒂𝒏𝒅 𝑹𝟐 → 𝑹𝟐 − 𝑹𝟏 , 𝒘𝒆 𝒈𝒆𝒕:
𝟏 𝒂 𝒂𝟐 𝒂𝟑
𝟏 𝒃 − 𝒂 𝒃 − 𝒂𝟐
𝟐
𝒃 − 𝒂𝟑 | = (𝒃 − 𝒂)(𝒄 − 𝒃)(𝒅 − 𝒄)𝚫 , 𝒘𝒉𝒆𝒓𝒆
𝟑
𝚫=| 𝟏
𝟏 𝒄 − 𝒃 𝒄𝟐 − 𝒃𝟐 𝒄𝟑 − 𝒃𝟑
𝟏 𝒅 − 𝒄 𝒅𝟐 − 𝒄𝟐 𝒅𝟑 − 𝒄𝟑
𝟏 𝒃 + 𝒂 𝒃𝟐 + 𝒂𝟐 + 𝒃𝒂
𝚫𝟏 = |𝟏 𝒄 + 𝒃 𝒄𝟐 + 𝒃𝟐 + 𝒃𝒄 |
𝟏 𝒅 + 𝒄 𝒅𝟐 + 𝒄𝟐 + 𝒅𝒄
𝑹𝟑 → 𝑹𝟑 − 𝑹𝟐 , 𝑹𝟐 → 𝑹𝟐 − 𝑹𝟏
𝟏 𝒃+𝒂 𝒃𝟐 + 𝒂𝟐 + 𝒃𝒂
𝚫𝟏 = |𝟎 𝒄 − 𝒂 𝒄𝟐 − 𝒂𝟐 + 𝒃(𝒄 − 𝒂) | =
𝟎 𝒅 − 𝒃 𝒅𝟐 − 𝒃𝟐 + 𝒄(𝒅 − 𝒃)
𝟏 𝒂+𝒃+𝒄
= (𝒄 − 𝒂)(𝒅 − 𝒃) | | = (𝒄 − 𝒂)(𝒅 − 𝒃)(𝒅 − 𝒂). 𝑻𝒉𝒖𝒔,
𝟏 𝒃+𝒄+𝒅
𝚫 = (𝒃 − 𝒂)(𝒄 − 𝒂)(𝒄 − 𝒃)(𝒅 − 𝒂)(𝒅 − 𝒃)(𝒅 − 𝒄).
𝛀 = 𝐬𝐢𝐧𝟑 𝒙 𝐬𝐢𝐧𝟑 𝟐𝒙 𝐬𝐢𝐧𝟑 𝟑𝒙 𝐬𝐢𝐧𝟑 𝟒𝒙 (𝐜𝐨𝐭 𝟐𝒙 − 𝐜𝐨𝐭 𝒙)(𝐜𝐨𝐭 𝟑𝒙 − 𝐜𝐨𝐭 𝒙)(𝐜𝐨𝐭 𝟒𝒙
− 𝐜𝐨𝐭 𝒙)(𝐜𝐨𝐭 𝟑𝒙 − 𝐜𝐨𝐭 𝟐𝒙)(𝐜𝐨𝐭 𝟒𝒙 − 𝐜𝐨𝐭 𝟐𝒙)(𝐜𝐨𝐭 𝟒𝒙 − 𝐜𝐨𝐭 𝟑𝒙) =
= 𝐬𝐢𝐧𝟑 𝒙 𝐬𝐢𝐧𝟑 𝟐𝒙 𝐬𝐢𝐧𝟑 𝟑𝒙 𝐬𝐢𝐧𝟑 𝟒𝒙 ×

3 RMM-ABSTRACT ALGEBRA MARATHON 601-700


www.ssmrmh.ro
𝐬𝐢𝐧 𝒙 𝐬𝐢𝐧 𝟐𝒙 𝐬𝐢𝐧 𝟑𝒙 𝐬𝐢𝐧 𝒙 𝐬𝐢𝐧 𝟐𝒙 𝐬𝐢𝐧 𝒙
× (− ) (− ) (− ) (− ) (− ) (− )
𝐬𝐢𝐧 𝒙 𝐬𝐢𝐧 𝟐𝒙 𝐬𝐢𝐧 𝟑𝒙 𝐬𝐢𝐧 𝒙 𝐬𝐢𝐧 𝟒𝒙 𝐬𝐢𝐧 𝒙 𝐬𝐢𝐧 𝟑𝒙 𝐬𝐢𝐧 𝟐𝒙 𝐬𝐢𝐧 𝟒𝒙 𝐬𝐢𝐧 𝟐𝒙 𝐬𝐢𝐧 𝟒𝒙 𝐬𝐢𝐧 𝟑𝒙
= 𝐬𝐢𝐧𝟑 𝒙 𝐬𝐢𝐧𝟐 𝟐𝒙 𝐬𝐢𝐧 𝟑𝒙
𝑵𝒐𝒘, 𝛀 = 𝟎 ⇒ 𝐬𝐢𝐧 𝒙 = 𝟎 𝒐𝒓 𝐬𝐢𝐧 𝟐𝒙 = 𝟎 𝒐𝒓 𝐬𝐢𝐧 𝟑𝒙 = 𝟎, 𝒕𝒉𝒆𝒏:
𝒏𝝅 𝒏𝝅
𝒙 ∈ { 𝟐 , 𝟑 |𝒏 ∈ ℤ}.

Solution 2 by Tapas Das-India


𝑽𝒐𝒏𝒅𝒆𝒓𝒎𝒐𝒏𝒅𝒆 𝒅𝒆𝒕𝒆𝒓𝒎𝒊𝒏𝒂𝒏𝒕:
𝒙𝒏−𝟏
𝟏 𝒙𝒏−𝟐
𝟏 … 𝒙𝟏 𝟏
𝒙
| 𝟐
𝒏−𝟏 𝒏−𝟐
𝒙𝟐 … 𝒙𝟐 𝟏 | = ∏ (𝒙 − 𝒙 )
… …… … … 𝒊 𝒋
𝒏−𝟏 𝒏−𝟐 𝟏<𝑖<𝑗≤𝑛
𝒙𝒏 𝒙𝒏 … 𝒙𝒏 𝟏
𝐬𝐢𝐧𝟑 𝒙 𝐬𝐢𝐧𝟐 𝒙 𝐜𝐨𝐬 𝒙 𝐬𝐢𝐧 𝒙 𝐜𝐨𝐬 𝟐 𝒙 𝐜𝐨𝐬 𝟑 𝒙
𝟑 𝟐
𝛀 = |𝐬𝐢𝐧𝟑 𝟐𝒙 𝐬𝐢𝐧𝟐 𝟐𝒙 𝐜𝐨𝐬 𝟐𝒙 𝐬𝐢𝐧 𝟐𝒙 𝐜𝐨𝐬 𝟐 𝟐𝒙 𝐜𝐨𝐬 𝟑 𝟐𝒙| =
𝐬𝐢𝐧 𝟑𝒙 𝐬𝐢𝐧 𝟑𝒙 𝐜𝐨𝐬 𝟑𝒙 𝐬𝐢𝐧 𝟑𝒙 𝐜𝐨𝐬 𝟐 𝟑𝒙 𝐜𝐨𝐬 𝟑 𝟑𝒙
𝐬𝐢𝐧𝟑 𝟒𝒙 𝐬𝐢𝐧𝟐 𝟒𝒙 𝐜𝐨𝐬 𝟒𝒙 𝐬𝐢𝐧 𝟒𝒙 𝐜𝐨𝐬 𝟐 𝟒𝒙 𝐜𝐨𝐬 𝟑 𝟒𝒙
𝐭𝐚𝐧𝟑 𝒙 𝐭𝐚𝐧𝟐 𝒙 𝐭𝐚𝐧 𝒙 𝟏
𝟑
= 𝐜𝐨𝐬 𝟑 𝒙 𝐜𝐨𝐬 𝟑 𝟐𝒙 𝐜𝐨𝐬 𝟑 𝟑𝒙 𝐜𝐨𝐬 𝟑 𝟒𝒙 |𝐭𝐚𝐧𝟑 𝟐𝒙 𝐭𝐚𝐧𝟐 𝟐𝒙 𝐭𝐚𝐧 𝟐𝒙 𝟏
|=
𝐭𝐚𝐧 𝟑𝒙 𝐭𝐚𝐧𝟐 𝟑𝒙 𝐭𝐚𝐧 𝟑𝒙 𝟏
𝐭𝐚𝐧𝟑 𝟒𝒙 𝐭𝐚𝐧𝟐 𝟒𝒙 𝐭𝐚𝐧 𝟒𝒙 𝟏
= 𝐜𝐨𝐬 𝟑 𝒙 𝐜𝐨𝐬 𝟑 𝟐𝒙 𝐜𝐨𝐬 𝟑 𝟑𝒙 𝐜𝐨𝐬 𝟑 𝟒𝒙 ×
𝐬𝐢𝐧 𝒙 𝐬𝐢𝐧 𝟐𝒙 𝐬𝐢𝐧 𝟑𝒙 𝐬𝐢𝐧 𝒙 𝐬𝐢𝐧 𝟐𝒙 𝐬𝐢𝐧 𝒙
× (− ) (− ) (− ) (− ) (− ) (− )
𝐜𝐨𝐬 𝒙 𝐜𝐨𝐬 𝟐𝒙 𝐜𝐨𝐬 𝒙 𝐜𝐨𝐬 𝟑𝒙 𝐜𝐨𝐬 𝒙 𝐜𝐨𝐬 𝟒𝒙 𝐜𝐨𝐬 𝟐𝒙 𝐜𝐨𝐬 𝟑𝒙 𝐜𝐨𝐬 𝟐𝒙 𝐜𝐨𝐬 𝟒𝒙 𝐜𝐨𝐬 𝟑𝒙 𝐜𝐨𝐬 𝟒𝒙
= 𝐬𝐢𝐧𝟑 𝒙 𝐬𝐢𝐧𝟐 𝟐𝒙 𝐬𝐢𝐧 𝟑𝒙
𝑵𝒐𝒘, 𝛀 = 𝟎 ⇒ 𝐬𝐢𝐧 𝒙 = 𝟎 𝒐𝒓 𝐬𝐢𝐧 𝟐𝒙 = 𝟎 𝒐𝒓 𝐬𝐢𝐧 𝟑𝒙 = 𝟎, 𝒕𝒉𝒆𝒏:
𝒏𝝅 𝒏𝝅
𝒙∈{ , |𝒏 ∈ ℤ}.
𝟐 𝟑
602. If 𝑨 ∈ 𝑴𝟑,𝟐 (ℂ); 𝑩 ∈ 𝑴𝟐,𝟒 (ℂ); 𝑪 ∈ 𝑴𝟒,𝟑 (ℂ), 𝑨, 𝑩, 𝑪 −fixed, then find all
𝒙 ∈ ℂ such that:
𝟑 𝟐 𝒙
𝑨𝑩𝑪 = (𝟐 𝒙 𝟑)
𝒙 𝟑 𝟐
Proposed by Daniel Sitaru-Romania
Solution by Hikmat Mammadov-Azerbaijan
𝐝𝐞𝐭(𝑨𝑩𝑪) = 𝟑(𝟐𝒙 − 𝟗) − 𝟐(𝟒 − 𝟑𝒙) + 𝒙(𝟔 − 𝒙𝟐 ) = −𝒙𝟑 + 𝟏𝟖𝒙 − 𝟑𝟓
𝒓𝒂𝒏𝒌(𝑨𝑩𝑪) ≤ 𝟐 ⇒ 𝐝𝐞𝐭(𝑨𝑩𝑪) = 𝟎

4 RMM-ABSTRACT ALGEBRA MARATHON 601-700


www.ssmrmh.ro
𝒙 − 𝟏𝟖𝒙 + 𝟑𝟓 = 𝟎 ⇔ (𝒙 + 𝟓)(𝒙𝟐 − 𝟓𝒙 + 𝟕) = 𝟎 ⇒
𝟑

𝟓 ± 𝒊√𝟑
𝒙 ∈ {−𝟓, }
𝟐

603. 𝐈𝐟 𝒂 𝒂𝐧𝐝 𝐛 𝒂𝐫𝐞 𝐩𝐨𝐬𝐢𝐭𝐢𝐯𝐞 𝐫𝐞𝒂𝒍 𝐧𝐮𝐦𝐛𝐞𝐫𝐬, 𝐭𝐡𝐞𝐧 ∶


(𝟑𝒂 − 𝐛)𝟐 (𝟑𝐛 − 𝒂)𝟐 𝟏
+ ≥
𝟐(𝟑𝒂 + 𝐛)𝟐 + 𝟖𝐛 𝟐 𝟐(𝟑𝐛 + 𝒂)𝟐 + 𝟖𝒂𝟐 𝟓
Proposed by Neculai Stanciu-Romania
Solution by Soumava Chakraborty-Kolkata-India

(𝟑𝒂 − 𝐛)𝟐 𝟏 (𝟑𝐛 − 𝒂)𝟐 𝟏


𝟐 𝟐
− + 𝟐 𝟐

𝟐(𝟑𝒂 + 𝐛) + 𝟖𝐛 𝟏𝟎 𝟐(𝟑𝐛 + 𝒂) + 𝟖𝒂 𝟏𝟎
𝟏𝟎(𝟑𝒂 − 𝐛)𝟐 − 𝟐(𝟑𝒂 + 𝐛)𝟐 − 𝟖𝐛𝟐 𝟏𝟎(𝟑𝐛 − 𝒂)𝟐 − 𝟐(𝟑𝐛 + 𝒂)𝟐 − 𝟖𝒂𝟐
= +
𝟏𝟎(𝟐(𝟑𝒂 + 𝐛)𝟐 + 𝟖𝐛 𝟐 ) 𝟏𝟎(𝟐(𝟑𝐛 + 𝒂)𝟐 + 𝟖𝒂𝟐 )
𝟕𝟐𝒂(𝒂 − 𝐛) 𝟕𝟐𝐛(𝒂 − 𝐛)
= −
𝟏𝟎(𝟐(𝟑𝒂 + 𝐛)𝟐 + 𝟖𝐛 𝟐 ) 𝟏𝟎(𝟐(𝟑𝐛 + 𝒂)𝟐 + 𝟖𝒂𝟐 )
𝟕𝟐(𝒂 − 𝐛) 𝒂 𝐛
= .( 𝟐 𝟐
− )
𝟐𝟎 (𝟑𝒂 + 𝐛) + 𝟒𝐛 (𝟑𝐛 + 𝒂)𝟐 + 𝟒𝒂𝟐
𝟏𝟖(𝒂 − 𝐛) 𝟒𝒂𝟑 − 𝟒𝐛𝟑 + 𝒂(𝒂𝟐 + 𝟗𝐛𝟐 + 𝟔𝒂𝐛) − 𝐛(𝟗𝒂𝟐 + 𝐛𝟐 + 𝟔𝒂𝐛)
= .
𝟓 ((𝟑𝒂 + 𝐛)𝟐 + 𝟒𝐛 𝟐 )((𝟑𝐛 + 𝒂)𝟐 + 𝟒𝒂𝟐 )
𝟏𝟖(𝒂 − 𝐛) 𝟓(𝒂 − 𝐛)(𝒂𝟐 + 𝒂𝐛 + 𝐛𝟐 ) − 𝟑𝒂𝐛(𝒂 − 𝐛)
= .
𝟓 ((𝟑𝒂 + 𝐛)𝟐 + 𝟒𝐛 𝟐 )((𝟑𝐛 + 𝒂)𝟐 + 𝟒𝒂𝟐 )
𝟏𝟖(𝒂 − 𝐛)𝟐 𝟓𝒂𝟐 + 𝟐𝒂𝐛 + 𝟓𝐛𝟐
= . ≥𝟎
𝟓 ((𝟑𝒂 + 𝐛)𝟐 + 𝟒𝐛 𝟐 )((𝟑𝐛 + 𝒂)𝟐 + 𝟒𝒂𝟐 )
(𝟑𝒂 − 𝐛)𝟐 (𝟑𝐛 − 𝒂)𝟐 𝟏
∴ 𝟐 𝟐
+ 𝟐 𝟐
≥ , ′′ =′′ 𝐢𝐟𝐟 𝒂 = 𝐛 (𝐐𝐄𝐃)
𝟐(𝟑𝒂 + 𝐛) + 𝟖𝐛 𝟐(𝟑𝐛 + 𝒂) + 𝟖𝒂 𝟓
604. Solve for real numbers:
𝒙𝟑 (𝒙 + 𝟏)𝟑 + 𝟖𝒙𝟑 + 𝟖 = 𝟏𝟐𝒙𝟐 (𝒙 + 𝟏)
Proposed by Daniel Sitaru-Romania
Solution 1 by Imamverdi Badalov-Azerbaijan
𝒙𝟑 (𝒙 + 𝟏)𝟑 + 𝟖𝒙𝟑 + 𝟖 = 𝟏𝟐𝒙𝟐 (𝒙 + 𝟏)
(𝒙 + 𝟏)(𝒙𝟓 + 𝟐𝒙𝟒 + 𝒙𝟑 − 𝟒𝒙𝟐 − 𝟖𝒙 + 𝟖) = 𝟎

5 RMM-ABSTRACT ALGEBRA MARATHON 601-700


www.ssmrmh.ro
𝒙𝟏 = −𝟏
𝒙 − 𝒙 + 𝟐𝒙 − 𝟐𝒙𝟐 + 𝒙𝟑 − 𝒙𝟐 − 𝟖(𝒙 − 𝟏) = 𝟎
𝟓 𝟐 𝟒

𝒙𝟐 (𝒙 − 𝟏)(𝒙𝟐 + 𝒙 + 𝟏) + 𝟐𝒙𝟐 (𝒙 − 𝟏)(𝒙 + 𝟏) + 𝒙𝟐 (𝒙 − 𝟏) − 𝟖(𝒙 − 𝟏) = 𝟎


(𝒙 − 𝟏)(𝒙𝟒 + 𝟑𝒙𝟑 + 𝟒𝒙𝟐 − 𝟖) = 𝟎
𝒙𝟐 = 𝟏
𝟑( 𝟐
𝒙 𝒙 + 𝟐 + 𝒙 + 𝟐 𝒙 − 𝟐𝒙 + 𝟒) + 𝟒(𝒙 − 𝟐)(𝒙 + 𝟐) = 𝟎
) ( )(
(𝒙 + 𝟐)(𝒙𝟑 + 𝒙𝟐 + 𝟐𝒙 − 𝟒 = 𝟎
𝒙𝟑 = −𝟐
𝒙𝟑 − 𝟏 + 𝒙𝟐 − 𝟏 + 𝟐𝒙 − 𝟐 = 𝟎
(𝒙 − 𝟏)(𝒙𝟐 + 𝒙 + 𝟏) + (𝒙 − 𝟏)(𝒙 + 𝟏) + 𝟐(𝒙 − 𝟏) = 𝟎
(𝒙 − 𝟏)(𝒙𝟐 + 𝟐𝒙 + 𝟑) = 𝟎
𝒙𝟑,𝟒 = 𝟏
𝑺 = {−𝟐, −𝟏, 𝟏}
Solution 2 by Christos Tsifakis-Greece
𝒙𝟑 (𝒙 + 𝟏)𝟑 + 𝟖𝒙𝟑 + 𝟖 = 𝟏𝟐𝒙𝟐 (𝒙 + 𝟏)
(𝒙 + 𝟏)(𝒙𝟓 + 𝟐𝒙𝟒 + 𝒙𝟑 − 𝟒𝒙𝟐 − 𝟖𝒙 + 𝟖) = 𝟎
𝒙𝟏 = −𝟏 𝒐𝒓 𝒙𝟓 + 𝟐𝒙𝟒 + 𝒙𝟑 − 𝟒𝒙𝟐 − 𝟖𝒙 + 𝟖 = 𝟎
𝒙𝟐 = 𝟏 𝒐𝒓 𝒙𝟒 + 𝟑𝒙𝟑 + 𝟒𝒙 − 𝟖 = 𝟎
𝒙𝟑 + 𝟒𝒙𝟐 + 𝟖𝒙 + 𝟖 = 𝟎 ⇔ 𝒙(𝒙 + 𝟐)𝟐 + 𝟒(𝒙 + 𝟐) = 𝟎 ⇔ 𝒙𝟑 = −𝟐
𝑺 = {−𝟐, −𝟏, 𝟏}.
Solution 3 by Bui Hong Suc-Vietnam
𝒙𝟑 (𝒙 + 𝟏)𝟑 + 𝟖𝒙𝟑 + 𝟖 = 𝟏𝟐𝒙𝟐 (𝒙 + 𝟏) ⇔ 𝒙𝟑 (𝒙 + 𝟏)𝟑 + 𝟖(𝒙𝟑 + 𝟏) − 𝟏𝟐𝒙𝟐 (𝒙 + 𝟏) = 𝟎
𝒙𝟑 (𝒙 + 𝟏)𝟑 + 𝟖(𝒙 + 𝟏)(𝒙𝟐 − 𝒙 + 𝟏) − 𝟏𝟐𝒙𝟐 (𝒙 + 𝟏) = 𝟎
(𝒙 + 𝟏)[𝒙𝟑 (𝒙 + 𝟏)𝟐 − 𝟒𝒙𝟐 − 𝟖𝒙 + 𝟖] = 𝟎
(𝒙 + 𝟏)[𝒙𝟑 (𝒙 + 𝟏)𝟐 + 𝟖(𝒙 + 𝟏)𝟐 − 𝟏𝟐𝒙(𝒙 + 𝟐)] = 𝟎
(𝒙 + 𝟏)[(𝒙 + 𝟏)𝟐 (𝒙𝟑 + 𝟖) − 𝟏𝟐𝒙(𝒙 + 𝟐)] = 𝟎
(𝒙 + 𝟏)[(𝒙 + 𝟏)𝟐 (𝒙 + 𝟐)(𝒙𝟐 − 𝟐𝒙 + 𝟒) − 𝟏𝟐𝒙(𝒙 + 𝟐)] = 𝟎
(𝒙 + 𝟏)(𝒙 + 𝟐)[(𝒙 + 𝟏)𝟐 (𝒙𝟐 − 𝟐𝒙 + 𝟒) − 𝟏𝟐𝒙] = 𝟎
(𝒙 + 𝟏)(𝒙 + 𝟐)(𝒙𝟒 + 𝒙𝟐 − 𝟔𝒙 + 𝟒) = 𝟎
(𝒙 + 𝟏)(𝒙 + 𝟐)(𝒙𝟒 − 𝟏 + 𝒙𝟐 − 𝟏 − 𝟔𝒙 + 𝟔) = 𝟎
(𝒙 + 𝟏)(𝒙 + 𝟐)(𝒙 − 𝟏)𝟐 (𝒙𝟐 + 𝟐𝒙 + 𝟒) = 𝟎
𝑺{−𝟐, −𝟏, 𝟏}.
Solution 4 by Fayssal Abdelli-Bejaia-Algerie
𝒙𝟑 (𝒙 + 𝟏)𝟑 + 𝟖𝒙𝟑 + 𝟖 = 𝟏𝟐𝒙𝟐 (𝒙 + 𝟏); (𝑨)
𝒙𝟏 = −𝟏 𝒉𝒂𝒔 𝒂𝒏 𝒔𝒐𝒍𝒖𝒕𝒊𝒐𝒏 𝒐𝒇 𝒕𝒉𝒆 𝒆𝒒𝒖𝒂𝒕𝒊𝒐𝒏.
𝑭𝒐𝒓 𝒙 ≠ −𝟏, 𝒘𝒆 𝒉𝒂𝒗𝒆: 𝒙𝟑 (𝒙 + 𝟏)𝟑 + 𝟖(𝒙𝟑 + 𝟏) = 𝟏𝟐𝒙𝟐 (𝒙 + 𝟏)
𝒙𝟑 (𝒙 + 𝟏)𝟑 + 𝟖(𝒙 + 𝟏)(𝒙𝟐 − 𝒙 + 𝟏) = 𝟏𝟐𝒙𝟐 (𝒙 + 𝟏)
𝒙𝟑 (𝒙 + 𝟏)𝟐 + 𝟖(𝒙𝟐 − 𝒙 + 𝟏) = 𝟏𝟐𝒙𝟐 (𝒙 + 𝟏)
𝒙𝟑 (𝒙 + 𝟏)𝟐 − 𝟒𝒙𝟐 − 𝟖𝒙 + 𝟖 = 𝟎
𝒙 𝒙 − 𝟏)(𝒙𝟐 + 𝟑𝒙 + 𝟒) − 𝟖(𝒙 − 𝟏) = 𝟎
𝟐(

6 RMM-ABSTRACT ALGEBRA MARATHON 601-700


www.ssmrmh.ro
𝒙𝟐 = 𝟏 𝒐𝒓 𝒙𝟐 (𝒙𝟐 + 𝟑𝒙 + 𝟒) − 𝟖 = 𝟎
𝒙𝟒 + 𝟑𝒙𝟑 + 𝟒𝒙𝟐 − 𝟖 = 𝟎
(𝒙 + 𝟐)(𝒙𝟐 − 𝟐𝒙 + 𝟒) + 𝟒𝒙(𝒙 + 𝟐) = 𝟎
𝒙𝟑 = −𝟐 𝒐𝒓 𝒙𝟐 + 𝟐𝒙 + 𝟒 = 𝟎
𝑺 = {−𝟐, −𝟏, 𝟏}.
Solution 5 by Pham Duc Nam-Vietnam
𝒙𝟑 (𝒙 + 𝟏)𝟑 + 𝟖𝒙𝟑 + 𝟖 = 𝟏𝟐𝒙𝟐 (𝒙 + 𝟏)
𝒙𝟑 (𝒙 + 𝟏)𝟑 − 𝟖 = 𝟏𝟐𝒙𝟑 + 𝟏𝟐𝒙𝟐 − 𝟖𝒙𝟑 − 𝟏𝟔
(𝒙𝟐 + 𝒙)𝟑 − 𝟖 = 𝟒𝒙𝟑 + 𝟏𝟐𝒙𝟐 − 𝟏𝟔
(𝒙𝟐 + 𝒙 − 𝟐)((𝒙𝟐 + 𝒙)𝟐 + 𝟐(𝒙𝟐 + 𝒙) + 𝟒) = 𝟒(𝒙 + 𝟐)(𝒙𝟐 + 𝒙 − 𝟐)
(𝒙𝟐 + 𝒙 − 𝟐)(𝒙𝟒 + 𝟐𝒙𝟑 + 𝟑𝒙𝟐 − 𝟐𝒙 − 𝟒) = 𝟎
(𝒙𝟐 + 𝒙 − 𝟐) (𝒙𝟐 (𝒙𝟐 − 𝟏) + 𝟐𝒙(𝒙𝟐 − 𝟏) + 𝟒(𝒙𝟐 − 𝟏)) = 𝟎
(𝒙𝟐 + 𝒙 − 𝟐)(𝒙𝟐 − 𝟏)(𝒙𝟐 + 𝟐𝒙 + 𝟒) = 𝟎
𝑺 = {−𝟐, −𝟏, 𝟏}.
Solution 6 by Amin Hajiyev-Azerbaijan
𝒙𝟑 (𝒙 + 𝟏)𝟑 + 𝟖𝒙𝟑 + 𝟖 = 𝟏𝟐𝒙𝟐 (𝟏 + 𝒙)
𝒙𝟑 (𝟏 + 𝒙)𝟑 + 𝟖(𝟏 + 𝒙)(𝒙𝟐 − 𝒙 + 𝟏) − 𝟏𝟐𝒙𝟐 (𝟏 + 𝒙) = 𝟎
(𝒙 + 𝟏)(𝒙𝟓 + 𝟐𝒙𝟒 + 𝒙𝟑 + 𝟖𝒙𝟐 − 𝟖𝒙 + 𝟖 − 𝟏𝟐𝒙𝟐 ) = 𝟎
𝒙𝟏 = −𝟏 𝒐𝒓 𝒙𝟓 + 𝟐𝒙𝟒 + 𝒙𝟑 − 𝟒𝒙𝟐 − 𝟖𝒙 + 𝟖 = 𝟎; (𝑯𝒐𝒓𝒏𝒆𝒓) ⇔
(𝒙 + 𝟐)(𝒙𝟒 + 𝒙𝟐 − 𝟔𝒙 + 𝟒) = 𝟎
𝒙𝟐 = −𝟐 𝒐𝒓 𝒙𝟒 + 𝒙𝟐 − 𝟔𝒙 + 𝟒 = 𝟎
𝒙𝟒 − 𝒙𝟐 + 𝟐𝒙𝟐 − 𝟔𝒙 + 𝟒 = 𝟎
𝒙𝟐 (𝒙 − 𝟏)(𝒙 + 𝟏) + (𝒙 − 𝟏)(𝟐𝒙 − 𝟒) = 𝟎
(𝒙 − 𝟏)(𝒙𝟑 + 𝒙𝟐 + 𝟐𝒙 − 𝟒) = 𝟎
𝒙𝟑 = 𝟏 𝒐𝒓 𝒙𝟑 + 𝒙𝟐 + 𝟐𝒙 − 𝟒 = 𝟎
𝒙𝟑 − 𝒙𝟐 + 𝟐𝒙𝟐 + 𝟐𝒙 − 𝟒 = 𝟎
𝒙𝟐 (𝒙 − 𝟏) + 𝟐(𝒙 − 𝟏)(𝒙 + 𝟐) = 𝟎
𝒙𝟐 + 𝟐𝒙 + 𝟒 = 𝟎 𝒏𝒐 𝒓𝒆𝒂𝒍 𝒔𝒐𝒍𝒖𝒕𝒊𝒐𝒏.
𝑺 = {−𝟐, −𝟏, 𝟏}.
605. Solve for real numbers:
𝒙 𝟐
𝟓 + 𝒍𝒐𝒈𝟏𝟐 = 𝒙 +
𝒙𝟑 + 𝟏𝟔 √𝒙 − 𝟏
Proposed by Marin Chirciu-Romania
Solution by Pham Duc Nam-Vietnam
∗ 𝑪𝒐𝒏𝒅𝒊𝒕𝒊𝒐𝒏: 𝒙 > 1
𝒙 ′(𝒙)
𝟐( 𝒙 𝟑 − 𝟖)
∗ 𝑳𝒆𝒕: 𝒇(𝒙) = 𝒍𝒐𝒈𝟏𝟐 𝟑 ,𝒙 > 1 ⇒ 𝒇 = − , 𝒇′(𝒙) = 𝟎 ⇔ 𝒙 = 𝟐
𝒙 + 𝟏𝟔 𝒙(𝒙𝟑 + 𝟏𝟔) 𝒍𝒐𝒈(𝟏𝟐)

7 RMM-ABSTRACT ALGEBRA MARATHON 601-700


www.ssmrmh.ro
𝑥 1 2 ∞
′(𝒙)
𝒇 + 0 -
| ⇒
𝟏
( )
𝑓 𝒙 log𝟏𝟐 ( ) ↗ -1 ↘ -∞
𝟏𝟕
𝒙
𝑴𝒂𝒙(𝒇(𝒙)) = −𝟏 ⇒ 𝟓 + 𝒍𝒐𝒈𝟏𝟐 𝟑 ≤ 𝟒( 𝟏)
𝒙 + 𝟏𝟔
𝟐
∗ 𝑳𝒆𝒕: 𝒈(𝒙) = 𝒙 + ,𝒙 > 1 ⇒
√𝒙 − 𝟏
𝟏 (√𝒙 − 𝟏 − 𝟏)(√𝒙 − 𝟏 + 𝟏) ′(𝒙)
𝒈′(𝒙) =𝟏− = ,𝒈 = 𝟎 ⇔ 𝒙 = 𝟐
(𝒙 − 𝟏)√𝒙 − 𝟏 (𝒙 − 𝟏)√𝒙 − 𝟏
𝑥 1 2 ∞∞
′(𝒙)
|𝒈 - 0 + ⇒
𝑔 ( 𝒙) + ∞ ↘ 4 ↗ ∞
𝟐
𝑴𝒊𝒏(𝒈(𝒙)) = 𝟒 ⇒ 𝒙 + ≥ 𝟒( 𝟐)
√𝒙 − 𝟏
𝒙 𝟐
𝑭𝒓𝒐𝒎(𝟏), (𝟐) ⇒ 𝟓 + 𝒍𝒐𝒈𝟏𝟐 𝟑 =𝒙+ ⇔
𝒙 + 𝟏𝟔 √𝒙 − 𝟏
𝒙 𝟐
𝟓 + 𝒍𝒐𝒈𝟏𝟐 𝟑 =𝒙+ =𝟒⇔𝒙=𝟐
𝒙 + 𝟏𝟔 √𝒙 − 𝟏
606. Solve for complex numbers:
𝝅 𝟏𝟏𝝅 𝟓𝝅 𝟕𝝅
𝟒𝒙𝟒 + 𝟓𝒙𝟐 + 𝟒 = 𝒙 (𝒕𝒂𝒏 𝒕𝒂𝒏 + 𝒕𝒂𝒏 𝒕𝒂𝒏 )
𝟐𝟒 𝟐𝟒 𝟐𝟒 𝟐𝟒

Proposed by Daniel Sitaru-Romania


Solution by Pham Duc Nam-Vietnam
𝝅 𝟏𝟏𝝅 𝟓𝝅 𝟕𝝅
𝟒𝒙𝟒 + 𝟓𝒙𝟐 + 𝟒 = 𝒙 (𝒕𝒂𝒏 𝒕𝒂𝒏 + 𝒕𝒂𝒏 𝒕𝒂𝒏 ) (𝟏)
𝟐𝟒 𝟐𝟒 𝟐𝟒 𝟐𝟒
𝝅 𝟏𝟏𝝅 𝝅 𝟏𝟏𝝅 𝝅
∗ 𝒄𝒐𝒕 ( − 𝒙) = 𝒕𝒂𝒏(𝒙) , 𝒕𝒂𝒏(𝒙) 𝒄𝒐𝒕(𝒙) = 𝟏,then: 𝒕𝒂𝒏 = 𝒄𝒐𝒕 ( − ) = 𝒄𝒐𝒕
𝟐 𝟐𝟒 𝟐 𝟐𝟒 𝟐𝟒
𝟕𝝅 𝝅 𝟕𝝅 𝟓𝝅 𝝅 𝟏𝟏𝝅 𝟓𝝅 𝟕𝝅
and: 𝒕𝒂𝒏 = 𝒄𝒐𝒕 ( − ) = 𝒄𝒐𝒕 ⇒ 𝒕𝒂𝒏 𝒕𝒂𝒏 + 𝒕𝒂𝒏 𝒕𝒂𝒏 = 𝟐 ⇒ (𝟏)
𝟐𝟒 𝟐 𝟐𝟒 𝟐𝟒 𝟐𝟒 𝟐𝟒 𝟐𝟒 𝟐𝟒
⇔ 𝟒𝒙𝟒 + 𝟓𝒙𝟐 − 𝟐𝒙 + 𝟒 = 𝟎
𝟓 𝟏 𝟓 𝟏
⇔ 𝒙𝟒 + 𝒙𝟐 − 𝒙 + 𝟏 = 𝟎,now : use Ferrari's method : 𝒙𝟒 + 𝒙𝟐 − 𝒙 + 𝟏
𝟒 𝟐 𝟒 𝟐
𝟐
𝟓 𝟏 𝟓 𝟐𝟓
= (𝒙𝟐 + + 𝜶) − [𝟐𝜶𝒙𝟐 + 𝒙 + (𝜶𝟐 + 𝜶 + − 𝟏)]
𝟖 𝟐 𝟒 𝟔𝟒
𝟐
𝟓 𝟏 𝟓 𝟑𝟗
= (𝒙𝟐 + + 𝜶) − [𝟐𝜶𝒙𝟐 + 𝒙 + (𝜶𝟐 + 𝜶 − )]
𝟖 𝟐 𝟒 𝟔𝟒
𝟏 𝟓 𝟑𝟗 𝟑𝟗 𝟏
Solve for 𝜶: − 𝟒. 𝟐𝜶 (𝜶 + 𝜶 − ) = 𝟎 ⇔ −𝟖𝜶 − 𝟏𝟎𝜶𝟐 +
𝟐 𝟑
𝜶 + = 𝟎,
𝟒 𝟒 𝟔𝟒 𝟖 𝟒

8 RMM-ABSTRACT ALGEBRA MARATHON 601-700


www.ssmrmh.ro
𝟏 𝟐𝟏𝟕 𝟑 𝟏
we pick: 𝜶𝟎 = (−𝟏𝟎 + 𝟑 + √−𝟐𝟓𝟑𝟗 + 𝟏𝟐𝒊√𝟐𝟔𝟏𝟗𝟑) ⇒ 𝒙𝟎 = −
𝟐𝟒 √−𝟐𝟓𝟑𝟗 + 𝟏𝟐𝒊√𝟐𝟔𝟏𝟗𝟑 𝟖𝜶𝟎
𝟐
𝟓 𝟓
⇒ (𝒙 + + 𝜶𝟎 ) − 𝟐𝜶𝟎 (𝒙 − 𝒙𝟎 )𝟐 = 𝟎 ⇒ (𝒙𝟐 + + 𝜶𝟎 ) = ±√𝟐𝜶𝟎 (𝒙 − 𝒙𝟎 )
𝟐
𝟖 𝟖
𝟓
𝒙𝟐 − 𝒙√𝟐𝜶𝟎 + + 𝜶𝟎 + 𝒙𝟎 √𝟐𝜶𝟎 = 𝟎
⇔[ 𝟖
𝟓
𝒙𝟐 + 𝒙√𝟐𝜶𝟎 + + 𝜶𝟎 − 𝒙𝟎 √𝟐𝜶𝟎 = 𝟎
𝟖
𝟓
√𝟐𝜶𝟎 ± √−𝟐𝜶𝟎 − 𝟐 − 𝟒𝒙𝟎 √𝟐𝜶𝟎
𝑥=
⇔ 𝟐
𝟓
−√𝟐𝜶𝟎 ± √−𝟐𝜶𝟎 − + 𝟒𝒙𝟎 √𝟐𝜶𝟎
𝟐
[𝑥 = 𝟐
𝟓 𝟏
𝟓 𝟏 √𝟐𝜶𝟎 ± √−𝟐𝜶𝟎 − 𝟐 +
√𝟐𝜶𝟎 ± √−𝟐𝜶𝟎 − 𝟐 + 𝟐𝜶 √𝟐𝜶𝟎 √𝟐𝜶𝟎
𝟎
𝑥= 𝑥=
⇔ 𝟐 ⇔ 𝟐
𝟓 𝟏 𝟓 𝟏
−√𝟐𝜶𝟎 ± √−𝟐𝜶𝟎 − 𝟐 + 𝟐𝜶 √𝟐𝜶𝟎 −√𝟐𝜶𝟎 ± √−𝟐𝜶𝟎 − 𝟐 +
𝟎
[𝑥 = √𝟐𝜶 𝟎
𝟐 𝑥 =
[ 𝟐
607. If 𝒙𝟏 , 𝒙𝟐 −are roots of the equation 𝒙𝟐 − 𝟕𝒙 + 𝟏 = 𝟎 then:
𝟖 𝟖
√𝒙𝟏 + √𝒙𝟐 = √𝟏 + 𝟐𝝋, 𝝋 −golden ratio.
Proposed by Togrul Ehmedov-Azerbaijan
Solution 1 by Adrian Popa-Romania
𝒙 + 𝒙𝟐 = 𝟕 > 0
{ 𝟏 ⇒ 𝒙𝟏 > 0, 𝒙𝟐 > 0
𝒙𝟏 𝒙𝟐 = 𝟏 > 0
𝟐
(√𝒙𝟏 + √𝒙𝟐 ) = 𝒙𝟏 + 𝒙𝟐 + 𝟐√𝒙𝟏 𝒙𝟐 = 𝟕 + 𝟐 ∙ 𝟏 = 𝟗 ⇒ √𝒙𝟏 + √𝒙𝟐 = 𝟑
𝟐
( 𝟒√𝒙𝟏 + 𝟒√ 𝒙𝟐 ) = √𝒙𝟏 + √𝒙𝟐 + 𝟐 𝟒√𝒙𝟏 𝒙𝟐 = 𝟑 + 𝟐 ∙ 𝟏 = 𝟓 ⇒
𝟒 𝟒
√ 𝒙𝟏 + √𝒙𝟐 = √𝟓
𝟐
( 𝟖√ 𝒙𝟏 + 𝟖√ 𝒙𝟐 ) = 𝟒√ 𝒙𝟏 + 𝟒√ 𝒙𝟐 + 𝟐 𝟖√𝒙𝟏 𝒙𝟐 = √𝟓 + 𝟐 ∙ 𝟏 = 𝟐 + √𝟓

𝟏 + √𝟓
√𝒙𝟏 + √𝒙𝟐 = √𝟐 + √𝟓√𝟏 + (𝟏 + √𝟓) = √𝟏 + 𝟐 ∙
𝟖 𝟖
= √𝟏 + 𝟐𝝋
𝟐

9 RMM-ABSTRACT ALGEBRA MARATHON 601-700


www.ssmrmh.ro
Solution 2 by Pham Duc Nam-Vietnam
𝟐
𝟏 𝟏 𝟏 𝟑 + √𝟓
𝑰𝒇 𝒙𝟏 𝒊𝒔 𝒓𝒐𝒐𝒕 𝒕𝒉𝒆𝒏 𝒙𝟐 = , 𝒍𝒆𝒕 𝒙𝟏 = (𝟕 + 𝟑√𝟓) = (𝟏𝟒 + 𝟔√𝟓) = ( )
𝒙𝟏 𝟐 𝟒 𝟐

𝟑 +𝟒
𝟏 √𝟑 + √𝟓 + 𝟏 √𝟔 + 𝟐√𝟓 + 𝟏
√𝟓 𝟐 𝟒
√ 𝒙𝟏 + √ 𝒙𝟐 = √
𝟖 𝟖
+ = = =
𝟐 𝟒
𝟑 + √𝟓 𝟒
𝟑 + √𝟓 𝟒
𝟔 + 𝟐√𝟓
√ √ √
𝟐 𝟐 𝟒
𝟐
√(√𝟓 + 𝟏) + 𝟏 √𝟓 + 𝟏
𝟐 + 𝟏 = 𝝋 + 𝟏 = 𝝋 = √𝝋𝟑 = √𝝋𝟐 + 𝝋 = √𝟏 + 𝟐𝝋
𝟒 𝟐
= =
𝟒 𝟐
√√𝟓 + 𝟏 √𝝋 √𝝋
√(√𝟓 + 𝟏) 𝟐
𝟒
Solution 3 by Samir Zaakouni-Morocco
𝑺 = 𝟖√ 𝒙𝟏 + 𝟖√𝒙𝟐 ⇒ 𝑺𝟐 − 𝟐 = 𝟒√𝒙𝟏 + 𝟒√𝒙𝟐
(𝑺𝟐 − 𝟐)𝟐 − 𝟐 = √𝒙𝟏 + √𝒙𝟐
((𝑺𝟐 − 𝟐)𝟐 − 𝟐)𝟐 = 𝒙𝟏 + 𝒙𝟐 + 𝟐
((𝑺𝟐 − 𝟐)𝟐 − 𝟐)𝟐 = 𝟗, (𝑺𝟐 − 𝟐)𝟐 − 𝟐 = 𝟑
(𝑺𝟐 − 𝟐)𝟐 = 𝟓, 𝑺𝟐 − 𝟐 = √𝟓, 𝑺𝟐 = 𝟐 + √𝟓
𝟏 + √𝟓
𝑺 = √𝟏 + 𝟐 ∙ = √𝟏 + 𝟐𝝋
𝟐
Solution 4 by Ahmad Ishnawahyudi-Indonesia
𝒙𝟏 + 𝒙𝟐 = 𝟕 𝒂𝒏𝒅 𝒙𝟏 𝒙𝟐 = 𝟏
𝑳𝒆𝒕 √ 𝒙𝟏 + √ 𝒙𝟐 = 𝒂, 𝟒√ 𝒙𝟏 + 𝟒√𝒙𝟐 = 𝒃 𝒂𝒏𝒅 𝒄 = √𝒙𝟏 + √𝒙𝟐 , 𝒕𝒉𝒆𝒏
𝟖 𝟖

𝒂𝟐 = 𝟒√𝒙𝟏 + 𝟒√𝒙𝟐 + 𝟐, 𝒃𝟐 = √𝒙𝟏 + √𝒙𝟐 + 𝟐 𝒂𝒏𝒅 𝒄𝟐 = 𝒙𝟏 + 𝒙𝟐 + 𝟐


𝟒 + 𝟐√𝟓 𝟐(𝟏 + √𝟓)
𝒄 = √𝟗 = 𝟑, 𝒃𝟐 = 𝟓 ⇒ 𝒃 = √𝟓 𝒂𝒏𝒅 𝒂𝟐 = 𝟐 + √𝟓 = = (𝟏 + )
𝟐 𝟐
= 𝟏 + 𝟐𝝋
Solution 5 by Ali Babazadeh-Azerbaijan
𝒙𝟐 − 𝟕𝒙 + 𝟏 = 𝟎
√𝒙𝟏 + √𝒙𝟐 = √𝒙𝟏 + 𝒙𝟐 + 𝟐√𝒙𝟏 𝒙𝟐 = √𝟕 + 𝟐 ∙ 𝟏 = 𝟑
𝟒 𝟒 𝟒
√𝒙𝟏 + √𝒙𝟐 = √√𝒙𝟏 + √𝒙𝟐 + 𝟐 √𝒙𝟏 𝒙𝟐 = √𝟑 + 𝟐 ∙ 𝟏 = √𝟓

10 RMM-ABSTRACT ALGEBRA MARATHON 601-700


www.ssmrmh.ro
𝟏 + √𝟓
√𝒙𝟏 + √𝒙𝟐 = √ √𝒙𝟏 + √𝒙𝟐 + 𝟐 √𝒙𝟏 𝒙𝟐 = √𝟐 + √𝟓 = √𝟏 + 𝟐 ∙
𝟖 𝟖 𝟒 𝟒 𝟖
= √𝟏 + 𝟐𝝋
𝟐
Solution 6 by Ravi Prakash-New Delhi-India
𝒙𝟏 + 𝒙𝟐 = 𝟕 𝒂𝒏𝒅 𝒙𝟏 𝒙𝟐 = 𝟏
𝑵𝒐𝒕𝒆 𝒕𝒉𝒂𝒕: 𝒙𝟏 , 𝒙𝟐 > 0
𝟐
(√𝒙𝟏 + √𝒙𝟐 ) = 𝒙𝟏 + 𝒙𝟐 + 𝟐√𝒙𝟏 𝒙𝟐 = 𝟕 + 𝟐 = 𝟗
√𝒙𝟏 + √𝒙𝟐 = 𝟑
𝟐
𝑵𝒆𝒙𝒕, ( 𝟒√𝒙𝟏 + 𝟒√𝒙𝟐 ) = √𝒙𝟏 + √𝒙𝟐 + 𝟐 𝟒√𝒙𝟏 𝒙𝟐 = 𝟐 + 𝟑 = 𝟓
𝟒 𝟒
√ 𝒙𝟏 + √𝒙𝟐 = √𝟓
𝟐
𝑵𝒐𝒘, ( 𝟖√𝒙𝟏 + 𝟖√𝒙𝟐 ) = 𝟒√𝒙𝟏 + 𝟒√𝒙𝟐 + 𝟐 𝟖√𝒙𝟏 𝒙𝟐 = √𝟓 + 𝟐

𝟐(√𝟓 + 𝟏)
√𝒙𝟏 + √𝒙𝟐 = √𝟐 + √𝟓 = √𝟏 +
𝟖 𝟖
= √𝟏 + 𝟐𝝋
𝟐
Solution 7 by Tapas Das-India
𝒙𝟐 − 𝟕𝒙 + 𝟏 = 𝟎
𝟒 𝟒
𝟕 + 𝟑√𝟓 √𝟓 + 𝟏 𝟕 − 𝟑√𝟓 √𝟓 − 𝟏
𝒙𝟏 = =( ) , 𝒙𝟐 = =( )
𝟐 𝟐 𝟐 𝟐

√𝟓 + 𝟏 √√𝟓 − 𝟏 √ 𝟐(√𝟓 + 𝟏)
√ 𝒙𝟏 + √ 𝒙 𝟐 = √ = 𝟐 + √𝟓 = √𝟏 +
𝟖 𝟖
+ = √𝟏 + 𝟐𝝋
𝟐 𝟐 𝟐

608. Solve for real numbers:


𝒙𝟑 + 𝒚𝟑 + 𝟐 = 𝟐(𝒙 + 𝒚) + 𝐥𝐨𝐠(𝒙𝒙 𝒚𝒚 )
Proposed by Daniel Sitaru-Romania
Solution by Adrian Popa-Romania
𝑳𝒆𝒕 𝒃𝒆 𝒕𝒉𝒆 𝒇𝒖𝒏𝒄𝒕𝒊𝒐𝒏 𝒇(𝒙) = 𝒙𝟑 + 𝟏 − 𝟐𝒙 − 𝒙 𝐥𝐨𝐠 𝒙 , 𝒙 > 0, 𝑡ℎ𝑒𝑛

′( 𝟐
𝟔𝒙𝟐 − 𝟏
′′ (
𝒇 𝒙) = 𝟑𝒙 − 𝐥𝐨𝐠 𝒙 − 𝟑 𝒂𝒏𝒅 𝒇 𝒙) =
𝒙
𝟏
𝒇′′ (𝒙) = 𝟎 ⇔ 𝒙 = , (𝒙 > 0)
√𝟔
𝟏
𝐥𝐨𝐠 𝒙
𝒇(𝟎) = 𝟏 − 𝐥𝐢𝐦 𝒙 𝐥𝐨𝐠 𝒙 = 𝟏 − 𝐥𝐢𝐦 = 𝟏 − 𝐥𝐢𝐦 𝒙 = 𝟏
𝒙→𝟎 𝒙→𝟎 𝟏 𝒙→𝟎 𝟏

𝒙 𝒙𝟐

11 RMM-ABSTRACT ALGEBRA MARATHON 601-700


www.ssmrmh.ro
𝒇(𝟏) = 𝟎

𝒇(𝒙) ≥ 𝟎, (∀)𝒙 > 0 ⇒ 𝒙𝟑 + 𝟏 ≥ 𝟐𝒙 + 𝒙 𝐥𝐨𝐠 𝒙 , (∀)𝒙 > 0


𝑬𝒒𝒖𝒂𝒍𝒊𝒕𝒚 𝒉𝒐𝒍𝒅𝒔 𝒇𝒐𝒓 𝒙 = 𝟏.
𝒇(𝒚) ≥ 𝟎, (∀)𝒚 > 0. 𝐸𝑞𝑢𝑎𝑙𝑖𝑡𝑦 ℎ𝑜𝑙𝑑𝑠 𝑓𝑜𝑟 𝑦 = 1.
𝑻𝒉𝒆𝒓𝒆𝒇𝒐𝒓𝒆, 𝒙𝟑 + 𝒚𝟑 + 𝟐 = 𝟐(𝒙 + 𝒚) + 𝐥𝐨𝐠(𝒙𝒙 𝒚𝒚 ) 𝒉𝒂𝒔 𝒔𝒐𝒍𝒖𝒕𝒊𝒐𝒏 (𝒙, 𝒚) = (𝟏, 𝟏).
609. Solve for real numbers:
𝟓
𝒙𝟐 + 𝝀 √𝒙𝟔 − 𝒙𝟒 = (𝝀 + 𝟏)𝒙 + 𝟏, 𝝀 ≥ 𝟎 − 𝒇𝒊𝒙𝒆𝒅
Proposed by Marin Chirciu-Romania
Solution by Pham Duc Nam-Vietnam
𝟓
𝐅𝐢𝐱𝐞𝐝: 𝝀 ≥ 𝟎, 𝐬𝐨𝐥𝐯𝐞: 𝒙𝟐 + 𝝀 √𝒙𝟔 − 𝒙𝟒 = (𝝀 + 𝟏)𝒙 + 𝟏(𝒆𝒒𝟎)
∗ 𝒙 = 𝟎 ⇒ (𝒆𝒒𝟎) ⇔ 𝟎 = 𝟏(Impossible)
𝟓 𝟏 𝟏
∗ Divide both sides by 𝒙: (𝒆𝒒𝟎) ⇔ 𝒙 + 𝝀 √𝒙 − = ( 𝝀 + 𝟏) +
𝒙 𝒙

𝟏 𝟓 𝟏 𝟓 𝟏
⇔ (𝒙 − ) − 𝟏 + 𝝀 √𝒙 − − 𝝀 = 𝟎, 𝒍𝒆𝒕: 𝒕 = √𝒙 −
𝒙 𝒙 𝒙
⇔ 𝒕𝟓 − 𝟏 + 𝝀(𝒕 − 𝟏) = 𝟎 ⇔ (𝒕 − 𝟏)(𝒕𝟒 + 𝒕𝟑 + 𝒕𝟐 + 𝒕 + 𝟏) + 𝝀(𝒕 − 𝟏) = 𝟎
⇔ (𝒕 − 𝟏)(𝒕𝟒 + 𝒕𝟑 + 𝒕𝟐 + 𝒕 + 𝟏 + 𝝀) = 𝟎
𝑡=1
⇔[ 𝟒
𝒕 + 𝒕 𝟑 + 𝒕𝟐 + 𝒕 + 𝟏 + 𝝀 = 𝟎
𝟏 𝟏 ± √𝟓
∗ 𝒕 = 𝟏 ⇔ 𝒙 − = 𝟏 ⇔ 𝒙𝟐 − 𝒙 − 𝟏 = 𝟎 ⇔ 𝒙 =
𝒙 𝟐
∗ 𝒕𝟒 + 𝒕𝟑 + 𝒕𝟐 + 𝒕 + 𝟏 + 𝝀 = (𝒕𝟐 + 𝒕)(𝒕𝟐 + 𝟏) + 𝟏 + 𝝀
𝑡≥𝟎
𝑪𝒂𝒔𝒆𝟏: [ ⇒ (𝒕𝟐 + 𝒕)(𝒕𝟐 + 𝟏) + 𝟏 + 𝝀 > 0 ⇒ 𝒕𝟒 + 𝒕𝟑 + 𝒕𝟐 + 𝒕 + 𝟏 + 𝝀 ≠ 𝟎
𝑡 ≤ −𝟏

12 RMM-ABSTRACT ALGEBRA MARATHON 601-700


www.ssmrmh.ro
𝑪𝒂𝒔𝒆𝟐: −𝟏 < 𝑡 < 0 ⇒ −𝟏 < 𝒕 + 𝒕)(𝒕𝟐 + 𝟏) < 0 ⇒ (𝒕𝟐 + 𝒕)(𝒕𝟐 + 𝟏) + 𝟏 + 𝝀 > 0
( 𝟐

⇒ 𝒕𝟒 + 𝒕𝟑 + 𝒕𝟐 + 𝒕 + 𝟏 + 𝝀 ≠ 𝟎
𝟏 ± √𝟓
⇒𝒙= 𝐚𝐫𝐞 𝐬𝐨𝐥𝐮𝐭𝐢𝐨𝐧𝐬
𝟐
610. Solve for real numbers:
𝟐 𝟏
𝒙𝟐 + = 𝟐√𝟐 (𝒙 − )
𝒙𝟐 + 𝟐𝒙 𝒙
Proposed by Amir Sofi-Kosovo
Solution by Pham Duc Nam-Vietnam
𝟐 𝟏
𝒙𝟐 + = 𝟐√𝟐 (𝒙 − ) (𝒆𝒒𝟎)(𝒙 ∈ ℝ)
𝒙𝟐 + 𝟐𝒙 𝒙
∗ 𝒙 ≠ 𝟎, 𝒙 ≠ −𝟐
𝒙𝟒 + 𝟐𝒙𝟑 + 𝟐 𝒙𝟐 − 𝟏 𝒙𝟒 + 𝟐𝒙𝟑 + 𝟐
∗ (𝒆𝒒𝟎) ⇔ = 𝟐√𝟐 ( )⇔ = 𝟐√𝟐(𝒙𝟐 − 𝟏)
𝒙(𝒙 + 𝟐) 𝒙 𝒙+𝟐

⇔ 𝒙𝟒 + 𝟐𝒙𝟑 + 𝟐 = 𝟐√𝟐(𝒙𝟑 + 𝟐𝒙𝟐 − 𝒙 − 𝟐)


⇔ 𝒙𝟒 + (𝟐 − 𝟐√𝟐)𝒙𝟑 − 𝟒√𝟐𝒙𝟐 + 𝟐√𝟐𝒙 + 𝟒√𝟐 + 𝟐 = 𝟎

⇔ 𝒙𝟒 + (𝟐 − 𝟐√𝟐)𝒙𝟑 + (𝟐 − 𝟑√𝟐)𝒙𝟐 − (𝟐 + √𝟐)𝒙𝟐 + 𝟐√𝟐𝒙 + 𝟒√𝟐 + 𝟐 = 𝟎

⇔ 𝒙𝟐 (𝒙𝟐 + (𝟐 − 𝟐√𝟐)𝒙 + (𝟐 − 𝟑√𝟐)) − (𝟐 + √𝟐)𝒙𝟐 − (𝟐 + √𝟐)(𝟐 − 𝟐√𝟐)𝒙

− (𝟐 + √𝟐)(𝟐 − 𝟑√𝟐) = 𝟎

⇔ 𝒙𝟐 (𝒙𝟐 + (𝟐 − 𝟐√𝟐)𝒙 + (𝟐 − 𝟑√𝟐)) − (𝟐 + √𝟐) (𝒙𝟐 + (𝟐 − 𝟐√𝟐)𝒙 + (𝟐 − 𝟑√𝟐)) = 𝟎

⇔ (𝒙𝟐 + (𝟐 − 𝟐√𝟐)𝒙 + (𝟐 − 𝟑√𝟐)) (𝒙𝟐 − (𝟐 + √𝟐)) = 𝟎

𝒙𝟐 − (𝟐 + √𝟐) = 𝟎 𝑥 = ±√𝟐 + √𝟐
⇔[ ⇔
𝒙𝟐 + (𝟐 − 𝟐√𝟐)𝒙 + (𝟐 − 𝟑√𝟐) = 𝟎

{𝑥 = −𝟏 + √𝟐 ± 𝟏 + √𝟐

611. Solve for real numbers:


(𝟏𝒙 + 𝟕𝒙 )(𝟏𝟐𝟔𝒙 + 𝟏𝟐𝟕𝒙 ) = 𝟐𝟎𝟐𝟒𝒙
Proposed by Marin Chirciu-Romania
Solution by Pham Duc Nam-Vietnam
(𝟏𝒙 + 𝟕𝒙 )(𝟏𝟐𝟔𝒙 + 𝟏𝟐𝟕𝒙 ) = 𝟐𝟎𝟐𝟒𝒙 (𝒆𝒒𝟎) ∴ 𝒙 ∈ ℝ
∗ We prove:let 𝒇(𝒙) and 𝒈(𝒙) are two monotone strictly decreasing functions on (𝒂, 𝒃),then:
𝒇(𝒙) + 𝒈(𝒙) is also a monotone strictly decreasing function on (𝒂, 𝒃).
By definition of monotone strictly decreasing function:

13 RMM-ABSTRACT ALGEBRA MARATHON 601-700


www.ssmrmh.ro
let any 𝒎, 𝒏 ∈ (𝒂, 𝒃)and 𝒎 > 𝒏 ⇒ 𝒇(𝒎) < 𝑓 (𝒏) and 𝒈(𝒎) < 𝑔(𝒏)
⇒ 𝒇(𝒎) + 𝒈(𝒎) < 𝑓(𝒏) + 𝒈(𝒏)(∴ Basic rule of inequality)
⇒ 𝒇(𝒙) + 𝒈(𝒙) is also a monotone strictly decreasing function on (𝒂, 𝒃)
(𝟏𝒙 + 𝟕𝒙 )(𝟏𝟐𝟔𝒙 + 𝟏𝟐𝟕𝒙 )
∗ 𝟐𝟎𝟐𝟒𝒙 ≠ 𝟎∀𝒙 ∈ ℝ ⇒ (𝒆𝒒𝟎) ⇔ =𝟏
𝟐𝟎𝟐𝟒𝒙
𝟏𝟐𝟔𝒙 + 𝟏𝟐𝟕𝒙 + 𝟖𝟖𝟗𝒙 + 𝟖𝟖𝟐𝒙 𝟏𝟐𝟔 𝒙 𝟏𝟐𝟕 𝒙 𝟖𝟖𝟐 𝒙 𝟖𝟖𝟗 𝒙
⇔ = 𝟏 ⇔ ( ) + ( ) + ( ) + ( ) =𝟏
𝟐𝟎𝟐𝟒𝒙 𝟐𝟎𝟐𝟒 𝟐𝟎𝟐𝟒 𝟐𝟎𝟐𝟒 𝟐𝟎𝟐𝟒
𝟏𝟐𝟔 𝟏𝟐𝟕 𝟖𝟖𝟐 𝟖𝟖𝟗
𝟎< , , , <1⇒
𝟐𝟎𝟐𝟒 𝟐𝟎𝟐𝟒 𝟐𝟎𝟐𝟒 𝟐𝟎𝟐𝟒
𝟏𝟐𝟔 𝒙 𝟏𝟐𝟕 𝒙 𝟖𝟖𝟐 𝒙 𝟖𝟖𝟗 𝒙
𝒚=( ) ,𝒚 = ( ) ,𝒚 = ( ) ,𝒚 = ( )
𝟐𝟎𝟐𝟒 𝟐𝟎𝟐𝟒 𝟐𝟎𝟐𝟒 𝟐𝟎𝟐𝟒
are all monotone strictly decreasing functions
𝟏𝟐𝟔 𝒙 𝟏𝟐𝟕 𝒙 𝟖𝟖𝟐 𝒙 𝟖𝟖𝟗 𝒙
⇒ 𝒇(𝒙) = ( ) +( ) +( ) +( )
𝟐𝟎𝟐𝟒 𝟐𝟎𝟐𝟒 𝟐𝟎𝟐𝟒 𝟐𝟎𝟐𝟒
is also a monotone strictly decreasing function, and 𝒈(𝒙) = 𝟏 is constant function
⇒ if: 𝒇(𝒙) = 𝒈(𝒙)has root then it is unique.
∗ We clearly see that 𝒙 = 𝟏 is root, then the given equation has only one root 𝒙 = 𝟏
612. Solve for real numbers:

𝟗𝟐𝒙𝟐 − 𝟖𝟎 𝟗𝟐𝒙𝟐 + 𝟐𝟖𝟕 𝟒 𝟐𝟑 + 𝟏𝟐𝒙


√ + √ =√ (𝒙 ∈ ℝ)
𝟑𝟔𝟕 𝟑𝟔𝟕 𝟐𝟑 − 𝟏𝟐𝒙
Proposed by Amir Sofi-Kosovo
Solution by Pham Duc Nam-Vietnam
𝟐𝟑 𝟓
𝑥 ∈ [− , −𝟐√ ]
𝟏𝟐 𝟐𝟑
𝟗𝟐𝒙𝟐 − 𝟖𝟎 𝟗𝟐𝒙𝟐 + 𝟐𝟖𝟕 𝟒 𝟐𝟑 + 𝟏𝟐𝒙
√ +√ =√ (𝒙 ∈ ℝ),
𝟑𝟔𝟕 𝟑𝟔𝟕 𝟐𝟑 − 𝟏𝟐𝒙
𝟓 𝟐𝟑
𝑥 ∈ [𝟐√ , )
𝟐𝟑 𝟏𝟐
[
𝟗𝟐𝒙𝟐 − 𝟖𝟎
𝑎=√
𝟑𝟔𝟕 𝟗𝟐𝒙𝟐 − 𝟖𝟎 𝟗𝟐𝒙𝟐 + 𝟐𝟖𝟕
𝟐 𝟐
∗ Let: ⇒𝒂 −𝒃 = − = −𝟏
𝟑𝟔𝟕 𝟑𝟔𝟕
𝟗𝟐𝒙𝟐 + 𝟐𝟖𝟕
𝑏=√
{ 𝟑𝟔𝟕
𝟒𝟐𝟑 + 𝟏𝟐𝒙 𝟒 𝟐𝟑 + 𝟏𝟐𝒙
𝟒 𝟐𝟑 − 𝟏𝟐𝒙 𝟐𝟑
⇔ {𝑎 + 𝑏 =
√ 𝑎+𝑏= √
𝟐𝟑 − 𝟏𝟐𝒙 ⇔ { 𝟐𝟑 − 𝟏𝟐𝒙 ⇒ 𝒂 − 𝒃 = − √𝟐𝟑 + 𝟏𝟐𝒙 (𝒙 ≠ − 𝟏𝟐)
𝒂𝟐 − 𝒃𝟐 = −𝟏 (𝒂 + 𝒃)(𝒂 − 𝒃) = −𝟏

14 RMM-ABSTRACT ALGEBRA MARATHON 601-700


www.ssmrmh.ro
𝟒 𝟐𝟑 + 𝟏𝟐𝒙
𝑎+𝑏 = √
𝟐𝟑 − 𝟏𝟐𝒙 𝟒 𝟐𝟑 + 𝟏𝟐𝒙 𝟒 𝟐𝟑 − 𝟏𝟐𝒙
⇔ ⇒ 𝟐𝒂 = √ −√ ⇔
𝟒 𝟐𝟑 − 𝟏𝟐𝒙
𝟐𝟑 − 𝟏𝟐𝒙 𝟐𝟑 + 𝟏𝟐𝒙
𝑎 − 𝒃 = −√
{ 𝟐𝟑 + 𝟏𝟐𝒙

𝟗𝟐𝒙𝟐 − 𝟖𝟎 𝟒 𝟐𝟑 + 𝟏𝟐𝒙 𝟒 𝟐𝟑 − 𝟏𝟐𝒙


𝟐√ =√ −√
𝟑𝟔𝟕 𝟐𝟑 − 𝟏𝟐𝒙 𝟐𝟑 + 𝟏𝟐𝒙

𝟗𝟐𝒙𝟐 − 𝟖𝟎 𝟐𝟑 + 𝟏𝟐𝒙 𝟐𝟑 − 𝟏𝟐𝒙 𝟑𝟔𝟖𝒙𝟐 + 𝟒𝟏𝟒


⇔ 𝟒( )=√ +√ −𝟐⇔
𝟑𝟔𝟕 𝟐𝟑 − 𝟏𝟐𝒙 𝟐𝟑 + 𝟏𝟐𝒙 𝟑𝟔𝟕
𝟐
𝟐𝟑 + 𝟏𝟐𝒙 𝟐𝟑 − 𝟏𝟐𝒙 𝟑𝟔𝟖𝒙𝟐 + 𝟒𝟏𝟒 𝟐𝟑 + 𝟏𝟐𝒙 𝟐𝟑 − 𝟏𝟐𝒙
=√ +√ ⇔( ) = + +𝟐
𝟐𝟑 − 𝟏𝟐𝒙 𝟐𝟑 + 𝟏𝟐𝒙 𝟑𝟔𝟕 𝟐𝟑 − 𝟏𝟐𝒙 𝟐𝟑 + 𝟏𝟐𝒙
𝟐 𝟐
𝟑𝟔𝟖𝒙𝟐 + 𝟒𝟏𝟒 𝟐𝟏𝟏𝟔 𝟖𝒙𝟐 + 𝟗 𝟏
⇔( ) = ⇔ ( ) =
𝟑𝟔𝟕 𝟓𝟐𝟗 − 𝟏𝟒𝟒𝒙𝟐 𝟑𝟔𝟕 𝟓𝟐𝟗 − 𝟏𝟒𝟒𝒙𝟐
⇔ 𝟏𝟒𝟒𝒙 − 𝟐𝟎𝟓𝒙 − 𝟏𝟎𝟎𝟖𝒙 + 𝟏𝟒𝟑𝟓 = 𝟎 ⇔ 𝟏𝟒𝟒𝒕 − 𝟐𝟎𝟓𝒕𝟐 − 𝟏𝟎𝟎𝟖𝒕 + 𝟏𝟒𝟑𝟓
𝟔 𝟒 𝟐 𝟑

= 𝟎 ∴ 𝒕 = 𝒙𝟐 > 0
𝒕𝟐 = 𝟕 𝒙𝟒 = 𝟕
𝟐 ( ) 𝟐𝟎𝟓
⇔ (𝒕 − 𝟕) 𝟏𝟒𝟒𝒕 − 𝟐𝟎𝟓 = 𝟎 ⇔ [ ⇔ [ 𝟐 𝟐𝟎𝟓
𝑡= 𝒙 =
𝟏𝟒𝟒 𝟏𝟒𝟒
𝟒 𝟒
𝑥 = ± √𝟕 𝑪𝒉𝒆𝒄𝒌 𝒃𝒚 𝒅𝒊𝒓𝒆𝒄𝒕𝒍𝒚 𝒄𝒐𝒎𝒑𝒖𝒕𝒆
𝑥 = √𝟕
⇔[ √𝟐𝟎𝟓 → [ √𝟐𝟎𝟓
𝑥=± 𝑥=
𝟏𝟐 𝟏𝟐
613. Solve for real numbers:
𝟑 𝟑
√𝒙𝟑 + 𝒙𝟐 + 𝟗𝒙 − 𝟏 + √𝒙𝟑 − 𝟓𝒙𝟐 + 𝟗𝒙 − 𝟑 = 𝟐𝒙
Proposed by Amir Sofi-Kosovo
Solution by Pham Duc Nam-Vietnam
𝟑 𝟑
√𝒙𝟑 + 𝒙𝟐 + 𝟗𝒙 − 𝟏 + √𝒙𝟑 − 𝟓𝒙𝟐 + 𝟗𝒙 − 𝟑 = 𝟐𝒙
𝟑 𝟑
⇔ √𝒙𝟑 + 𝒙𝟐 + 𝟗𝒙 − 𝟏 − (𝒙 + 𝟏) + √𝒙𝟑 − 𝟓𝒙𝟐 + 𝟗𝒙 − 𝟑 − (𝒙 − 𝟏) = 𝟎
𝒙𝟑 + 𝒙𝟐 + 𝟗𝒙 − 𝟏 − (𝒙 + 𝟏)𝟑
⇔ 𝟑 𝟑
√(𝒙𝟑 + 𝒙𝟐 + 𝟗𝒙 − 𝟏)𝟐 + (𝒙 + 𝟏) √𝒙𝟑 + 𝒙𝟐 + 𝟗𝒙 − 𝟏 + (𝒙 + 𝟏)𝟐
𝒙𝟑 − 𝟓𝒙𝟐 + 𝟗𝒙 − 𝟑 − (𝒙 − 𝟏)𝟑
+𝟑 𝟑
=𝟎
√(𝒙𝟑 − 𝟓𝒙𝟐 + 𝟗𝒙 − 𝟑)𝟐 + (𝒙 − 𝟏) √𝒙𝟑 − 𝟓𝒙𝟐 + 𝟗𝒙 − 𝟑 + (𝒙 − 𝟏)𝟐

15 RMM-ABSTRACT ALGEBRA MARATHON 601-700


www.ssmrmh.ro
−𝟐(𝒙𝟐 − 𝟑𝒙 + 𝟏)
⇔ 𝟑 𝟑
√(𝒙𝟑 + 𝒙𝟐 + 𝟗𝒙 − 𝟏)𝟐 + (𝒙 + 𝟏) √𝒙𝟑 + 𝒙𝟐 + 𝟗𝒙 − 𝟏 + (𝒙 + 𝟏)𝟐
−𝟐(𝒙𝟐 − 𝟑𝒙 + 𝟏)
+𝟑 𝟑
=𝟎
√(𝒙𝟑 − 𝟓𝒙𝟐 + 𝟗𝒙 − 𝟑)𝟐 + (𝒙 − 𝟏) √𝒙𝟑 − 𝟓𝒙𝟐 + 𝟗𝒙 − 𝟑 + (𝒙 − 𝟏)𝟐
𝟏
𝟑 𝟑
+
√(𝒙𝟑 + 𝒙𝟐 + 𝟗𝒙 − 𝟏)𝟐 + (𝒙 + 𝟏) √𝒙𝟑 + 𝒙𝟐 + 𝟗𝒙 − 𝟏 + (𝒙 + 𝟏)𝟐
⇔ −𝟐(𝒙𝟐 − 𝟑𝒙 + 𝟏) =𝟎
𝟏
𝟑 𝟑
[ √(𝒙𝟑 − 𝟓𝒙𝟐 + 𝟗𝒙 − 𝟑)𝟐 + (𝒙 − 𝟏) √𝒙𝟑 − 𝟓𝒙𝟐 + 𝟗𝒙 − 𝟑 + (𝒙 − 𝟏)𝟐 ]

≠𝟎

𝟑 ± √𝟓
⇔ 𝒙𝟐 − 𝟑𝒙 + 𝟏 = 𝟎 ⇔ 𝒙 =
𝟐
614. Solve for natural numbers:
𝒙𝟒 − 𝒚𝟔 − 𝒛𝟔 − 𝟏𝟒𝒙𝟐 + 𝟐𝒚𝟑 + 𝟐𝒛𝟑 − 𝟐𝒚𝟑 𝒛𝟑 + 𝟒𝟓 = 𝟎
Proposed by Elsen Kerimov-Azerbaijan
Solution 1 by Bedri Hajrizi-Mitrovica-Kosovo
𝑳𝒆𝒕 𝒙𝟐 = 𝒂, 𝒚𝟑 = 𝒃, 𝒛𝟑 = 𝒄, 𝒕𝒉𝒆𝒏
𝒂𝟐 − 𝟏𝟒𝒂 − 𝒃𝟐 − 𝟐𝒃𝒄 − 𝒄𝟐 + 𝟐𝒃 + 𝟐𝒄 + 𝟒𝟓 = 𝟎
(𝒂 − 𝟕)𝟐 + 𝟐(𝒃 + 𝒄) = (𝒃 + 𝒄)𝟐 + 𝟒
(𝒂 − 𝟕)𝟐 − 𝟐𝟐 = (𝒃 + 𝒄)(𝒃 + 𝒄 − 𝟐)
(𝒂 − 𝟕 + 𝟐)(𝒂 − 𝟕 − 𝟐) = (𝒃 + 𝒄)(𝒃 + 𝒄 − 𝟐)
(𝒂 − 𝟓)(𝒂 − 𝟗) = (𝒃 + 𝒄)(𝒃 + 𝒄 − 𝟐)
𝑳𝒆𝒕: 𝒂 − 𝟗 = 𝒑, 𝒃 + 𝒄 − 𝟐 = 𝒒, 𝒕𝒉𝒆𝒏: 𝒑(𝒑 + 𝟒) = 𝒒(𝒒 + 𝟐)
𝒑𝟐 + 𝟒𝒑 + 𝟒 − 𝟒 = 𝒒𝟐 + 𝟐𝒒 + 𝟏 − 𝟏
(𝒑 + 𝟐)𝟐 − 𝟒 = (𝒒 + 𝟏)𝟐 − 𝟏, ( 𝒑 + 𝟐) 𝟐 − ( 𝒒 + 𝟏) 𝟐 = 𝟑
(𝒑 + 𝟐 + 𝒒 + 𝟏)(𝒑 + 𝟐 − 𝒒 − 𝟏) = 𝟑
(𝒑 + 𝒒 + 𝟑)(𝒑 − 𝒒 + 𝟏) = 𝟑
𝒑+𝒒+𝟑 =𝟑
𝟏) { ⇒ 𝒑 = 𝒒 = 𝟎 𝒂𝒏𝒅 𝒉𝒆𝒏𝒄𝒆,
𝒑−𝒒+𝟏 =𝟏
𝒂 = 𝟗, 𝒃 = 𝒄 = 𝟏 𝒐𝒓 𝒙 = 𝟑, 𝒚 = 𝒛 = 𝟏.
𝒑+𝒒+𝟑 =𝟏 𝒑+𝒒=𝟐 𝒑=𝟎
𝟐) { ⇒{ ⇒{ 𝒂𝒏𝒅 𝒉𝒆𝒏𝒄𝒆,
𝒑−𝒒+𝟏 =𝟑 𝒑−𝒒=𝟐 𝒒 = −𝟐

16 RMM-ABSTRACT ALGEBRA MARATHON 601-700


www.ssmrmh.ro
𝒂 = 𝟗, 𝒃 + 𝒄 = 𝟎. 𝑵𝒐 𝒔𝒐𝒍𝒖𝒕𝒊𝒐𝒏!
𝒑 + 𝒒 + 𝟑 = −𝟑 𝒑 + 𝒒 = −𝟔 𝒑 = −𝟒
𝟑) { ⇒{ ⇒{ 𝒂𝒏𝒅 𝒉𝒆𝒏𝒄𝒆,
𝒑 − 𝒒 + 𝟏 = −𝟏 𝒑 − 𝒒 = −𝟐 𝒒 = −𝟐
𝒂 = 𝟓, 𝒃 + 𝒄 = 𝟎. 𝑵𝒐 𝒔𝒐𝒍𝒖𝒕𝒊𝒐𝒏!
𝒑 + 𝒒 + 𝟑 = −𝟏 𝒑 + 𝒒 = −𝟐 𝒑 = −𝟒
𝟒) { ⇒{ ⇒{ 𝒂𝒏𝒅 𝒉𝒆𝒏𝒄𝒆,
𝒑 − 𝒒 + 𝟏 = −𝟑 𝒑 − 𝒒 = −𝟒 𝒒=𝟎
𝒂 = 𝟓, 𝒃 + 𝒄 = 𝟎. 𝑵𝒐 𝒔𝒐𝒍𝒖𝒕𝒊𝒐𝒏.
Solution 2 by Jose Ferreira Queiroz-Olinda-Brazil
𝒙𝟒 − 𝒚𝟔 − 𝒛𝟔 − 𝟏𝟒𝒙𝟐 + 𝟐𝒚𝟑 + 𝟐𝒛𝟑 − 𝟐𝒚𝟑 𝒛𝟑 + 𝟒𝟓 = 𝟎
(𝒙𝟐 − 𝟕)𝟐 − (𝒚𝟑 + 𝒛𝟑 − 𝟏)𝟐 = 𝟑; 𝒙, 𝒚, 𝒛 ∈ ℕ
( 𝒙 𝟐 − 𝟕) 𝟐 = 𝟐𝟐 𝒙=𝟑
{ ⇒{ 𝟑
( 𝒚𝟑 + 𝒛 𝟐 − 𝟏) 𝟐 = 𝟏𝟐 𝒚 + 𝒛𝟑 − 𝟏 = ±𝟏
𝟏) 𝒚𝟑 + 𝒛𝟑 = 𝟐 ⇔ (𝒚 + 𝒛)(𝒚𝟐 − 𝒚𝒛 + 𝒛𝟐 ) = 𝟐 ⇒ 𝒚 = 𝒛 = 𝟏.
𝟐) 𝒚𝟑 + 𝒛𝟑 = 𝟎 ⇒ 𝒚 = 𝒛 = 𝟎. 𝑺𝒐𝒍𝒖𝒕𝒊𝒐𝒏𝒔 (𝟑, 𝟏, 𝟏).
615. 𝐅𝐢𝐧𝐝 𝐚𝐥𝐥 (𝒙, 𝒚, 𝒛) ∈ [𝟎,∞) × [𝟎,∞) × [𝟎,∞) 𝐬𝐮𝐜𝐡 𝐭𝐡𝐚𝐭 ∶

√𝒙𝒚(𝒙 + 𝒚) + √𝒚𝒛(𝒚 + 𝒛) + √𝒛𝒙(𝒛 + 𝒙) = √(𝒙 + 𝒚)(𝒚 + 𝒛)(𝒛 + 𝒙) + √𝟐𝒙𝒚𝒛

Proposed by Daniel Sitaru-Romania


Solution by Mohamed Amine Ben Ajiba-Tanger-Morocco
𝐈𝐟 𝒙𝒚𝒛 = 𝟎, 𝑾𝑳𝑶𝑮, 𝐚𝐬𝐬𝐮𝐦𝐞 𝐭𝐡𝐚𝐭 𝒛 = 𝟎, 𝐭𝐡𝐞 𝐞𝐪𝐮𝐚𝐭𝐢𝐨𝐧 𝐛𝐞𝐜𝐨𝐦𝐞𝐬 ∶

√𝒙𝒚(𝒙 + 𝒚) = √𝒙𝒚(𝒙 + 𝒚)

𝐭𝐡𝐞𝐧 ∶ (𝒖, 𝒗, 𝟎), (𝒖, 𝟎, 𝒗), (𝟎, 𝒖, 𝒗), 𝒖, 𝒗 ≥ 𝟎, 𝐚𝐫𝐞 𝐬𝐨𝐥𝐮𝐭𝐢𝐨𝐧𝐬 𝐨𝐟 𝐭𝐡𝐞 𝐞𝐪𝐮𝐚𝐭𝐢𝐨𝐧.

𝐀𝐬𝐬𝐮𝐦𝐞 𝐧𝐨𝐰 𝐭𝐡𝐚𝐭 𝒙𝒚𝒛 > 0. 𝐓𝐡𝐞 𝐝𝐞𝐬𝐢𝐫𝐞𝐝 𝐞𝐪𝐮𝐚𝐭𝐢𝐨𝐧 𝐢𝐬 𝐞𝐪𝐮𝐢𝐯𝐚𝐥𝐞𝐧𝐭 𝐭𝐨:

𝒙𝒚 𝒚𝒛 𝒛𝒙 𝟐𝒙𝒚𝒛
√( +√ +√ = 𝟏+√
𝒚 + 𝒛)(𝒛 + 𝒙) (𝒛 + 𝒙)(𝒙 + 𝒚) (𝒙 + 𝒚)(𝒚 + 𝒛) (𝒙 + 𝒚)(𝒚 + 𝒛)(𝒛 + 𝒙)

𝐋𝐞𝐭

𝟏 𝟏 𝟏 𝟏 𝟏 𝟏
𝒂 = √ + ,𝒃 = √ + ,𝒄 = √ + .
𝒚 𝒛 𝒛 𝒙 𝒙 𝒚

17 RMM-ABSTRACT ALGEBRA MARATHON 601-700


www.ssmrmh.ro
𝟏 𝒃𝟐 + 𝒄𝟐 − 𝒂𝟐
𝐖𝐞 𝐡𝐚𝐯𝐞 ∶ = > 0 (𝐚𝐧𝐝 𝐚𝐧𝐚𝐥𝐨𝐠𝐬),
𝒙 𝟐

𝐭𝐡𝐞𝐧 𝒂, 𝒃, 𝒄 𝐜𝐚𝐧 𝐛𝐞 𝐭𝐡𝐞 𝐬𝐢𝐝𝐞𝐬 𝐨𝐟 𝐚𝐧 𝐚𝐜𝐮𝐭𝐞 𝐭𝐫𝐢𝐚𝐧𝐠𝐥𝐞 𝑨𝑩𝑪 𝐚𝐧𝐝,

𝒙𝒚 𝟏 𝒚𝒛 𝒛𝒙 𝒂𝟐 + 𝒃𝟐 − 𝒄𝟐
√( = .√ .√ = = 𝐜𝐨𝐬 𝑪 (𝐚𝐧𝐝 𝐚𝐧𝐚𝐥𝐨𝐠𝐬)
𝒚 + 𝒛)(𝒛 + 𝒙) 𝒛 𝒚 + 𝒛 𝒛 + 𝒙 𝟐𝒂𝒃

𝐒𝐨 𝐭𝐡𝐞 𝐝𝐞𝐬𝐢𝐫𝐞𝐝 𝐞𝐪𝐮𝐚𝐭𝐢𝐨𝐧 𝐛𝐞𝐜𝐨𝐦𝐞𝐬 ∶

𝐜𝐨𝐬 𝑨 + 𝐜𝐨𝐬 𝑩 + 𝐜𝐨𝐬 𝑪 = 𝟏 + √𝟐 𝐜𝐨𝐬 𝑨 𝐜𝐨𝐬 𝑩 𝐜𝐨𝐬 𝑪.

𝑩𝒖𝒕 𝒘𝒆 𝒉𝒂𝒗𝒆:

𝑮𝒆𝒓𝒓𝒆𝒕𝒔𝒆𝒏
𝒔𝟐 − (𝟐𝑹 + 𝒓)𝟐 (𝟒𝑹𝟐 + 𝟒𝑹𝒓 + 𝟑𝒓𝟐 ) − (𝟐𝑹 + 𝒓)𝟐 𝒓𝟐
𝐜𝐨𝐬 𝑨 𝐜𝐨𝐬 𝑩 𝐜𝐨𝐬 𝑪 = ⏞
≤ = ,
𝟒𝑹𝟐 𝟒𝑹𝟐 𝟐𝑹𝟐
𝒓
𝒕𝒉𝒆𝒏 ∶ 𝟏 + √𝟐 𝐜𝐨𝐬 𝑨 𝐜𝐨𝐬 𝑩 𝐜𝐨𝐬 𝑪 ≤ 𝟏 + = 𝐜𝐨𝐬 𝑨 + 𝐜𝐨𝐬 𝑩 + 𝐜𝐨𝐬 𝑪,
𝑹

𝒘𝒊𝒕𝒉 𝒆𝒒𝒖𝒂𝒍𝒊𝒕𝒚 𝒘𝒉𝒆𝒏 ∆𝑨𝑩𝑪 𝒊𝒔 𝒆𝒒𝒖𝒊𝒍𝒂𝒕𝒆𝒓𝒂𝒍 ⇔ 𝒂 = 𝒃 = 𝒄 ⇔ 𝒙 = 𝒚 = 𝒛.

𝑻𝒉𝒆𝒓𝒆𝒇𝒐𝒓𝒆, 𝑺 = {(𝒖, 𝒗, 𝟎), (𝒖, 𝟎, 𝒗), (𝟎, 𝒖, 𝒗), (𝒖, 𝒖, 𝒖)|𝒖, 𝒗 ≥ 𝟎}.

616. Solve for real numbers:


𝟑√𝟑
𝟔𝒙 − 𝒙𝟐 =
√𝒙𝟒 − 𝟏𝟐𝒙𝟑 + 𝟑𝟔𝒙𝟐 + 𝟏 + 𝟏
Proposed by Amir Sofi-Kosovo
Solution by Pham Duc Nam-Vietnam
𝟑√𝟑
𝟔𝒙 − 𝒙𝟐 =
√𝒙𝟒 − 𝟏𝟐𝒙𝟑 + 𝟑𝟔𝒙𝟐 + 𝟏 + 𝟏
𝟑√𝟑 𝟑√𝟑
⇔ 𝟔𝒙 − 𝒙𝟐 = ⇔ let: 𝒕 = 𝟔𝒙 − 𝒙𝟐 ⇔ 𝒕 = ⇒𝒕>0
√(𝟔𝒙 − 𝒙𝟐 )𝟐 + 𝟏 + 𝟏 √𝒕𝟐 + 𝟏 + 𝟏
𝟐 𝟐
⇔ 𝒕 (√𝒕𝟐 + 𝟏 + 𝟏) = 𝟑√𝟑 ⇔ (𝒕√𝒕𝟐 + 𝟏) = (𝟑√𝟑 − 𝒕)
⇔ 𝒕𝟒 + 𝟔√𝟑𝒕 − 𝟐𝟕 = 𝟎 ⇔ (𝒕 − √𝟑) (𝒕 𝟑 𝟐
⏟ + √𝟑𝒕 + 𝟑𝒕 + 𝟗√𝟑) = 𝟎
>0∀𝒕>0

⇔ 𝒕 = √𝟑 ⇔ −𝒙𝟐 + 𝟔𝒙 − √𝟑 = 𝟎 ⇔ 𝒙 = 𝟑 ± √𝟗 − √𝟑

18 RMM-ABSTRACT ALGEBRA MARATHON 601-700


www.ssmrmh.ro
617. Solve for real numbers:
𝟏 + √𝟏 − 𝒙
= 𝟐𝒙
𝒙 − √𝟏 − 𝒙𝟐
Proposed by Amir Sofi-Kosovo
Solution by Pham Duc Nam-Vietnam
∗ 𝑫𝒐𝒎𝒂𝒊𝒏: −𝟏 < 𝑥 < 1
𝒕
∗ −𝟏 < 𝑥 < 1 ⇒ ∃𝒕 ∈ (𝟎, 𝝅): 𝒙 = 𝒄𝒐𝒔 𝒕 , 𝒕 ∈ (𝟎, 𝝅) ⇒ 𝒔𝒊𝒏 𝒕 , 𝒔𝒊𝒏 >0
𝟐
𝒕
𝟏 + √𝟏 − 𝒄𝒐𝒔 𝒕 𝟏 + √𝟐 𝒔𝒊𝒏𝟐 𝟐 𝝅
⇔ = 𝟐 𝒄𝒐𝒔 𝒕 ⇔ = 𝟐 𝒄𝒐𝒔 𝒕 ∴ 𝒕 ≠
𝒄𝒐𝒔 𝒕 − √𝟏 − 𝒄𝒐𝒔𝟐 𝒕 𝒄𝒐𝒔 𝒕 − 𝒔𝒊𝒏 𝒕 𝟒
𝒕 𝒕
⇔ 𝟏 + √𝟐 𝒔𝒊𝒏𝟐 = 𝟐 𝒄𝒐𝒔 𝒕 (𝒄𝒐𝒔 𝒕 − 𝒔𝒊𝒏 𝒕) ⇔ √𝟐 𝒔𝒊𝒏 = 𝟐 𝒄𝒐𝒔𝟐 𝒕 − 𝟏 − 𝒔𝒊𝒏 𝟐 𝒕
𝟐 𝟐
𝒕 𝝅 𝒕 𝝅
⇔ √𝟐 𝒔𝒊𝒏 = 𝒄𝒐𝒔 𝟐 𝒕 − 𝒔𝒊𝒏 𝟐 𝒕 = √𝟐 𝒔𝒊𝒏 ( − 𝟐𝒕) ⇔ 𝒔𝒊𝒏 = 𝒔𝒊𝒏 ( − 𝟐𝒕)
𝟐 𝟒 𝟐 𝟒
𝒕 𝝅 𝝅 𝒌𝟒𝝅
= − 𝟐𝒕 + 𝒌𝟐𝝅 𝑡= +
⇔ [𝒕𝟐 𝟒 ⇔[ 𝟏𝟎 𝟓 (𝒌 ∈ ℤ)
𝝅 𝝅 𝒌𝟒𝝅
= 𝝅 − ( − 𝟐𝒕) + 𝒌𝟐𝝅 𝑡=− −
𝟐 𝟒 𝟐 𝟑
𝝅 𝟗𝝅 𝟓𝝅 𝝅 𝟗𝝅 𝟓𝝅
∗ 𝒕 ∈ (𝟎, 𝝅) ⇒ 𝒕 = , , ⇒ 𝒙 = 𝒄𝒐𝒔 , 𝒙 = 𝒄𝒐𝒔 , 𝒙 = 𝒄𝒐𝒔
𝟏𝟎 𝟏𝟎 𝟔 𝟏𝟎 𝟏𝟎 𝟔
𝟏 𝟓 + √𝟓 √𝟑
⇔𝒙=± √ ,𝒙 = −
𝟐 𝟐 𝟐
618. Solve for real numbers:
𝟏 𝟏 𝟏 𝟏
+ 𝟒 = 𝟔 +
𝒙(𝒙 + 𝟏) 𝒙 + 𝟏 𝒙 + 𝒙𝟓 − 𝒙𝟒 + 𝟏 𝟐
Proposed by Daniel Sitaru-Romania
Solution 1 by Christos Tsifakis-Greece
𝟏 𝟏 𝟏 𝟏
+ 𝟒 = 𝟔 + ; (𝒙 ∉ {𝟎, 𝟏}) ⇔
𝒙 ( 𝒙 + 𝟏) 𝒙 + 𝟏 𝒙 + 𝒙 𝟓 − 𝒙 𝟒 + 𝟏 𝟐
𝒙𝟏𝟐 + 𝟐𝒙𝟏𝟏 − 𝟐𝒙𝟏𝟎 − 𝟑𝒙𝟗 + 𝒙𝟖 − 𝟐𝒙𝟕 + 𝒙𝟔 + 𝟐𝒙𝟓 + 𝒙𝟐 + 𝒙 − 𝟐 = 𝟎 ⇔
(𝒙 − 𝟏)𝟑 (𝒙 + 𝟏)(𝒙𝟖 + 𝟒𝒙𝟕 + 𝟔𝒙𝟔 + 𝟕𝒙𝟓 + 𝟖𝒙𝟒 + 𝟔𝒙𝟑 + 𝟓𝒙𝟐 + 𝟑𝒙 + 𝟐) = 𝟎 ⇔
(𝒙 − 𝟏)𝟑 (𝒙 + 𝟏)(𝒙 + 𝟐)(𝒙𝟕 + 𝟐𝒙𝟔 + 𝟐𝒙𝟓 + 𝟑𝒙𝟒 + 𝟐𝒙𝟑 + 𝟐𝒙𝟐 + 𝒙 + 𝟏) = 𝟎 ⇔
(𝒙 − 𝟏)𝟑 (𝒙 + 𝟏)(𝒙 + 𝟐)(𝒙𝟐 + 𝟏)(𝒙𝟓 + 𝟐𝒙𝟒 + 𝒙𝟑 + 𝒙𝟐 + 𝒙 + 𝟏) = 𝟎 ⇔
𝒙 = 𝟏 𝒕𝒓𝒊𝒑𝒍𝒆, 𝒙 = −𝟐

19 RMM-ABSTRACT ALGEBRA MARATHON 601-700


www.ssmrmh.ro
𝑳𝒆𝒕 𝒇 𝒙 = 𝒙 + 𝟐𝒙𝟒 + 𝒙𝟑 + 𝒙𝟐 + 𝒙 + 𝟏, 𝒙 ∈ ℝ
( ) 𝟓

𝟑 𝟐𝟗
𝒇(−𝟐) = −𝟓 < 0 𝑎𝑛𝑑 𝑓 (− ) = > 0 𝑓𝑟𝑜𝑚 𝐵𝑜𝑙𝑧𝑎𝑛𝒐′ 𝒔 𝒍𝒆𝒎𝒎𝒂
𝟐 𝟑𝟐
𝟑
(∃)𝒙𝟎 ∈ (−𝟐, − ) , 𝒇(𝒙𝟎 ) = 𝟎
𝟐
𝑺𝒐, 𝑺 = {−𝟐, 𝒙𝟎 , 𝟏}.
Solution 2 by Pham Duc Nam-Vietnam
𝟏 𝟏 𝟏 𝟏
+ 𝟒 = 𝟔 +
𝒙 ( 𝒙 + 𝟏) 𝒙 + 𝟏 𝒙 + 𝒙 𝟓 − 𝒙 𝟒 + 𝟏 𝟐
𝟏. 𝑪𝒐𝒏𝒅𝒊𝒕𝒊𝒐𝒏: 𝒙 ≠ 𝟎, 𝒙 ≠ 𝟏, 𝒙 ≠ 𝒙(∗) , 𝒘𝒉𝒆𝒓𝒆 𝒙(∗) − 𝒊𝒔 𝒓𝒐𝒐𝒕 𝒐𝒇 𝒙𝟓 − 𝒙𝟑 + 𝒙𝟐 − 𝒙 + 𝟏 = 𝟎
𝟏 𝟏 𝟏 𝟏
𝟐. + 𝟒 = 𝟔 + ⇔
𝒙 ( 𝒙 + 𝟏) 𝒙 + 𝟏 𝒙 + 𝒙 𝟓 − 𝒙 𝟒 + 𝟏 𝟐
𝒙𝟒 + 𝟏 + 𝒙𝟐 + 𝒙 𝟐 + 𝒙𝟔 + 𝒙𝟓 − 𝒙𝟒 + 𝟏
+ ⇔
𝒙(𝒙 + 𝟏)(𝒙𝟒 + 𝟏) 𝟐(𝒙 + 𝟏)(𝒙𝟓 − 𝒙𝟑 + 𝒙𝟐 − 𝒙 + 𝟏)
𝒙𝟒 + 𝒙 𝟐 + 𝒙 + 𝟏 𝒙𝟔 + 𝒙 𝟓 − 𝒙𝟒 + 𝟑
= ⇔
𝒙 ( 𝒙 𝟒 + 𝟏) 𝟐( 𝒙 𝟓 − 𝒙 𝟑 + 𝒙 𝟐 − 𝒙 + 𝟏)
𝒙𝟕 + 𝟑𝒙𝟔 + 𝒙𝟓 − 𝟐𝒙𝟒 − 𝒙 − 𝟐 = 𝟎
(𝒙 − 𝟏)(𝒙 + 𝟐)(𝒙𝟓 + 𝟐𝒙𝟒 + 𝒙𝟑 + 𝒙𝟐 + 𝒙 + 𝟏) = 𝟎
{ 𝒙=𝟏
𝒙 = −𝟐
𝒙𝟓 + 𝟐𝒙𝟒 + 𝒙𝟑 + 𝒙𝟐 + 𝒙 + 𝟏 = 𝟎
𝟑. 𝑳𝒆𝒕 𝒇(𝒙) = 𝒙𝟓 + 𝟐𝒙𝟒 + 𝒙𝟑 + 𝒙𝟐 + 𝒙 + 𝟏 𝒉𝒂𝒔 𝑮𝒂𝒍𝒐𝒊𝒔 𝒈𝒓𝒐𝒖𝒑 𝒊𝒔 𝑺𝟓 , 𝒂𝒏𝒅 𝒐𝒓𝒅𝒆𝒓 𝟏𝟐𝟎
⇒ 𝒇(𝒙) 𝒊𝒔 𝒊𝒓𝒓𝒆𝒅𝒖𝒄𝒊𝒃𝒍𝒆 𝒂𝒏𝒅 𝒖𝒏𝒔𝒐𝒍𝒗𝒂𝒃𝒍𝒆, 𝒕𝒉𝒆𝒏 𝒇(𝒙) = 𝟎 𝒉𝒂𝒔 𝒐𝒏𝒆 𝒓𝒐𝒐𝒕 𝒂𝒏𝒅 𝒕𝒉𝒊𝒔 𝒓𝒐𝒐𝒕
𝒄𝒂𝒏𝒏𝒐𝒕 𝒃𝒆 𝒆𝒙𝒑𝒓𝒆𝒔𝒔𝒆𝒅 𝒃𝒚 𝒓𝒂𝒅𝒊𝒄𝒂𝒍𝒔.
𝒇(−𝟏) = 𝟏, 𝒇(−𝟐) = −𝟓 ⇒ 𝒇(−𝟏)𝒇(−𝟐) < 0 𝑡ℎ𝑒𝑛 𝑡ℎ𝑠 𝑟𝑜𝑜𝑡 𝑙𝑖𝑒
𝒊𝒏 (−𝟐, −𝟏), 𝒘𝒆 𝒄𝒂𝒍𝒍 𝒕𝒉𝒊𝒔 𝒓𝒐𝒐𝒕 𝒊𝒔 𝒙(∗∗).
𝟑
𝑩𝒚 𝒖𝒔𝒊𝒏𝒈 𝑵𝒆𝒘𝒕𝒐𝒏 − 𝑹𝒂𝒑𝒉𝒔𝒐𝒏 𝒎𝒆𝒕𝒉𝒐𝒅, 𝒘𝒆 𝒑𝒊𝒄𝒌 𝒙𝟎 = − ∈ (−𝟐, −𝟏) 𝒂𝒏𝒅 𝒔𝒆𝒕:
𝟐
𝒇(𝒙𝟎 ) 𝒇(𝒙𝒏 )
𝒙𝟏 = 𝒙𝟎 − , 𝒕𝒉𝒆𝒏 𝒙𝒏+𝟏 = 𝒙𝒏 − ′
𝒇′ (𝒙𝟎 ) 𝒇 ( 𝒙𝒏 )
𝑨 𝒏𝒖𝒎𝒆𝒓𝒊𝒄𝒂𝒍 𝒔𝒐𝒍𝒖𝒕𝒊𝒐𝒏 𝒊𝒔 𝒙 ≅ −𝟏. 𝟔𝟕𝟑𝟔𝟒𝟖𝟓𝟒𝟔.
𝑻𝒉𝒆𝒏 𝒕𝒉𝒆 𝒈𝒊𝒗𝒆𝒏 𝒆𝒒𝒖𝒂𝒍𝒕𝒊𝒐𝒏 𝒉𝒂𝒔 𝟑 𝒓𝒐𝒐𝒕𝒔: 𝒙 = 𝟏, 𝒙 = −𝟐, 𝒙(∗∗) .

20 RMM-ABSTRACT ALGEBRA MARATHON 601-700


www.ssmrmh.ro
619. Let 𝟎 ≤ 𝒂 ≤ 𝟗; 𝟎 ≤ 𝒃 ≤ 𝟏𝟓 fixed. Solve for real numbers:

√(𝟑𝒙 − 𝒂)(𝟗 − 𝒂) + √(𝟓𝒙 − 𝒃)(𝟏𝟓 − 𝒃) = 𝟒𝒙 + 𝟏𝟐 − 𝒂 − 𝒃


Proposed by Marin Chirciu-Romania
Solution 1 by Amir Sofi-Kosovo
𝟑𝒙 + 𝟗 − 𝟐𝒂
√(𝟑𝒙 − 𝒂)(𝟗 − 𝒂) ≤
𝟐
𝟓𝒙 + 𝟏𝟓 − 𝟐𝒃
√(𝟓𝒙 − 𝒃)(𝟏𝟓 − 𝒃) ≤
𝟐
⇒ √(𝟑𝒙 − 𝒂)(𝟗 − 𝒂) + √(𝟓𝒙 − 𝒃)(𝟏𝟓 − 𝒃) ≤ 𝟒𝒙 + 𝟏𝟐 − 𝒂 − 𝒃
𝟑𝒙 − 𝒂 = 𝟗 − 𝒂
𝑬𝒒𝒖𝒂𝒍𝒊𝒕𝒚 𝒉𝒐𝒍𝒅𝒔 𝒇𝒐𝒓 { ⇔ 𝒙 = 𝟑.
𝟓𝒙 − 𝒃 = 𝟏𝟓 − 𝒃
Solution 2 by Myagmarsuren Yadamsuren-Darkhan-Mongolia
𝟑𝒙 + 𝟗 − 𝟐𝒂
√(𝟑𝒙 − 𝒂)(𝟗 − 𝒂) ≤
𝟐
𝟓𝒙 + 𝟏𝟓 − 𝟐𝒃
√(𝟓𝒙 − 𝒃)(𝟏𝟓 − 𝒃) ≤
𝟐
⇒ √(𝟑𝒙 − 𝒂)(𝟗 − 𝒂) + √(𝟓𝒙 − 𝒃)(𝟏𝟓 − 𝒃) ≤ 𝟒𝒙 + 𝟏𝟐 − 𝒂 − 𝒃
𝟑𝒙 − 𝒂 = 𝟗 − 𝒂
𝑬𝒒𝒖𝒂𝒍𝒊𝒕𝒚 𝒉𝒐𝒍𝒅𝒔 𝒇𝒐𝒓 { ⇔ 𝒙 = 𝟑.
𝟓𝒙 − 𝒃 = 𝟏𝟓 − 𝒃
620. Solve for real numbers:
𝟏 𝟏
−𝟏 + √ (𝟐𝟒 + 𝟖√𝟐𝟏 𝒄𝒐𝒔 ( (𝟐𝝅 + 𝒂𝒓𝒄𝒕𝒂𝒏 𝒙)))
𝝅 𝟑 𝟑
𝒕𝒂𝒏 =
𝟏𝟒
𝟏 𝟏
√ (𝟐𝟒 + 𝟖√𝟐𝟏 𝒄𝒐𝒔 ( (𝟐𝝅 + 𝒂𝒓𝒄𝒕𝒂𝒏 𝒙)))
𝟑 𝟑
Proposed by Carlos Paiva-Brazil
Solution by Pham Duc Nam-Vietnam
𝟏 𝟏 𝝅 𝒚−𝟏 𝝅
∗ 𝑳𝒆𝒕: 𝒚 = √ (𝟐𝟒 + 𝟖√𝟐𝟏 𝒄𝒐𝒔 ( (𝟐𝝅 + 𝒂𝒓𝒄𝒕𝒂𝒏 𝒙))) ⇒ 𝒕𝒂𝒏 = ⇒ 𝒚 𝒕𝒂𝒏 = 𝒚−𝟏
𝟑 𝟑 𝟏𝟒 𝒚 𝟏𝟒

𝟏
𝒚= 𝝅
𝟏 − 𝒕𝒂𝒏 𝟏𝟒

21 RMM-ABSTRACT ALGEBRA MARATHON 601-700


www.ssmrmh.ro
𝟐
𝟏 𝟏 𝟏
⇒ (𝟐𝟒 + 𝟖√𝟐𝟏 𝒄𝒐𝒔 ( (𝟐𝝅 + 𝒂𝒓𝒄𝒕𝒂𝒏 𝒙))) = ( 𝝅)
𝟑 𝟑 𝟏 − 𝒕𝒂𝒏 𝟏𝟒
𝟐
𝟏 𝟏
⇔ 𝟐𝟒 + 𝟖√𝟐𝟏 𝒄𝒐𝒔 ( (𝟐𝝅 + 𝒂𝒓𝒄𝒕𝒂𝒏 𝒙)) = 𝟑 ( 𝝅)
𝟑 𝟏 − 𝒕𝒂𝒏 𝟏𝟒
𝟐
𝟏
𝟑( 𝝅 ) − 𝟐𝟒 𝟐 𝝅 𝝅
𝟏 𝟏 − 𝒕𝒂𝒏 𝟏𝟒 𝟏 𝟑 −𝟖 𝒕𝒂𝒏 𝟏𝟒 + 𝟏𝟔 𝒕𝒂𝒏 𝟏𝟒 − 𝟕
⇒ 𝒄𝒐𝒔 ( (𝟐𝝅 + 𝒂𝒓𝒄𝒕𝒂𝒏 𝒙)) = = √ ( )
𝟑 𝟖√𝟐𝟏 𝟖 𝟕 𝝅 𝟐
(𝟏 − 𝒕𝒂𝒏 )
𝟏𝟒
𝟐 𝝅 𝝅
𝟏 𝟏 𝟑 −𝟖 𝒕𝒂𝒏 𝟏𝟒 + 𝟏𝟔 𝒕𝒂𝒏 𝟏𝟒 − 𝟕
( )
⇒ 𝟐𝝅 + 𝒂𝒓𝒄𝒕𝒂𝒏 𝒙 = 𝒂𝒓𝒄𝒄𝒐𝒔 [ √ ( )] + 𝒌𝟐𝝅
𝟑 𝟖 𝟕 𝝅 𝟐
(𝟏 − 𝒕𝒂𝒏 )
𝟏𝟒
𝟏
∴ 𝒌 ∈ ℤ and (𝟐𝝅 + 𝒂𝒓𝒄𝒕𝒂𝒏 𝒙) > 0∀𝒙 ∈ ℝ
𝟑
𝟐 𝝅 𝝅
𝟏 𝟑 −𝟖 𝒕𝒂𝒏 𝟏𝟒 + 𝟏𝟔 𝒕𝒂𝒏 𝟏𝟒 − 𝟕
⇒ 𝒂𝒓𝒄𝒕𝒂𝒏 𝒙 = 𝟑 𝒂𝒓𝒄𝒄𝒐𝒔 [ √ ( )] + 𝒌𝟔𝝅 − 𝟐𝝅 ⇒ 𝒙
𝟖 𝟕 𝝅 𝟐
(𝟏 − 𝒕𝒂𝒏 )
𝟏𝟒
𝟐 𝝅 𝝅
𝟏 𝟑 −𝟖 𝒕𝒂𝒏 𝟏𝟒 + 𝟏𝟔 𝒕𝒂𝒏 𝟏𝟒 − 𝟕
= 𝒕𝒂𝒏 [𝟑 𝒂𝒓𝒄𝒄𝒐𝒔 [ √ ( )]] ∴ 𝒕𝒂𝒏(𝒙 ± 𝟐𝒌𝝅)
𝟖 𝟕 𝝅 𝟐
(𝟏 − 𝒕𝒂𝒏 )
𝟏𝟒
= 𝒕𝒂𝒏 𝒙
𝟐 𝝅 𝝅
𝟏 𝟑 −𝟖 𝒕𝒂𝒏 𝟏𝟒 + 𝟏𝟔 𝒕𝒂𝒏 𝟏𝟒 − 𝟕
⇒ 𝒙 = 𝒕𝒂𝒏 [𝟑 𝒂𝒓𝒄𝒄𝒐𝒔 [ √ ( )]] is solution
𝟖 𝟕 𝝅 𝟐
(𝟏 − 𝒕𝒂𝒏 )
𝟏𝟒
621. Solve for real numbers:
𝟓 𝟓 𝟒 𝟒
𝟓( √𝟏 − 𝒙 + √𝟏 + 𝒙) = 𝟐 + 𝟒( √𝟏 − 𝒙 + √𝟏 + 𝒙)
Proposed by Daniel Sitaru-Romania
Solution by Pham Duc Nam-Vietnam
𝟓 𝟓 𝟒 𝟒
𝟓(√𝟏 − 𝒙 + √𝟏 + 𝒙) = 𝟐 + 𝟒(√𝟏 − 𝒙 + √𝟏 + 𝒙)(𝟏)
∗ Domain: −𝟏 ≤ 𝒙 ≤ 𝟏
∗ Note that: if 𝒙𝟎 ∈ [−𝟏, 𝟏] is a root,then − 𝒙𝟎 is also a root ⇒ Consider: 𝟎 ≤ 𝒙 ≤ 𝟏
−𝟏 ≤ 𝒂 ≤ 𝟏
𝟐𝟎
∗ Let: {1 − 𝒙 = 𝒂 {
𝟐𝟎
⇒ (𝟏) ⇔ 𝟓(𝒂𝟒 + 𝒃𝟒 ) = 𝟐 + 𝟒(𝒂𝟓 + 𝒃𝟓 )
𝟐𝟎 ⇒ [1 ≤ 𝒃 ≤ √𝟐
1+𝑥 =𝒃 𝟐𝟎
− √𝟐 ≤ 𝒃 ≤ −𝟏
⇔ −𝟒𝒂𝟓 + 𝟓𝒂𝟒 − 𝟏 = 𝟒𝒃𝟓 − 𝟓𝒃𝟒 + 𝟏

22 RMM-ABSTRACT ALGEBRA MARATHON 601-700


www.ssmrmh.ro
⇔ − 𝒂 − 𝟏 𝟒𝒂 + 𝟑𝒂 + 𝟐𝒂 + 𝟏) = (𝒃 − 𝟏)𝟐 (𝟒𝒃𝟑 + 𝟑𝒃𝟐 + 𝟐𝒃 + 𝟏)
( ) 𝟐( 𝟑 𝟐

⇔ (𝒃 − 𝟏)𝟐 (𝟒𝒃𝟑 + 𝟑𝒃𝟐 + 𝟐𝒃 + 𝟏) + (𝒂 − 𝟏)𝟐 (𝟒𝒂𝟑 + 𝟑𝒂𝟐 + 𝟐𝒂 + 𝟏) = 𝟎


𝑎−𝟏=𝟎 𝑎=1 1−𝒙=𝟏
⇒{ ⇒{ ⇒{ ⇔𝒙=𝟎
𝑏−𝟏=𝟎 𝑏=1 1+𝑥 =1
622. 𝐅𝐢𝐧𝐝 𝒙, 𝐲, 𝐳 ∈ ℝ+ 𝐬𝐮𝐜𝐡 𝐭𝐡𝒂𝐭 ∶
𝟑
√(𝒙𝟐 𝐲 + 𝐲 𝟐 𝐳 + 𝐳 𝟐 𝒙)(𝒙𝐲 𝟐 + 𝐲𝐳 𝟐 + 𝐳𝒙𝟐 ) = 𝒙𝐲𝐳 + √(𝒙𝟑 + 𝒙𝐲𝐳)(𝐲 𝟑 + 𝒙𝐲𝐳)(𝐳 𝟑 + 𝒙𝐲𝐳)

𝟐𝒙𝟒 𝐲 + 𝟑𝐲𝟒 𝒛 + 𝟒𝐳 𝟒 𝐲 = 𝟗
Proposed by Samed Ahmedov-Azerbaijan
Solution 1 by Mohamed Amine Ben Ajiba-Tanger-Morocco

𝒙, 𝒚, 𝒛 ∈ ℝ+
{√(𝒙𝟐 𝒚 + 𝒚𝟐 𝒛 + 𝒛𝟐 𝒙)(𝒙𝒚𝟐 + 𝒚𝒛𝟐 + 𝒛𝒙𝟐 ) = 𝒙𝒚𝒛 + 𝟑√(𝒙𝟑 + 𝒙𝒚𝒛)(𝒚𝟑 + 𝒙𝒚𝒛)(𝒛𝟑 + 𝒙𝒚𝒛) (𝟏)
𝟐𝒙𝟒 𝒚 + 𝟑𝒚𝟒 𝒛 + 𝟒𝒛𝟒 𝒚 = 𝟗 (𝟐)

𝑨𝑴−𝑮𝑴
(𝒙𝟑 + 𝒙𝒚𝒛)(𝒚𝟑 + 𝒙𝒚𝒛)(𝒛𝟑 + 𝒙𝒚𝒛) 𝟐√𝒙𝟑 . 𝒙𝒚𝒛. 𝟐√𝒚𝟑 . 𝒙𝒚𝒛. 𝟐√𝒛𝟑 . 𝒙𝒚𝒛
𝐋𝐞𝐭 𝒌 ≔ ⏞
≥ = 𝟏.
𝟖(𝒙𝒚𝒛)𝟑 𝟖(𝒙𝒚𝒛)𝟑

𝐒𝐢𝐧𝐜𝐞 (𝒙𝟐 𝒚 + 𝒚𝟐 𝒛 + 𝒛𝟐 𝒙)(𝒙𝒚𝟐 + 𝒚𝒛𝟐 + 𝒛𝒙𝟐 ) = (𝒙𝟐 + 𝒚𝒛)(𝒚𝟐 + 𝒛𝒙)(𝒛𝟐 + 𝒙𝒚) + (𝒙𝒚𝒛)𝟐 ,

𝐭𝐡𝐞𝐧 𝐭𝐡𝐞 𝐞𝐪𝐮𝐚𝐭𝐢𝐨𝐧 (𝟏) 𝐢𝐬 𝐞𝐪𝐮𝐢𝐯𝐚𝐥𝐞𝐧𝐭 𝐭𝐨 ∶

𝟑 𝟖𝒌 𝟑 𝟑
√𝟖𝒌 + 𝟏 = 𝟏 + √𝟖𝒌 ⇔ = 𝟐√𝒌 ⇔ 𝟒 √ 𝒌𝟐 = √𝟖𝒌 + 𝟏 + 𝟏,
√𝟖𝒌 + 𝟏 + 𝟏
𝟑 𝟑 𝟑
𝒃𝒖𝒕 𝒔𝒊𝒏𝒄𝒆 𝒌 ≥ 𝟏 𝒕𝒉𝒆𝒏 ∶ 𝟒 √𝒌𝟐 = 𝟑 √ 𝒌𝟐 + √𝒌𝟐 ≥ 𝟑√𝒌 + 𝟏 = √𝟗𝒌 + 𝟏 ≥ √𝟖𝒌 + 𝟏 + 𝟏,

𝒘𝒊𝒕𝒉 𝒆𝒒𝒖𝒂𝒍𝒊𝒕𝒚 𝒘𝒉𝒆𝒏 𝒌 = 𝟏 ⇔ 𝒙𝟑 = 𝒚𝟑 = 𝒛𝟑 = 𝒙𝒚𝒛 ⇔ 𝒙 = 𝒚 = 𝒛.

𝑹𝒆𝒑𝒍𝒂𝒄𝒊𝒏𝒈 𝒊𝒏 (𝟐), 𝒘𝒆 𝒉𝒂𝒗𝒆 ∶ 𝒙𝟓 = 𝟏 ⇔ 𝒙 = 𝟏.

𝑻𝒉𝒆𝒓𝒆𝒇𝒐𝒓𝒆, 𝒙 = 𝒚 = 𝒛 = 𝟏.

Solution 2 by Soumava Chakraborty-Kolkata-India

√(𝒙𝟐 𝐲 + 𝐲 𝟐 𝐳 + 𝐳 𝟐 𝒙)(𝒙𝐲 𝟐 + 𝐲𝐳 𝟐 + 𝐳𝒙𝟐 )

23 RMM-ABSTRACT ALGEBRA MARATHON 601-700


www.ssmrmh.ro
𝟑
= 𝒙𝐲𝐳 + √(𝒙𝟑 + 𝒙𝐲𝐳)(𝐲 𝟑 + 𝒙𝐲𝐳)(𝐳 𝟑 + 𝒙𝐲𝐳) ⇒ √∑ 𝒙𝟑 𝐲 𝟑 + 𝒙𝐲𝐳 ∑ 𝒙𝟑 + 𝟑𝒙𝟐 𝐲 𝟐 𝐳 𝟐
𝐜𝐲𝐜 𝐜𝐲𝐜

= 𝒙𝐲𝐳 + 𝟑√𝒙𝐲𝐳 ∑ 𝒙𝟑 𝐲 𝟑 + 𝒙𝟐 𝐲 𝟐 𝐳 𝟐 ∑ 𝒙𝟑 + 𝟐𝒙𝟑 𝐲 𝟑 𝐳 𝟑


𝐜𝐲𝐜 𝐜𝐲𝐜

∑𝐜𝐲𝐜 𝒙𝟑 𝐲 𝟑 ∑𝐜𝐲𝐜 𝒙𝟑 𝟑 ∑
𝐜𝐲𝐜 𝒙 𝐲
𝟑 𝟑 ∑𝐜𝐲𝐜 𝒙𝟑
⇒ 𝒙𝐲𝐳. √ + + 𝟑 = 𝒙𝐲𝐳 + 𝒙𝐲𝐳. √ + +𝟐
𝒙𝟐 𝐲 𝟐 𝐳 𝟐 𝒙𝐲𝐳 𝒙𝟐 𝐲 𝟐 𝐳 𝟐 𝒙𝐲𝐳

(∗) 𝟑
∑𝐜𝐲𝐜 𝒙𝟑 𝐲 𝟑 ∑𝐜𝐲𝐜 𝒙𝟑 𝐀−𝐆
⇒ √𝐭 + 𝟑 = 𝟏 + √𝐭 + 𝟐 (𝐭 = + ≥ 𝟑 + 𝟑 = 𝟔 → (𝟏))
𝒙𝟐 𝐲 𝟐 𝐳 𝟐 𝒙𝐲𝐳
𝟑
𝐋𝐞𝐭 √𝐭 + 𝟐 = 𝐦 ≥ 𝟐 > 0 (∵ 𝐭 ≥ 𝟔) ∴ 𝐭 + 𝟐 = 𝐦𝟑 ⇒ 𝐭 + 𝟑 = 𝐦𝟑 + 𝟏
∴ (∗) 𝐛𝐞𝐜𝐨𝐦𝐞𝐬 ∶ √𝐦𝟑 + 𝟏 = 𝟏 + 𝐦 ⇒ 𝐦𝟑 + 𝟏 = 𝐦𝟐 + 𝟐𝐦 + 𝟏
⇒ 𝐦(𝐦𝟐 − 𝐦 − 𝟐) = 𝟎 ⇒ 𝐦(𝐦 − 𝟐)(𝐦 + 𝟏) = 𝟎 ⇒ 𝐦 = 𝟐 (∵ 𝐦 > 0) ⇒ 𝐭 = 𝟔
∑𝐜𝐲𝐜 𝒙𝟑 𝐲 𝟑 ∑𝐜𝐲𝐜 𝒙𝟑 ∑𝐜𝐲𝐜 𝒙𝟑 𝐲 𝟑 ∑𝐜𝐲𝐜 𝒙𝟑 𝐯𝐢𝒂 (𝟏) ′′ ′′
⇒ 𝟐 𝟐 𝟐 + = 𝟔, 𝐛𝐮𝐭 𝟐 𝟐 𝟐 + ≥ 𝟔, = 𝐢𝐟𝐟 𝒙 = 𝐲 = 𝐳
𝒙 𝐲 𝐳 𝒙𝐲𝐳 𝒙 𝐲 𝐳 𝒙𝐲𝐳
∴ 𝒙 = 𝐲 = 𝐳 𝒂𝐧𝐝 𝐯𝐢𝒂 𝒙 = 𝐲 = 𝐳, 𝟐𝒙𝟒 𝐲 + 𝟑𝐲 𝟒 𝒛 + 𝟒𝐳 𝟒 𝐲 = 𝟗 ⇒ 𝟗𝒙𝟓 = 𝟗 ⇒ 𝒙 = 𝟏
∴ 𝒙 = 𝐲 = 𝐳 = 𝟏 (𝒂𝒏𝒔)
623. Solve for real numbers:
𝑥𝒚𝟒 𝒕 + 𝟔𝒙𝟐 𝒚 = 𝟐𝒕𝟐
{𝑥𝑡 + 𝒚𝟑 𝒕𝟐 = 𝟔𝒙𝟐 𝒚𝟒
𝒚𝟑 𝒕 − 𝒙𝒚𝟔 𝒕𝟐 = 𝟐𝒙𝟐
Proposed by Hikmat Mammadov-Azerbaijan
Solution by Pham Duc Nam-Vietnam
𝑥𝒚𝟒 𝒕 + 𝟔𝒙𝟐 𝒚 = 𝟐𝒕𝟐 𝑥𝒚𝟒 𝒕 + 𝟔𝒙𝟐 𝒚 − 𝟐𝒕𝟐 = 𝟎(𝟏)
{𝑥𝑡 + 𝒚𝟑 𝒕𝟐 = 𝟔𝒙𝟐 𝒚𝟒 ⇔ {𝑥𝑡 + 𝒚𝟑 𝒕𝟐 − 𝟔𝒙𝟐 𝒚𝟒 = 𝟎(𝟐)
𝒚𝟑 𝒕 − 𝒙𝒚𝟔 𝒕𝟐 = 𝟐𝒙𝟐 𝒚𝟑 𝒕 − 𝒙𝒚𝟔 𝒕𝟐 − 𝟐𝒙𝟐 = 𝟎(𝟑)
∗ 𝑪𝒍𝒆𝒂𝒓𝒍𝒚: (𝒙, 𝒚, 𝒕) = (𝟎, 𝒚, 𝟎) 𝒊𝒔 𝒂 𝒔𝒐𝒍𝒖𝒕𝒊𝒐𝒏
∗ 𝑨𝒔𝒔𝒖𝒎𝒆: 𝒙, 𝒚, 𝒕 ≠ 𝟎
⊕ 𝒚 . 𝟏 + (𝟐) = 𝟎 ⇔ −𝒕 𝒚 − 𝒙𝒕𝒚𝟕 + 𝒙𝒕 = 𝟎 ⇔ 𝒙 + 𝒙𝒚𝟕 − 𝒚𝟑 𝒕 = 𝟎 ⇔ 𝒙(𝟏 + 𝒚𝟕 ) − 𝒕𝒚𝟑 = 𝟎
𝟑 ( ) 𝟐 𝟑

𝒕 𝟏 + 𝒚𝟕
⇒ = (∗)
𝒙 𝒚𝟑
𝒙𝒕 + 𝒚𝟑 𝒕𝟐 − 𝟔𝒙𝟐 𝒚𝟒 𝒕 𝒙
⊕ 𝑭𝒓𝒐𝒎 (𝟐) ⇒ = 𝟎 ⇔ 𝟏 + 𝒚𝟑 − 𝟔𝒚𝟒 = 𝟎(∗∗)
𝒙𝒕 𝒙 𝒕
𝟑
𝟏 + 𝒚𝟕 𝟒
𝒚𝟑 𝟕
𝟔𝒚𝟕
𝑷𝒖𝒕 (∗) 𝒕𝒐 (∗∗) ⇒ 𝟏 + 𝒚 − 𝟔𝒚 = 𝟎 ⇔ 𝟐 + 𝒚 − =𝟎
𝒚𝟑 𝟏 + 𝒚𝟕 𝟏 + 𝒚𝟕
𝒚𝟕 = 𝟏 𝑦=1
⇔ 𝒚𝟏𝟒 − 𝟑𝒚𝟕 + 𝟐 = 𝟎 ⇔ [ 𝟕 ⇔[ 𝟕
𝒚 =𝟐 𝑦 = √𝟐

24 RMM-ABSTRACT ALGEBRA MARATHON 601-700


www.ssmrmh.ro
𝑡 − 𝒙𝒕𝟐 − 𝟐𝒙𝟐 = 𝟎 𝟐) 𝟐
𝟐 𝟐
∗ 𝒚 = 𝟏 ⇒ (𝟑) ⇔ 𝒕 − 𝒙𝒕 − 𝟐𝒙 = 𝟎 ⇒ { 𝒕 ⇔ {2𝒙 − 𝒙(𝟒𝒙 − 𝟐𝒙 = 𝟎
= 𝟐 ⇒ 𝒕 = 𝟐𝒙 𝑡 = 2𝒙
𝒙
𝟑 𝟐
⇔ {−𝟒𝒙 − 𝟐𝒙 + 𝟐𝒙 = 𝟎
𝑡 = 2𝒙
𝑥 = 0 (𝑵𝒐𝒕 𝒔𝒂𝒕𝒊𝒔𝒇𝒊𝒆𝒅) 𝑥 = −𝟏
{
𝑥 = −𝟏 𝑡 = −𝟐 𝟏
[ 𝟏 𝟏 ⇒ (𝒙, 𝒚, 𝒕) = (−𝟏, 𝟏, −𝟐), ( , 𝟏, 𝟏)
⇔ ⇔
𝑥=
𝟐 {𝑥 = 𝟐 𝟐
{𝑡 = 2𝒙 [ 𝑡=1
𝟕 𝟕
√𝟖𝒕 − √𝟔𝟒𝒙𝒕𝟐 − 𝟐𝒙𝟐 = 𝟎
𝟕 𝟕 𝟕 𝟐 𝟐
∗ 𝒚 = √𝟐 ⇒ (𝟑) ⇔ √𝟖𝒕 − √𝟔𝟒𝒙𝒕 − 𝟐𝒙 = 𝟎 ⇒ { 𝒕 𝟑 𝟑𝒙
=𝟕 ⇒𝒕=𝟕
𝒙 √𝟖 √𝟖
𝟐
𝟕 𝟑𝒙 𝟕 𝟑𝒙
√𝟖 𝟕 − √𝟔𝟒𝒙 ( 𝟕 ) − 𝟐𝒙𝟐 −𝟗𝒙𝟑 − 𝟐𝒙𝟐 + 𝟑𝒙 = 𝟎
⇔ √𝟖 √𝟖 ⇔{ 𝟑𝒙
𝟑𝒙 𝑡=𝟕
𝑡=𝟕 √𝟖
{ √𝟖
−𝟏 − 𝟐√𝟕
𝑥 = 0 (𝑵𝒐𝒕 𝒔𝒂𝒕𝒊𝒔𝒇𝒊𝒆𝒅) 𝑥=
𝟗
−𝟏 − 𝟐√𝟕 −𝟏 − 𝟐√𝟕
𝑥= 𝑡=
𝟗 𝟕
𝟑 √𝟖
⇔ −𝟏 + 𝟐√𝟕 ⇔ { ⇒ (𝒙, 𝒚, 𝒕)
𝑥 = −𝟏 + 𝟐√𝟕
[ 𝟗 𝑥=
𝟑𝒙 𝟗
𝑡=𝟕 −𝟏 + 𝟐√𝟕
{ √𝟖 𝑡= 𝟕
[{ 𝟑 √𝟖
−𝟏 − 𝟐√𝟕 𝟕 −𝟏 − 𝟐√𝟕 −𝟏 + 𝟐√𝟕 𝟕 −𝟏 + 𝟐√𝟕
=( , √𝟐, 𝟕 ),( , √𝟐, 𝟕 )
𝟗 𝟑 √𝟖 𝟗 𝟑 √𝟖
𝟏
⇒ (𝒙, 𝒚, 𝒕) = (𝟎, 𝒚, 𝟎), (−𝟏, 𝟏, −𝟐), ( , 𝟏, 𝟏),
𝟐
−𝟏 − 𝟐√𝟕 𝟕 −𝟏 − 𝟐√𝟕 −𝟏 + 𝟐√𝟕 𝟕 −𝟏 + 𝟐√𝟕
( , √𝟐, 𝟕 ),( , √𝟐, 𝟕 ) 𝒂𝒓𝒆 𝒔𝒐𝒍𝒖𝒕𝒊𝒐𝒏𝒔
𝟗 𝟑 √𝟖 𝟗 𝟑 √𝟖
624. Solve for real numbers:
𝑎 − 𝟑𝒃 + 𝟑𝒄 = 𝒂𝒃𝒄
{3𝒂𝟐 + 𝟐𝒃 − 𝟑𝒂𝒃 = 𝟏
4𝒃𝟐 − 𝟑𝒂 − 𝒂𝟐 = 𝟔𝒃
Proposed by Soumava Chakraborty-Kolkata-India
Solution by Pham Duc Nam-Vietnam
𝑎 − 𝟑𝒃 + 𝟑𝒄 = 𝒂𝒃𝒄(𝟏)
{3𝒂𝟐 + 𝟐𝒃 − 𝟑𝒂𝒃 = 𝟏(𝟐)
4𝒃𝟐 − 𝟑𝒂 − 𝒂𝟐 = 𝟔𝒃(𝟑)

∗ From(𝟑): 𝟒𝒃𝟐 − 𝟑𝒂 − 𝒂𝟐 = 𝟔𝒃 ⇔ 𝟒𝒃𝟐 − 𝟔𝒃 − 𝟑𝒂 − 𝒂𝟐 = 𝟎

25 RMM-ABSTRACT ALGEBRA MARATHON 601-700


www.ssmrmh.ro
𝟑 + 𝟐𝒂 + 𝟑 𝒂+𝟑
𝑏= 𝑏=
𝜟′ = (−𝟑)𝟐 + 𝟒(𝟑𝒂 + 𝒂𝟐 ) = 𝟒𝒂𝟐 + 𝟏𝟐𝒂 + 𝟗 = (𝟐𝒂 + 𝟑)𝟐 ⇒ [ 𝟒 ⇔[ 𝟐
𝟑 − 𝟐𝒂 − 𝟑 𝒂
𝑏= 𝑏=−
𝟒 𝟐
𝒂+𝟑 𝒂 + 𝟑
⊕ 𝑪𝑨𝑺𝑬: 𝒃 = ⇒ (𝟐) ⇔ 𝟑𝒂𝟐 + 𝒂 + 𝟑 − 𝟑𝒂 ( )−𝟏 = 𝟎
𝟐 𝟐
𝑎=1⇒𝒃=𝟐
⇔ 𝟑𝒂𝟐 − 𝟕𝒂 + 𝟒 = 𝟎 ⇔ [ 𝟒 𝟏𝟑
𝑎= ⇒𝒃=
𝟑 𝟔
−𝒂 𝟐
𝒂
⊕ 𝑪𝑨𝑺𝑬: 𝒃 = ( )
⇒ 𝟐 ⇔ 𝟑𝒂 − 𝒂 − 𝟑𝒂. (− ) − 𝟏 = 𝟎 ⇔ 𝟗𝒂𝟐 − 𝟐𝒂 − 𝟐 = 𝟎
𝟐 𝟐
𝟏 − √𝟏𝟗 √𝟏𝟗 − 𝟏
𝑎= ⇒𝒃=
⇔ 𝟗 𝟏𝟖
𝟏 + √𝟏𝟗 𝟏 + √𝟏𝟗
[𝑎 = 𝟗
⇒𝒃=−
𝟏𝟖
𝒂 − 𝟑𝒃
∗ From(𝟏): 𝒂 − 𝟑𝒃 + 𝟑𝒄 = 𝒂𝒃𝒄 ⇔ 𝒄 =
𝒂𝒃 − 𝟑
𝟒 𝟏𝟑 𝟗𝟑 𝟏 − √𝟏𝟗 √𝟏𝟗 − 𝟏
(𝒂, 𝒃) = (𝟏, 𝟐) ⇒ 𝒄 = 𝟓, (𝒂, 𝒃) = ( , ) ⇒ 𝒄 = , (𝒂, 𝒃) = ( , )⇒𝒄
𝟑 𝟔 𝟐 𝟗 𝟏𝟖
𝟏 𝟏 + √𝟏𝟗 √𝟏𝟗 + 𝟏 𝟏
= (𝟏𝟒√𝟏𝟗 − 𝟏𝟑), (𝒂, 𝒃) = ( ,− )⇒𝒄= (−𝟏𝟒√𝟏𝟗 − 𝟏𝟑)
𝟏𝟓𝟖 𝟗 𝟏𝟖 𝟏𝟓𝟖
𝟒 𝟏𝟑 𝟗𝟑
⇒ (𝒂, 𝒃, 𝒄) = (𝟏, 𝟐, 𝟓), ( , , ),
𝟑 𝟔 𝟐
𝟏 − √𝟏𝟗 √𝟏𝟗 − 𝟏 𝟏 𝟏 + √𝟏𝟗 √𝟏𝟗 + 𝟏 𝟏
( , , (𝟏𝟒√𝟏𝟗 − 𝟏𝟑)) , ( ,− , (−𝟏𝟒√𝟏𝟗 − 𝟏𝟑)) are solutions
𝟗 𝟏𝟖 𝟏𝟓𝟖 𝟗 𝟏𝟖 𝟏𝟓𝟖

𝝅
625. If 𝟎 ≤ 𝒂 ≤ 𝒃 < then:
𝟐

| |

𝒃 𝒂
| − | ≥ |𝒃 − 𝒂|
𝒃𝟐 𝒂𝟐
𝟏− 𝟏−
𝒃𝟐 𝒂𝟐
𝟑− 𝟑−
| 𝒃𝟐 𝒂𝟐 |
𝟓− 𝟓−
𝒃𝟐 𝒂𝟐
𝟕− 𝟕−
… …
Proposed by Daniel Sitaru-Romania
Solution 1 by Toubal Fethi-Algerie
𝑩𝒚 𝒖𝒔𝒊𝒏𝒈 𝑮𝒂𝒖𝒔𝒔′ 𝒔 𝒄𝒐𝒏𝒕𝒊𝒏𝒖𝒆𝒅 𝒇𝒓𝒂𝒄𝒕𝒊𝒐𝒏, 𝒘𝒆 𝒈𝒆𝒕:

26 RMM-ABSTRACT ALGEBRA MARATHON 601-700


www.ssmrmh.ro
𝒃
= 𝐭𝐚𝐧 𝒃
𝒃𝟐
𝟏−
𝒃𝟐
𝟑−
𝒃𝟐
𝟓−
𝒃𝟐
𝟕− …
𝒂
= 𝐭𝐚𝐧 𝒂
𝒂𝟐
𝟏−
𝒂𝟐
𝟑−
𝒂𝟐
𝟓−
𝒂𝟐
𝟕− …

𝑵𝒐𝒘, 𝒖𝒔𝒊𝒏𝒈 𝑴𝑽𝑻 𝒇𝒐𝒓 𝒇(𝒙) = 𝐭𝐚𝐧 𝒙 , 𝒔𝒊𝒏𝒄𝒆 𝒇′ (𝒙) = 𝟏 + 𝐭𝐚𝐧𝟐 𝒙 ≥ 𝟏,


𝒘𝒆 𝒈𝒆𝒕: |𝐭𝐚𝐧 𝒃 − 𝐭𝐚𝐧 𝒂| ≥ |𝒃 − 𝒂| 𝒉𝒆𝒏𝒄𝒆 𝒑𝒓𝒐𝒗𝒆𝒅.
Solution 2 by Hikmat Mammadov-Azerbaijan
𝑻𝒉𝒆 𝒄𝒐𝒏𝒕𝒊𝒏𝒖𝒐𝒖𝒔 𝒇𝒓𝒂𝒄𝒕𝒊𝒐𝒏 𝒓𝒆𝒑𝒓𝒆𝒔𝒆𝒏𝒕𝒂𝒕𝒊𝒐𝒏 𝒐𝒇 𝒙 → 𝐭𝐚𝐧 𝒙 , 𝒊𝒔:
𝒙
𝐭𝐚𝐧 𝒙 =
𝒙𝟐
𝟏−
𝒙𝟐
𝟑−
𝒙𝟐
𝟓−
𝒙𝟐
𝟕− …
𝝅
𝑻𝒉𝒆𝒏 𝒕𝒉𝒆 𝒊𝒏𝒆𝒒𝒖𝒂𝒍𝒊𝒕𝒚 𝒃𝒆𝒄𝒐𝒎𝒆𝒔: |𝐭𝐚𝐧 𝒃 − 𝐭𝐚𝐧 𝒂| ≥ |𝒃 − 𝒂|, (∀)𝒂, 𝒃 ∈ (𝟎, )
𝟐
(𝒊) 𝑰𝒇 𝒂 = 𝒃 𝒏𝒐𝒕𝒉𝒊𝒏𝒈 𝒕𝒐 𝒑𝒓𝒐𝒗𝒆.
(𝒊𝒊) 𝑰𝒇 𝒂 < 𝑏 𝑓𝑟𝑜𝑚 𝑀𝑉𝑇 (∃)𝒄 ∈ (𝒂, 𝒃) 𝒔𝒖𝒄𝒉 𝒕𝒉𝒂𝒕:
𝐭𝐚𝐧 𝒃 − 𝐭𝐚𝐧 𝒂 𝟏
=
𝒃−𝒂 𝐜𝐨𝐬 𝟐 𝒄
𝟏
|𝐭𝐚𝐧 𝒃 − 𝐭𝐚𝐧 𝒂| = |𝒃 − 𝒂| ⋅ ≥ |𝒃 − 𝒂|; (𝐜𝐨𝐬 𝟐 𝒄 < 1)
𝐜𝐨𝐬 𝟐 𝒄
626. 𝐒𝐨𝐥𝐯𝐞 𝐟𝐨𝐫 𝐢𝐧𝐭𝐞𝐠𝐞𝐫𝐬:

√𝒙𝟑 + 𝒚𝟐 + √𝒙𝟐 + 𝒚𝟑 = 𝒙 + 𝒚
Proposed by Sakthi Vel-India
Solution by Mohamed Amine Ben Ajiba-Tanger-Morocco
𝐖𝐋𝐎𝐆, 𝐰𝐞 𝐚𝐬𝐬𝐮𝐦𝐞 𝐭𝐡𝐚𝐭 𝒙 ≥ 𝒚.

27 RMM-ABSTRACT ALGEBRA MARATHON 601-700


www.ssmrmh.ro
𝐈𝐟 𝒚 ≥ 𝟎, 𝐰𝐞 𝐡𝐚𝐯𝐞: √𝒙𝟑 + 𝒚𝟐 ≥ 𝒚, √𝒙𝟐 + 𝒚𝟑 ≥ 𝒙, 𝐭𝐡𝐞𝐧

√𝒙𝟑 + 𝒚𝟐 + √𝒙𝟐 + 𝒚𝟑 ≥ 𝒙 + 𝒚,

𝐞𝐪𝐮𝐚𝐥𝐢𝐭𝐲 𝐡𝐨𝐥𝐝𝐬 𝐢𝐟𝐟 𝒙 = 𝒚 = 𝟎.

𝐈𝐟 𝐱 ≥ 𝟎 ≥ 𝐲, 𝐰𝐞 𝐡𝐚𝐯𝐞: √𝒙𝟑 + 𝒚𝟐 ≥ √𝒙𝟑 ≥ 𝒙 𝒂𝒏𝒅 √𝒙𝟐 + 𝒚𝟑 ≥ 𝟎 ≥ 𝒚, 𝐭𝐡𝐞𝐧

√𝒙𝟑 + 𝒚𝟐 + √𝒙𝟐 + 𝒚𝟑 ≥ 𝒙 + 𝒚,

𝐞𝐪𝐮𝐚𝐥𝐢𝐭𝐲 𝐡𝐨𝐥𝐝𝐬 𝐢𝐟𝐟 𝒙 = 𝒚 = 𝟎.

𝐈𝐟 𝐱 ≤ 𝟎, 𝐰𝐞 𝐡𝐚𝐯𝐞: √𝒙𝟑 + 𝒚𝟐 + √𝒙𝟐 + 𝒚𝟑 ≥ 𝟎 ≥ 𝒙 + 𝒚, 𝐰𝐢𝐭𝐡 𝐞𝐪𝐮𝐚𝐥𝐢𝐭𝐲 𝐢𝐟 𝒙 = 𝒚 = 𝟎.

𝐓𝐡𝐞𝐫𝐞𝐟𝐨𝐫𝐞, 𝐭𝐡𝐞 𝐨𝐧𝐥𝐲 𝐬𝐨𝐥𝐮𝐭𝐢𝐨𝐧 𝐨𝐟 𝐭𝐡𝐞 𝐠𝐢𝐯𝐞𝐧 𝐞𝐪𝐮𝐚𝐭𝐢𝐨𝐧 𝐢𝐬 𝒙 = 𝒚 = 𝟎.

627. 𝐒𝐨𝐥𝐯𝐞 𝐟𝐨𝐫 𝐢𝐧𝐭𝐞𝐠𝐞𝐫𝐬:

√𝒂𝟑 + 𝒃𝟐 + √𝒃𝟑 + 𝒄𝟐 + √𝒄𝟑 + 𝒂𝟐 = 𝒂 + 𝒃 + 𝒄


Proposed by Mehmet Șahin-Ankara-Turkiye
Solution by Mohamed Amine Ben Ajiba-Tanger-Morocco
𝐈𝐟 𝒂, 𝒃, 𝒄 ≥ 𝟎, 𝐰𝐞 𝐡𝐚𝐯𝐞, √𝒂𝟑 + 𝒃𝟐 ≥ 𝒃, √𝒃𝟑 + 𝒄𝟐 ≥ 𝒄, √𝒄𝟑 + 𝒂𝟐 ≥ 𝒂, 𝐭𝐡𝐞𝐧

√𝒂𝟑 + 𝒃𝟐 + √𝒃𝟑 + 𝒄𝟐 + √𝒄𝟑 + 𝒂𝟐 ≥ 𝒂 + 𝒃 + 𝒄,


𝐄𝐪𝐮𝐚𝐥𝐢𝐭𝐲 𝐡𝐨𝐥𝐝𝐬 𝐢𝐟𝐟 𝒂 = 𝒃 = 𝒄 = 𝟎.
𝐈𝐟 𝐨𝐧𝐞 𝐨𝐟 𝒂, 𝒃, 𝒄 𝐢𝐬 𝐧𝐞𝐠𝐚𝐭𝐢𝐯𝐞, 𝐟𝐨𝐫 𝐞𝐱𝐚𝐦𝐩𝐥𝐞 𝒄, 𝐰𝐞 𝐡𝐚𝐯𝐞:

√𝒂𝟑 + 𝒃𝟐 ≥ √𝒂𝟑 ≥ 𝒂, √𝒃𝟑 + 𝒄𝟐 ≥ 𝒃 𝐚𝐧𝐝 √𝒄𝟑 + 𝒂𝟐 ≥ 𝟎 ≥ 𝒄, 𝐭𝐡𝐞𝐧

√𝒂𝟑 + 𝒃𝟐 + √𝒃𝟑 + 𝒄𝟐 + √𝒄𝟑 + 𝒂𝟐 ≥ 𝒂 + 𝒃 + 𝒄,


𝐄𝐪𝐮𝐚𝐥𝐢𝐭𝐲 𝐡𝐨𝐥𝐝𝐬 𝐢𝐟𝐟 𝒂 = 𝒃 = 𝒄 = 𝟎.
𝐈𝐟 𝐨𝐧𝐞 𝐨𝐟 𝒂, 𝒃, 𝒄 𝐢𝐬 𝐧𝐨𝐧𝐧𝐞𝐠𝐚𝐭𝐢𝐯𝐞, 𝐟𝐨𝐫 𝐞𝐱𝐚𝐦𝐩𝐥𝐞 𝒂, 𝐰𝐞 𝐡𝐚𝐯𝐞:

√𝒂𝟑 + 𝒃𝟐 ≥ √𝒂𝟑 ≥ 𝒂, √𝒃𝟑 + 𝒄𝟐 ≥ 𝟎 ≥ 𝒃 𝐚𝐧𝐝 √𝒄𝟑 + 𝒂𝟐 ≥ 𝟎 ≥ 𝒄, 𝐭𝐡𝐞𝐧

√𝒂𝟑 + 𝒃𝟐 + √𝒃𝟑 + 𝒄𝟐 + √𝒄𝟑 + 𝒂𝟐 ≥ 𝒂 + 𝒃 + 𝒄,


𝐄𝐪𝐮𝐚𝐥𝐢𝐭𝐲 𝐡𝐨𝐥𝐝𝐬 𝐢𝐟𝐟 𝒂 = 𝒃 = 𝒄 = 𝟎.

𝐈𝐟 𝒂, 𝒃, 𝒄 ≤ 𝟎, 𝐰𝐞 𝐡𝐚𝐯𝐞: √𝒂𝟑 + 𝒃𝟐 + √𝒃𝟑 + 𝒄𝟐 + √𝒄𝟑 + 𝒂𝟐 ≥ 𝟎 ≥ 𝒂 + 𝒃 + 𝒄,


𝐄𝐪𝐮𝐚𝐥𝐢𝐭𝐲 𝐡𝐨𝐥𝐝𝐬 𝐢𝐟 𝒂 = 𝒃 = 𝒄 = 𝟎. 𝐓𝐡𝐞𝐫𝐞𝐟𝐨𝐫𝐞,
𝐭𝐡𝐞 𝐨𝐧𝐥𝐲 𝐬𝐨𝐥𝐮𝐭𝐢𝐨𝐧 𝐨𝐟 𝐭𝐡𝐞 𝐠𝐢𝐯𝐞𝐧 𝐞𝐪𝐮𝐚𝐭𝐢𝐨𝐧 𝐢𝐬 𝒂 = 𝒃 = 𝒄 = 𝟎.

28 RMM-ABSTRACT ALGEBRA MARATHON 601-700


www.ssmrmh.ro
628. Solve for integers:
𝟐 𝒙 + 𝟑 𝒚 = 𝒛𝟐
Proposed by Elsen Kerimov-Azerbaijan
Solution by Hikmat Mammadov-Azerbaijan
𝑰𝒇 𝒙 < 0 𝑜𝑟 𝑦 < 0, 𝐿𝐻𝑆 𝑖𝑠 𝑛𝑜𝑡 𝑖𝑛𝑡𝑒𝑔𝑒𝑟.
𝑰𝒇 𝒙 = 𝟎 ⇒ 𝟑𝒚 = 𝒛𝟐 − 𝟏 = (𝒛 − 𝟏)(𝒛 + 𝟏)
𝒛 − 𝟏 = 𝟑𝒂 , 𝒛 + 𝟏 = 𝟑𝒃 , 𝒂 + 𝒃 = 𝒚, 𝒂 < 𝑏 ⇒ 𝟑𝒃 − 𝟑𝒂 = 𝟐 ⇒
𝒂 = 𝟎, 𝟑𝒃 − 𝟏 = 𝟐 ⇒ 𝒃 = 𝟏 ⇒ (𝒙, 𝒚, 𝒛) ∈ {(𝟎, 𝟏, 𝟐), (𝟎, 𝟏, −𝟐)}
𝑰𝒇 𝒚 = 𝟎 ⇒ 𝟐𝒙 = 𝒛𝟐 − 𝟏 = (𝒛 − 𝟏)(𝒛 + 𝟏)
𝒛 − 𝟏 = 𝟑𝒂 , 𝒛 + 𝟏 = 𝟑𝒃 , 𝒂 < 𝑏, 𝑎 + 𝑏 = 𝑥 ⇒ 𝟐𝒃 − 𝟐𝒂 = 𝟐
𝟐𝒂 (𝟐𝒃−𝒂 − 𝟏) = 𝟐 ⇒ 𝒂 = 𝟏, 𝟐𝒃 − 𝟏 = 𝟏 ⇒ 𝒃 = 𝟐
(𝒙, 𝒚, 𝒛) ∈ {(𝟑, 𝟎, 𝟑), (𝟑, 𝟎, −𝟑)}
𝒙 > 0, 𝑦 > 0. 𝑀𝑜𝑑 3: (−𝟏)𝒙 ≡ 𝒛𝒙 ⇒ 𝒙 𝒊𝒔 𝒆𝒗𝒆𝒏, 𝒙 = 𝟐𝒙
̅⇒
𝑴𝒐𝒅 𝟒: (−𝟏)𝒚 ≡ 𝒛𝟐 , 𝒚 𝒊𝒔 𝒆𝒗𝒆𝒏, 𝒚 = 𝟐𝒚
̅
𝟑𝒚 = 𝒛𝟐 − (𝟐𝒙̅ )𝟐 = (𝒛 − 𝟐𝒙̅ )(𝒛 + 𝟐𝒙̅ )
𝒛 − 𝟐𝒙̅ = 𝟑𝒂 , 𝒛 + 𝟐𝒙̅ = 𝟑𝒃 , 𝒂 < 𝑏, 𝑎 + 𝑏 = 𝑦 = 2𝒚
̅ ⇒ 𝟑𝒂 (𝟑𝒃−𝒂 − 𝟏) = 𝟐𝒙̅+𝟏 ⇒ 𝒂 = 𝟎
𝟑𝟐𝒚̅ − 𝟏 = 𝟐𝒙̅+𝟏 ⇔ (𝟑𝒚̅ − 𝟏)(𝟑𝒚̅ + 𝟏) = 𝟐𝒙̅+𝟏
𝟑𝒚̅ − 𝟏 = 𝟐𝒂 , 𝟑𝒚̅ + 𝟏 = 𝟐𝒃 , 𝒂 < 𝑏, +𝑏 = 𝒙
̅ + 𝟏 ⇒ 𝟐𝒂 (𝟐𝒃−𝒂 − 𝟏) = 𝟐 ⇒ 𝒂 = 𝟏, 𝒚
̅=𝟏
𝟐𝒃−𝟏 − 𝟏 = 𝟏 ⇒ 𝒃 = 𝟐 ⇒ 𝒙
̅ = 𝟐 ⇒ (𝒙, 𝒚, 𝒛) ∈ {(𝟒, 𝟐, 𝟓), (𝟒, 𝟐, −𝟓)}
𝑺𝒐, (𝒙, 𝒚, 𝒛) ∈ {(𝟎, 𝟏 ± 𝟐), (𝟒, 𝟐, ±𝟓)}.
629. 𝐒𝐨𝐥𝐯𝐞 𝐟𝐨𝐫 𝐫𝐞𝐚𝐥 𝐧𝐮𝐦𝐛𝐞𝐫𝐬 ∶
𝒙, 𝐲, 𝐳 > 0,
𝟑 𝟑
√(𝟏 + 𝒙)(𝟏 + 𝐲)(𝟏 + 𝐳) + √(𝟏 − 𝒙)(𝟏 − 𝐲)(𝟏 − 𝐳) = 𝟐,
𝒙+𝐲+𝐳=𝟏
Proposed by Daniel Sitaru-Romania
Solution 1 by Pham Duc Nam-Vietnam
𝒙, 𝒚, 𝒛 > 0, 𝑥 + 𝑦 + 𝑧 = 1 ⇒ 0 < 𝑥, 𝑦, 𝑧 < 1
𝟑 𝒙+𝒚+𝒛+𝟑 𝟒
√(𝟏 + 𝒙)(𝟏 + 𝒚)(𝟏 + 𝒛) ≤ =
𝟑 𝟑
29 RMM-ABSTRACT ALGEBRA MARATHON 601-700
www.ssmrmh.ro
𝟑 𝟑 − ( 𝒙 + 𝒚 + 𝒛) 𝟐
√(𝟏 − 𝒙)(𝟏 − 𝒚)(𝟏 − 𝒛) ≤ =
𝟑 𝟑
𝑩𝒚 𝒂𝒅𝒅𝒊𝒏𝒈, 𝒘𝒆 𝒈𝒆𝒕:
𝟑 𝟑
√ (𝟏 + 𝒙)(𝟏 + 𝒚)(𝟏 + 𝒛) + √(𝟏 − 𝒙)(𝟏 − 𝒚)(𝟏 − 𝒛) ≤ 𝟐
𝟏
𝑳𝑯𝑺 = 𝑹𝑯 ⇔ 𝒙 = 𝒚 = 𝒛 = .
𝟑
Solution 2 by Soumava Chakraborty-Kolkata-India

𝟑 𝟑
√ (𝟏 + 𝒙)(𝟏 + 𝐲)(𝟏 + 𝐳) + √(𝟏 − 𝒙)(𝟏 − 𝐲)(𝟏 − 𝐳) = 𝟐
𝐜𝐮𝐛𝐢𝐧𝐠
⇒ ∏(𝟏 + 𝒙) + ∏(𝟏 − 𝒙)
𝐜𝐲𝐜 𝐜𝐲𝐜

+𝟑. 𝟑√∏(𝟏 + 𝒙) . 𝟑√ ∏(𝟏 − 𝒙) . ( 𝟑√∏(𝟏 + 𝒙) + 𝟑√∏(𝟏 − 𝒙)) = 𝟖


𝐜𝐲𝐜 𝐜𝐲𝐜 𝐜𝐲𝐜 𝐜𝐲𝐜

⇒ 𝟐 ∑ 𝒙𝐲 + 𝟐 + 𝟔. 𝟑√∏(𝟏 − 𝒙𝟐 ) = 𝟖 (∵ 𝟑√∏(𝟏 + 𝒙) + 𝟑√∏(𝟏 − 𝒙) = 𝟐)


𝐜𝐲𝐜 𝐜𝐲𝐜 𝐜𝐲𝐜 𝐜𝐲𝐜

(∗)
⇒ 𝟑 − ∑ 𝒙𝐲 = 𝟑. 𝟑√ ∏(𝟏 − 𝒙𝟐 )
𝐜𝐲𝐜 𝐜𝐲𝐜
𝟐
𝒙+𝐲+𝐳 = 𝟏 𝟏
𝐍𝐨𝐰, ∀ 𝒙, 𝐲, 𝐳 ∈ ℝ, 𝟑 ∑ 𝒙𝐲 ≤ (∑ 𝒙) = 𝟏 ⇒ − ∑ 𝒙𝐲 ≥ − ⇒ 𝟑 − ∑ 𝒙𝐲
𝟑
𝐜𝐲𝐜 𝐜𝐲𝐜 𝐜𝐲𝐜 𝐜𝐲𝐜

𝟏 𝟖 𝐯𝐢𝒂 (∗) 𝟑 𝟖 𝟖
≥ 𝟑− = ⇒ 𝟑. √ ∏(𝟏 − 𝒙𝟐 ) ≥ ⇒ 𝟑√ ∏(𝟏 − 𝒙𝟐 ) ≥
𝟑 𝟑 𝟑 𝟗
𝐜𝐲𝐜 𝐜𝐲𝐜

𝟖 𝟏 𝟖
⇒ 𝐥𝐧 ( 𝟑√∏(𝟏 − 𝒙𝟐 )) ≥ 𝐥𝐧 ( ) ⇒ ∑ 𝐥𝐧(𝟏 − 𝒙𝟐 ) ≥ 𝐥𝐧 ( )
𝟗 𝟑 𝟗
𝐜𝐲𝐜 𝐜𝐲𝐜
(∗∗) 𝟖
⇒ ∑ 𝐥𝐧(𝟏 − 𝒙𝟐 ) ≥ 𝟑𝐥𝐧 ( )
𝟗
𝐜𝐲𝐜
( 𝒙𝟐 + 𝟏)
𝐋𝐞𝐭 𝐟(𝒙) = 𝐥𝐧(𝟏 − 𝒙𝟐 ) ∀ 𝒙 ∈ ℝ+ 𝒂𝐧𝐝 𝐭𝐡𝐞𝐧 ∶ 𝐟 ′′(𝒙) = − <0
( 𝒙𝟐 − 𝟏) 𝟐
(∵ 𝒙, 𝐲, 𝐳 > 0 𝑎𝐧𝐝 𝒙 + 𝐲 + 𝐳 = 𝟏 ⇒ 𝒙, 𝐲, 𝐳 < 1 ⇒ 𝒙, 𝐲, 𝐳 ≠ 𝟏)
𝐉𝐞𝐧𝐬𝐞𝐧 ∑𝐜𝐲𝐜 𝒙 𝟐
+ 𝟐
⇒ 𝐟(𝒙) 𝐢𝐬 𝐜𝐨𝐧𝐜𝒂𝐯𝐞 ∀ 𝒙 ∈ ℝ ⇒ ∑ 𝐥𝐧(𝟏 − 𝒙 ) ≤ 𝟑𝐥𝐧 (𝟏 − ( ) )
𝟑
𝐜𝐲𝐜

30 RMM-ABSTRACT ALGEBRA MARATHON 601-700


www.ssmrmh.ro
𝒙+𝐲+𝐳 = 𝟏 𝟏 (∗∗∗) 𝟖
= 𝟑𝐥𝐧 (𝟏 − ) ⇒ ∑ 𝐥𝐧(𝟏 − 𝒙𝟐 ) ≤ 𝟑𝐥𝐧 ( ) ,′′ =′′ 𝐢𝐟𝐟 𝒙 = 𝐲 = 𝐳
𝟗 𝟗
𝐜𝐲𝐜
𝟖
∴ (∗∗), (∗∗∗) ⇒ ∑ 𝐥𝐧(𝟏 − 𝒙𝟐 ) = 𝟑𝐥𝐧 ( ) ⇒ 𝐞𝐪𝐮𝒂𝒍𝐢𝐭𝐲 𝐜𝒂𝐬𝐞 𝐟𝐨𝐫 (∗∗∗)
𝟗
𝐜𝐲𝐜

𝟏
⇒ 𝒙 = 𝐲 = 𝐳 𝒂𝐧𝐝 ∵ 𝒙 + 𝐲 + 𝐳 = 𝟏 ∴ 𝒙 = 𝐲 = 𝐳 = (𝒂𝐧𝐬)
𝟑
Solution 3 by Khanh Hai-Vietnam
𝒙, 𝒚, 𝒛 > 0; (𝒊)
{ √(𝟏 + 𝒙)(𝟏 + 𝒚)(𝟏 + 𝒛) + 𝟑√ (𝟏 − 𝒙)(𝟏 − 𝒚)(𝟏 − 𝒛) = 𝟐; (𝒊𝒊)
𝟑

𝒙 + 𝒚 + 𝒛 = 𝟏; (𝒊𝒊𝒊)
𝑭𝒓𝒐𝒎 (𝒊) 𝒂𝒏𝒅 (𝒊𝒊𝒊), 𝒘𝒆 𝒉𝒂𝒗𝒆: 𝟎 < 𝑥, 𝑦, 𝑧 < 1 ⇒ 1 − 𝑥, 1 − 𝑦, 1 − 𝑧 > 0
𝟑 + ( 𝒙 + 𝒚 + 𝒛) 𝟑 − ( 𝒙 + 𝒚 + 𝒛)
𝑩𝒚 𝑨𝑴 − 𝑮𝑴: 𝑳𝑯𝑺(𝒊𝒊) ≤ + = 𝟐 = 𝑹𝑯𝑺(𝒊𝒊)
𝟑 𝟑
𝒙=𝒚=𝒛>0 𝟏
𝑬𝒒𝒖𝒂𝒍𝒊𝒕𝒚 𝒉𝒐𝒍𝒅𝒔 𝒊𝒇 𝒂𝒏𝒅 𝒐𝒏𝒍𝒚 𝒊𝒇 { ⇔𝒙=𝒚=𝒛= .
𝒙+𝒚+𝒛 =𝟏 𝟑
Solution 4 by Myagmarsuren Yadamsuren-Darkhan-Mongolia
∑𝒙=𝟏
𝑴 = ∏(𝟏 + 𝒙) = 𝟐 + ∑ 𝒙𝒚 + 𝒙𝒚𝒛
𝒄𝒚𝒄 𝒄𝒚𝒄

𝑵 = ∏(𝟏 − 𝒙) = ∑ 𝒙𝒚 − 𝒙𝒚𝒛
𝒄𝒚𝒄 𝒄𝒚𝒄
𝟐
𝟏 𝟐 𝟖
𝑴 + 𝑵 = 𝟐 + 𝟐 ∑ 𝒙𝒚 = 𝟐 + 𝟐 ⋅ (∑ 𝒙) = 𝟐 + =
𝟑 𝟑 𝟑
𝒄𝒚𝒄 𝒄𝒚𝒄

𝟖
{𝑴 + 𝑵 = 𝟑
𝑴>𝑁
𝟑 𝟑 𝟑
( √𝑴 + √𝑵) = 𝟐𝟑
𝟑 𝟑 𝟑 𝟑 𝟖 𝟑 𝟖
𝟖 = 𝑴 + 𝑵 + 𝟑√𝑴𝑵( √𝑴 + √𝑵) ⇒ 𝟑 ⋅ √𝑴𝑵 = ⇒ √𝑴𝑵 =
𝟑 𝟗
𝑨+𝑩=𝟐
𝟑 𝟑 𝟖 𝟒 𝟐
𝑳𝒆𝒕 √𝑴 = 𝑨 𝒂𝒏𝒅 √𝑵 = 𝑩 ⇒ { 𝑨𝑩 = ⇒ (𝑨, 𝑩) = ( , ),
𝟗 𝟑 𝟑
𝑨>𝐵

31 RMM-ABSTRACT ALGEBRA MARATHON 601-700


www.ssmrmh.ro
𝟒 𝟐 𝟐 𝟒
(𝑨𝟏 , 𝑩𝟏 ) = ( , ) , (𝑨𝟐 , 𝑩𝟐 ) = ( , )
𝟑 𝟑 𝟑 𝟑
𝟐 𝟑 𝟖 𝟏 𝟏
𝑩= = √∑ 𝒙𝒚 − 𝒙𝒚𝒛 ⇒ = ∑ 𝒙𝒚 − 𝒙𝒚𝒛 = − 𝒙𝒚𝒛 ⇒ 𝒙𝒚𝒛 =
𝟑 𝟐𝟕 𝟑 𝟐𝟕
𝒄𝒚𝒄 𝒄𝒚𝒄

𝟏 𝟏
𝒑 = 𝟏, 𝒒 = ,𝒓 =
𝟑 𝟐𝟕

𝟑 𝟐
𝟏 𝟏 𝟏 𝟑 𝟏 𝟏
( )
𝒕 = 𝒙, 𝒚, 𝒛 ⇒ 𝒕 − 𝒕 + 𝒕 − = 𝟎 ⇔ (𝒕 − ) = 𝟎 ⇒ 𝒕 = ⇒ 𝒙 = 𝒚 = 𝒛 = .
𝟑 𝟐𝟕 𝟑 𝟑 𝟑
𝟏𝟓
630. 𝒂 < 𝑏 = 𝑐 < 𝑑 < 𝑒, 𝛼 = √𝜷 = 𝒆𝒊 𝒂𝒓𝒄𝒔𝒊𝒏 𝟖𝒊 ,
√𝒂+√𝒃+√𝒆 𝒂𝒅
𝒙𝜶 + 𝜷𝒙𝜶−𝟏 − 𝟐𝜷𝒙𝜶−𝟑 = 𝜶𝟐 for: 𝒙 = . Find:
√𝒄+√𝒅 𝒆

Proposed by Samir Cabiyev-Azerbaijan


Solution by Pham Duc Nam-Vietnam

𝟏𝟓 𝟏𝟓 𝟏𝟓𝒊 𝟏𝟓
∗ 𝜶 = √𝜷 = 𝒆𝒊 𝒂𝒓𝒄𝒔𝒊𝒏 𝟖𝒊 , 𝒊 𝒂𝒓𝒄𝒔𝒊𝒏 = 𝒊 𝒂𝒓𝒄𝒔𝒊𝒏 (− ) = 𝒊. 𝒊 𝒔𝒊𝒏𝒉−𝟏 (− )
𝟖𝒊 𝟖 𝟖
𝟏𝟓 𝟐 𝟏𝟓
= − 𝒍𝒐𝒈 (√(− ) +𝟏− )
𝟖 𝟖

𝟐𝟖𝟗 𝟏𝟓 𝟏𝟕 𝟏𝟓 𝟏𝟓
= − 𝒍𝒐𝒈 (√ − ) = − 𝒍𝒐𝒈 ( − ) = 𝒍𝒐𝒈(𝟒) ⇒ 𝜶 = √𝜷 = 𝒆𝒊 𝒂𝒓𝒄𝒔𝒊𝒏 𝟖𝒊 = 𝟒
𝟔𝟒 𝟖 𝟖 𝟖
∗ 𝒙𝜶 + 𝜷𝒙𝜶−𝟏 − 𝟐𝜷𝒙𝜶−𝟑 = 𝜶𝟐 ⇔ 𝒙𝟒 + 𝟏𝟔𝒙𝟑 − 𝟑𝟐𝒙 = 𝟏𝟔 ⇔ 𝒙𝟒 + 𝟏𝟔𝒙𝟑 − 𝟑𝟐𝒙 − 𝟏𝟔 = 𝟎
Sub: 𝒙 = 𝒕 − 𝟒 ⇔ 𝒕𝟒 − 𝟗𝟔𝒕𝟐 + 𝟒𝟖𝟎𝒕 − 𝟔𝟓𝟔 = 𝟎
𝟐 𝟐 𝒕𝟐 − 𝟖 = 𝟒√𝟓(𝒕 − 𝟑)
⇔ (𝒕𝟐 − 𝟖) − 𝟖𝟎(𝒕 − 𝟑)𝟐 = 𝟎 ⇔ (𝒕𝟐 − 𝟖) = 𝟖𝟎(𝒕 − 𝟑)𝟐 ⇔ [ 𝟐
𝒕 − 𝟖 = −𝟒√𝟓(𝒕 − 𝟑)

𝑡 = 2 (√𝟓 − √𝟕 − 𝟑√𝟓) ⇒ 𝒙 = 𝟐 (√𝟓 − √𝟕 − 𝟑√𝟓) − 𝟒


⊕ 𝒕𝟐 − 𝟖 = 𝟒√𝟓(𝒕 − 𝟑) ⇔
𝑡 = 2 (√𝟓 + √𝟕 − 𝟑√𝟓) ⇒ 𝒙 = 𝟐 (√𝟓 + √𝟕 − 𝟑√𝟓) − 𝟒
[

𝑡 = −𝟐 (√𝟓 + √𝟕 + 𝟑√𝟓) ⇒ 𝒙 = −𝟐 (√𝟓 + √𝟕 + 𝟑√𝟓) − 𝟒


𝟐
⊕ 𝒕 − 𝟖 = −𝟒√𝟓(𝒕 − 𝟑) ⇔
𝑡 = −𝟐 (√𝟓 − √𝟕 + 𝟑√𝟓) ⇒ 𝒙 = −𝟐 (√𝟓 − √𝟕 + 𝟑√𝟓) − 𝟒
[

32 RMM-ABSTRACT ALGEBRA MARATHON 601-700


www.ssmrmh.ro
√𝒂 + √𝒃 + √𝒆 √𝟐 + 𝟐 + √𝟏𝟎 √𝟐 + √𝟒 + √𝟏𝟎
∗𝒙 = > 0 ⇒ 𝒙 = 𝟐 (√𝟓 + √𝟕 − 𝟑√𝟓) − 𝟒 = =
√𝒄 + √𝒅 𝟐 + √𝟓 √𝟒 + √𝟓
√𝒂 + √𝒃 + √𝒆 𝒂𝒅
= ⇒ 𝒂 = 𝟐, 𝒅 = 𝟓, 𝒆 = 𝟏𝟎 ⇒ =𝟏
√𝒄 + √𝒅 𝒆
631. Solve for real numbers:
𝟏
𝟑 𝟒𝟐𝒙𝟒 +𝟏𝟐𝟏
𝟏 𝒙 + 𝟏𝟏𝒙
(𝟑𝒙𝟔 + 𝟐𝒙𝟑 + 𝟏𝟑𝟓𝒙𝟐 + 𝟐𝟐𝒙). + 𝒍𝒐𝒈 𝟔 ( ) =𝟏
𝟒𝟐𝒙𝟒 + 𝟏𝟐𝟏 𝒙𝟐 + 𝟏
Proposed by Neculai Stanciu-Romania
Solution by Pham Duc Nam-Vietnam
∗ Domain: 𝒙 > 𝟎
𝟏 𝟏 𝒙𝟑 + 𝟏𝟏𝒙
⇔ (𝟑𝒙𝟔 + 𝟐𝒙𝟑 + 𝟏𝟑𝟓𝒙𝟐 + 𝟐𝟐𝒙). + . 𝒍𝒐𝒈 𝟔 ( )=𝟏
𝟒𝟐𝒙𝟒 + 𝟏𝟐𝟏 𝟒𝟐𝒙𝟒 + 𝟏𝟐𝟏 𝒙𝟐 + 𝟏
⇔ (𝟑𝒙𝟔 + 𝟐𝒙𝟑 + 𝟏𝟑𝟓𝒙𝟐 + 𝟐𝟐𝒙) + 𝒍𝒐𝒈𝟔 (𝒙𝟑 + 𝟏𝟏𝒙) − 𝒍𝒐𝒈𝟔 (𝒙𝟐 + 𝟏) = 𝟒𝟐𝒙𝟒 + 𝟏𝟐𝟏
⇔ 𝟑(𝒙𝟑 + 𝟏𝟏𝒙)𝟐 + 𝟐(𝒙𝟑 + 𝟏𝟏𝒙) + 𝒍𝒐𝒈𝟔(𝒙𝟑 + 𝟏𝟏𝒙)
𝟐
= 𝟑. (𝟔(𝒙𝟐 + 𝟏)) + 𝟐. 𝟔(𝒙𝟐 + 𝟏) + 𝒍𝒐𝒈𝟔 𝟔 (𝒙𝟐 + 𝟏)
𝟏
∗ Let: 𝒇(𝒕) = 𝟑𝒕𝟐 + 𝟐𝒕 + 𝒍𝒐𝒈𝟔 𝒕 , 𝒕 > 0 ⇒ 𝒇′(𝒕) = 𝟔𝒕 + 𝟐 + > 0∀𝒕 > 0
𝒕 𝒍𝒐𝒈(𝟔)

⇒ 𝒇(𝒕) is always increasing ⇒ 𝒇(𝒙𝟑 + 𝟏𝟏𝒙) = 𝒇 (𝟔(𝒙𝟐 + 𝟏)) ⇔ 𝒙𝟑 + 𝟏𝟏𝒙 = 𝟔(𝒙𝟐 + 𝟏)

⇔ 𝒙𝟑 − 𝟔𝒙𝟐 + 𝟏𝟏𝒙 − 𝟔 = 𝟎 ⇔ (𝒙 − 𝟏)(𝒙 − 𝟐)(𝒙 − 𝟑) = 𝟎


⇒ 𝒙 = 𝟏, 𝒙 = 𝟐, 𝒙 = 𝟑 are solutions
632. Solve for 𝒙 ∈ ℝ

𝟒 −𝟒𝒙𝟐 −𝟑𝒙+𝟔 𝟑𝒙
𝟐𝒙 =√ 𝟐
𝒙 +𝟐

Proposed by Neculai Stanciu-Romania


Solution by Pham Duc Nam-Vietnam
𝟒 −𝟒𝒙𝟐 −𝟑𝒙+𝟔 𝟑𝒙
𝟐𝒙 =√ 𝟐 ( 𝟏)
𝒙 +𝟐

∗ Domain: 𝒙 > 0
∗ 𝒙𝟒 − 𝟒𝒙𝟐 − 𝟑𝒙 + 𝟔 = (𝒙𝟐 + 𝟐)𝟐 − (𝟑𝒙)𝟐 + (𝒙𝟐 + 𝟐 − 𝟑𝒙)
33 RMM-ABSTRACT ALGEBRA MARATHON 601-700
www.ssmrmh.ro
𝟏 𝟏
⇒ (𝟏) ⇔ (𝒙𝟐 + 𝟐)𝟐 − (𝟑𝒙)𝟐 + (𝒙𝟐 + 𝟐 − 𝟑𝒙) = 𝒍𝒐𝒈𝟐 (𝟑𝒙) − 𝒍𝒐𝒈𝟐(𝒙𝟐 + 𝟐)
𝟐 𝟐
𝟏 𝟏
⇔ (𝒙𝟐 + 𝟐)𝟐 + (𝒙𝟐 + 𝟐) + 𝒍𝒐𝒈𝟐 (𝒙𝟐 + 𝟐) = (𝟑𝒙)𝟐 + 𝟑𝒙 + 𝒍𝒐𝒈𝟐 (𝟑𝒙)
𝟐 𝟐
𝟏
∗ Consider : 𝒇(𝒕) = 𝒕𝟐 + 𝒕 + 𝒍𝒐𝒈𝟐 (𝒕) , 𝒕 > 0
𝟐
𝟏
⇒ 𝒇′(𝒕) = 𝟐𝒕 + + 𝟏 > 0∀𝒕 > 0 ⇒ 𝒇(𝒕) is always increasing
𝟐𝒕 𝒍𝒐𝒈(𝟐)
𝑥 = 1(
⇒ 𝒇(𝒙𝟐 + 𝟐) = 𝒇(𝟑𝒙) ⇔ 𝒙𝟐 + 𝟐 = 𝟑𝒙 ⇔ [ Satisfied)
𝑥=2
⇒ 𝒙 = 𝟏, 𝒙 = 𝟐 are solutions
𝐧+𝟒 𝐧+𝟒
633. 𝐏𝐫𝐨𝐯𝐞 𝐭𝐡𝒂𝐭 ∶ 𝐅𝐧+𝟐 − 𝐅𝐧 𝐅𝐧+𝟏 < 𝒂𝐧𝐝 𝐋𝐧+𝟐 − 𝐋𝐧 𝐋𝐧+𝟏 <
𝟒 𝟒

Proposed by D.M. Bătinețu-Giurgiu, Neculai Stanciu-Romania


Solution by Soumava Chakraborty-Kolkata-India
𝐧 𝐧
𝐧+𝟒 𝐧
𝐅𝐧+𝟐 − 𝐅𝐧 𝐅𝐧+𝟏 < ⇔ 𝟏 + ∑ 𝐅𝐤 − ∑ 𝐅𝐤𝟐 < + 𝟏
𝟒 𝟒
𝐤=𝟏 𝐤=𝟏
𝐧 𝐧
𝟏 (∗)
⇔ ∑ (𝐅𝐤𝟐 + ) > ∑ 𝐅𝐤
𝟒
𝐤=𝟏 𝐤=𝟏
𝐧 𝐧 𝐧
𝟏 𝐀−𝐆 𝟏
𝐍𝐨𝐰, ∑ (𝐅𝐤𝟐 + ) > 𝟐. ∑ √𝐅𝐤𝟐 . = ∑ 𝐅𝐤 ⇒ (∗) 𝐢𝐬 𝐭𝐫𝐮𝐞
𝟒 𝟒
𝐤=𝟏 𝐤=𝟏 𝐤=𝟏
𝐧+𝟒
∴ 𝐅𝐧+𝟐 − 𝐅𝐧 𝐅𝐧+𝟏 <
𝟒
𝐧+𝟒 𝐧
𝐀𝐠𝒂𝐢𝐧, 𝐋𝐧+𝟐 − 𝐋𝐧 𝐋𝐧+𝟏 < ⇔ 𝐋𝐧 𝐋𝐧+𝟏 + + 𝟏 > 𝐋𝐧+𝟐
𝟒 𝟒
𝐧 𝐧 𝐧 𝐧
𝐧
⇔ ∑ 𝐋𝟐𝐤 + 𝟐 + + 𝟏 > ∑ 𝐋𝐤 + 𝟏 = 𝐋𝟎 + ∑ 𝐋𝐤 + 𝟏 = 𝟐 + ∑ 𝐋𝐤 + 𝟏
𝟒
𝐤=𝟏 𝐤=𝟎 𝐤=𝟏 𝐤=𝟏
𝐧 𝐧 𝐧 𝐧 𝐧
𝟏 𝟏 𝐀−𝐆 𝟏
⇔ ∑ (𝐋𝟐𝐤 + ) > ∑ 𝐋𝐤 → 𝐭𝐫𝐮𝐞 ∵ ∑ (𝐋𝟐𝐤 + ) > 𝟐. ∑ √𝐋𝟐𝐤 . = ∑ 𝐋𝐤
𝟒 𝟒 𝟒
𝐤=𝟏 𝐤=𝟏 𝐤=𝟏 𝐤=𝟏 𝐤=𝟏
𝐧+𝟒
∴ 𝐋𝐧+𝟐 − 𝐋𝐧 𝐋𝐧+𝟏 < (𝐐𝐄𝐃)
𝟒

34 RMM-ABSTRACT ALGEBRA MARATHON 601-700


www.ssmrmh.ro
634.𝐏𝐫𝐨𝐯𝐞 ∶
𝐦 𝐦
𝐅𝐧+𝟐 . 𝐬𝐢𝐧𝐦+𝟐 𝐭 𝐅𝐧+𝟐 . 𝐜𝐨𝐬𝐦+𝟐 𝐭
+ ≥ 𝟏;
(𝐅𝐧 𝐬𝐢𝐧 𝐭 + 𝐅𝐧+𝟏 𝐜𝐨𝐬 𝐭)𝐦 (𝐅𝐧 𝐜𝐨𝐬 𝐭 + 𝐅𝐧+𝟏 𝐬𝐢𝐧 𝐭)𝐦
𝐋𝐦 𝟐
𝐧+𝟐 . 𝐜𝐨𝐬 𝐭 𝐋𝐦
𝐧+𝟐 . 𝐭𝒂𝐧
𝐦+𝟐
𝐭 . 𝐜𝐨𝐬𝟐 𝐭
+ ≥ 𝟏,
(𝐋𝐧 + 𝐋𝐧+𝟏 𝐭𝒂𝐧 𝐭)𝐦 (𝐋𝐧 𝐭𝒂𝐧 𝐭 + 𝐋𝐧+𝟏 )𝐦
𝛑
𝐰𝐡𝐞𝐫𝐞 𝐭 ∈ (𝟎, ) , 𝐦 ≥ 𝟎, 𝒂𝐧𝐝
𝟐
(𝐅𝐧 )𝐧 ≥ 𝟎 𝐢𝐬 𝐅𝐢𝐛𝐨𝐧𝒂𝐜𝐜𝐢′ 𝐬 𝐬𝐞𝐪𝐮𝐞𝐧𝐜𝐞 𝒂𝐧𝐝 (𝐋𝐧 )𝐧 ≥ 𝟎 𝐢𝐬 𝐋𝐮𝐜𝒂𝐬′ 𝐬𝐞𝐪𝐮𝐞𝐧𝐜𝐞
Proposed by D.M. Bătinețu-Giurgiu, Neculai Stanciu-Romania
Solution 1 by Soumava Chakraborty-Kolkata-India

𝐖𝐞 𝐬𝐡𝒂𝒍𝒍 𝐟𝐢𝐫𝐬𝐭 𝐩𝐫𝐨𝐯𝐞 ∶


𝐦 𝐦
(𝒙 + 𝐲). 𝐬𝐢𝐧 𝐭 𝟐
(𝒙 + 𝐲). 𝐜𝐨𝐬 𝐭 (∗)
( ) . 𝐬𝐢𝐧 𝐭 + ( ) . 𝐜𝐨𝐬 𝟐 𝐭 ≥ 𝟏
𝒙 𝐬𝐢𝐧 𝐭 + 𝐲 𝐜𝐨𝐬 𝐭 𝒙 𝐜𝐨𝐬 𝐭 + 𝐲 𝐬𝐢𝐧 𝐭
𝛑
∀ 𝒙, 𝐲 > 0, ∀ 𝐭│𝐭 ∈ (𝟎, ) 𝒂𝐧𝐝 ∀ 𝐦 ≥ 𝟎
𝟐
𝐂𝒂𝐬𝐞 𝟏 𝐦 = 𝟎, 𝒂𝐧𝐝 𝐭𝐡𝐞𝐧 ∶ 𝐋𝐇𝐒 𝐨𝐟 (∗) = 𝐬𝐢𝐧𝟐 𝐭 + 𝐜𝐨𝐬 𝟐 𝐭 = 𝟏
𝐁𝐞𝐫𝐧𝐨𝐮𝐥𝐥𝐢
𝐂𝒂𝐬𝐞 𝟐 𝐦 ≥ 𝟏 𝒂𝐧𝐝 𝐭𝐡𝐞𝐧 ∶ 𝐋𝐇𝐒 𝐨𝐟 (∗) ≥
(𝒙 + 𝐲). 𝐬𝐢𝐧 𝐭 (𝒙 + 𝐲). 𝐜𝐨𝐬 𝐭
(𝟏 + 𝐦 ( − 𝟏)) 𝐬𝐢𝐧𝟐 𝐭 + (𝟏 + 𝐦 ( − 𝟏)) 𝐜𝐨𝐬 𝟐 𝐭
𝒙 𝐬𝐢𝐧 𝐭 + 𝐲 𝐜𝐨𝐬 𝐭 𝒙 𝐜𝐨𝐬 𝐭 + 𝐲 𝐬𝐢𝐧 𝐭
𝐲(𝐬𝐢𝐧 𝐭 − 𝐜𝐨𝐬 𝐭) 𝐲(𝐬𝐢𝐧 𝐭 − 𝐜𝐨𝐬 𝐭)
= 𝐬𝐢𝐧𝟐 𝐭 + 𝐜𝐨𝐬 𝟐 𝐭 + 𝐦 ( . 𝐬𝐢𝐧𝟐 𝐭 − . 𝐜𝐨𝐬 𝟐 𝐭)
𝒙 𝐬𝐢𝐧 𝐭 + 𝐲 𝐜𝐨𝐬 𝐭 𝒙 𝐜𝐨𝐬 𝐭 + 𝐲 𝐬𝐢𝐧 𝐭
𝐬𝐢𝐧𝟐 𝐭 𝐜𝐨𝐬 𝟐 𝐭
= 𝟏 + 𝐦𝐲(𝐬𝐢𝐧 𝐭 − 𝐜𝐨𝐬 𝐭) ( − )
𝒙 𝐬𝐢𝐧 𝐭 + 𝐲 𝐜𝐨𝐬 𝐭 𝒙 𝐜𝐨𝐬 𝐭 + 𝐲 𝐬𝐢𝐧 𝐭
𝐦𝐲(𝐬𝐢𝐧 𝐭 − 𝐜𝐨𝐬 𝐭) ((𝒙 𝐜𝐨𝐬 𝐭 + 𝐲 𝐬𝐢𝐧 𝐭) 𝐬𝐢𝐧𝟐 𝐭 − (𝒙 𝐬𝐢𝐧 𝐭 + 𝐲 𝐜𝐨𝐬 𝐭) 𝐜𝐨𝐬 𝟐 𝐭)
=𝟏+ =𝟏+
(𝒙 𝐬𝐢𝐧 𝐭 + 𝐲 𝐜𝐨𝐬 𝐭)(𝒙 𝐜𝐨𝐬 𝐭 + 𝐲 𝐬𝐢𝐧 𝐭)
𝐦𝐲(𝐬𝐢𝐧 𝐭 − 𝐜𝐨𝐬 𝐭) (𝒙 𝐬𝐢𝐧 𝐭 𝐜𝐨𝐬 𝐭 (𝐬𝐢𝐧 𝐭 − 𝐜𝐨𝐬 𝐭) + 𝐲(𝐬𝐢𝐧 𝐭 − 𝐜𝐨𝐬 𝐭)(𝐬𝐢𝐧𝟐 𝐭 + 𝐜𝐨𝐬 𝟐 𝐭 − 𝐬𝐢𝐧 𝐭 𝐜𝐨𝐬 𝐭))
(𝒙 𝐬𝐢𝐧 𝐭 + 𝐲 𝐜𝐨𝐬 𝐭)(𝒙 𝐜𝐨𝐬 𝐭 + 𝐲 𝐬𝐢𝐧 𝐭)
𝐦𝐲 𝐬𝐢𝐧 𝐭 − 𝐜𝐨𝐬 𝐭)𝟐 (𝒙 𝐬𝐢𝐧 𝐭 𝐜𝐨𝐬 𝐭 + 𝐲(𝟏 − 𝐬𝐢𝐧 𝐭 𝐜𝐨𝐬 𝐭))
(
=𝟏+ ≥𝟏
(𝒙 𝐬𝐢𝐧 𝐭 + 𝐲 𝐜𝐨𝐬 𝐭)(𝒙 𝐜𝐨𝐬 𝐭 + 𝐲 𝐬𝐢𝐧 𝐭)
𝛑
∵ 𝐦, 𝒙, 𝐲 > 0, 𝐭 ∈ (𝟎, ) 𝒂𝐧𝐝 𝟏 − 𝐬𝐢𝐧 𝐭 𝐜𝐨𝐬 𝐭 > 0 ∴ 𝐜𝐨𝐦𝐛𝐢𝐧𝐢𝐧𝐠 𝐛𝐨𝐭𝐡 𝐜𝒂𝐬𝐞𝐬,
𝟐
𝐦 𝐦
(𝒙 + 𝐲). 𝐬𝐢𝐧 𝐭 𝟐
(𝒙 + 𝐲). 𝐜𝐨𝐬 𝐭
( ) . 𝐬𝐢𝐧 𝐭 + ( ) . 𝐜𝐨𝐬 𝟐 𝐭 ≥ 𝟏
𝒙 𝐬𝐢𝐧 𝐭 + 𝐲 𝐜𝐨𝐬 𝐭 𝒙 𝐜𝐨𝐬 𝐭 + 𝐲 𝐬𝐢𝐧 𝐭

35 RMM-ABSTRACT ALGEBRA MARATHON 601-700


www.ssmrmh.ro
𝛑
∀ 𝒙, 𝐲 > 0, ∀ 𝐭│𝐭 ∈ (𝟎, ) 𝒂𝐧𝐝 ∀ 𝐦 ≥ 𝟎
𝟐
𝐦 𝐦
𝐅𝐧+𝟐 . 𝐬𝐢𝐧𝐦+𝟐 𝐭 𝐅𝐧+𝟐 . 𝐜𝐨𝐬 𝐦+𝟐 𝐭
𝐍𝐨𝐰, +
(𝐅𝐧 𝐬𝐢𝐧 𝐭 + 𝐅𝐧+𝟏 𝐜𝐨𝐬 𝐭)𝐦 (𝐅𝐧 𝐜𝐨𝐬 𝐭 + 𝐅𝐧+𝟏 𝐬𝐢𝐧 𝐭)𝐦
𝐦 𝐦
(𝐅𝐧 + 𝐅𝐧+𝟏 ). 𝐬𝐢𝐧𝐭 𝟐
(𝐅𝐧 + 𝐅𝐧+𝟏 ). 𝐜𝐨𝐬𝐭
=( ) . 𝐬𝐢𝐧 𝐭 + ( ) . 𝐜𝐨𝐬 𝟐 𝐭
𝐅𝐧 𝐬𝐢𝐧 𝐭 + 𝐅𝐧+𝟏 𝐜𝐨𝐬 𝐭 𝐅𝐧 𝐜𝐨𝐬 𝐭 + 𝐅𝐧+𝟏 𝐬𝐢𝐧 𝐭
𝐯𝐢𝒂 (∗)
≥ 𝟏 (∵ 𝐅𝐧 , 𝐅𝐧+𝟏 > 0 ∀ 𝐧 ∈ ℕ)
𝐦 𝐦
𝐅𝐧+𝟐 . 𝐬𝐢𝐧𝐦+𝟐 𝐭 𝐅𝐧+𝟐 . 𝐜𝐨𝐬 𝐦+𝟐 𝐭
∴ + ≥𝟏
(𝐅𝐧 𝐬𝐢𝐧 𝐭 + 𝐅𝐧+𝟏 𝐜𝐨𝐬 𝐭)𝐦 (𝐅𝐧 𝐜𝐨𝐬 𝐭 + 𝐅𝐧+𝟏 𝐬𝐢𝐧 𝐭)𝐦
𝛑 𝛑
∀ 𝐧 ∈ ℕ, ∀ 𝐭│𝐭 ∈ (𝟎, ) 𝒂𝐧𝐝 ∀ 𝐦 ≥ 𝟎,′′ =′′ 𝐟𝐨𝐫 𝐦 = 𝟎 𝐨𝐫 𝐟𝐨𝐫 𝐭 =
𝟐 𝟒
𝐋𝐦 𝟐
𝐧+𝟐 . 𝐜𝐨𝐬 𝐭 𝐋𝐦𝐧+𝟐 . 𝐭𝒂𝐧
𝐦+𝟐
𝐭 . 𝐜𝐨𝐬 𝟐 𝐭
𝐀𝐠𝒂𝐢𝐧, +
(𝐋𝐧 + 𝐋𝐧+𝟏 𝐭𝒂𝐧 𝐭)𝐦 (𝐋𝐧 𝐭𝒂𝐧 𝐭 + 𝐋𝐧+𝟏 )𝐦
𝐦 𝐦
(𝐋𝐧 + 𝐋𝐧+𝟏 ). 𝐜𝐨𝐬𝐭 𝟐
(𝐋𝐧 + 𝐋𝐧+𝟏 ). 𝐬𝐢𝐧𝐭
=( ) . 𝐜𝐨𝐬 𝐭 + ( ) . 𝐬𝐢𝐧𝟐 𝐭
𝐋𝐧 𝐜𝐨𝐬 𝐭 + 𝐋𝐧+𝟏 𝐬𝐢𝐧 𝐭 𝐋𝐧 𝐬𝐢𝐧 𝐭 + 𝐋𝐧+𝟏 𝐜𝐨𝐬 𝐭
𝐯𝐢𝒂 (∗)
≥ 𝟏 (∵ 𝐋𝐧 , 𝐋𝐧+𝟏 > 0 ∀ 𝐧 ∈ ℕ)
𝐋𝐦 𝟐
𝐧+𝟐 . 𝐜𝐨𝐬 𝐭 𝐋𝐦
𝐧+𝟐 . 𝐭𝒂𝐧
𝐦+𝟐
𝐭 . 𝐜𝐨𝐬 𝟐 𝐭
∴ + ≥𝟏
(𝐋𝐧 + 𝐋𝐧+𝟏 𝐭𝒂𝐧 𝐭)𝐦 (𝐋𝐧 𝐭𝒂𝐧 𝐭 + 𝐋𝐧+𝟏 )𝐦
𝛑 𝛑
∀ 𝐧 ∈ ℕ, ∀ 𝐭│𝐭 ∈ (𝟎, ) 𝒂𝐧𝐝 ∀ 𝐦 ≥ 𝟎,′′ =′′ 𝐟𝐨𝐫 𝐦 = 𝟎 𝐨𝐫 𝐟𝐨𝐫 𝐭 = (𝐐𝐄𝐃)
𝟐 𝟒
Solution 2 by Mohamed Amine Ben Ajiba-Tanger-Morocco
𝐁𝐲 𝑹𝒂𝒅𝒐𝒏′𝒔 𝒊𝒏𝒆𝒒𝒖𝒂𝒍𝒊𝒕𝒚 𝐚𝐧𝐝 𝑨𝑴 − 𝑮𝑴 𝒊𝒏𝒆𝒒𝒖𝒂𝒍𝒊𝒕𝒚, 𝐰𝐞 𝐡𝐚𝐯𝐞:

𝐬𝐢𝐧𝒎+𝟐 𝒕 𝐜𝐨𝐬𝒎+𝟐 𝒕
+
(𝑭𝒏 𝐬𝐢𝐧 𝒕 + 𝑭𝒏+𝟏 𝐜𝐨𝐬 𝒕)𝒎 (𝑭𝒏 𝐜𝐨𝐬 𝒕 + 𝑭𝒏+𝟏 𝐬𝐢𝐧 𝒕)𝒎
𝒎+𝟏 𝒎+𝟏
(𝐬𝐢𝐧𝟐 𝒕) (𝐜𝐨𝐬𝟐 𝒕)
= +
(𝑭𝒏 𝐬𝐢𝐧𝟐 𝒕 + 𝑭𝒏+𝟏 𝐜𝐨𝐬 𝒕 𝐬𝐢𝐧 𝒕)𝒎 (𝑭𝒏 𝐜𝐨𝐬𝟐 𝒕 + 𝑭𝒏+𝟏 𝐜𝐨𝐬 𝒕 𝐬𝐢𝐧 𝒕)𝒎
𝒎+𝟏
(𝐬𝐢𝐧𝟐 𝒕 + 𝐜𝐨𝐬𝟐 𝒕) 𝟏
≥ 𝟐 𝟐 𝒎

[𝑭𝒏 (𝐬𝐢𝐧 𝒕 + 𝐜𝐨𝐬 𝒕) + 𝟐𝑭𝒏+𝟏 𝐜𝐨𝐬 𝒕 𝐬𝐢𝐧 𝒕] [𝑭𝒏 + 𝑭𝒏+𝟏 (𝐬𝐢𝐧𝟐 𝒕 + 𝐜𝐨𝐬 𝟐 𝒕)]𝒎

𝟏 𝟏 𝑭𝒏+𝟐 𝒎 𝐬𝐢𝐧𝒎+𝟐 𝒕 𝑭𝒏+𝟐 𝒎 𝐜𝐨𝐬𝒎+𝟐 𝒕


= = ⇒ + ≥ 𝟏.
(𝑭𝒏 + 𝑭𝒏+𝟏 )𝒎 𝑭𝒏+𝟐 𝒎 (𝑭𝒏 𝐬𝐢𝐧 𝒕 + 𝑭𝒏+𝟏 𝐜𝐨𝐬 𝒕)𝒎 (𝑭𝒏 𝐜𝐨𝐬 𝒕 + 𝑭𝒏+𝟏 𝐬𝐢𝐧 𝒕)𝒎
𝒎+𝟏
𝟏 𝐭𝐚𝐧𝒎+𝟐 𝒕 𝟏 (𝐭𝐚𝐧𝟐 𝒕)
+ = +
(𝑳𝒏 + 𝑳𝒏+𝟏 𝐭𝐚𝐧 𝒕)𝒎 (𝑳𝒏 𝐭𝐚𝐧 𝒕 + 𝑳𝒏+𝟏 )𝒎 (𝑳𝒏 + 𝑳𝒏+𝟏 𝐭𝐚𝐧 𝒕)𝒎 (𝑳𝒏 𝐭𝐚𝐧𝟐 𝒕 + 𝑳𝒏+𝟏 𝐭𝐚𝐧 𝒕)𝒎
𝒎+𝟏 𝒎+𝟏
(𝟏 + 𝐭𝐚𝐧𝟐 𝒕) (𝟏 + 𝐭𝐚𝐧𝟐 𝒕)
≥ ≥
[𝑳𝒏 (𝟏 + 𝐭𝐚𝐧𝟐 𝒕) + 𝟐𝑳𝒏+𝟏 𝐭𝐚𝐧 𝒕]𝒎 [𝑳𝒏 (𝟏 + 𝐭𝐚𝐧𝟐 𝒕) + 𝑳𝒏+𝟏 (𝟏 + 𝐭𝐚𝐧𝟐 𝒕)]𝒎

36 RMM-ABSTRACT ALGEBRA MARATHON 601-700


www.ssmrmh.ro
𝟏 + 𝐭𝐚𝐧𝟐 𝒕 𝟏 𝑳𝒏+𝟐 𝒎 𝐜𝐨𝐬 𝟐 𝒕 𝑳𝒏+𝟐 𝒎 𝐭𝐚𝐧𝒎+𝟐 𝒕 𝐜𝐨𝐬𝟐 𝒕
= = ⇒ + ≥𝟏
(𝑳𝒏 + 𝑳𝒏+𝟏 )𝒎 𝑳𝒏+𝟐 𝒎 𝐜𝐨𝐬𝟐 𝒕 (𝑳𝒏 + 𝑳𝒏+𝟏 𝐭𝐚𝐧 𝒕)𝒎 (𝑳𝒏 𝐭𝐚𝐧 𝒕 + 𝑳𝒏+𝟏 )𝒎
𝝅
𝐒𝐨 𝐭𝐡𝐞 𝐩𝐫𝐨𝐨𝐟 𝐢𝐬 𝐜𝐨𝐦𝐩𝐥𝐞𝐭𝐞𝐝. 𝐄𝐪𝐮𝐚𝐥𝐢𝐭𝐲 𝐡𝐨𝐥𝐝𝐬 𝐢𝐟 𝐚𝐧𝐝 𝐨𝐧𝐥𝐲 𝐢𝐟 𝒕 = .
𝟒
635. 𝐏𝐫𝐨𝐯𝐞 𝐭𝐡𝒂𝐭 ∶
𝐧
𝟏 𝟒 𝟐
𝐅𝐤𝟐 − 𝟏
∑ (√𝐅𝐤 − 𝐅𝐤 + 𝟏 + 𝟒 )≤𝟏
𝐅𝐧 𝐅𝐧+𝟏 𝐅𝐤 + 𝟏
𝐤=𝟏
Proposed by D.M.Bătinețu-Giurgiu, Neculai Stanciu
Solution by Soumava Chakraborty-Kolkata-India
𝒙𝟐 − 𝟏 ? 𝟐 ?
𝟐
𝒙𝟐 − 𝟏
∀ 𝒙 ≥ 𝟏, √𝒙𝟒 − 𝒙𝟐 + 𝟏 + √ 𝟒 𝟐
≤𝒙 ⇔ 𝒙 −𝒙 +𝟏 ≤𝒙 − 𝟒
𝒙𝟒 + 𝟏 𝒙 +𝟏
𝟔 𝟐 𝟒 𝟐
𝒙 + 𝟏 (𝒙 + 𝟏)(𝒙 − 𝒙 + 𝟏) 𝒙𝟒 + 𝟏 ?
= 𝟒 = ⇔ 𝟐 − 𝟏 ≤ √𝒙𝟒 − 𝒙𝟐 + 𝟏 − 𝟏
𝒙 +𝟏 𝒙𝟒 + 𝟏 𝒙 +𝟏
𝒙𝟒 − 𝒙𝟐 ? 𝒙𝟒 − 𝒙𝟐 + 𝟏 − 𝟏 𝟏 𝟏 ?
⇔ 𝟐 ≤ ⇔ (𝒙𝟒 − 𝒙𝟐 ) ( − 𝟐 )≥𝟎
𝒙 +𝟏 √𝒙𝟒 − 𝒙𝟐 + 𝟏 + 𝟏 √𝒙𝟒 − 𝒙𝟐 + 𝟏 + 𝟏 𝒙 + 𝟏
𝒙𝟐 − √𝒙𝟒 − 𝒙𝟐 + 𝟏 ?
⇔ (𝒙𝟒 − 𝒙𝟐 ) ( )≥𝟎
(𝒙𝟐 + 𝟏) (√𝒙𝟒 − 𝒙𝟐 + 𝟏 + 𝟏)
(𝒙𝟒 − 𝒙𝟐 ) 𝒙𝟒 − (𝒙𝟒 − 𝒙𝟐 + 𝟏) ?
⇔ . ≥𝟎
(𝒙𝟐 + 𝟏) (√𝒙𝟒 − 𝒙𝟐 + 𝟏 + 𝟏) 𝒙𝟐 + √𝒙𝟒 − 𝒙𝟐 + 𝟏
𝟐
𝒙𝟐 (𝒙𝟐 − 𝟏) ?
⇔ ≥ 𝟎 → 𝐭𝐫𝐮𝐞
(𝒙𝟐 + 𝟏) (√𝒙𝟒 − 𝒙𝟐 + 𝟏 + 𝟏) (𝒙𝟐 + √𝒙𝟒 − 𝒙𝟐 + 𝟏)
𝒙𝟐 − 𝟏
∴ √𝒙𝟒 − 𝒙𝟐 + 𝟏 +
𝟒 +𝟏
≤ 𝒙𝟐 ∀ 𝒙 ≥ 𝟏,′′ =′′ 𝐢𝐟𝐟 𝒙 = 𝟏 → (𝟏)
𝐧
𝒙 𝐧
𝟏 𝐅𝐤𝟐 − 𝟏 𝟏 𝐅𝐧 𝐅𝐧+𝟏
∴ 𝐯𝐢𝒂 (𝟏), 𝟒 𝟐
∑ (√𝐅𝐤 − 𝐅𝐤 + 𝟏 + 𝟒 )≤ ∑ 𝐅𝐤𝟐 = =𝟏
𝐅𝐧 𝐅𝐧+𝟏 𝐅𝐤 + 𝟏 𝐅𝐧 𝐅𝐧+𝟏 𝐅𝐧 𝐅𝐧+𝟏
𝐤=𝟏 𝐤=𝟏
𝐧
𝟏 𝐅𝟐 − 𝟏
∴ ∑ (√𝐅𝐤𝟒 − 𝐅𝐤𝟐 + 𝟏 + 𝐤𝟒 ) ≤ 𝟏,′′ =′′ 𝐢𝐟𝐟 𝐧 = 𝟏 (𝐐𝐄𝐃)
𝐅𝐧 𝐅𝐧+𝟏 𝐅𝐤 + 𝟏
𝐤=𝟏

636. 𝐏𝐫𝐨𝐯𝐞 𝐭𝐡𝒂𝐭 ∶


𝐧 𝐧
𝟐𝐅𝐤 𝟐𝐅𝐤+𝟏 𝟐𝐋𝐤 𝟐𝐋𝐤+𝟏
∑( + ) > 𝐧 𝒂𝐧𝐝 ∑ ( + )>𝐧
𝐅𝐤+𝟑 𝟐𝐅𝐤 + 𝐅𝐤+𝟏 𝐋𝐤+𝟑 𝟐𝐋𝐤 + 𝐋𝐤+𝟏
𝐤=𝟏 𝐤=𝟏
𝐰𝐡𝐞𝐫𝐞 𝐅𝐧 𝒂𝐧𝐝 𝐋𝐧 𝒂𝐫𝐞 𝐭𝐡𝐞 𝐧 − 𝐭𝐡 𝐅𝐢𝐛𝐨𝐧𝒂𝐜𝐜𝐢 𝒂𝐧𝐝 𝐧 − 𝐭𝐡 𝐋𝐮𝐜𝒂𝐬 𝐧𝐮𝐦𝐛𝐞𝐫

Proposed by D.M.Bătinețu-Giurgiu, Neculai Stanciu-Romania

37 RMM-ABSTRACT ALGEBRA MARATHON 601-700


www.ssmrmh.ro
Solution by Soumava Chakraborty-Kolkata-India
𝟐𝒙 𝟐𝐲 ?
+ > 𝟏∀𝒙 >0
𝒙 + 𝟐𝐲 𝟐𝒙 + 𝐲
? ?
⇔ 𝟒𝒙𝟐 + 𝟐𝒙𝐲 + 𝟐𝒙𝐲 + 𝟒𝐲 𝟐 > 𝟐𝒙𝟐 + 𝟐𝐲 𝟐 + 𝟓𝒙𝐲 ⇔ 𝟐𝒙𝟐 − 𝒙𝐲 + 𝟐𝐲 𝟐 > 𝟎
? 𝟐𝒙 𝟐𝐲 (∗)
⇔ 𝟐(𝒙 − 𝐲)𝟐 + 𝟑𝒙𝐲 > 𝟎 → 𝐭𝐫𝐮𝐞 ∴ + > 𝟏∀𝒙 >0
𝒙 + 𝟐𝐲 𝟐𝒙 + 𝐲
𝟐𝐅𝐤 𝟐𝐅𝐤+𝟏 𝟐𝐅𝐤 𝟐𝐅𝐤+𝟏
𝐍𝐨𝐰, + = +
𝐅𝐤+𝟑 𝟐𝐅𝐤 + 𝐅𝐤+𝟏 𝐅𝐤+𝟏 + 𝐅𝐤+𝟐 𝟐𝐅𝐤 + 𝐅𝐤+𝟏
𝟐𝐅𝐤 𝟐𝐅𝐤+𝟏 𝟐𝐅𝐤 𝟐𝐅𝐤+𝟏 𝐯𝐢𝒂 (∗)
= + = + > 𝟏
𝐅𝐤+𝟏 + 𝐅𝐤 + 𝐅𝐤+𝟏 𝟐𝐅𝐤 + 𝐅𝐤+𝟏 𝐅𝐤 + 𝟐𝐅𝐤+𝟏 𝟐𝐅𝐤 + 𝐅𝐤+𝟏
𝐧
𝟐𝐅𝐤 𝟐𝐅𝐤+𝟏
⇒ ∑( + )>𝐧
𝐅𝐤+𝟑 𝟐𝐅𝐤 + 𝐅𝐤+𝟏
𝐤=𝟏
𝟐𝐋𝐤 𝟐𝐋𝐤+𝟏 𝟐𝐋𝐤 𝟐𝐋𝐤+𝟏
𝐀𝐥𝐬𝐨, + = +
𝐋𝐤+𝟑 𝟐𝐋𝐤 + 𝐋𝐤+𝟏 𝐋𝐤+𝟏 + 𝐋𝐤+𝟐 𝟐𝐋𝐤 + 𝐋𝐤+𝟏
𝟐𝐋𝐤 𝟐𝐋𝐤+𝟏 𝟐𝐋𝐤 𝟐𝐋𝐤+𝟏 𝐯𝐢𝒂 (∗)
= + = + > 𝟏
𝐋𝐤+𝟏 + 𝐋𝐤 + 𝐋𝐤+𝟏 𝟐𝐋𝐤 + 𝐋𝐤+𝟏 𝐋𝐤 + 𝟐𝐋𝐤+𝟏 𝟐𝐋𝐤 + 𝐋𝐤+𝟏
𝐧
𝟐𝐋𝐤 𝟐𝐋𝐤+𝟏
⇒ ∑( + ) > 𝐧 (𝐐𝐄𝐃)
𝐋𝐤+𝟑 𝟐𝐋𝐤 + 𝐋𝐤+𝟏
𝐤=𝟏
637. 𝐏𝐫𝐨𝐯𝐞 ∶
𝟐(𝐋𝐧+𝟐 − 𝟑)𝟐 + 𝟒𝐧 − 𝟓𝐧𝐅𝟐𝐧+𝟏 ≤ 𝟎 𝒂𝐧𝐝 𝟐(𝐅𝐧+𝟐 − 𝟏)𝟐 − 𝐧𝐅𝟐𝐧+𝟏 ≤ 𝟎
Proposed by D.M. Bătinețu-Giurgiu, Neculai Stanciu-Romania
Solution by Soumava Chakraborty-Kolkata-India
𝐋𝐞𝐭 𝐅𝐧 = 𝒙 𝒂𝐧𝐝 𝐋𝐧 = 𝐲 𝒂𝐧𝐝 𝐯𝐢𝒂 𝒂𝐝𝐝𝐢𝐭𝐢𝐨𝐧 𝐟𝐨𝐫𝐦𝐮𝒍𝒂𝐞,
(∗) 𝟏 (∗∗) 𝟏
𝐅𝐦+𝐧 = (𝐅𝐦 𝐋𝐧 + 𝐋𝐦 𝐅𝐧 ) 𝒂𝐧𝐝 𝐋𝐦+𝐧 = (𝟓𝐅𝐦 𝐅𝐧 + 𝐋𝐦 𝐋𝐧 )
𝟐 𝟐
𝟏 𝟏 (⦁) 𝒙 + 𝐲
𝐏𝐮𝐭𝐭𝐢𝐧𝐠 𝐦 = 𝟏 𝐢𝐧 (∗), 𝐅𝐧+𝟏 = (𝐅𝟏 𝐋𝐧 + 𝐋𝟏 𝐅𝐧 ) = (𝒙 + 𝐲) ⇒ 𝐅𝐧+𝟏 =
𝟐 𝟐 𝟐
𝟏 𝟏
𝒂𝐧𝐝 𝐩𝐮𝐭𝐭𝐢𝐧𝐠 𝐦 = 𝟏 𝐢𝐧 (∗∗), 𝐋𝐧+𝟏 = (𝟓𝐅𝟏 𝐅𝐧 + 𝐋𝟏 𝐋𝐧 ) = (𝟓𝒙 + 𝐲)
𝟐 𝟐
(⦁⦁) 𝟓𝒙 + 𝐲
⇒ 𝐋𝐧+𝟏 =
𝟐
𝐯𝐢𝒂 (∗) 𝟏 𝐯𝐢𝒂 (⦁),(⦁⦁) 𝟏 𝒙 + 𝐲 𝟓𝒙 + 𝐲
𝐀𝐠𝒂𝐢𝐧, 𝐅𝟐𝐧+𝟏 = (𝐅𝐧+𝟏 𝐋𝐧 + 𝐋𝐧+𝟏 𝐅𝐧 ) = ( .𝐲+ . 𝒙)
𝟐 𝟐 𝟐 𝟐
(⦁⦁⦁) 𝟓𝒙𝟐 + 𝟐𝒙𝐲 + 𝐲 𝟐
⇒ 𝐅𝟐𝐧+𝟏 =
𝟒
𝐧 𝟐

𝐍𝐨𝐰, 𝟐(𝐋𝐧+𝟐 − 𝟑 )𝟐 + 𝟒𝐧 − 𝟓𝐧𝐅𝟐𝐧+𝟏 = 𝟐 (𝟏 + ∑ 𝐋𝐤 − 𝟑) + 𝟒𝐧 − 𝟓𝐧𝐅𝟐𝐧+𝟏


𝐤=𝟎

38 RMM-ABSTRACT ALGEBRA MARATHON 601-700


www.ssmrmh.ro
𝐧 𝟐 𝐧 𝟐

= 𝟐 (−𝟐 + 𝐋𝟎 + ∑ 𝐋𝐤 ) + 𝟒𝐧 − 𝟓𝐧𝐅𝟐𝐧+𝟏 = 𝟐 (∑ 𝐋𝐤 ) + 𝟒𝐧 − 𝟓𝐧𝐅𝟐𝐧+𝟏


𝐤=𝟏 𝐤=𝟏
𝐧 𝐧
𝐂𝐁𝐒
≤ 𝟐𝐧 ∑ 𝐋𝟐𝐤 + 𝟒𝐧 − 𝟓𝐧𝐅𝟐𝐧+𝟏 = 𝟐𝐧 (∑ 𝐋𝟐𝐤 − 𝟒) + 𝟒𝐧 − 𝟓𝐧𝐅𝟐𝐧+𝟏
𝐤=𝟏 𝐤=𝟎
= 𝟐𝐧(𝐋𝐧 𝐋𝐧+𝟏 + 𝟐 − 𝟒) + 𝟒𝐧 − 𝟓𝐧𝐅𝟐𝐧+𝟏 = 𝟐𝐧𝐋𝐧 𝐋𝐧+𝟏 − 𝟓𝐧𝐅𝟐𝐧+𝟏
𝐯𝐢𝒂 (⦁⦁),(⦁⦁⦁) 𝟓𝒙 + 𝐲 𝟓𝒙𝟐 + 𝟐𝒙𝐲 + 𝐲 𝟐
= 𝐧 (𝟐𝐲. − 𝟓. )
𝟐 𝟒
𝐧 𝐧
= − (𝟐𝟓𝒙𝟐 + 𝟏𝟎𝒙𝐲 + 𝟓𝐲 𝟐 − 𝟒𝐲(𝟓𝒙 + 𝐲)) = − (𝟐𝟓𝒙𝟐 − 𝟏𝟎𝒙𝐲 + 𝐲 𝟐 )
𝟒 𝟒
𝐧 𝟐 𝟐
= − (𝟓𝒙 − 𝐲) ≤ 𝟎 ∴ 𝟐(𝐋𝐧+𝟐 − 𝟑) + 𝟒𝐧 − 𝟓𝐧𝐅𝟐𝐧+𝟏 ≤ 𝟎
𝟒
𝐧 𝟐 𝐧
𝐂𝐁𝐒
𝐀𝐥𝐬𝐨, 𝟐(𝐅𝐧+𝟐 − 𝟏)𝟐 − 𝐧𝐅𝟐𝐧+𝟏 = 𝟐 (∑ 𝐅𝐤 ) − 𝐧𝐅𝟐𝐧+𝟏 ≤ 𝟐𝐧 ∑ 𝐅𝐤𝟐 − 𝐧𝐅𝟐𝐧+𝟏
𝐤=𝟏 𝐤=𝟏
𝒙+𝐲 𝟓𝒙𝟐 + 𝟐𝒙𝐲 + 𝐲 𝟐
𝐯𝐢𝒂 (⦁),(⦁⦁⦁)
= 𝟐𝐧𝐅𝐧 𝐅𝐧+𝟏 − 𝐧𝐅𝟐𝐧+𝟏 = 𝟐𝐧𝒙. − 𝐧.
𝟐 𝟒
𝐧 𝐧
= − (𝟓𝒙 + 𝟐𝒙𝐲 + 𝐲 − 𝟒𝒙(𝒙 + 𝐲)) = − (𝒙 − 𝐲)𝟐 ≤ 𝟎
𝟐 𝟐
𝟒 𝟒
𝟐
∴ 𝟐 𝐅𝐧+𝟐 − 𝟏 − 𝐧𝐅𝟐𝐧+𝟏 ≤ 𝟎 𝐐𝐄𝐃)
( ) (

𝐦 𝐦 𝟏
638. 𝐏𝐫𝐨𝐯𝐞 𝐭𝐡𝒂𝐭 ∶ ( ) . ∑𝐧+𝟏
𝐤=𝟏 ( (𝟏 + 𝐅𝟐𝐤−𝟏 )𝐦+𝟏 ) ≥ 𝐅𝟐𝐧+𝟐 ,
𝐦+𝟏 𝐦+𝟏

𝐰𝐡𝐞𝐫𝐞 𝐦 ∈ [𝟏, ∞) 𝒂𝐧𝐝 𝐅𝐤 𝐢𝐬 𝐤 − 𝐭𝐡 𝐅𝐢𝐛𝐨𝐧𝒂𝐜𝐜𝐢 𝐧𝐮𝐦𝐛𝐞𝐫


Proposed by D.M. Bătinețu-Giurgiu, Neculai Stanciu-Romania
Solution by Soumava Chakraborty-Kolkata-India
𝐧 𝐧
(∗)
∑ 𝐅𝟐𝐤 = 𝐅𝟐𝐧+𝟏 − 𝟏 𝒂𝐧𝐝 ∵ 𝐅𝟎 = 𝟎 ∴ ∑ 𝐅𝟐𝐤 = 𝐅𝟐𝐧+𝟏 − 𝟏 𝒂𝐧𝐝 𝒂𝒍𝐬𝐨,
𝐤=𝟎 𝐤=𝟏
𝐧 𝐧

∑ 𝐅𝟐𝐤+𝟏 = 𝐅𝟐𝐧+𝟐 ⇒ 𝐅𝟏 + ∑(𝐅𝟐𝐤 + 𝐅𝟐𝐤−𝟏 ) = 𝐅𝟐𝐧+𝟐


𝐤=𝟎 𝐤=𝟏
𝐧
𝐯𝐢𝒂 (∗)
⇒ 𝟏 + (𝐅𝟐𝐧+𝟏 − 𝟏) + ∑ 𝐅𝟐𝐤−𝟏 = 𝐅𝟐𝐧+𝟐
𝐤=𝟏
𝐧+𝟏 𝐧+𝟏
𝐦 𝐦 𝟏
⇒ ∑ 𝐅𝟐𝐤−𝟏 = 𝐅𝟐𝐧+𝟐 ∴ ( ) .∑( (𝟏 + 𝐅𝟐𝐤−𝟏 )𝐦+𝟏 ) ≥ 𝐅𝟐𝐧+𝟐
𝐦+𝟏 𝐦+𝟏
𝐤=𝟏 𝐤=𝟏

39 RMM-ABSTRACT ALGEBRA MARATHON 601-700


www.ssmrmh.ro
𝐧+𝟏 𝐧+𝟏
𝐦+𝟏 𝐦
⇔ ∑ (𝟏 + 𝐅𝟐𝐤−𝟏 )𝐦+𝟏 ≥ ∑ ((𝐦 + 𝟏) ( ) 𝐅𝟐𝐤−𝟏 ) , 𝐭𝐨 𝐩𝐫𝐨𝐯𝐞 𝐰𝐡𝐢𝐜𝐡, 𝐢𝐭: 𝐬𝐮𝐟𝐟𝐢𝐜𝐞𝐬 𝐭𝐨 𝐩𝐫𝐨𝐯𝐞
𝐦
𝐤=𝟏 𝐤=𝟏

𝐦+𝟏 𝐦
(𝟏 + 𝐅𝟐𝐤−𝟏 )𝐦+𝟏
≥ ( 𝐦 + 𝟏) ( ) . 𝐅𝟐𝐤−𝟏
𝐦
𝐦
𝐦+𝟏 𝐦 𝐦 + 𝐦𝒙 𝐦 (⦁) 𝒙𝐦 + 𝒙
⇔ (𝟏 + 𝒙)(𝟏 + 𝒙) ≥ (𝒙𝐦 + 𝒙) ( ) (𝒙 = 𝐅𝟐𝐤−𝟏 ) ⇔ ( ) ≥
𝐦 𝐦+𝟏 𝟏+𝒙
𝐦
𝐦 + 𝐦𝒙 𝐦 𝐦 + 𝐦𝒙 𝐁𝐞𝐫𝐧𝐨𝐮𝐥𝐥𝐢 𝐦(𝐦𝒙 − 𝟏) ? 𝒙𝐦 + 𝒙
𝐍𝐨𝐰, ( ) = (𝟏 + ( − 𝟏)) ≥ 𝟏+ ≥
𝐦+𝟏 𝐦+𝟏 𝐦+𝟏 𝟏+𝒙
𝐦(𝐦𝒙 − 𝟏) ? 𝒙𝐦 + 𝒙 − 𝟏 − 𝒙 𝐦𝒙 − 𝟏 𝐦 𝟏 ?
⇔ ≥ = ⇔ (𝐦𝒙 − 𝟏) ( − )≥𝟎
𝐦+𝟏 𝟏+𝒙 𝟏+𝒙 𝐦+𝟏 𝟏+𝒙
(𝐦𝒙 − 𝟏) 𝟐
𝐦 + 𝐦𝒙 − 𝐦 − 𝟏 ? ?
⇔ (𝐦𝒙 − 𝟏). ≥𝟎 ⇔ ≥ 𝟎 → 𝐭𝐫𝐮𝐞
(𝐦 + 𝟏)(𝟏 + 𝒙) (𝐦 + 𝟏)(𝟏 + 𝒙)
𝐧+𝟏
𝐦 𝐦 𝟏
∴( ) .∑( (𝟏 + 𝐅𝟐𝐤−𝟏 )𝐦+𝟏 ) ≥ 𝐅𝟐𝐧+𝟐 ∀ 𝐦 ∈ [𝟏, ∞) (𝐐𝐄𝐃)
𝐦+𝟏 𝐦+𝟏
𝐤=𝟏

639. 𝐈𝐟 (𝐋𝐧 )𝐧≥𝟎 𝐢𝐬 𝐋𝐮𝐜𝒂𝐬′ 𝐬𝐞𝐪𝐮𝐞𝐧𝐜𝐞, 𝐧 ∈ ℕ, 𝐭𝐡𝐞𝐧 𝐩𝐫𝐨𝐯𝐞 𝐭𝐡𝒂𝐭 ∶


𝐋𝐧 𝐋𝟐𝐧+𝟐 𝐋𝐧+𝟏 𝐋𝟐𝐧+𝟑
+ + (𝐋𝐧 + 𝐋𝐧+𝟐 )𝟐 − 𝟐√𝟔. √𝐋𝐧 𝐋𝐧+𝟏 . 𝐋𝐧+𝟐 ≥ 𝟎
𝐋𝐧+𝟑 𝐋𝐧 + 𝐋𝐧+𝟐
Proposed by D.M. Bătinețu-Giurgiu, Neculai Stanciu-Romania
Solution by Soumava Chakraborty-Kolkata-India
𝐋𝐧 𝐋𝟐𝐧+𝟐 𝐋𝐧+𝟏 𝐋𝟐𝐧+𝟑
+ + (𝐋𝐧 + 𝐋𝐧+𝟐 )𝟐 − 𝟐√𝟔. √𝐋𝐧 𝐋𝐧+𝟏 . 𝐋𝐧+𝟐
𝐋𝐧+𝟑 𝐋𝐧 + 𝐋𝐧+𝟐
𝐋𝐧 (𝐋𝐧 + 𝐋𝐧+𝟏 )𝟐 𝐋𝐧+𝟏 (𝐋𝐧+𝟏 + 𝐋𝐧+𝟐 )𝟐
= + + (𝟐𝐋𝐧 + 𝐋𝐧+𝟏 )𝟐
𝐋𝐧+𝟏 + 𝐋𝐧+𝟐 𝐋𝐧 + 𝐋𝐧 + 𝐋𝐧+𝟏
−𝟐√𝟔. √𝐋𝐧 𝐋𝐧+𝟏 . (𝐋𝐧 + 𝐋𝐧+𝟏 )
𝟐
𝐋𝐧 (𝐋𝐧 + 𝐋𝐧+𝟏 ) 𝐋𝐧+𝟏 (𝐋𝐧+𝟏 + 𝐋𝐧 + 𝐋𝐧+𝟏 )𝟐
= + + (𝟐𝐋𝐧 + 𝐋𝐧+𝟏 )𝟐
𝐋𝐧+𝟏 + 𝐋𝐧 + 𝐋𝐧+𝟏 𝟐𝐋𝐧 + 𝐋𝐧+𝟏
−𝟐√𝟔. √𝐋𝐧 𝐋𝐧+𝟏 . (𝐋𝐧 + 𝐋𝐧+𝟏 )
𝐋𝐧 (𝐋𝐧 + 𝐋𝐧+𝟏 )𝟐 𝐋𝐧+𝟏 (𝐋𝐧 + 𝟐𝐋𝐧+𝟏 )𝟐
= + + (𝟐𝐋𝐧 + 𝐋𝐧+𝟏 )𝟐
𝐋𝐧 + 𝟐𝐋𝐧+𝟏 𝟐𝐋𝐧 + 𝐋𝐧+𝟏
−𝟐√𝟔. √𝐋𝐧 𝐋𝐧+𝟏 . (𝐋𝐧 + 𝐋𝐧+𝟏 ) ≥ 𝟎
𝟐
𝒙(𝒙 + 𝐲) 𝐲(𝒙 + 𝟐𝐲)𝟐
⇔ + + (𝟐𝒙 + 𝐲)𝟐 − 𝟐√𝟔. √𝒙𝐲. (𝒙 + 𝐲) ≥ 𝟎
𝒙 + 𝟐𝐲 𝟐𝒙 + 𝐲
(𝒙 = 𝐋𝐧 , 𝐲 = 𝐋𝐧+𝟏 )

40 RMM-ABSTRACT ALGEBRA MARATHON 601-700


www.ssmrmh.ro
𝒙(𝟐𝒙 + 𝐲)(𝒙 + 𝐲)𝟐 + 𝐲(𝒙 + 𝟐𝐲)𝟑 + (𝒙 + 𝟐𝐲)(𝟐𝒙 + 𝐲)𝟑
⇔ ≥ 𝟐√𝟔. √𝒙𝐲. (𝒙 + 𝐲)
(𝒙 + 𝟐𝐲)(𝟐𝒙 + 𝐲)
𝟏𝟎𝒙𝟒 + 𝟑𝟒𝒙𝟑 𝐲 + 𝟒𝟎𝒙𝟐 𝐲 𝟐 + 𝟐𝟔𝒙𝐲 𝟑 + 𝟏𝟎𝐲 𝟒
⇔ ≥ 𝟐√𝟔. √𝒙𝐲. (𝒙 + 𝐲)
(𝒙 + 𝟐𝐲)(𝟐𝒙 + 𝐲)
𝟐(𝒙 + 𝐲)(𝟓𝒙𝟑 + 𝟏𝟐𝒙𝟐 𝐲 + 𝟖𝒙𝐲 𝟐 + 𝟓𝐲 𝟑 )
⇔ ≥ 𝟐√𝟔. √𝒙𝐲. (𝒙 + 𝐲)
(𝒙 + 𝟐𝐲)(𝟐𝒙 + 𝐲)
⇔ (𝟓𝒙𝟑 + 𝟏𝟐𝒙𝟐 𝐲 + 𝟖𝒙𝐲 𝟐 + 𝟓𝐲 𝟑 )𝟐 ≥ 𝟔𝒙𝐲(𝒙 + 𝟐𝐲)𝟐 (𝟐𝒙 + 𝐲)𝟐
(∵ 𝒙 = 𝐋𝐧 , 𝐲 = 𝐋𝐧+𝟏 > 0 ⇒ 𝑥 + 𝐲 > 0 ∀ 𝑛 ∈ ℕ)
𝐋𝐧
⇔ 𝟐𝟓𝐭 𝟔 + 𝟗𝟔𝐭 𝟓 + 𝟏𝟎𝟒𝐭 𝟒 + 𝟒𝟒𝐭 𝟑 + 𝟔𝟒𝐭 𝟐 + 𝟓𝟔𝐭 + 𝟐𝟓 ≥ 𝟎 (𝐭 = )
𝐋𝐧+𝟏
𝐋𝐧
→ 𝐭𝐫𝐮𝐞 ∵ 𝐭 = > 0∀𝐧 ∈ ℕ
𝐋𝐧+𝟏
𝐋𝐧 𝐋𝟐𝐧+𝟐 𝐋𝐧+𝟏 𝐋𝟐𝐧+𝟑
∴ + + (𝐋𝐧 + 𝐋𝐧+𝟐 )𝟐 − 𝟐√𝟔. √𝐋𝐧 𝐋𝐧+𝟏 . 𝐋𝐧+𝟐 ≥ 𝟎 ∀ 𝐧 ∈ ℕ (𝐐𝐄𝐃)
𝐋𝐧+𝟑 𝐋𝐧 + 𝐋𝐧+𝟐

640. If 𝒙𝟏 , 𝒙𝟐 , … , 𝒙𝟒𝒌−𝟑 , 𝒌 ∈ ℕ∗ are in A.P. and 𝒎 = 𝒙𝟏 + 𝒙𝟓 + ⋯ + 𝒙𝟒𝒌−𝟑


then find
𝛀(𝒌, 𝒎) = 𝒔𝒖𝒑(𝒙𝟏 ∙ 𝒙𝟐 ∙ … ∙ 𝒙𝟒𝒌−𝟑 )
Proposed by SK Sabiruddin-India
Solution by Daniel Sitaru-Romania

𝒌 𝒌 𝒌

𝒎 = ∑ 𝒙𝟒𝒊−𝟑 = ∑(𝒙𝟏 + (𝟒𝒊 − 𝟒)𝒓) = 𝒌𝒙𝟏 + 𝟒𝒓 ∑(𝒊 − 𝟏) =


𝒊=𝟏 𝒊=𝟏 𝒊=𝟏

𝒌 ( 𝒌 − 𝟏)
= 𝒌𝒙𝟏 + 𝟒𝒓 ∙ = 𝒌𝒙𝟏 + 𝟐𝒓𝒌(𝒌 − 𝟏) = 𝒌(𝒙𝟏 + 𝟐𝒓(𝒌 − 𝟏))
𝟐
𝒎
𝒙𝟏 + 𝟐𝒓(𝒌 − 𝟏) =
𝒌
𝑨𝑴−𝑮𝑴
𝒙𝟏 + 𝒙𝟐 + ⋯ + 𝒙𝟒𝒌−𝟑 𝟒𝒌−𝟑
𝒙𝟏 ∙ 𝒙𝟐 ∙ … ∙ 𝒙𝟒𝒌−𝟑 ⏞
< ( ) =
𝟒𝒌 − 𝟑

(𝒙𝟏 + 𝒙𝟒𝒌−𝟑 )(𝟒𝒌 − 𝟑) 𝟒𝒌−𝟑 𝟒𝒌−𝟑


𝟐 𝒙𝟏 + 𝒙𝟏 + 𝒓(𝟒𝒌 − 𝟒)
=( ) =( ) =
𝟒𝒌 − 𝟑 𝟐
𝟒𝒌−𝟑 𝒎 𝟒𝒌−𝟑
= (𝒙𝟏 + 𝟐𝒓(𝒌 − 𝟏)) =( )
𝒌

41 RMM-ABSTRACT ALGEBRA MARATHON 601-700


www.ssmrmh.ro
𝒎 𝟒𝒌−𝟑
( ) ( )
𝛀 𝒌, 𝒎 = 𝒔𝒖𝒑 𝒙𝟏 ∙ 𝒙𝟐 ∙ … ∙ 𝒙𝟒𝒌−𝟑 = ( )
𝒌
641. Solve for real numbers:
𝒛𝟐 − 𝟏𝟐𝒕 = 𝟏𝟓
{𝒛𝟐 𝟐𝒛 𝒛𝟑 𝒛𝟐 𝒕 ∴ 𝒛, 𝒕 ∈ ℝ
+ =√ + −
𝟖𝒕 𝟑 𝟑𝒕 𝟒 𝟐
Proposed by Hikmat Mammadov-Azerbaijan
Solution by Pham Duc Nam-Vietnam

𝒛𝟐 − 𝟏𝟐𝒕 = 𝟏𝟓(𝟏)
𝒛𝟐 𝟐𝒛 𝒛𝟑 𝒛𝟐 𝒕 ∴ 𝒛, 𝒕 ∈ ℝ
+ = √ + − (𝟐)
{𝟖𝒕 𝟑 𝟑𝒕 𝟒 𝟐

𝒛𝟑 𝒛𝟒
∗ 𝑫𝒐𝒎𝒂𝒊𝒏: + ≥ 𝟎, 𝒕 ≠ 𝟎(∗)
𝟑𝒕 𝟒

𝒛𝟐 𝟐𝒛 𝒕 𝒛𝟑 𝒛𝟐
𝑭𝒓𝒐𝒎(𝟐) ⇔ + + =√ +
𝟖𝒕 𝟑 𝟐 𝟑𝒕 𝟒

𝟐 𝒛𝟑 𝒛𝟐 𝟐
⇔ (𝟑𝒛𝟐 + 𝟏𝟔𝒛𝒕 + 𝟏𝟐𝒕𝟐 ) = 𝟓𝟕𝟔𝒕𝟐 ( + ) ⇔ ((𝟑𝒛 + 𝟐𝒕)(𝟔𝒕 − 𝒛) + 𝟔𝒛𝟐 )
𝟑𝒕 𝟒
= 𝟏𝟗𝟐𝒛𝟑 𝒕 + 𝟏𝟒𝟒𝒛𝟐 𝒕𝟐
⇔ [(𝟑𝒛 + 𝟐𝒕)(𝟔𝒕 − 𝒛)]𝟐 + 𝟏𝟐𝒛𝟐(𝟑𝒛 + 𝟐𝒕)(𝟔𝒕 − 𝒛) + 𝟑𝟔𝒛𝟒 − 𝟏𝟗𝟐𝒛𝟑 𝒕 − 𝟏𝟒𝟒𝒛𝟐 𝒕𝟐 = 𝟎
⇔ [(𝟑𝒛 + 𝟐𝒕)(𝟔𝒕 − 𝒛)]𝟐 + 𝟏𝟐𝒛𝟐 [(𝟑𝒛 + 𝟐𝒕)(𝟔𝒕 − 𝒛) + 𝟑𝒛𝟐 − 𝟏𝟔𝒛𝒕 − 𝟏𝟐𝒕𝟐 ] = 𝟎
⇔ [(𝟑𝒛 + 𝟐𝒕)(𝟔𝒕 − 𝒛)]𝟐 + 𝟏𝟐𝒛𝟐 (−𝟑𝒛𝟐 + 𝟏𝟔𝒛𝒕 + 𝟏𝟐𝒕𝟐 + 𝟑𝒛𝟐 − 𝟏𝟔𝒛𝒕 − 𝟏𝟐𝒕𝟐 ) = 𝟎
6𝒕 − 𝒛 = 𝟎
⇔ (𝟑𝒛 + 𝟐𝒕)(𝟔𝒕 − 𝒛) = 𝟎 ⇔ [
3𝒛 + 𝟐𝒕 = 𝟎
∗ 𝟔𝒕 − 𝒛 = 𝟎 ⇔ 𝒛 = 𝟔𝒕 ⇒ (𝟏) ⇔ 𝟑𝟔𝒕𝟐 − 𝟏𝟐𝒕 − 𝟏𝟓 = 𝟎
𝟏
𝑡 = − ⇒ 𝒛 = −𝟑(𝑵𝒐𝒕 𝒔𝒂𝒕𝒊𝒔𝒇𝒊𝒆𝒅 𝒃𝒚 𝒓𝒆𝒄𝒉𝒆𝒄𝒌𝒊𝒏𝒈)
⇔[ 𝟐
𝟓
𝑡 = ⇒ 𝒛 = 𝟓(𝑺𝒂𝒕𝒊𝒔𝒇𝒊𝒆𝒅)
𝟔

42 RMM-ABSTRACT ALGEBRA MARATHON 601-700


www.ssmrmh.ro
𝟐𝒕 𝟒
∗ 𝟑𝒛 + 𝟐𝒕 = 𝟎 ⇔ 𝒛 = − (𝟏) ⇔ 𝒕𝟐 − 𝟏𝟐𝒕 − 𝟏𝟓 = 𝟎
𝟑 𝟗
𝟐𝟕 − 𝟏𝟐√𝟔
𝑡= ⇒ 𝒛 = 𝟒√𝟔 − 𝟗(𝑵𝒐𝒕 𝒔𝒂𝒕𝒊𝒔𝒇𝒊𝒆𝒅 𝒃𝒚 𝒓𝒆𝒄𝒉𝒆𝒄𝒌𝒊𝒏𝒈)
⇔ 𝟐
𝟐𝟕 + 𝟏𝟐√𝟔 𝟑
[𝑡 = = (𝟗 + 𝟒√𝟔) ⇒ 𝒛 = −𝟗 − 𝟒√𝟔(𝑺𝒂𝒕𝒊𝒔𝒇𝒊𝒆𝒅)
𝟐 𝟐
𝟓 𝟑
⇒ (𝒛, 𝒕) = (𝟓, ) , (−𝟗 − 𝟒√𝟔, (𝟗 + 𝟒√𝟔)) 𝒂𝒓𝒆 𝒔𝒐𝒍𝒖𝒕𝒊𝒐𝒏𝒔
𝟔 𝟐

642. Solve for natural numbers:


𝒙 + 𝒚 + 𝒛 + 𝒕 = 𝟏𝟒𝟒𝟔
{
𝒙! ∙ 𝒚! ∙ 𝒛! = 𝒕!
Proposed by Daniel Sitaru-Romania
Solution by Dragan Santrac-Sarajevo-Bosnia and Herzegovina
𝑳𝒆𝒕 𝒙 ≤ 𝒚 ≤ 𝒛 ≤ 𝒕 𝒂𝒏𝒅 𝒙 = 𝒚, 𝒛 = 𝒕 ⇒ 𝟐𝒙 + 𝟐𝒚 = 𝟏𝟒𝟒𝟔, 𝒙 + 𝒚 = 𝟕𝟐𝟑
𝒙! ∙ 𝒙! ∙ 𝒛! = 𝒛! ⇒ (𝒙!)𝟐 = 𝟏 ⇒ 𝒙 = 𝒚 = 𝟏, 𝒕𝒉𝒆𝒏 𝒕 = 𝒛 = 𝟕𝟐𝟐.
(𝒙, 𝒚, 𝒛, 𝒕) = (𝟏, 𝟏, 𝟕𝟐𝟐, 𝟕𝟐𝟐)
𝑭𝒐𝒓 𝒙 ≠ 𝒚 ≠ 𝒛 ≠ 𝒕 𝒂𝒏𝒅 𝒙 = 𝟏. 𝑵𝒐𝒕𝒊𝒄𝒆 𝟔! = 𝟕𝟐𝟎, 𝒍𝒆𝒕 𝒚 = 𝟔.
𝟏! ∙ 𝟔! ∙ 𝒛! = 𝒕!
𝟕𝟐𝟎 ∙ 𝟕𝟏𝟗! = 𝟕𝟐𝟎! ⇒ 𝒛 = 𝟕𝟏𝟗, 𝒕 = 𝟕𝟐𝟎
𝟏 + 𝟔 + 𝟕𝟏𝟗 + 𝟕𝟐𝟎 = 𝟏𝟒𝟒𝟔
𝑪𝒉𝒆𝒄𝒌: {
𝟏! ∙ 𝟔! ∙ 𝟕𝟏𝟗! = 𝟕𝟐𝟎!
(𝒙, 𝒚, 𝒛, 𝒕) = (𝟏, 𝟔, 𝟕𝟏𝟗, 𝟕𝟐𝟎)
𝑨𝒍𝒍 𝒔𝒐𝒍𝒖𝒕𝒊𝒐𝒏𝒔: (𝒙, 𝒚, 𝒛, 𝒕) ∈ {(𝟏, 𝟏, 𝟕𝟐𝟐, 𝟕𝟐𝟎), (𝟏, 𝟔, 𝟕𝟏𝟗, 𝟕𝟐𝟎)}.
643. 𝐒𝐨𝐥𝐯𝐞 𝐟𝐨𝐫 𝒙, 𝒚 ∈ ℝ
(𝒚𝟐 + 𝟑𝒚)√𝟐𝒙 + 𝟕 = (𝟑𝒙 − 𝟐)(𝒚 + 𝟏)
{
(𝟑𝒙 − 𝟐)𝟐 + 𝟐𝒙𝒚𝟐 = 𝟐𝒚𝟒 + 𝟖𝒚𝟑 + 𝟑𝒚𝟐
Proposed by Carlos Paiva-Brazil
Solution by Pham Duc Nam-Vietnam

𝟕 𝟕
∗ 𝐃𝐨𝐦𝐚𝐢𝐧: 𝒙 > − (𝒙 = − is not satisfied)
𝟐 𝟐
𝟐
∗ 𝒚 = 𝟎 ⇒ 𝒙 = , now assume: 𝒚 ≠ 𝟎
𝟑
𝟐
𝒚𝟐 + 𝟑𝒚 𝟑𝒙 − 𝟐
( )(
∗ (𝒚 + 𝟑𝒚)√𝟐𝒙 + 𝟕 = 𝟑𝒙 − 𝟐 𝒚 + 𝟏 ⇔ ) =
𝒚+𝟏 √𝟐𝒙 + 𝟕
43 RMM-ABSTRACT ALGEBRA MARATHON 601-700
www.ssmrmh.ro
∗ (𝟑𝒙 − 𝟐)𝟐 + 𝟐𝒙𝒚𝟐 = 𝟐𝒚𝟒 + 𝟖𝒚𝟑 + 𝟑𝒚𝟐 ⇔ (𝟑𝒙 − 𝟐)𝟐 + 𝟐𝒙𝒚𝟐 + 𝟕𝒚𝟐 = 𝟐𝒚𝟒 + 𝟖𝒚𝟑 + 𝟏𝟎𝒚𝟐
(𝟑𝒙 − 𝟐)𝟐 𝟐𝒚𝟒 + 𝟖𝒚𝟑 + 𝟏𝟎𝒚𝟐
⇔ (𝟑𝒙 − 𝟐)𝟐 + 𝒚𝟐 (𝟐𝒙 + 𝟕) = 𝟐𝒚𝟒 + 𝟖𝒚𝟑 + 𝟏𝟎𝒚𝟐 ⇔ + 𝒚𝟐 =
𝟐𝒙 + 𝟕 𝟐𝒙 + 𝟕
𝟐 𝟐 𝟒 𝟑 𝟐 𝟐 𝟐 𝟐
𝒚 + 𝟑𝒚 𝟐𝒚 + 𝟖𝒚 + 𝟏𝟎𝒚 𝒚 + 𝟑𝒚 𝒚(𝒚 + 𝟑𝒚)
⇔( ) + 𝒚𝟐 = ⇔( − 𝒚) + 𝟐.
𝒚+𝟏 𝟐𝒙 + 𝟕 𝒚+𝟏 𝒚+𝟏
𝟐𝒚𝟒 + 𝟖𝒚𝟑 + 𝟏𝟎𝒚𝟐
=
𝟐𝒙 + 𝟕
𝟐 𝟐
𝟒𝒚 𝒚(𝒚 + 𝟑𝒚) 𝟐𝒚𝟒 + 𝟖𝒚𝟑 + 𝟏𝟎𝒚𝟐 𝟒 𝟐(𝒚 + 𝟑) 𝟐𝒚𝟐 + 𝟖𝒚 + 𝟏𝟎
⇔ + 𝟐. = ⇔ + =
(𝒚 + 𝟏)𝟐 𝒚+𝟏 𝟐𝒙 + 𝟕 (𝒚 + 𝟏)𝟐 (𝒚 + 𝟏) 𝟐𝒙 + 𝟕
𝟐𝒙 + 𝟕 − (𝒚 + 𝟏)𝟐
⇔ =𝟎
(𝟐𝒙 + 𝟕)(𝒚 + 𝟏)𝟐
𝟕
∗ 𝒙 > − , 𝒚 = −𝟏 is not satisfied ⇒ 𝟐𝒙 + 𝟕 − (𝒚 + 𝟏)𝟐 = 𝟎 ⇒ 𝟐𝒙 + 𝟕 = (𝒚 + 𝟏)𝟐
𝟐
𝟑𝒚𝟐 + 𝟔𝒚 − 𝟐𝟐
⇒ (𝒚𝟐 + 𝟑𝒚)√𝟐𝒙 + 𝟕 = (𝟑𝒙 − 𝟐)(𝒚 + 𝟏) ⇔ (𝒚𝟐 + 𝟑𝒚)|𝒚 + 𝟏| = ( ) (𝒚 + 𝟏)
𝟐
𝟑𝒚𝟐 + 𝟔𝒚 − 𝟐𝟐
⊕ 𝐂𝐚𝐬𝐞 𝟏: 𝒚 + 𝟏 > 0 ⇒ 𝒚 > −𝟏 ⇔ 𝒚𝟐 + 𝟑𝒚 =
𝟐
𝑦 = √𝟐𝟐 ⇒ 𝒙 = 𝟖 + √𝟐𝟐
⇔[
𝑦 = −√𝟐𝟐(𝐫𝐞𝐣𝐞𝐜𝐭 𝐛𝐲 𝐜𝐨𝐧𝐝𝐢𝐭𝐢𝐨𝐧 𝒚 > −𝟏)
𝟑𝒚𝟐 + 𝟔𝒚 − 𝟐𝟐
⊕ 𝐂𝐚𝐬𝐞 𝟐: 𝒚 + 𝟏 < 0 ⇒ 𝒚 < −𝟏 ⇔ 𝒚𝟐 + 𝟑𝒚 = −
𝟐
−√𝟏𝟒𝟔 − 𝟔 √𝟏𝟒𝟔 − 𝟏𝟒
𝑦= ⇒𝒙=
⇔ 𝟓 𝟐𝟓
√𝟏𝟒𝟔 − 𝟔
(𝐫𝐞𝐣𝐞𝐜𝐭 𝐛𝐲 𝐜𝐨𝐧𝐝𝐢𝐭𝐢𝐨𝐧 𝒚 < −𝟏)
[𝑦 = 𝟓
𝟐 √𝟏𝟒𝟔 − 𝟏𝟒 −√𝟏𝟒𝟔 − 𝟔
⇒ (𝒙, 𝒚) = ( , 𝟎) , (𝟖 + √𝟐𝟐, √𝟐𝟐), ( , )
𝟑 𝟐𝟓 𝟓
644. Solve for real numbers:
𝑥, 𝑦, 𝑧 > 0
{49(𝒙𝒚 + 𝒚𝒛 + 𝒙𝒛) = 𝟔𝟖𝟔 + 𝟗𝒙𝒚𝒛 , (𝒙, 𝒚, 𝒛 ∈ ℝ)
𝑥+𝑦+𝑧=7
Proposed by Daniel Sitaru-Romania
Solution by Pham Duc Nam-Vietnam
Use: 𝒑, 𝒒, 𝒓 transformation
𝒑 = 𝒙+𝒚+𝒛 = 𝟕
𝒒 = 𝒙𝒚 + 𝒚𝒛 + 𝒙𝒛
𝒓 = 𝒙𝒚𝒛

44 RMM-ABSTRACT ALGEBRA MARATHON 601-700


www.ssmrmh.ro
𝑥, 𝑦, 𝑧 > 0
⇔ {49𝒒 = 𝟔𝟖𝟔 + 𝟗𝒓
𝑝=7
We have: 𝟐𝒑𝟑 + 𝟗𝒓 ≥ 𝟕𝒑𝒒 ⇔ 𝟐. 𝟕𝟑 + 𝟗𝒓 ≥ 𝟕. 𝟕. 𝒒 ⇔ 𝟔𝟖𝟔 + 𝟗𝒙𝒚𝒛 ≥ 𝟒𝟗(𝒙𝒚 + 𝒚𝒛 + 𝒙𝒛)
⇔ 𝟒𝟗(𝒙𝒚 + 𝒚𝒛 + 𝒙𝒛) ≤ 𝟔𝟖𝟔 + 𝟗𝒙𝒚𝒛 ⇒ 𝟒𝟗(𝒙𝒚 + 𝒚𝒛 + 𝒙𝒛) = 𝟔𝟖𝟔 + 𝟗𝒙𝒚𝒛 ⇔ 𝒙 = 𝒚 = 𝒛
𝟕
⇒𝒙=𝒚=𝒛=
𝟑
𝑵𝑶𝑻𝑬:
Prove: 𝟐𝒑𝟑 + 𝟗𝒓 ≥ 𝟕𝒑𝒒
∗ 𝒑𝒒 ≥ 𝟗𝒓 ⇔ (𝒙 + 𝒚 + 𝒛)(𝒙𝒚 + 𝒚𝒛 + 𝒙𝒛) ≥ 𝟗𝒙𝒚𝒛
𝟑
𝑨𝑴 − 𝑮𝑴: (𝒙 + 𝒚 + 𝒛)(𝒙𝒚 + 𝒚𝒛 + 𝒙𝒛) ≥ 𝟑 𝟑√𝒙𝒚𝒛. 𝟑 √(𝒙𝒚𝒛)𝟐 = 𝟗𝒙𝒚𝒛
∗ 𝒑𝟑 + 𝟗𝒓 ≥ 𝟒𝒑𝒒 ⇔ (𝒙 + 𝒚 + 𝒛)𝟑 + 𝟗𝒙𝒚𝒛 ≥ 𝟒(𝒙 + 𝒚 + 𝒛)(𝒙𝒚 + 𝒚𝒛 + 𝒙𝒛)
WLOG: assume: 𝒙 ≥ 𝒚 ≥ 𝒛 ⇒ (𝒙 + 𝒚 + 𝒛)𝟑 + 𝟗𝒙𝒚𝒛 ≥ 𝟒(𝒙 + 𝒚 + 𝒛)(𝒙𝒚 + 𝒚𝒛 + 𝒙𝒛)
⇔ 𝒙(𝒙 − 𝒚)(𝒙 − 𝒛) + 𝒚(𝒚 − 𝒛)(𝒚 − 𝒙) + 𝒛(𝒛 − 𝒙)(𝒛 − 𝒚) ≥ 𝟎, which is true by: 𝒙 ≥ 𝒚 ≥ 𝒛 > 0
𝐍𝐨𝐰: 𝟐𝒑𝟑 + 𝟗𝒓 ≥ 𝟕𝒑𝒒 ⇔ 𝟐(𝒑𝟑 − 𝟒𝒑𝒒 + 𝟗𝒓) + (𝒑𝒒 − 𝟗𝒓) ≥ 𝟎, which is true by: 𝒑𝒒
≥ 𝟗𝒓 and 𝒑𝟑 + 𝟗𝒓 ≥ 𝟒𝒑𝒒
645. Solve for integers:
𝒙𝟒 + 𝟐𝒙𝒚𝟑 + 𝟐𝒙𝒛𝟑 + 𝟐𝒚𝟑 𝒛 + 𝟐𝒚𝒛𝟑 + 𝟑 = 𝒚𝟒 + 𝒛𝟒 + 𝒙𝟐 𝒚𝟐 + 𝟐𝒙𝟐 𝒚𝒛 + 𝒙𝟐 𝒛𝟐 + 𝟑𝒚𝟐 𝒛𝟐
Proposed by Elsen Kerimov, Kenan Rustemov-Azerbaijan
Solution 1 by Pham Duc Nam-Vietnam

𝒙𝟒 + 𝟐𝒙𝒚𝟑 + 𝟐𝒙𝒛𝟑 + 𝟐𝒚𝟑 𝒛 + 𝟐𝒚𝒛𝟑 + 𝟑 = 𝒚𝟒 + 𝒛𝟒 + 𝒙𝟐 𝒚𝟐 + 𝟐𝒙𝟐 𝒚𝒛 + 𝒙𝟐 𝒛𝟐 + 𝟑𝒚𝟐 𝒛𝟐


⇔ 𝒙𝟒 + 𝟐𝒙𝒚𝟑 + 𝟐𝒙𝒛𝟑 + 𝟐𝒚𝟑 𝒛 + 𝟐𝒚𝒛𝟑 − 𝒚𝟒 − 𝒛𝟒 − 𝒙𝟐 𝒚𝟐 − 𝟐𝒙𝟐 𝒚𝒛 − 𝒙𝟐 𝒛𝟐 − 𝟑𝒚𝟐 𝒛𝟐 = −𝟑
⇔ 𝒙𝟒 − (𝒙𝟐 𝒚𝟐 + 𝟐𝒙𝟐 𝒚𝒛 + 𝒙𝟐 𝒛𝟐 ) − (𝒚𝟒 + 𝒚𝟐 𝒛𝟐 + 𝒛𝟒 − 𝟐𝒚𝟐 𝒚𝒛 + 𝟐𝒚𝟐 𝒛𝟐 − 𝟐𝒚𝒛𝒛𝟐 )
+ 𝒚𝟐 (𝒙𝟐 + 𝒙𝒚 + 𝒙𝒛)
−𝒚𝒛(𝒙𝟐 + 𝒙𝒚 + 𝒙𝒛) + 𝒛𝟐 (𝒙𝟐 + 𝒙𝒚 + 𝒙𝒛) − 𝒚𝟐 (𝒙𝟐 − 𝒙𝒚 − 𝒙𝒛) + 𝒚𝒛(𝒙𝟐 − 𝒙𝒚 − 𝒙𝒛)
− 𝒛𝟐 (𝒙𝟐 − 𝒙𝒚 − 𝒙𝒛) = −𝟑
𝟐
⇔ (𝒙𝟐 + 𝒙𝒚 + 𝒙𝒛)(𝒙𝟐 − 𝒙𝒚 − 𝒙𝒛) − (𝒚𝟐 − 𝒚𝒛 + 𝒛𝟐 ) − (𝒙𝟐 − 𝒙𝒚 − 𝒙𝒛)(𝒚𝟐 − 𝒚𝒛 + 𝒛𝟐 )
+ (𝒙𝟐 + 𝒙𝒚 + 𝒙𝒛)(𝒚𝟐 − 𝒚𝒛 + 𝒛𝟐 ) = −𝟑
⇔ (𝒙𝟐 + 𝒙𝒚 + 𝒙𝒛 − 𝒚𝟐 + 𝒚𝒛 − 𝒛𝟐 )(𝒙𝟐 − 𝒙𝒚 − 𝒙𝒛 + 𝒚𝟐 − 𝒚𝒛 + 𝒛𝟐 ) = −𝟑
𝒙𝟐 + 𝒙𝒚 + 𝒙𝒛 − 𝒚𝟐 + 𝒚𝒛 − 𝒛𝟐 = −𝟏
(𝟏) { 𝟐
𝒙 − 𝒙𝒚 − 𝒙𝒛 + 𝒚𝟐 − 𝒚𝒛 + 𝒛𝟐 = 𝟑
𝒙, 𝒚, 𝒛 ∈ ℤ ⇒
𝒙𝟐 + 𝒙𝒚 + 𝒙𝒛 − 𝒚𝟐 + 𝒚𝒛 − 𝒛𝟐 = 𝟑
(𝟐) { 𝟐
[ 𝒙 − 𝒙𝒚 − 𝒙𝒛 + 𝒚𝟐 − 𝒚𝒛 + 𝒛𝟐 = −𝟏
Case: −𝟑 = 𝟏(−𝟑)is not satisfied, because it leads to: 𝟐𝒙𝟐 = −𝟐 ⇒ 𝒙 ∉ ℤ

45 RMM-ABSTRACT ALGEBRA MARATHON 601-700


www.ssmrmh.ro
From: (𝟏), (𝟐) ⇒ 𝟐𝒙𝟐 = 𝟐 ⇒ 𝒙 = ±𝟏
−𝒚𝟐 − 𝒛𝟐 + 𝒚𝒛 + 𝒚 + 𝒛 = −𝟐
∗ 𝒙 = 𝟏, (𝟏) ⇔ { 𝟐 ⇒ 𝒚𝟐 − 𝒚(𝒛 + 𝟏) + 𝒛𝟐 − 𝒛 − 𝟐 = 𝟎, 𝜟
𝒚 + 𝒛𝟐 − 𝒚𝒛 − 𝒚 − 𝒛 = 𝟐
= (𝒛 + 𝟏)𝟐 − 𝟒(𝒛𝟐 − 𝒛 − 𝟐) = 𝟏𝟐 − 𝟑(𝒛 − 𝟏)𝟐
12 − 𝟑(𝒛 − 𝟏)𝟐 ≥ 𝟎 −𝟏 ≤ 𝒛 ≤ 𝟑 ⇒ 𝒛 = −𝟏, 𝟎, 𝟏, 𝟐, 𝟑
𝑾𝒂𝒏𝒕: 𝒚 ∈ ℤ ⇒ { ⇔{
𝟐
12 − 𝟑(𝒛 − 𝟏) = 𝒌 𝟐 12 − 𝟑(𝒛 − 𝟏)𝟐 = 𝒌𝟐
𝑧 = 0 ⇒ 𝒚 = −𝟏, 𝒚 = 𝟐
𝑧 = −𝟏 ⇒ 𝒚 = 𝟎
⇒[
𝑧 = 2 ⇒ 𝒚 = 𝟑, 𝒚 = 𝟎
𝑧=3⇒𝒚=𝟐
(𝟐)𝒊. 𝒆(𝟏) ⇒ (𝒙, 𝒚, 𝒛) = (𝟏, −𝟏, 𝟎), (𝟏, 𝟐, 𝟎), (𝟏, 𝟎, −𝟏), (𝟏, 𝟎, 𝟐), (𝟏, 𝟑, 𝟐), (𝟏, 𝟐, 𝟑)
−𝒚𝟐 − 𝒛𝟐 + 𝒚𝒛 − 𝒚 − 𝒛 = 𝟐
∗ 𝒙 = −𝟏, (𝟏) ⇔ { 𝟐 ⇒ 𝒚𝟐 + 𝒚(𝟏 − 𝒛) + 𝒛𝟐 + 𝒛 − 𝟐 = 𝟎, 𝜟
𝒚 + 𝒛𝟐 − 𝒚𝒛 + 𝒚 + 𝒛 = 𝟐
= (𝒛 − 𝟏)𝟐 − 𝟒(𝒛𝟐 + 𝒛 − 𝟐) = 𝟏𝟐 − 𝟑(𝒛 + 𝟏)𝟐
12 − 𝟑(𝒛 + 𝟏)𝟐 ≥ 𝟎 −𝟑 ≤ 𝒛 ≤ 𝟏 ⇒ 𝒛 = −𝟑, −𝟐, −𝟏, 𝟎, 𝟏
𝑾𝒂𝒏𝒕: 𝒚 ∈ ℤ ⇒ { 𝟐
{
𝟐 ⇔ 12 − 𝟑(𝒛 + 𝟏)𝟐 = 𝒌𝟐
( )
12 − 𝟑 𝒛 + 𝟏 = 𝒌
𝑧 = −𝟑 ⇒ 𝒚 = −𝟐
𝑧 = −𝟐 ⇒ 𝒚 = −𝟑, 𝒚 = 𝟎
⇒[
𝑧 = 0 ⇒ 𝒚 = −𝟐, 𝒚 = 𝟏
𝑧=1⇒𝒚=𝟎
(𝟐)𝒊. 𝒆(𝟏) ⇒ (𝒙, 𝒚, 𝒛) = (−𝟏, −𝟐, −𝟑), (−𝟏, −𝟑, −𝟐), (−𝟏, 𝟎, −𝟐), (−𝟏, −𝟐, 𝟎), (−𝟏, 𝟏, 𝟎), (−𝟏, 𝟎, 𝟏)
There are 12 solutions satisfied
Solution 2 by George Florin Șerban-Romania
x 4  2 x( y 3  z 3 )  2 yz ( y 2  z 2 )  3  x 2 ( y  z )2  ( y 2  z 2 )2  y 2 z 2 ,
x 2 ( y  z )2  ( y 2  z 2 )2  y 2 z 2  x 4  2 x( y  z )( y 2  yz  z 2 )  2 yz ( y 2  z 2 )  3,
x 2 ( y  z )2  ( y 2  yz  z 2 )2  x 4  2 x( y  z )( y 2  yz  z 2 )  3,
[ x( y  z )  ( y 2  yz  z 2 )]2  ( x 2 )2  3,
( xy  xz  yz  y 2  z 2  x 2 )( xy  xz  yz  y 2  z 2  x 2 )  3. Case 1.

 xy  xz  yz  y  z  x  1
2 2 2

 then 2 x 2  2, 2 x 2  0, 2  0, false. Case 2.


 xy  xz  yz  y  z  x  3

2 2 2


 xy  xz  yz  y  z  x  1
2 2 2

 then 2 x 2  2, x 2  1, x {1,1}. If x  1
 xy  xz  yz  y  z  x  3

2 2 2

 y  z  yz  y  z  1  1 
 2 2
 y  z  yz  y  z  1  1 2
2 2

 ,  , y  z 2  y  z  yz  2,
 y  z  yz  y  z  1  3 
  y  z  yz  y  z  1  3
2 2 2 2

y 2  y(1  z )  z 2  z  2  0, y 2  y(z 1)  ( z  1)( z  2)  0,


  ( z  1)2  4( z  1)( z  2)  ( z  1)( z  1  4 z  8)  ( z  1)(3z  9)  3(z 1)(z 3)  0,
4
z {3, 2, 1, 0,1},  perfect square.If z  3, y   2, solution (1, 2, 3). If z  2,
2
46 RMM-ABSTRACT ALGEBRA MARATHON 601-700
www.ssmrmh.ro
3  3 3  3
y  0 or y   3, solutions (1, 0, 2), (1, 3, 2). If z  1,   12 is not perfect
2 2
1  3 1  3
square.If z  0, y   1 or y   2, solutions (1,1, 0), (1, 2, 0). If z  1, y  0,
2 2
solution ( 1, 0,1). If x  1, then y  z  yz  y 2  z 2  1  1, y 2  z 2  y  z  yz  2,
y 2  ( z  1) y  z 2  z  2  0, y 2  ( z  1) y  ( z  1)( z  2)  0,
  ( z  1)2  4( z  1)( z  2)  ( z  1)( z  1  4 z  8)  ( z  1)(3z  9)  3(z 1)(z 3)  0,
4 33
z {3, 2,1, 0, 1},  perfect square.If z  3, y   2, solution (1, 2,3). If z  2, y   3, or
2 2
33
y  0, solutions (1,3, 2), (1, 0, 2). If z  1,   12 is not perfect square.If
2
1 3 1 3
z  0, y   2 y   1, solutions (1, 2, 0), (1, 1, 0). If z  1, y  0, solution (1, 0, 1).
2 2

 xy  xz  yz  y  z  x  3
2 2 2

Case 3.  then 2 x 2  2, x 2  1, x {1,1}. If x  1,


 xy  xz  yz  y  z  x  1

2 2 2

y 2  z 2  y  z  yz  2, 2 y 2  2 z 2  2 y  2 z  2 yz  4, ( y  z )2  ( y  1)2  ( z  1)2  2 false


because ( y  z )2  ( y  1)2  ( z  1) 2  0, 2  0. If x  1, y 2  z 2  y  z  yz  2,
2 y 2  2 z 2  2 y  2 z  2 yz  4, ( y  z )2  ( y  1)2  ( z  1)2  2 false because

 xy  xz  yz  y  z  x  3
2 2 2

( y  z )2  ( y  1)2  ( z  1)2  0, 2  0. Case 4.  then 2 x 2  2,


 xy  xz  yz  y  z  x  1

2 2 2

2 x 2  0, 2  0, false.In conclusion
S  {(1, 2, 3), (1, 0, 2), (1, 3, 2), (1,1, 0), (1, 2, 0), (1, 0,1), (1, 2,3), (1,3, 2), (1, 0, 2), (1, 2, 0),
(1, 1, 0), (1, 0, 1)}.

646. 𝐈𝐟 𝐅𝐤 𝐢𝐬 𝐤 − 𝐭𝐡 𝐅𝐢𝐛𝐨𝐧𝒂𝐜𝐜𝐢 𝐧𝐮𝐦𝐛𝐞𝐫 𝒂𝐧𝐝 𝐧 ∈ ℕ∗ − {𝟏}, 𝐭𝐡𝐞𝐧 𝐩𝐫𝐨𝐯𝐞 𝐭𝐡𝒂𝐭 ∶


𝐧−𝟏
𝟏 𝐅𝐧𝟒 + 𝐅𝐧𝟐 + 𝟏 𝐅𝐤𝟒 + 𝐅𝐤𝟐 𝐅𝐤+𝟏
𝟐 𝟒
+ 𝐅𝐤+𝟏
( +∑ ) ≥ 𝟏 𝒂𝐧𝐝
𝟑𝐅𝐧 𝐅𝐧+𝟏 𝐅𝐧 𝐅𝐤 𝐅𝐤+𝟏
𝐤=𝟏
𝐧−𝟏
𝟏 𝐅𝐧𝟒 + 𝟏 𝐅𝐤𝟒 + 𝐅𝐤+𝟏
𝟒
( +∑ 𝟐 𝟐
)≥𝟏
𝟐𝐅𝐧 𝐅𝐧+𝟏 𝐅𝐧𝟐 − 𝐅𝐧 + 𝟏 𝐅𝐤 − 𝐅𝐤 𝐅𝐤+𝟏 + 𝐅𝐤+𝟏
𝐤=𝟏

Proposed by D.M. Bătinețu-Giurgiu, Neculai Stanciu-Romania

47 RMM-ABSTRACT ALGEBRA MARATHON 601-700


www.ssmrmh.ro
Solution by Soumava Chakraborty-Kolkata-India
𝐧−𝟏
(∗)
𝐅𝐢𝐫𝐬𝐭𝐥𝐲, 𝐰𝐞 𝐩𝐫𝐨𝐯𝐞 ∶ ∑ 𝐅𝐤 𝐅𝐤+𝟏 ≥ 𝐅𝐧 𝐅𝐧−𝟏 ∀ ∈ ℕ∗ − {𝟏}
𝐤=𝟏
𝐂𝒂𝐬𝐞 𝟏 𝐧 = 𝟐 𝒂𝐧𝐝 𝐭𝐡𝐞𝐧 ∶ 𝐋𝐇𝐒 𝐨𝐟 (∗) = 𝐑𝐇𝐒 𝐨𝐟 (∗) = 𝐅𝟏 𝐅𝟐
⇒ (∗) 𝐢𝐬 𝐭𝐫𝐮𝐞 (𝐞𝐪𝐮𝒂𝐥𝐢𝐭𝐲 𝐜𝒂𝐬𝐞)
𝐂𝒂𝐬𝐞 𝟐 𝐧 ≥ 𝟑 𝒂𝐧𝐝 𝐭𝐡𝐞𝐧 ∶ 𝐋𝐇𝐒 𝐨𝐟 (∗) = 𝐅𝟏 𝐅𝟐 + 𝐅𝟐 𝐅𝟑 + ⋯ + 𝐅𝐧−𝟏 𝐅𝐧 > 𝐅𝐧−𝟏 𝐅𝐧
= 𝐑𝐇𝐒 𝐨𝐟 (∗) ⇒ (∗) 𝐢𝐬 𝐭𝐫𝐮𝐞 (𝐬𝐭𝐫𝐢𝐜𝐭 𝐢𝐧𝐞𝐪𝐮𝒂𝐥𝐢𝐭𝐲)
𝐧−𝟏

∴ 𝐜𝐨𝐦𝐛𝐢𝐧𝐢𝐧𝐠 𝐛𝐨𝐭𝐡 𝐜𝒂𝐬𝐞𝐬, ∑ 𝐅𝐤 𝐅𝐤+𝟏 ≥ 𝐅𝐧 𝐅𝐧−𝟏 ∀ ∈ ℕ∗ − {𝟏}


𝐤=𝟏
𝐧−𝟏
𝟏 𝐅𝐧𝟒 + 𝐅𝐧𝟐 + 𝟏 𝐅𝐤𝟒 + 𝐅𝐤𝟐 𝐅𝐤+𝟏
𝟐 𝟒
+ 𝐅𝐤+𝟏
𝐍𝐨𝐰, ( +∑ )
𝟑𝐅𝐧 𝐅𝐧+𝟏 𝐅𝐧 𝐅𝐤 𝐅𝐤+𝟏
𝐤=𝟏
𝐧−𝟏 𝐧−𝟏
𝐀−𝐆 𝟏 𝟑𝐅𝐧𝟐 ? 𝐅𝐤𝟐 𝐅𝐤+𝟏
𝟐 ?
≥ ( +𝟑∑ ) ≥ 𝟏 ⇔ 𝐅𝐧 + ∑ 𝐅𝐤 𝐅𝐤+𝟏 ≥ 𝐅𝐧 𝐅𝐧+𝟏
𝟑𝐅𝐧 𝐅𝐧+𝟏 𝐅𝐧 𝐅𝐤 𝐅𝐤+𝟏
𝐤=𝟏 𝐤=𝟏
𝐧−𝟏
?
⇔ ∑ 𝐅𝐤 𝐅𝐤+𝟏 ≥ 𝐅𝐧 (𝐅𝐧+𝟏 − 𝐅𝐧 ) = 𝐅𝐧 𝐅𝐧−𝟏 → 𝐭𝐫𝐮𝐞 𝐯𝐢𝒂 (∗)
𝐤=𝟏
𝐧−𝟏
𝟏 𝐅𝐧𝟒 + 𝐅𝐧𝟐 + 𝟏 𝐅𝐤𝟒 + 𝐅𝐤𝟐 𝐅𝐤+𝟏
𝟐 𝟒
+ 𝐅𝐤+𝟏
∴ ( +∑ )≥𝟏
𝟑𝐅𝐧 𝐅𝐧+𝟏 𝐅𝐧 𝐅𝐤 𝐅𝐤+𝟏
𝐤=𝟏
𝐂𝐡𝐞𝐛𝐲𝐬𝐡𝐞𝐯 𝟏
𝐀𝐠𝒂𝐢𝐧, ∀ 𝒙, 𝐲 > 0, 𝒙𝟒 + 𝐲 𝟒 ≥ (𝒙 + 𝐲)(𝒙𝟑 + 𝐲 𝟑 )
𝟐
𝟏 𝐀−𝐆
= (𝒙 + 𝐲)𝟐 (𝒙𝟐 − 𝒙𝐲 + 𝐲 𝟐 ) ≥ 𝟐𝒙𝐲(𝒙𝟐 − 𝒙𝐲 + 𝐲 𝟐 )
𝟐
𝒙𝟒 + 𝐲 𝟒 (∗∗)
⇒ 𝟐 ≥ 𝟐𝒙𝐲 ∀ 𝒙, 𝐲 > 0
𝒙 − 𝒙𝐲 + 𝐲 𝟐
𝐧−𝟏
𝟏 𝐅𝐧𝟒 + 𝟏 𝐅𝐤𝟒 + 𝐅𝐤+𝟏
𝟒
𝐕𝐢𝒂 (∗∗), ( 𝟐 +∑ 𝟐 𝟐
)
𝟐𝐅𝐧 𝐅𝐧+𝟏 𝐅𝐧 − 𝐅𝐧 + 𝟏 𝐅𝐤 − 𝐅𝐤 𝐅𝐤+𝟏 + 𝐅𝐤+𝟏
𝐤=𝟏
𝐧−𝟏 𝐧−𝟏
𝟏 ? ?
≥ (𝟐𝐅𝐧 + 𝟐 ∑ 𝐅𝐤 𝐅𝐤+𝟏 ) ≥ 𝟏 ⇔ 𝐅𝐧 + ∑ 𝐅𝐤 𝐅𝐤+𝟏 ≥ 𝐅𝐧 𝐅𝐧+𝟏
𝟐𝐅𝐧 𝐅𝐧+𝟏
𝐤=𝟏 𝐤=𝟏
𝐧−𝟏
?
⇔ ∑ 𝐅𝐤 𝐅𝐤+𝟏 ≥ 𝐅𝐧 (𝐅𝐧+𝟏 − 𝐅𝐧 ) = 𝐅𝐧 𝐅𝐧−𝟏
𝐤=𝟏
𝐧−𝟏
𝟏 𝐅𝐧𝟒 + 𝟏 𝐅𝐤𝟒 + 𝐅𝐤+𝟏
𝟒
→ 𝐭𝐫𝐮𝐞 𝐯𝐢𝒂 (∗) ∴ ( 𝟐 +∑ 𝟐 𝟐
) ≥ 𝟏 (𝐐𝐄𝐃)
𝟐𝐅𝐧 𝐅𝐧+𝟏 𝐅𝐧 − 𝐅𝐧 + 𝟏 𝐅𝐤 − 𝐅𝐤 𝐅𝐤+𝟏 + 𝐅𝐤+𝟏
𝐤=𝟏

48 RMM-ABSTRACT ALGEBRA MARATHON 601-700


www.ssmrmh.ro
647. Solve for real numbers:
𝑥 + 2𝒚 + 𝟑𝒛 + 𝟒𝒙𝒚 + 𝟓𝒚𝒛 = 𝟓𝟐
{5𝒙 − 𝟒𝒚 + 𝟑𝒛 − 𝟐𝒙𝒚 + 𝒚𝒛 = 𝟖
𝑥 + 𝑦 + 𝑧 + 𝑥𝑦 + 𝑦𝑧 = 14
Proposed by Sakthi Vel-India
Solution by Pham Duc Nam-Vietnam
𝑳𝒆𝒕: 𝒎 = 𝒙𝒚, 𝒏 = 𝒚𝒛
𝑚 = 𝑥𝑦 = 6 − 𝟐𝒚
4𝒎 + 𝟓𝒑 + 𝟑𝒛 = 𝟓𝟐 − (𝒙 + 𝟐𝒚) 𝟑𝒚
⇔ {−𝟐𝒎 + 𝒏 + 𝟑𝒛 = 𝟖 − (𝟓𝒙 − 𝟒𝒚) ⇔ 𝑛 = 𝑦𝑧 = 𝑥 + 𝟐 + 𝟐
𝑚 + 𝑛 + 𝑧 = 14 − (𝒙 + 𝒚) 𝒚
𝑧 = −𝟐𝒙 − + 𝟔
{ 𝟐
𝟔
∗ 𝒙𝒚 = 𝟔 − 𝟐𝒚 ⇒ 𝒚(𝒙 + 𝟐) = 𝟔 ⇒ 𝒚 =
𝒙+𝟐
𝟑𝒚 𝟔𝒛 𝟗 𝒙(𝒙 + 𝟐) + 𝟗 + 𝟐(𝒙 + 𝟐)
∗ 𝒚𝒛 = 𝒙 + +𝟐⇒ = 𝒙+ +𝟐⇒𝒛=
𝟐 𝒙+𝟐 (𝒙 + 𝟐) 𝟔
𝒚 𝒙(𝒙 + 𝟐) + 𝟗 + 𝟐(𝒙 + 𝟐) 𝟑
𝒛 = −𝟐𝒙 − + 𝟔 ⇔ = −𝟐𝒙 − + 𝟔 ⇔ 𝒙𝟑 + 𝟏𝟖𝒙𝟐 + 𝟗𝒙 − 𝟐𝟖 = 𝟎
𝟐 𝟔 𝒙+𝟐
𝑥=1
𝟏
𝑥 = − (𝟏𝟗 + √𝟐𝟒𝟗)
⇔ 𝟐
𝟏
[𝑥 = 𝟐 (√𝟐𝟒𝟗 − 𝟏𝟗)
∗𝒙= 𝟏⇒ 𝒚=𝟐⇒𝒛 =𝟑
𝟏 𝟏 𝟓
∗ 𝒙 = − (𝟏𝟗 + √𝟐𝟒𝟗) ⇒ 𝒚 = (𝟏𝟓 − √𝟐𝟒𝟗) ⇒ 𝒛 = (𝟏𝟕 + √𝟐𝟒𝟗)
𝟐 𝟐 𝟒
𝟏 𝟏 𝟓
∗ 𝒙 = (√𝟐𝟒𝟗 − 𝟏𝟗) ⇒ 𝒚 = (𝟏𝟓 + √𝟐𝟒𝟗) ⇒ 𝒛 = (𝟏𝟕 − √𝟐𝟒𝟗)
𝟐 𝟐 𝟒
648. 𝑭𝟎 = 𝟎, 𝑭𝟏 = 𝟏, 𝑭𝒏+𝟐 = 𝑭𝒏+𝟏 + 𝑭𝒏 , 𝒏 ≥ 𝟎. Prove that:
𝒏
𝑭𝟒𝒌−𝟏
∑ = 𝟏 , ∀𝒏 ∈ ℕ\{𝟎}
𝑭𝟐𝒏+𝟏 𝑭𝟐𝒏
𝒌=𝟏
Proposed by Neculai Stanciu-Romania
Solution by Pham Duc Nam-Vietnam
𝒏 𝒏
𝑭𝟒𝒌−𝟏 𝟏
∗𝑺 = ∑ = ∑ 𝑭𝟒𝒌−𝟏
𝑭𝟐𝒏+𝟏 𝑭𝟐𝒏 𝑭𝟐𝒏+𝟏 𝑭𝟐𝒏
𝒌=𝟏 𝒌=𝟏
𝟏 𝟏 𝒏 𝑭 𝑭𝒏
∗ 𝑭𝒓𝒐𝒎: 𝑭𝒊𝒃𝒐𝒏𝒂𝒄𝒄𝒊 𝒎𝒂𝒕𝒓𝒊𝒙: 𝑨 = (𝒏
) = ( 𝒏+𝟏 )
𝟏 𝟎 𝑭𝒏 𝑭𝒏−𝟏
𝑭 𝑭𝒎
𝒕𝒉𝒆𝒏: 𝑨𝒎 = ( 𝒎+𝟏 ) ⇒ 𝒃𝒚 𝒎𝒂𝒕𝒓𝒊𝒙 𝒎𝒖𝒍𝒕𝒊𝒑𝒍𝒚: 𝑨𝒏 𝑨𝒎 = 𝑨𝒏+𝒎
𝑭𝒎 𝑭𝒎−𝟏
𝑭 𝑭𝒏 𝑭 𝑭𝒎 𝑭 𝑭𝒏+𝒎
⇒ ( 𝒏+𝟏 ) ( 𝒎+𝟏 ) = ( 𝒏+𝒎+𝟏 )
𝑭𝒏 𝑭𝒏−𝟏 𝑭𝒎 𝑭𝒎−𝟏 𝑭𝒏+𝒎 𝑭𝒏+𝒎−𝟏

49 RMM-ABSTRACT ALGEBRA MARATHON 601-700


www.ssmrmh.ro
𝑭𝒎 𝑭𝒏 + 𝑭𝒎−𝟏 𝑭𝒏−𝟏 = 𝑭𝒎+𝒏−𝟏
⇒{ , 𝒔𝒆𝒕: 𝒎 = 𝒏 ⇒ 𝑭𝟐𝒏−𝟏 = 𝑭𝟐𝒏 + 𝑭𝟐𝒏−𝟏 ⇒ 𝑭𝟒𝒌−𝟏
𝑭𝒎 𝑭𝒏+𝟏 + 𝑭𝒎−𝟏 𝑭𝒏 = 𝑭𝒎+𝒏
= 𝑭𝟐𝟐𝒌 + 𝑭𝟐𝟐𝒌−𝟏
𝟏 𝟐 𝟐
𝑩𝒚: 𝑩𝒊𝒏𝒆𝒕′𝒔 𝒇𝒐𝒓𝒎𝒖𝒍𝒂: 𝑭𝟐𝟐𝒌 + 𝑭𝟐𝟐𝒌−𝟏 = [(𝝋𝟐𝒌 − 𝝍𝟐𝒌 ) + (𝝋𝟐𝒌−𝟏 − 𝝍𝟐𝒌−𝟏 ) ]
𝟓
𝟏 𝟒𝒌 𝟒𝒌 𝟒𝒌−𝟐 𝟒𝒌−𝟐
= (𝝋 + 𝝍 + 𝝋 +𝝍 − 𝟐(𝝋𝝍)𝟐𝒌 − 𝟐(𝝋𝝍)𝟐𝒌−𝟏 )
𝟓
𝒏

𝑻𝒂𝒌𝒆 𝒔𝒖𝒎 𝒂𝒏𝒅 𝒄𝒐𝒏𝒔𝒊𝒅𝒆𝒓 𝒔𝒖𝒎𝒔 𝒐𝒇 𝒈𝒆𝒐𝒎𝒆𝒕𝒓𝒊𝒄 𝒔𝒆𝒓𝒊𝒆𝒔 ⇒ ∑ 𝑭𝟒𝒌−𝟏


𝒌=𝟏
𝒏
𝟏
= ∑(𝝋𝟒𝒌 + 𝝍𝟒𝒌 + 𝝋𝟒𝒌−𝟐 + 𝝍𝟒𝒌−𝟐 − 𝟐(𝝋𝝍)𝟐𝒌
𝟓
𝒌=𝟏
𝟏 + √𝟓
− 𝟐(𝝋𝝍)𝟐𝒌−𝟏 ) , 𝒘𝒉𝒆𝒓𝒆: 𝝋 = ,𝝍 = 𝟏 − 𝝋
𝟐
𝟏 𝝋𝟒𝒏+𝟐 − 𝝋𝟐 𝝍𝟒𝒏+𝟐 − 𝝍𝟐
= ( + ) , 𝒌𝒏𝒐𝒘𝒏: 𝝋𝟐 = 𝝋 + 𝟏 ⇒ 𝝍𝟐 − 𝟏 = 𝝍, 𝝋𝝍 = −𝟏
𝟓 𝝋𝟐 − 𝟏 𝝍𝟐 − 𝟏
𝒏
𝟏 𝝋𝟒𝒏+𝟐 − 𝝋𝟐 𝝍𝟒𝒏+𝟐 − 𝝍𝟐
⇒ ∑ 𝑭𝟒𝒌−𝟏 = ( + )
𝟓 𝝋 𝝍
𝒌=𝟏
𝟏
= − (𝝍(𝝋𝟒𝒏+𝟐 − 𝝋𝟐 ) + 𝝋(𝝍𝟒𝒏+𝟐 − 𝝍𝟐 ))
𝟓
𝟏
= ((𝝋𝟒𝒏+𝟏 − 𝝋) + (𝝍𝟒𝒏+𝟏 − 𝝍))
𝟓
𝟏
= (𝝋𝟐𝒏 𝝋𝟐𝒏+𝟏 − 𝝋(𝝋𝝍)𝟐𝒏 − 𝝍(𝝋𝝍)𝟐𝒏 + 𝝍𝟐𝒏 𝝍𝟐𝒏+𝟏 )
𝟓
𝟏 𝟐𝒏 𝟏
= (𝝋 − 𝝍𝟐𝒏 )(𝝋𝟐𝒏+𝟏 − 𝝍𝟐𝒏+𝟏 ) = 𝑭𝟐𝒏+𝟏 𝑭𝟐𝒏 ⇒ 𝑺 = .𝑭 𝑭 = 𝟏(𝑸. 𝑬. 𝑫)
𝟓 𝑭𝟐𝒏+𝟏 𝑭𝟐𝒏 𝟐𝒏+𝟏 𝟐𝒏
649. Solve for real numbers:
𝒙𝟒 + 𝟔𝒛𝟐 + 𝟏 = 𝟒𝒚(𝒚 + 𝟏)
𝟐
{ 𝟒𝒙𝟐 𝒚𝟐 (𝒙 + 𝒚)𝟐 𝟑𝒙𝒚(√𝒙 + √𝒚)
+ 𝒙𝒚 + =
(𝒙 + 𝒚)𝟐 𝟒 𝟐(𝒙 + 𝒚)
Proposed by Daniel Sitaru-Romania
Solution by Pham Duc Nam-Vietnam
𝒙𝟒 + 𝟔𝒛𝟐 + 𝟏 = 𝟒𝒚(𝒚 + 𝟏)(𝟏)
𝟐
{ 𝟒𝒙𝟐 𝒚𝟐 (𝒙 + 𝒚)𝟐 𝟑𝒙𝒚(√𝒙 + √𝒚)
+ 𝒙𝒚 + = ( 𝟐)
(𝒙 + 𝒚)𝟐 𝟒 𝟐( 𝒙 + 𝒚 )
∗ 𝑫𝒐𝒎𝒂𝒊𝒏: 𝒙, 𝒚 ≥ 𝟎, 𝒙 + 𝒚 ≠ 𝟎
∗ 𝑭𝒓𝒐𝒎: (𝟐)
𝟐
𝟒𝑷𝟐 𝑺𝟐 𝟑𝑷(𝑺 + 𝟐√𝑷)
𝑳𝒆𝒕: 𝑺 = 𝒙 + 𝒚, 𝑷 = 𝒙𝒚(𝑺 ≥ 𝟒𝑷): (𝟐) ⇔ 𝟐 + 𝑷 + =
𝑺 𝟒 𝟐𝑺

50 RMM-ABSTRACT ALGEBRA MARATHON 601-700


www.ssmrmh.ro
⇔ 𝟏𝟔𝑷 − 𝟏𝟐𝑷√𝑷𝑺 − 𝟐𝑷𝑺 + 𝑺 = 𝟎 ⇔ (𝟐√𝑷 − 𝑺)(𝟖𝑷√𝑷 − 𝟐√𝑷𝑺 − 𝟐𝑷𝑺 − 𝑺𝟑 ) = 𝟎
𝟐 𝟐 𝟒

∗ 𝑺𝟐 ≥ 𝟒𝑷, 𝑺 >, 𝑃 ≥ 𝟎 ⇒ 𝟐√𝑷 ≤ 𝑺 ⇒ 𝟐𝑷√𝑷 ≤ 𝑷𝑺,


⇒ 𝟖𝑷√𝑷 − 𝟐√𝑷𝑺 − 𝟐𝑷𝑺 − 𝑺𝟑 ≤ 𝟐𝑷𝑺 − 𝟐√𝑷𝑺 − 𝑺𝟑 = 𝑺(𝟐√𝑷(√𝑷 − 𝑺) − 𝑺𝟐 )
𝑺 𝟑
≤ 𝑺 (𝑺 ( − 𝑺) − 𝑺𝟐 ) = 𝑺 (− 𝑺𝟐 ) < 0
𝟐 𝟐
⇒ 𝟐√𝑷 − 𝑺 = 𝟎 ⇒ (𝒙 + 𝒚) = 𝟐√𝒙𝒚 ⇔ (𝒙 + 𝒚)𝟐 = 𝟒𝒙𝒚 ⇔ (𝒙 − 𝒚)𝟐 = 𝟎 ⇒ 𝒙 = 𝒚
∗ 𝒙 = 𝒚 ⇒ (𝟏) ⇔ 𝒙𝟒 + 𝟔𝒛𝟐 + 𝟏 = 𝟒𝒙(𝒙 + 𝟏) ⇔ 𝒙𝟒 + 𝟔𝒛𝟐 + 𝟏 = 𝟒𝒙𝟐 + 𝟒𝒙 ⇔ 𝟔𝒛𝟐
= −𝒙𝟒 + 𝟒𝒙𝟐 + 𝟒𝒙 − 𝟏
𝟏
⇒𝒛=± √−𝒙𝟒 + 𝟒𝒙𝟐 + 𝟒𝒙 − 𝟏, 𝒘𝒉𝒆𝒓𝒆: −𝒙𝟒 + 𝟒𝒙𝟐 + 𝟒𝒙 − 𝟏 ≥ 𝟎
√𝟔
𝟏
⇒ 𝒙 = 𝒚, 𝒛 = ± √−𝒙𝟒 + 𝟒𝒙𝟐 + 𝟒𝒙 − 𝟏 is solution
√𝟔
650. 𝐈𝐟 𝐅𝐧 𝐢𝐬 𝐧𝐭𝐡 𝐅𝐢𝐛𝐨𝐧𝒂𝐜𝐜𝐢 𝐧𝐮𝐦𝐛𝐞𝐫, 𝐭𝐡𝐞𝐧 𝐩𝐫𝐨𝐯𝐞 𝐭𝐡𝒂𝐭 ∶
𝐧
𝐧 𝐦
( ) . ∑ 𝐅𝐤𝟐𝐦 ≥ 𝐧 ∀ 𝐧 ∈ ℕ∗ 𝒂𝐧𝐝 ∀ 𝐦 ≥ 𝟏
𝐅𝐧 𝐅𝐧+𝟏
𝐤=𝟏

Proposed by D.M. Batinetu-Giurgiu, Neculai Stanciu


Solution by Soumava Chakraborty-Kolkata-India
𝐧
𝐧 𝐦
( ) . ∑ 𝐅𝐤𝟐𝐦 ≥ 𝐧
𝐅𝐧 𝐅𝐧+𝟏
𝐤=𝟏
𝐦 (∗)
𝐧 𝟐 𝐦 𝟐 𝐦 𝟐 )𝐦 )
⇔( 𝟐 ) . ((𝐅𝟏 ) + (𝐅𝟐 ) + ⋯ + ( 𝐅𝐧 ≥𝐧
𝐅𝟏 + 𝐅𝟐𝟐 + ⋯ + 𝐅𝐧𝟐
∵ 𝐦 ≥ 𝟏 ∴ 𝐯𝐢𝒂 𝐩𝐨𝐰𝐞𝐫 − 𝐦𝐞𝒂𝐧 𝐢𝐧𝐞𝐪𝐮𝒂𝒍𝐢𝐭𝐲,
𝟐 𝐦 𝟐𝐦 𝟏 𝟏
( 𝟐 )𝐦 𝟏√(𝐅𝟏𝟐 ) + (𝐅𝟐𝟐 ) + ⋯ + (𝐅𝐧𝟐 )𝟏
√ (𝐅𝟏 ) + (𝐅𝟐 ) + ⋯ + 𝐅𝐧
𝐦

𝐧 𝐧
𝟐 𝐦 𝟐 𝐦 𝟐 𝐦 𝟐 𝟐 𝐦
(𝐅𝟏 ) + (𝐅𝟐 ) + ⋯ + (𝐅𝐧 ) 𝐅𝟏 + 𝐅𝟐 + ⋯ + 𝐅𝐧𝟐
⇒ ≥( )
𝐧 𝐧
𝐦
𝟐 𝐦 𝟐 𝐦 𝟐 𝐦
(𝐅𝟏𝟐 + 𝐅𝟐𝟐 + ⋯ + 𝐅𝐧𝟐 )
⇒ (𝐅𝟏 ) + (𝐅𝟐 ) + ⋯ + (𝐅𝐧 ) ≥
𝐧𝐦−𝟏
𝐦
𝐧 𝐦 𝐦
⇒( 𝟐 𝟐 𝟐
) . ((𝐅𝟏𝟐 ) + (𝐅𝟐𝟐 ) + ⋯ + (𝐅𝐧𝟐 )𝐦 )
𝐅𝟏 + 𝐅𝟐 + ⋯ + 𝐅𝐧
𝐦 𝐦
𝐧 (𝐅𝟏𝟐 + 𝐅𝟐𝟐 + ⋯ + 𝐅𝐧𝟐 )
≥( 𝟐 ) . =𝐧
𝐅𝟏 + 𝐅𝟐𝟐 + ⋯ + 𝐅𝐧𝟐 𝐧𝐦−𝟏
𝐧
𝐧 𝐦
⇒ (∗) 𝐢𝐬 𝐭𝐫𝐮𝐞 ∴ ( ) . ∑ 𝐅𝐤𝟐𝐦 ≥ 𝐧 ∀ 𝐧 ∈ ℕ∗ 𝒂𝐧𝐝 ∀ 𝐦 ≥ 𝟏 (𝐐𝐄𝐃)
𝐅𝐧 𝐅𝐧+𝟏
𝐤=𝟏

51 RMM-ABSTRACT ALGEBRA MARATHON 601-700


www.ssmrmh.ro
651.
𝐈𝐟 𝐅𝐧 𝐢𝐬 𝐧𝐭𝐡 𝐅𝐢𝐛𝐨𝐧𝒂𝐜𝐜𝐢 𝐧𝐮𝐦𝐛𝐞𝐫 𝒂𝐧𝐝 𝐋𝐧 𝐢𝐬 𝐧𝐭𝐡 𝐋𝐮𝐜𝒂𝐬 𝐧𝐮𝐦𝐛𝐞𝐫, 𝐭𝐡𝐞𝐧 𝐩𝐫𝐨𝐯𝐞 𝐭𝐡𝒂𝐭
𝟐 𝟐 𝟐
(𝐅𝟏 − √𝐅𝟏 𝐅𝟐 + 𝐅𝟐) + (𝐅𝟐 − √𝐅𝟐 𝐅𝟑 + 𝐅𝟑 ) + ⋯ + (𝐅𝐧 − √𝐅𝐧𝐅𝟏 + 𝐅𝟏 )
≥ 𝟏;
𝐅𝐧𝐅𝐧+𝟏
𝟐 𝟐 𝟐
(𝐋𝟏 − √𝐋𝟏 𝐋𝟐 + 𝐋𝟐 ) + (𝐋𝟐 − √𝐋𝟐 𝐋𝟑 + 𝐋𝟑 ) + ⋯ + (𝐋𝐧 − √𝐋𝐧𝐋𝟏 + 𝐋𝟏 )
≥𝟏
𝐋𝐧𝐋𝐧+𝟏 − 𝟐
Proposed by D.M. Bătinețu-Giurgiu, Neculai Stanciu
Solution by Soumava Chakraborty-Kolkata-India

𝟐 𝒙𝟐 + 𝐲 𝟐
∀ 𝒙, 𝐲 > 0, (𝒙 − √𝒙𝐲 + 𝐲) ≥ ⇔ (𝒙 + 𝐲)𝟐 + 𝒙𝐲 − 𝟐√𝒙𝐲(𝒙 + 𝐲)
𝟐
𝒙𝟐 + 𝐲 𝟐 𝒙𝟐 + 𝐲 𝟐
≥ ⇔ + 𝒙𝐲 + 𝟐𝒙𝐲 − 𝟐√𝒙𝐲(𝒙 + 𝐲) ≥ 𝟎
𝟐 𝟐
𝟐
⇔ (𝒙 + 𝐲)𝟐 + 𝟒𝒙𝐲 − 𝟒√𝒙𝐲(𝒙 + 𝐲) ≥ 𝟎 ⇔ (𝒙 + 𝐲 − 𝟐√𝒙𝐲) ≥ 𝟎
𝟐 𝟐 𝒙𝟐 + 𝐲 𝟐
⇔ (√𝒙 − √𝐲) ≥ 𝟎 → 𝐭𝐫𝐮𝐞 ∴ (𝒙 − √𝒙𝐲 + 𝐲) ≥ ∀ 𝒙, 𝐲 > 0 → (𝟏)
𝟐
𝟐 𝟐 𝟐
𝐕𝐢𝒂 (𝟏), (𝐅𝟏 − √𝐅𝟏 𝐅𝟐 + 𝐅𝟐 ) + (𝐅𝟐 − √𝐅𝟐 𝐅𝟑 + 𝐅𝟑 ) + ⋯ + (𝐅𝐧 − √𝐅𝐧 𝐅𝟏 + 𝐅𝟏 )
𝐧
𝐅𝟏𝟐 + 𝐅𝟐𝟐 𝐅𝟐𝟐 + 𝐅𝟑𝟐 𝐅𝐧𝟐 + 𝐅𝟏𝟐
≥ + +⋯+ = ∑ 𝐅𝐤𝟐 = 𝐅𝐧 𝐅𝐧+𝟏
𝟐 𝟐 𝟐
𝐤=𝟏
𝟐 𝟐 𝟐
(𝐅𝟏 − √𝐅𝟏 𝐅𝟐 + 𝐅𝟐 ) + (𝐅𝟐 − √𝐅𝟐 𝐅𝟑 + 𝐅𝟑 ) + ⋯ + (𝐅𝐧 − √𝐅𝐧 𝐅𝟏 + 𝐅𝟏 )
⇒ ≥𝟏
𝐅𝐧 𝐅𝐧+𝟏
𝟐 𝟐
𝐀𝐠𝒂𝐢𝐧, 𝐯𝐢𝒂 (𝟏), (𝐋𝟏 − √𝐋𝟏 𝐋𝟐 + 𝐋𝟐 ) + (𝐋𝟐 − √𝐋𝟐 𝐋𝟑 + 𝐋𝟑 ) + ⋯
𝐧
𝐋𝟐𝟏 + 𝐋𝟐𝟐 𝐋𝟐𝟐 + 𝐋𝟐𝟑
𝟐 𝐋𝟐𝐧 + 𝐋𝟐𝟏
)
+(𝐋𝐧 − √𝐋𝐧 𝐋𝟏 + 𝐋𝟏 ≥ + +⋯+ = ∑ 𝐋𝟐𝐤 = 𝐋𝐧 𝐋𝐧+𝟏 − 𝟐
𝟐 𝟐 𝟐
𝐤=𝟏
𝟐 𝟐 𝟐
(𝐋𝟏 − √𝐋𝟏 𝐋𝟐 + 𝐋𝟐 ) + (𝐋𝟐 − √𝐋𝟐 𝐋𝟑 + 𝐋𝟑 ) + ⋯ + (𝐋𝐧 − √𝐋𝐧 𝐋𝟏 + 𝐋𝟏 )
⇒ ≥𝟏
𝐋𝐧 𝐋𝐧+𝟏 − 𝟐
𝟏 𝟐 𝟑
652. 𝐈𝐟 𝒙, 𝒚, 𝒛 ∈ ℕ∗, 𝐟𝐢𝐧𝐝 𝐚𝐥𝐥 𝐬𝐨𝐥𝐮𝐭𝐢𝐨𝐧𝐬 𝐨𝐟 ∶ + + =𝟐
𝒙 𝒚 𝒛
Proposed by Sidi Abdallah Lemrabott-Mauritania
Solution by Mohamed Amine Ben Ajiba-Tanger-Morocco
𝟐 𝟑
𝐅𝐢𝐫𝐬𝐭𝐥𝐲, 𝐰𝐞 𝐡𝐚𝐯𝐞, 𝟐 > , 𝐭𝐡𝐞𝐧, 𝒚, 𝒛 ≥ 𝟐.
𝒚 𝒛
𝟏 𝟐 𝟑 𝟏 𝟐 𝟑 𝟔
𝐋𝐞𝐭 𝒎 = 𝒎𝒊𝒏{𝒙, 𝒚, 𝒛}. 𝐖𝐞 𝐡𝐚𝐯𝐞, 𝟐 = + + ≤ + + = ,
𝒙 𝒚 𝒛 𝒎 𝒎 𝒎 𝒎

𝐭𝐡𝐞𝐧 𝒎 ≤ 𝟑, 𝐢. 𝐞. 𝒎 ∈ {𝟏, 𝟐, 𝟑}.

52 RMM-ABSTRACT ALGEBRA MARATHON 601-700


www.ssmrmh.ro
𝟐 𝟑 𝟓
• 𝐈𝐟 𝒎 = 𝒙, 𝐭𝐡𝐞𝐧, 𝐟𝐨𝐫 𝒙 = 𝟏, 𝐰𝐞 𝐡𝐚𝐯𝐞, 𝟏 = + ≤ ⇒ 𝒎𝒊𝒏{𝒚, 𝒛} ∈ {𝟐, 𝟑, 𝟒, 𝟓}.
𝒚 𝒛 𝒎𝒊𝒏{𝒚, 𝒛}

𝐂𝐡𝐞𝐜𝐤𝐢𝐧𝐠 𝐟𝐨𝐫 𝐲 ∈ {𝟐, 𝟑, 𝟒, 𝟓} 𝐚𝐧𝐝 𝐟𝐨𝐫 𝒛 ∈ {𝟐, 𝟑, 𝟒, 𝟓} 𝐰𝐞 𝐠𝐞𝐭 𝐭𝐡𝐞 𝐬𝐨𝐥𝐮𝐭𝐢𝐨𝐧𝐬,

𝒙 = 𝟏, 𝒚 = 𝟑, 𝒛 = 𝟗 ; 𝒙 = 𝟏, 𝒚 = 𝟒, 𝒛 = 𝟔 ; 𝒙 = 𝟏, 𝒚 = 𝟓, 𝒛 = 𝟓 ; 𝒙 = 𝟏, 𝒚 = 𝟖, 𝒛 = 𝟒.
𝟑 𝟐 𝟑 𝟓
𝐅𝐨𝐫 𝒙 = 𝟐, 𝐰𝐞 𝐡𝐚𝐯𝐞, = + ≤ ⇒ 𝒎𝒊𝒏{𝒚, 𝒛} ∈ {𝟐, 𝟑}.
𝟐 𝒚 𝒛 𝒎𝒊𝒏{𝒚, 𝒛}

𝐂𝐡𝐞𝐜𝐤𝐢𝐧𝐠 𝐟𝐨𝐫 𝐲 ∈ {𝟐, 𝟑} 𝐚𝐧𝐝 𝐟𝐨𝐫 𝒛 ∈ {𝟐, 𝟑} 𝐰𝐞 𝐠𝐞𝐭 𝐭𝐡𝐞 𝐬𝐨𝐥𝐮𝐭𝐢𝐨𝐧𝐬,

𝒙 = 𝟐, 𝒚 = 𝟐, 𝒛 = 𝟔 ; 𝒙 = 𝟐, 𝒚 = 𝟒, 𝒛 = 𝟑.
𝟓 𝟐 𝟑 𝟓
𝐅𝐨𝐫 𝒙 = 𝟑, 𝐰𝐞 𝐡𝐚𝐯𝐞, = + ≤ ⇒ 𝒎𝒊𝒏{𝒚, 𝒛} ∈ {𝟑} ⇒ 𝒙 = 𝟑, 𝒚 = 𝟑, 𝒛 = 𝟑.
𝟑 𝒚 𝒛 𝒎𝒊𝒏{𝒚, 𝒛}
𝟏 𝟑 𝟒
• 𝐈𝐟 𝒎 = 𝒚, 𝐭𝐡𝐞𝐧, 𝐟𝐨𝐫 𝒚 = 𝟐, 𝐰𝐞 𝐡𝐚𝐯𝐞, 𝟏 = + ≤ ⇒ 𝒎𝒊𝒏{𝒙, 𝒛} ∈ {𝟐, 𝟑, 𝟒}.
𝒙 𝒛 𝒎𝒊𝒏{𝒙, 𝒛}
𝐂𝐡𝐞𝐜𝐤𝐢𝐧𝐠 𝐟𝐨𝐫 𝐱 ∈ {𝟐, 𝟑, 𝟒} 𝐚𝐧𝐝 𝐟𝐨𝐫 𝒛 ∈ {𝟐, 𝟑, 𝟒} 𝐰𝐞 𝐠𝐞𝐭 𝐭𝐡𝐞 𝐧𝐞𝐰 𝐬𝐨𝐥𝐮𝐭𝐢𝐨𝐧, 𝒙 = 𝟒, 𝒚 = 𝟐, 𝒛 = 𝟒.
𝟒 𝟏 𝟑 𝟒
𝐅𝐨𝐫 𝒚 = 𝟑, 𝐰𝐞 𝐡𝐚𝐯𝐞, = + ≤ ⇒ 𝒎𝒊𝒏{𝒙, 𝒛} ≤ 𝟑 ⇒ 𝒙 = 𝟑 𝐨𝐫 𝒛 = 𝟑 ⇒ 𝒙 = 𝒚 = 𝒛
𝟑 𝒙 𝒛 𝒎𝒊𝒏{𝒙, 𝒛}
= 𝟑.
𝟏 𝟏 𝟐 𝟑
• 𝐈𝐟 𝒎 = 𝒛, 𝐭𝐡𝐞𝐧, 𝐟𝐨𝐫 𝒛 = 𝟐, 𝐰𝐞 𝐡𝐚𝐯𝐞, = + ≤ ⇒ 𝒎𝒊𝒏{𝒙, 𝒚} ∈ {𝟐, 𝟑, 𝟒, 𝟓, 𝟔}.
𝟐 𝒙 𝒚 𝒎𝒊𝒏{𝒙, 𝒚}

𝐂𝐡𝐞𝐜𝐤𝐢𝐧𝐠 𝐟𝐨𝐫 𝐱 ∈ {𝟐, 𝟑, 𝟒, 𝟓, 𝟔} 𝐚𝐧𝐝 𝐟𝐨𝐫 𝒛 ∈ {𝟐, 𝟑, 𝟒, 𝟓, 𝟔} 𝐰𝐞 𝐠𝐞𝐭 𝐭𝐡𝐞 𝐧𝐞𝐰 𝐬𝐨𝐥𝐮𝐭𝐢𝐨𝐧𝐬,

𝒙 = 𝟑, 𝒚 = 𝟏𝟐, 𝒛 = 𝟐 ; 𝒙 = 𝟒, 𝒚 = 𝟖, 𝒛 = 𝟐 ; 𝒙 = 𝟏𝟎, 𝒚 = 𝟓, 𝒛 = 𝟐 ; 𝒙 = 𝟔, 𝒚 = 𝟔, 𝒛 = 𝟐.
𝟏 𝟐 𝟑
𝐅𝐨𝐫 𝒛 = 𝟑, 𝐰𝐞 𝐡𝐚𝐯𝐞, 𝟏 = + ≤ ⇒ 𝒎𝒊𝒏{𝒙, 𝒚} ≤ 𝟑 ⇒
𝒙 𝒚 𝒎𝒊𝒏{𝒙, 𝒚}

𝒙 = 𝟑 𝐨𝐫 𝒚 = 𝟑 ⇒ 𝒙 = 𝒚 = 𝒛 = 𝟑.

𝐓𝐡𝐞𝐫𝐞𝐟𝐨𝐫𝐞,
(𝟏, 𝟑, 𝟗); (𝟏, 𝟒, 𝟔); (𝟏, 𝟓, 𝟓); (𝟏, 𝟖, 𝟒); (𝟐, 𝟐, 𝟔); (𝟐, 𝟒, 𝟑); (𝟑, 𝟑, 𝟑); (𝟒, 𝟐, 𝟒);
𝐒={ }
(𝟑, 𝟏𝟐, 𝟐); (𝟒, 𝟖, 𝟐); (𝟏𝟎, 𝟓, 𝟐); (𝟔, 𝟔, 𝟐)/(𝐱, 𝐲, 𝐳)

653. Prove that:


𝒏
𝟏 𝟐
∑ 𝒕𝒂𝒏(𝟏 + 𝑭𝟐𝒌 ) > 1
𝟒𝑭𝒏 𝑭𝒏+𝟏
𝒌=𝟏

Proposed by D.M.Bătinețu-Giurgiu, Neculai Stanciu-Romania

53 RMM-ABSTRACT ALGEBRA MARATHON 601-700


www.ssmrmh.ro
Solution by Tapas Das-India

We know that 𝒕𝒂𝒏𝒙 > 𝑥, ∀𝑥 > 0.


𝑨𝑴−𝑮𝑴
𝟐 𝟐
𝒕𝒂𝒏(𝟏 + 𝑭𝟐𝒌 ) > (𝟏 + 𝑭𝟐𝒌 ) ⏞
≥ 𝟒𝑭𝟐𝒌
𝒏 𝒏
𝟏 𝟐 𝟏 𝑭𝒏 𝑭𝒏+𝟏
∑ 𝒕𝒂𝒏(𝟏 + 𝑭𝟐𝒌 ) > ∑ 𝟒𝑭𝟐𝒌 = =𝟏
𝟒𝑭𝒏 𝑭𝒏+𝟏 𝟒𝑭𝒏 𝑭𝒏+𝟏 𝑭𝒏 𝑭𝒏+𝟏
𝒌=𝟏 𝒌=𝟏
Observation:
We will prove by mathematical induction that:
𝒏

𝑷(𝒏) : ∑ 𝑭𝟐𝒌 = 𝑭𝒏 𝑭𝒏+𝟏


𝒌=𝟏
For 𝒏 = 𝟐: 𝑭𝟐𝟏 + 𝑭𝟐𝟐 = 𝑭𝟐 𝑭𝟑 ⟺ 𝟎𝟐 + 𝑭𝟐𝟐 = 𝑭𝟐 (𝟎 + 𝑭𝟐 ) ⟺ 𝑭𝟐𝟐 = 𝑭𝟐𝟐

𝒏+𝟏

𝑷(𝒏 + 𝟏) : ∑ 𝑭𝟐𝒌 = 𝑭𝒏+𝟏 𝑭𝒏+𝟐


𝒌=𝟏
𝒏

∑ 𝑭𝟐𝒌 + 𝑭𝟐𝒏+𝟏 = 𝑭𝒏+𝟏 (𝑭𝒏 + 𝑭𝒏+𝟏 )


𝒌=𝟏

𝑭𝒏 𝑭𝒏+𝟏 + 𝑭𝟐𝒏+𝟏 = 𝑭𝒏 𝑭𝒏+𝟏 + 𝑭𝟐𝒏+𝟏

𝑷(𝒏) → 𝑷(𝒏 + 𝟏)
654. 𝐈𝐟 𝒙, 𝒚, 𝒛 > 0, 𝐭𝐡𝐞𝐧 𝐩𝐫𝐨𝐯𝐞 𝐭𝐡𝐚𝐭
𝟐𝑭𝒏+𝟐 𝒙 𝒚 𝒛
( + + )≥𝟏
𝟑 𝒙𝑭𝒏 + 𝒚𝑭𝒏+𝟏 + 𝒛𝑭𝒏+𝟐 𝒚𝑭𝒏 + 𝒛𝑭𝒏+𝟏 + 𝒙𝑭𝒏+𝟐 𝒛𝑭𝒏 + 𝒙𝑭𝒏+𝟏 + 𝒚𝑭𝒏+𝟐
𝟐𝑳𝒏+𝟐 𝒙 𝒚 𝒛
( + + ) ≥ 𝟏,
𝟑 𝒙𝑳𝒏 + 𝒚𝑳𝒏+𝟏 + 𝒛𝑳𝒏+𝟐 𝒚𝑳𝒏 + 𝒛𝑳𝒏+𝟏 + 𝒙𝑳𝒏+𝟐 𝒛𝑳𝒏 + 𝒙𝑳𝒏+𝟏 + 𝒚𝑳𝒏+𝟐

𝐰𝐡𝐞𝐫𝐞 𝑭𝒏 , 𝑳𝒏 𝐫𝐞𝐩𝐫𝐞𝐬𝐞𝐧𝐭𝐬 𝐭𝐡𝐞 𝒏𝒕𝒉 𝐅𝐢𝐛𝐨𝐧𝐚𝐜𝐜𝐢 𝐧𝐮𝐦𝐛𝐞𝐫, 𝐋𝐮𝐜𝐚𝐬 𝐧𝐮𝐦𝐛𝐞𝐫.


Proposed by D.M.Bătinețu-Giurgiu,Neculai Stanciu-Romania
Solution by Mohamed Amine Ben Ajiba-Tanger-Morocco
𝐁𝐲 𝐂𝐁𝐒 𝐢𝐧𝐞𝐪𝐮𝐚𝐥𝐢𝐭𝐲, 𝐰𝐞 𝐡𝐚𝐯𝐞,

𝒙 (𝒙 + 𝒚 + 𝒛)𝟐 (𝒙 + 𝒚 + 𝒛)𝟐
∑ ≥ =
𝒙𝑭𝒏 + 𝒚𝑭𝒏+𝟏 + 𝒛𝑭𝒏+𝟐 ∑𝒄𝒚𝒄 𝒙(𝒙𝑭𝒏 + 𝒚𝑭𝒏+𝟏 + 𝒛𝑭𝒏+𝟐 ) 𝑭𝒏 . ∑𝒄𝒚𝒄 𝒙 + (𝑭𝒏 + 𝟐𝑭𝒏+𝟏 ) ∑𝒄𝒚𝒄 𝒚𝒛
𝟐
𝒄𝒚𝒄

54 RMM-ABSTRACT ALGEBRA MARATHON 601-700


www.ssmrmh.ro
𝟐𝑭𝒏+𝟏 ≥ 𝑭𝒏
(𝒙 + 𝒚 + 𝒛 )𝟐 (𝒙 + 𝒚 + 𝒛)𝟐
= ⏞

𝟐 𝟐
𝑭𝒏 (∑𝒄𝒚𝒄 𝒙) + (𝟐𝑭𝒏+𝟏 − 𝑭𝒏 ) ∑𝒄𝒚𝒄 𝒚𝒛 𝟐 (∑𝒄𝒚𝒄 𝒙)
𝑭𝒏 (∑𝒄𝒚𝒄 𝒙) + (𝟐𝑭𝒏+𝟏 − 𝑭𝒏 ) 𝟑
𝟑 𝟑
= = .
(
𝟐 𝑭𝒏 + 𝑭𝒏+𝟏 ) 𝟐𝑭𝒏+𝟐

𝟐𝑭𝒏+𝟐 𝒙 𝒚 𝒛
( + + ) ≥ 𝟏.
𝟑 𝒙𝑭𝒏 + 𝒚𝑭𝒏+𝟏 + 𝒛𝑭𝒏+𝟐 𝒚𝑭𝒏 + 𝒛𝑭𝒏+𝟏 + 𝒙𝑭𝒏+𝟐 𝒛𝑭𝒏 + 𝒙𝑭𝒏+𝟏 + 𝒚𝑭𝒏+𝟐

𝐄𝐪𝐮𝐚𝐥𝐢𝐭𝐲 𝐡𝐨𝐥𝐝𝐬 𝐢𝐟𝐟 𝒙 = 𝒚 = 𝒛.

𝐒𝐢𝐦𝐢𝐥𝐚𝐫𝐥𝐲, 𝐰𝐞 𝐡𝐚𝐯𝐞,

𝒙 (𝒙 + 𝒚 + 𝒛)𝟐 (𝒙 + 𝒚 + 𝒛)𝟐
∑ ≥ =
𝒙𝑳𝒏 + 𝒚𝑳𝒏+𝟏 + 𝒛𝑳𝒏+𝟐 ∑𝒄𝒚𝒄 𝒙(𝒙𝑳𝒏 + 𝒚𝑳𝒏+𝟏 + 𝒛𝑳𝒏+𝟐 ) 𝑳𝒏 . ∑𝒄𝒚𝒄 𝒙𝟐 + (𝑳𝒏 + 𝟐𝑳𝒏+𝟏 ) ∑𝒄𝒚𝒄 𝒚𝒛
𝒄𝒚𝒄

𝟐𝑳𝒏+𝟏 ≥ 𝑳𝒏
(𝒙 + 𝒚 + 𝒛)𝟐 (𝒙 + 𝒚 + 𝒛)𝟐
= ⏞

𝟐 𝟐
𝑳𝒏 (∑𝒄𝒚𝒄 𝒙) + (𝟐𝑳𝒏+𝟏 − 𝑳𝒏 ) ∑𝒄𝒚𝒄 𝒚𝒛 𝟐 (∑𝒄𝒚𝒄 𝒙)
𝑳𝒏 (∑𝒄𝒚𝒄 𝒙) + (𝟐𝑳𝒏+𝟏 − 𝑳𝒏 ) 𝟑
𝟑 𝟑
= = .
(
𝟐 𝑳𝒏 + 𝑳𝒏+𝟏 ) 𝟐𝑳𝒏+𝟐

𝟐𝑳𝒏+𝟐 𝒙 𝒚 𝒛
𝐓𝐡𝐞𝐧, ( + + ) ≥ 𝟏,
𝟑 𝒙𝑳𝒏 + 𝒚𝑳𝒏+𝟏 + 𝒛𝑳𝒏+𝟐 𝒚𝑳𝒏 + 𝒛𝑳𝒏+𝟏 + 𝒙𝑳𝒏+𝟐 𝒛𝑳𝒏 + 𝒙𝑳𝒏+𝟏 + 𝒚𝑳𝒏+𝟐

𝐄𝐪𝐮𝐚𝐥𝐢𝐭𝐲 𝐡𝐨𝐥𝐝𝐬 𝐢𝐟𝐟 𝒙 = 𝒚 = 𝒛.

655. Solve for real numbers:


𝒙+𝒚 𝒙 𝒙 𝒙
𝒙 𝒙+𝒚 𝒙 𝒙 𝟒
| 𝒙 |= 𝒚 +𝟖
𝒙 𝒙 𝒙+𝒚
𝒙 𝒙 𝒙 𝒙+𝒚
Proposed by Qudrat Muhammadi-Afghanistan
Solution by Daniel Sitaru-Romania

𝑳𝟏 +𝑳𝟐 +𝑳𝟑 +𝑳𝟒


𝒙+𝒚 𝒙 𝒙 𝒙 𝟒𝒙 + 𝒚 𝟒𝒙 + 𝒚 𝟒𝒙 + 𝒚 𝟒𝒙 + 𝒚
𝒙 𝒙+𝒚 𝒙 𝒙 𝒙 𝒙+𝒚 𝒙 𝒙
| 𝒙 | =|
𝒙 𝒙 𝒙+𝒚 𝒙 𝒙 𝒙+𝒚 𝒙 |=
𝒙 𝒙 𝒙 𝒙+𝒚 𝒙 𝒙 𝒙 𝒙+𝒚

55 RMM-ABSTRACT ALGEBRA MARATHON 601-700


www.ssmrmh.ro
𝑪𝟐 −𝑪𝟏
𝑪𝟑 −𝑪𝟏
𝟏 𝟏 𝟏 𝟏 𝑪𝟒 −𝑪𝟏
𝟏 𝟎 𝟎 𝟎
𝒙 𝒙+𝒚 𝒙 𝒙 𝒙 𝒚 𝟎 𝟎
= (𝟒𝒙 + 𝒚) | 𝒙 | = (𝟒𝒙 + 𝒚) | | = (𝟒𝒙 + 𝒚)𝒚𝟑
𝒙 𝒙 𝒙+𝒚 𝒙 𝟎 𝒚 𝟎
𝒙 𝒙 𝒙 𝒙+𝒚 𝒙 𝟎 𝟎 𝒚

𝟑 𝟐
(𝟒𝒙 + 𝒚)𝒚𝟑 = 𝒚𝟒 + 𝟖 ⟹ 𝟒𝒙𝒚𝟑 = 𝟖 ⟹ 𝒚 = √
𝒙
𝟑 𝟐
Solution:(𝒙, √𝒙 ) , 𝒙 ≠ 𝟎.
𝒂 𝒃 𝟎 𝟎 (
656. 𝑲 = {( )| 𝒂, 𝒃 ∈ ℝ} , 𝑶𝟐 = ( ) , 𝑲, +), (𝑲 − {𝑶𝟐 }) are abelian
−𝒃 𝒂 𝟎 𝟎
groups. Prove that: (𝑲, +) ≇ (𝑲 − {𝑶𝟐 }).
Proposed by Daniel Sitaru-Romania
Solution by Ravi Prakash-New Delhi-India

Suppose by absurdum that exists 𝒇: (𝑲, +) → (𝑲 − {𝑶𝟐 }) 𝒇 −isomorphism.

𝟏 𝟎 𝟎 𝟎
𝒇(𝑶𝟐 ) = 𝑰𝟐 = ( ), 𝑶𝟐 = ( )
𝟎 𝟏 𝟎 𝟎

𝒂 −𝒃 −𝟏 𝟎
𝒇 −bijectif, hence exists 𝑨 = ( ) ≠ 𝑶𝟐 such that: 𝒇(𝑨) = ( )
𝒃 𝒂 𝟎 −𝟏

−𝟏 𝟎 −𝟏 𝟎
𝒇(𝟐𝑨) = 𝒇(𝑨 + 𝑨) = 𝒇(𝑨) ∙ 𝒇(𝑨) = ( )∙( ) = 𝑰𝟐
𝟎 −𝟏 𝟎 −𝟏
𝒇−𝒊𝒏𝒋𝒆𝒄𝒕𝒊𝒇
𝒇(
𝟐𝒂 −𝟐𝒃
) = 𝒇(𝑶𝟐 ) ⟹ ⏞ (𝟐𝒂 −𝟐𝒃) = (𝟎 𝟎) ⟹
𝟐𝒃 𝟐𝒂 𝟐𝒃 𝟐𝒂 𝟎 𝟎
𝒂 −𝒃
⟹𝒂=𝒃=𝟎⟹ 𝑨=( ) = 𝑶𝟐 . False!
𝒃 𝒂
Hence:
(𝑲, +) ≇ (𝑲 − {𝑶𝟐 })
657.
𝒂𝟏 𝒃𝟏 𝒂 𝒃𝒏
𝑨=( ) , 𝑨𝒏 = ( 𝒏 ) , 𝒂𝟏 = 𝟓, 𝒃𝟏 = 𝟏
−𝒃𝟏 𝒂𝟏 −𝒃𝒏 𝒂𝒏

Find: 𝛀 = 𝟑√𝒂𝟐𝟎 + 𝒊𝒃𝟐𝟎


Proposed by Daniel Sitaru-Romania
Solution by Pham Duc Nam-Vietnam
𝟓 𝟏
∗𝑨=( )
−𝟏 𝟓

56 RMM-ABSTRACT ALGEBRA MARATHON 601-700


www.ssmrmh.ro
𝒊
Eigenvalue: 𝟓 − 𝒊,eigenvector ( ) 𝟏 −𝒊 𝟏
𝟏 𝒊 −𝒊
{ ⇒𝑷=( ) ⇒ 𝑷−𝟏 = ( ),𝑫
−𝒊 𝟏 𝟏 𝟐 𝒊 𝟏
Eigenvalue: 𝟓 + 𝒊,eigenvector ( )
𝟏
𝟓−𝒊 𝟎
=( )
𝟎 𝟓+𝒊
𝟏 𝒊 −𝒊 𝟓 − 𝒊 𝟎 −𝒊 𝟏
⇒ 𝑨 = 𝑷𝑫𝑷−𝟏 = ( )( )( ) ⇒ 𝑨𝒏 = 𝑷𝑫𝒏 𝑷−𝟏
𝟐 𝟏 𝟏 𝟎 𝟓+𝒊 𝒊 𝟏
𝟏 𝒊 −𝒊 (𝟓 − 𝒊)𝒏 𝟎 −𝒊 𝟏
= ( )( 𝒏) ( )
𝟐 𝟏 𝟏 𝟎 ( 𝟓 + 𝒊 ) 𝒊 𝟏
𝟏 𝒊(𝟓 − 𝒊)𝒏 −𝒊(𝟓 + 𝒊)𝒏 −𝒊 𝟏
= ( )( )
𝟐 (𝟓 − 𝒊 ) 𝒏 (𝟓 + 𝒊 )𝒏 𝒊 𝟏
𝟏 𝟏
((𝟓 + 𝒊)𝒏 + (𝟓 − 𝒊)𝒏 ) 𝒊((𝟓 − 𝒊)𝒏 − (𝟓 + 𝒊)𝒏 )
=( 𝟐 𝟐 )
𝟏 𝒏 𝒏
𝟏 𝒏 𝒏
− 𝒊((𝟓 − 𝒊) − (𝟓 + 𝒊) ) ((𝟓 + 𝒊) + (𝟓 − 𝒊) )
𝟐 𝟐
𝟏
𝒂𝟐𝟎 = ((𝟓 + 𝒊)𝟐𝟎 + (𝟓 − 𝒊)𝟐𝟎 )
⇒{ 𝟐
𝟏
𝒃𝟐𝟎 = 𝒊((𝟓 − 𝒊)𝟐𝟎 − (𝟓 + 𝒊)𝟐𝟎 )
𝟐
∗ 𝜴 = 𝟑√𝒂𝟐𝟎 + 𝒊𝒃𝟐𝟎
𝟏 𝟏
𝒂𝟐𝟎 + 𝒊𝒃𝟐𝟎 = ((𝟓 + 𝒊)𝟐𝟎 + (𝟓 − 𝒊)𝟐𝟎 ) − ((𝟓 − 𝒊)𝟐𝟎 − (𝟓 + 𝒊)𝟐𝟎 ) = (𝟓 + 𝒊)𝟐𝟎
𝟐 𝟐
𝟏 𝟏
= 𝟐𝟔𝟏𝟎 (𝒄𝒐𝒔 (𝟐𝟎 𝒕𝒂𝒏−𝟏 ( )) + 𝒊 𝒔𝒊𝒏 (𝟐𝟎 𝒕𝒂𝒏−𝟏 ( )))
𝟓 𝟓
𝟑
⇒ 𝜴 = √𝒂𝟐𝟎 + 𝒊𝒃𝟐𝟎
𝟏 𝟏
𝟏𝟎 𝟐𝟎 𝒕𝒂𝒏−𝟏 (𝟓) + 𝟐𝒌𝝅 𝟐𝟎 𝒕𝒂𝒏−𝟏 (𝟓) + 𝟐𝒌𝝅
= 𝟐𝟔 𝟑 (𝒄𝒐𝒔 ( ) + 𝒊 𝒔𝒊𝒏 ( )),
𝟑 𝟑
𝒌 = 𝟎, 𝟏, 𝟐

658. 𝐋𝐞𝐭 𝛌 ≥ 𝟎 𝐟𝐢𝒙𝐞𝐝. 𝐒𝐨𝐥𝐯𝐞 𝐢𝐧 ℝ ∶


(𝒙 − 𝛌)𝟒 𝟐 𝟒
𝟏
𝟐 𝟐
+ (𝒙 − 𝟐𝛌 − 𝟏 ) + 𝟐
= 𝟑𝒙𝟐 − 𝟐𝛌𝒙 + 𝛌𝟐 − 𝟒𝛌 − 𝟐
(𝒙 − 𝟐𝛌 − 𝟏) (𝒙 − 𝛌)
Proposed by Marin Chirciu-Romania
Solution 1 by Soumava Chakraborty-Kolkata-India
(𝒙 − 𝛌)𝟒 𝟒 𝟏
𝟐 𝟐
+ (𝒙𝟐 − 𝟐𝛌 − 𝟏) +
(𝒙 − 𝟐𝛌 − 𝟏) (𝒙 − 𝛌)𝟐
= 𝟑𝒙 − 𝟐𝛌𝒙 + 𝛌 − 𝟒𝛌 − 𝟐 = 𝒙 − 𝟐𝝀𝒙 + 𝛌 + 𝟐𝒙𝟐 − 𝟒𝛌 − 𝟐
𝟐 𝟐 𝟐 𝟐

𝒂𝟐 𝟏 (∗)
= (𝒙 − 𝛌)𝟐 + 𝟐(𝒙𝟐 − 𝟐𝛌 − 𝟏) ⇒ 𝟐 + 𝐛𝟒 + = 𝒂 + 𝟐𝐛
𝐛 𝒂
(𝒂 = (𝒙 − 𝛌)𝟐 , 𝐛 = 𝒙𝟐 − 𝟐𝛌 − 𝟏)
57 RMM-ABSTRACT ALGEBRA MARATHON 601-700
www.ssmrmh.ro
𝟐
𝒂𝟐 𝟏 𝐁𝐞𝐫𝐠𝐬𝐭𝐫𝐨𝐦 (𝒂 + 𝐛𝟐 + 𝟏)
∵ 𝒂, 𝐛 ≠ 𝟎 ∴ 𝒂 = (𝒙 − 𝛌 )𝟐 > 0, 𝐛 > 0 ∴ 𝟐 + 𝐛𝟒 +
𝟐

𝐛 𝒂 𝐛𝟐 + 𝟏 + 𝒂
𝐯𝐢𝒂 (∗)
= 𝒂 + 𝐛𝟐 + 𝟏 ⇒ 𝒂 + 𝟐𝐛 ≥ 𝒂 + 𝐛𝟐 + 𝟏 ⇒ (𝐛 − 𝟏)𝟐 ≤ 𝟎, 𝐛𝐮𝐭 ∵ (𝐛 − 𝟏)𝟐 ≥ 𝟎
(∗∗) 𝒂𝟐 𝟏 𝟏
∴ (𝐛 − 𝟏)𝟐 = 𝟎 ⇒ 𝐛 = 𝟏 ∴ (∗), (∗∗) ⇒ + 𝟏𝟒 + = 𝒂 + 𝟐 ⇒ 𝒂𝟐 − 𝟏 = 𝒂 −
𝟏 𝒂 𝒂
𝒂𝟐 − 𝟏 𝟏 (𝒂 + 𝟏 )(𝒂 − 𝟏 )𝟐
= ⇒ (𝒂𝟐 − 𝟏) (𝟏 − ) = 𝟎 ⇒ = 𝟎 ⇒ 𝒂 = ±𝟏 ⇒ 𝒂 = 𝟏
𝒂 𝒂 𝒂
(∵ 𝒂 = (𝒙 − 𝛌)𝟐 > 0) ⇒ (𝒙 − 𝛌)𝟐 = 𝟏 ⇒ 𝒙 = 𝛌 ± 𝟏 𝒂𝐧𝐝 ∵ 𝐛 = 𝟏
(∗∗∗)
∴ 𝒙𝟐 − 𝟐𝛌 − 𝟏 = 𝟏 ⇒ 𝒙𝟐 = 𝟐𝛌 + 𝟐
𝐯𝐢𝒂 (∗∗∗)
𝐂𝒂𝐬𝐞 𝟏 𝒙 = 𝛌 + 𝟏 ⇒ 𝒙𝟐 = 𝛌𝟐 + 𝟐𝛌 + 𝟏 ⇒ 𝟐𝛌 + 𝟐 = 𝛌𝟐 + 𝟐𝛌 + 𝟏 ⇒ 𝛌 = ±𝟏
⇒ 𝛌 = 𝟏 (∵ 𝛌 ≥ 𝟎) ∴ 𝒙 = 𝛌 + 𝟏 = 𝟐 ∴ 𝒙 = 𝟐
𝐯𝐢𝒂 (∗∗∗)
𝐂𝒂𝐬𝐞 𝟐 𝒙 = 𝛌 − 𝟏 ⇒ 𝒙𝟐 = 𝛌𝟐 − 𝟐𝛌 + 𝟏 ⇒ 𝟐𝛌 + 𝟐 = 𝛌𝟐 − 𝟐𝛌 + 𝟏
𝟒 ± √𝟏𝟔 + 𝟒
⇒ 𝛌𝟐 − 𝟒𝛌 − 𝟏 = 𝟎 ⇒ 𝛌 = ⇒ 𝛌 = 𝟐 ± √𝟓 𝒂𝐧𝐝 ∵ 𝛌 ≥ 𝟎 ∴ 𝛌 = 𝟐 + √𝟓
𝟐
⇒ 𝒙 = 𝛌 − 𝟏 = √𝟓 + 𝟏 ∴ 𝒙 = √𝟓 + 𝟏
∴ (𝒙 = 𝟐 𝐟𝐨𝐫 𝛌 = 𝟏) 𝒂𝐧𝐝 (𝒙 = √𝟓 + 𝟏 𝐟𝐨𝐫 𝛌 = √𝟓 + 𝟐)
𝒂𝐫𝐞 𝒂𝒍𝒍 𝐩𝐨𝐬𝐬𝐢𝐛𝐥𝐞 𝐬𝐨𝐥𝐮𝐭𝐢𝐨𝐧𝐬 𝒂𝐧𝐝 𝐢𝐟 𝛌 ≠ 𝟏, √𝟓 + 𝟐, 𝐭𝐡𝐞𝐧 𝐧𝐨 𝐬𝐨𝐥𝐮𝐭𝐢𝐨𝐧 𝐞𝒙𝐢𝐬𝐭𝐬 (𝒂𝒏𝒔)

Solution 2 by Mohamed Amine Ben Ajiba-Tanger-Morocco

𝐋𝐞𝐭 𝒂 ≔ (𝒙 − 𝝀)𝟐 > 0 𝐚𝐧𝐝 𝒃


≔ 𝒙𝟐 − 𝟐𝝀 − 𝟏. 𝐓𝐡𝐞 𝐠𝐢𝐯𝐞𝐧 𝐞𝐪𝐮𝐚𝐭𝐢𝐨𝐧 𝐜𝐚𝐧 𝐛𝐞 𝐫𝐞𝐰𝐫𝐢𝐭𝐭𝐞𝐧 𝐚𝐬 𝐟𝐨𝐥𝐥𝐨𝐰𝐬 ∶
𝟐
𝒂 𝟒
𝟏 𝒂𝟐 𝒃𝟒 𝟏 𝒃𝟒 𝟑 𝒂𝟐 𝒃𝟒 𝟏 𝟕
+ 𝒃 + = 𝒂 + 𝟐𝒃 𝐨𝐫 ( 𝟐 + + − 𝒂) + ( + − 𝟐𝒃) + ( 𝟐 + + − )
𝒃𝟐 𝒂 𝟐𝒃 𝟒 𝟒 𝟐 𝟐 𝟐𝒃 𝟒 𝒂 𝟒
= 𝟎 (∗)
𝐁𝐲 𝐭𝐡𝐞 𝐀𝐌 − 𝐆𝐌 𝐢𝐧𝐞𝐪𝐮𝐚𝐥𝐢𝐭𝐲, 𝐰𝐞 𝐡𝐚𝐯𝐞
𝟐
𝒂𝟐 𝒃𝟒 𝟏 𝟏 𝒂𝟐 𝟒 𝒂𝟐 𝒂𝟐
• + √
+ = (𝟐. 𝟐 + 𝒃 + 𝟏) ≥ ( 𝟐 ) . 𝒃 . 𝟏 = 𝒂, 𝐞𝐪𝐮𝐚𝐥𝐢𝐭𝐲 𝐢𝐟 𝟐 = 𝒃𝟒 = 𝟏.
𝟒 𝟒
𝟐𝒃𝟐 𝟒 𝟒 𝟒 𝒃 𝒃 𝒃
𝒃𝟒 𝟑 𝒃𝟒 + 𝟑. 𝟏 𝟒
• + = ≥ 𝟐. √𝒃𝟒 . 𝟏𝟑 = 𝟐|𝒃| ≥ 𝟐𝒃, 𝐞𝐪𝐮𝐚𝐥𝐢𝐭𝐲 𝐢𝐟 𝒃𝟒 = 𝟏 𝐚𝐧𝐝 𝒃 ≥ 𝟎 ⇔ 𝒃
𝟐 𝟐 𝟐
= 𝟏.
𝟐
𝒂𝟐 𝒃𝟒 𝟏 𝟏 𝒂𝟐 𝟏 𝟕 𝟕 𝒂𝟐 𝟏 𝟒 𝟕 𝒂𝟐
• + + = (𝟐. + 𝒃𝟒
+ 𝟒. ) ≥ . √ ( ) . 𝒃𝟒 . ( ) = , 𝐞𝐪𝐮𝐚𝐥𝐢𝐭𝐲 𝐢𝐟
𝟐𝒃𝟐 𝟒 𝒂 𝟒 𝒃𝟐 𝒂 𝟒 𝒃𝟐 𝒂 𝟒 𝒃𝟐
𝟏
= 𝒃𝟒 = .
𝒂
𝐅𝐫𝐨𝐦 𝐭𝐡𝐞𝐬𝐞 𝐫𝐞𝐬𝐮𝐥𝐭𝐬, 𝐰𝐞 𝐡𝐚𝐯𝐞 𝑳𝑯𝑺(∗) ≥ 𝟎 𝐰𝐢𝐭𝐡 𝐞𝐪𝐮𝐚𝐥𝐢𝐭𝐲 𝐢𝐟 𝒂 = 𝒃 = 𝟏 ⇔ (𝒙 − 𝝀)𝟐
= 𝒙𝟐 − 𝟐𝝀 − 𝟏 = 𝟏

58 RMM-ABSTRACT ALGEBRA MARATHON 601-700


www.ssmrmh.ro
𝟐 𝟐
( 𝝀 + 𝟏) 𝟐
𝐖𝐞 𝐡𝐚𝐯𝐞, (𝒙 − 𝝀) = 𝒙 − 𝟐𝝀 − 𝟏 ⇔ 𝒙 = 𝐚𝐧𝐝 (𝒙 − 𝝀)𝟐 = 𝟏 ⇔ 𝒙 = 𝝀 ± 𝟏,
𝟐𝝀
( 𝝀 + 𝟏) 𝟐 ( 𝝀 + 𝟏) 𝟐
𝐈𝐟 = 𝝀 + 𝟏 𝐭𝐡𝐞𝐧 𝝀 = 𝟏 𝐚𝐧𝐝 𝒙 = 𝟐, 𝒊𝒇 = 𝝀 − 𝟏 ⇔ 𝝀𝟐 − 𝟒𝝀 − 𝟏
𝟐𝝀 𝟐𝝀
= 𝟎 𝐚𝐧𝐝 𝝀 ≥ 𝟏, 𝐭𝐡𝐞𝐧 𝝀 = 𝟐 + √𝟓 𝐚𝐧𝐝 𝒙 = 𝟏 + √𝟓.
𝐓𝐡𝐞𝐫𝐞𝐟𝐨𝐫𝐞,
𝑰𝒇 𝝀 = 𝟏 𝐭𝐡𝐞𝐧 𝐭𝐡𝐞 𝐞𝐪𝐮𝐚𝐭𝐢𝐨𝐧 𝐚𝐝𝐦𝐢𝐭𝐬 𝐨𝐧𝐥𝐲 𝐨𝐧𝐞 𝐬𝐨𝐥𝐮𝐭𝐢𝐨𝐧 𝒙 = 𝟐.
{𝑰𝒇 𝝀 = 𝟐 + √𝟓 𝐭𝐡𝐞𝐧 𝐭𝐡𝐞 𝐞𝐪𝐮𝐚𝐭𝐢𝐨𝐧 𝐚𝐝𝐦𝐢𝐭𝐬 𝐨𝐧𝐥𝐲 𝐨𝐧𝐞 𝐬𝐨𝐥𝐮𝐭𝐢𝐨𝐧 𝒙 = 𝟏 + √𝟓.
𝑰𝒇 𝝀 ≠ 𝟏 𝐚𝐧𝐝 𝟐 + √𝟓 𝐭𝐡𝐞𝐧 𝐭𝐡𝐞 𝐠𝐢𝐞𝐧 𝐞𝐪𝐮𝐚𝐭𝐢𝐨𝐧 𝐡𝐚𝐬 𝐧𝐨 𝐬𝐨𝐥𝐮𝐭𝐢𝐨𝐧.
659. If 𝑨 ∈ 𝑴𝒏 (ℂ), 𝒏 ∈ ℕ, 𝒏 ≥ 𝟐 then:
𝒓𝒂𝒏𝒌 (𝑨𝟓 ) + 𝒓𝒂𝒏𝒌 (𝑨) ≥ 𝒓𝒂𝒏𝒌 (𝑨𝟒 ) + 𝒓𝒂𝒏𝒌 (𝑨𝟐 )
Proposed by Daniel Sitaru – Romania
Solution 1 by Hikmat Mammadov-Azerbaijan
If 𝑨 ∈ 𝑴𝒏 (ℂ) → 𝒏 ∈ ℕ → 𝒏 ≥ 𝟐
Then 𝒓𝒂𝒏𝒌 (𝑨𝟓 ) + 𝒓𝒂𝒏𝒌 (𝑨) ≥ 𝒓𝒂𝒏𝒌 (𝑨𝟒 ) + 𝒓𝒂𝒏𝒌 (𝑨𝟐 )
Note: 𝑹(𝑨) → range / column space of 𝑨 and 𝑵(𝑨) → null spacd / kernel of 𝑨
𝒓𝒂𝒏𝒌 {𝑷𝑸} = 𝒓𝒂𝒏𝒌 (𝑸) − 𝐝𝐢𝐦{𝑹(𝑸) ∩ 𝑵(𝑷)}
Then: 𝒓𝒂𝒏𝒌 {𝑿𝒀𝒁} = 𝒓𝒂𝒏𝒌 {𝒀𝒁} − 𝐝𝐢𝐦{𝑹(𝒀𝒁) ∩ 𝑵(𝒙)} (1)
Also: 𝒓𝒂𝒏𝒌 {𝑿𝒀} = 𝒓𝒂𝒏𝒌 {𝒀} − 𝐝𝐢𝐦{𝑹(𝒀) ∩ 𝑵(𝑿)} (2)
If vector 𝜶 ∈ 𝑹(𝒀𝒁) there exists vector 𝜷 such that 𝒀𝒁𝜷 = 𝜶
⇒ 𝒀(𝒁𝜷) = 𝜶 ⇒ 𝜶 ∈ 𝑹(𝒀) ⇒ 𝑹(𝒀𝒁) ⊆ 𝑹(𝒀) ⇒ 𝑹(𝒀𝒁) ∩ 𝑵(𝑿) ⊆ 𝑹(𝒀) ∩ 𝑵(𝑿)
𝐝𝐢𝐦{𝑹(𝒀𝒁) ∩ 𝑵(𝑿)} ≤ 𝐝𝐢𝐦{𝑹(𝒀) ∩ 𝑵(𝑿)} − 𝐝𝐢𝐦{𝑹(𝒀𝒁) ∩ 𝑵(𝑿)} ≥ − 𝐝𝐢𝐦{𝑹(𝒀) ∩ 𝑵(𝑿)} (3)

From (1) and (3) → 𝒓𝒂𝒏𝒌 {𝒀𝑿𝒁} ≥ 𝒓𝒂𝒏𝒌 {𝒀𝒁} − 𝐝𝐢𝐦{𝑹(𝒀) ∩ 𝑵(𝑿)}
(2) → 𝒓𝒂𝒏𝒌 {𝑿𝒀} = −𝒓𝒂𝒏𝒌 (𝒀) + 𝐝𝐢𝐦{𝑹(𝒀) ∩ 𝑵(𝑿)}
Adding: 𝒓𝒂𝒏𝒌 {𝑿𝒀𝒁} − 𝒓𝒂𝒏𝒌 {𝑿𝒀} ≥ 𝒓𝒂𝒏𝒌 {𝒀𝒁} − 𝒓𝒂𝒏𝒌 {𝒀}
⇒ 𝒓𝒂𝒏𝒌 {𝑿𝒀𝒁} ≥ 𝒓𝒂𝒏𝒌 {𝒀𝒁} + 𝒓𝒂𝒏𝒌 {𝑿𝒀} − 𝒓𝒂𝒏𝒌 {𝒀}
Let → 𝒁 = 𝑨𝟑 , 𝒀 = 𝑨 and 𝒁 = 𝑨
Then → 𝒓𝒂𝒏𝒌{𝑨𝟓 } ≥ 𝒓𝒂𝒏𝒌 {𝑨𝟒 } + 𝒓𝒂𝒏𝒌 {𝑨𝟐 } − 𝒓𝒂𝒏𝒌 {𝒀}
Therefore ⇒ 𝒓𝒂𝒏𝒌 {𝑨𝟓 } + 𝒓𝒂𝒏𝒌 {𝑨} ≥ 𝒓𝒂𝒏𝒌 {𝑨𝟒 } + 𝒓𝒂𝒏𝒌 {𝑨𝟐 }

59 RMM-ABSTRACT ALGEBRA MARATHON 601-700


www.ssmrmh.ro
Solution 2 by Adrian Popa-Romania
Using Frobenius inequality:
If 𝑨 ∈ 𝑴𝒏,𝒌 (ℂ), 𝑩 ∈ 𝑴𝒌,𝒑 (ℂ) and 𝑪 ∈ 𝑴𝒑,𝒏 (ℂ) ⇒
⇒ 𝒓𝒂𝒏𝒌 (𝑨𝑩) + 𝒓𝒂𝒏𝒌 (𝑩𝑪) ≤ 𝒓𝒂𝒏𝒌 (𝑩) + 𝒓𝒂𝒏𝒌 (𝑨𝑩𝑪)
Taking 𝒎 = 𝒌 = 𝒑 = 𝒏
𝑨 → 𝑨𝟑 , 𝑩 → 𝑨, 𝑪 → 𝑨
We have: 𝒓𝒂𝒏𝒌 (𝑨𝟑 ⋅ 𝑨) + 𝒓𝒂𝒏𝒌 (𝑨 ⋅ 𝑨) ≤ 𝒓𝒂𝒏𝒌 (𝑨) + 𝒓𝒂𝒏𝒌 (𝑨𝟑 ⋅ 𝑨 ⋅ 𝑨)
⇒ 𝒓𝒂𝒏𝒌 (𝑨𝟒 ) + 𝒓𝒂𝒏𝒌 (𝑨𝟐 ) ≤ 𝒓𝒂𝒏𝒌 (𝑨) + 𝒓𝒂𝒏𝒌 (𝑨𝟓 )

660. If 𝒂, 𝒃, 𝒄, 𝒅 ≥ 𝟎, 𝑨 ∈ 𝑴𝟕,𝟓 (ℂ), 𝑩 ∈ 𝑴𝟓,𝟗 (ℂ), 𝑪 ∈ 𝑴𝟕,𝟑 (ℂ), 𝑫 ∈ 𝑴𝟑,𝟗 (ℂ)


then:
𝟑𝟐(𝒂𝟐 + 𝒃𝟐 + 𝒄𝟐 + 𝒅𝟐 ) ≥ (𝒂 + 𝒃 + 𝒄 + 𝒅)𝟐 ⋅ 𝒓𝒂𝒏𝒌 (𝑨𝑩 + 𝑪𝑫)
Proposed by Daniel Sitaru – Romania
Solution by Ravi Prakash-New Delhi-India
𝑨𝑩, 𝑪𝑫 ∈ 𝑴𝟕,𝟗 (ℂ) ⇒ 𝑨𝑩 + 𝑪𝑫 ∈ 𝑴𝟕,𝟗 (ℂ)
⇒ 𝒓𝒂𝒏𝒌 (𝑨𝑩 + 𝑪𝑫) ≤ 𝐦𝐢𝐧(𝟕, 𝟗) = 𝟕
We first show that: 𝟑𝟐(𝒂𝟐 + 𝒃𝟐 + 𝒄𝟐 + 𝒅𝟐 ) ≥ 𝟕(𝒂 + 𝒃 + 𝒄 + 𝒅)𝟐
⇔ 𝟐𝟓(𝒂𝟐 + 𝒃𝟐 + 𝒄𝟐 + 𝒅𝟐 ) ≥ 𝟏𝟒[𝒂(𝒃 + 𝒄 + 𝒅) + 𝒃(𝒄 + 𝒅) + 𝒄𝒅]
⇔ 𝟕(𝒂𝟐 + 𝒃𝟐 − 𝟐𝒂𝒃) + 𝟕(𝒂𝟐 + 𝒄𝟐 − 𝟐𝒂𝒄) + 𝟕(𝒂𝟐 + 𝒄𝟐 − 𝟐𝒂𝒄) +
+𝟕(𝒃𝟐 + 𝒄𝟐 − 𝟐𝒃𝒄) + 𝟕(𝒃𝟐 + 𝒅𝟐 − 𝟐𝒃𝒅) + 𝟕(𝒄𝟐 + 𝒅𝟐 − 𝟐𝒄𝒅) +
+𝟒(𝒂𝟐 + 𝒃𝟐 + 𝒄𝟐 ) ≥ 𝟎
𝟕(𝒂 − 𝒃)𝟐 + 𝟕(𝒂 − 𝒄)𝟐 + 𝟕(𝒂 − 𝒅)𝟐 + 𝟕(𝒃 − 𝒄)𝟐 + 𝟕(𝒃 − 𝒅)𝟐 + 𝟕(𝒄 − 𝒅)𝟐 +
+𝟒(𝒂𝟐 + 𝒃𝟐 + 𝒄𝟐 + 𝒅𝟐 ) ≥ 𝟎
Now,
𝟑𝟐(𝒂𝟐 + 𝒃𝟐 + 𝒄𝟐 + 𝒅𝟐 ) ≥ 𝟕(𝒂 + 𝒃 + 𝒄 + 𝒅)𝟐 ≥ (𝒂 + 𝒃 + 𝒄 + 𝒅)𝟐 𝒓𝒂𝒏𝒌 (𝑨𝑩 + 𝑪𝑫)
𝝅
661. Solve for ∈ (𝟎, ) :
𝟐

𝒄𝒐𝒔(𝒙) + 𝒄𝒐𝒔 (𝟕𝒙) = (√𝟑 − 𝟏)(𝒄𝒐𝒔(𝟑𝒙) + 𝒄𝒐𝒔(𝟓𝒙))


Proposed by Daniel Sitaru-Romania

60 RMM-ABSTRACT ALGEBRA MARATHON 601-700


www.ssmrmh.ro
Solution 1 by Pham Duc Nam-Vietnam
𝒄𝒐𝒔(𝒙) + 𝒄𝒐𝒔(𝟕𝒙) = (√𝟑 − 𝟏)(𝒄𝒐𝒔(𝟑𝒙) + 𝒄𝒐𝒔(𝟓𝒙))

⇔ 𝟐 𝒄𝒐𝒔(𝟒𝒙) 𝒄𝒐𝒔(𝟑𝒙) = 𝟐(√𝟑 − 𝟏) 𝒄𝒐𝒔(𝟒𝒙) 𝒄𝒐𝒔(𝒙)

⇔ 𝟐 𝒄𝒐𝒔(𝟒𝒙) [𝒄𝒐𝒔(𝟑𝒙) − (√𝟑 − 𝟏) 𝒄𝒐𝒔(𝒙)] = 𝟎

⇔ 𝟐 𝒄𝒐𝒔(𝟒𝒙) [𝟒 𝒄𝒐𝒔𝟑 (𝒙) − 𝒄𝒐𝒔(𝒙) (𝟐 + √𝟑)] = 𝟎


𝝅 𝝅
+𝒌𝑥=
𝒄𝒐𝒔(𝟒𝒙) = 𝟎 𝟖 𝟒
𝝅
( )
⇔ [𝒄𝒐𝒔 𝒙 = 𝟎 ⇔ 𝑥 = + 𝒌𝝅 (𝒌 ∈ ℤ)
𝟐( ) 𝟐
4 𝒄𝒐𝒔 𝒙 − (𝟐 + √𝟑) = 𝟎 𝝅
[𝑥 = ± + 𝒌𝝅
𝟏𝟐
𝝅 𝝅 𝝅 𝟑𝝅
𝒙 ∈ (𝟎, ) ⇒ 𝒙 = , ,
𝟐 𝟖 𝟏𝟐 𝟖
Solution 2 by Tapas Das-India
𝒄𝒐𝒔(𝒙) + 𝒄𝒐𝒔(𝟕𝒙) = (√𝟑 − 𝟏)(𝒄𝒐𝒔(𝟑𝒙) + 𝒄𝒐𝒔(𝟓𝒙))

𝟐𝒄𝒐𝒔𝟒𝒙𝒄𝒐𝒔𝟑𝒙 = (√𝟑 − 𝟏) ∙ 𝟐𝒄𝒐𝒔𝟒𝒙𝒄𝒐𝒔𝒙


𝒄𝒐𝒔𝟒𝒙(𝒄𝒐𝒔𝟑𝒙 − (√𝟑 − 𝟏)𝒄𝒐𝒔𝒙) = 𝟎
𝒄𝒐𝒔𝟒𝒙 = 𝟎 ⟹ 𝟒𝒙 = ±𝒂𝒓𝒄𝒄𝒐𝒔𝟎 + 𝟐𝒏𝝅, 𝒏 ∈ ℤ
𝟏 𝟏 𝝅 𝝅 𝟑𝝅
𝒙 = ± 𝒂𝒓𝒄𝒄𝒐𝒔𝟎 + 𝒏𝝅, 𝒏 ∈ ℤ, 𝒙 ∈ (𝟎, ) ⟹ 𝒙 = , 𝒙 =
𝟒 𝟐 𝟐 𝟖 𝟖
𝒄𝒐𝒔𝟑𝒙 − (√𝟑 − 𝟏)𝒄𝒐𝒔𝒙 = 𝟎 ⟹ 𝒄𝒐𝒔𝒙(𝟒𝒄𝒐𝒔𝟐 𝒙 − 𝟑 − √𝟑 + 𝟏) = 𝟎

𝟐 + √𝟑 𝝅
𝟒𝒄𝒐𝒔𝟐 𝒙 − √𝟑 − 𝟐 = 𝟎 ⟹ 𝒄𝒐𝒔𝒙 = √ ⟹𝒙=
𝟒 𝟏𝟐

662. Solve for real numbers:

(𝟐𝒙 + 𝟑𝒙 ) ⋅ √𝟔𝟏−𝒙 = 𝟓
Proposed by Daniel Sitaru – Romania
Solution 1 by Israfilov Murad-Azerbaijan
(𝟐𝒙 + 𝟑𝒙 ) ⋅ √𝟔𝟏−𝒙 = 𝟓
𝟏−𝒙
( 𝟐𝒙 + 𝟑𝒙 ) ⋅ 𝟔 𝟐 =𝟓

61 RMM-ABSTRACT ALGEBRA MARATHON 601-700


www.ssmrmh.ro
𝟏−𝒙
{𝒊𝒇 𝟐 = 𝒕
𝒙 = 𝟏 − 𝟐𝒕
(𝟐𝟏−𝟐𝒕 + 𝟑𝟏−𝟐𝒕 ) ⋅ 𝟔𝒕 = 𝟓, 𝟑𝒕 ⋅ 𝟐𝟏−𝒕 + 𝟐𝒕 ⋅ 𝟑𝟏−𝒕 = 𝟓| × 𝟔𝒕

𝟐𝒕 𝟐𝒕 𝒕
𝟑 𝒕 𝟐 𝒕
𝟐⋅𝟑 +𝟑⋅𝟐 =𝟔⋅𝟔 , 𝟐 ⋅ ( ) + 𝟑( ) − 𝟓 = 𝟎
𝟐 𝟑
𝟑 𝒕
𝒊𝒇 ( ) = 𝒖
𝟐
𝟐 𝒕 𝟏
{ ( 𝟑) = 𝒖
𝟑
𝟐𝒖 + − 𝟓 = 𝟎, 𝟐𝒖𝟐 − 𝟓𝒖 + 𝟑 = 𝟎, (𝟐𝒖 − 𝟑)(𝒖 − 𝟏) = 𝟎
𝟒
𝟐 𝟑 𝒕 𝟐
𝒖𝟏 = , 𝒖𝟐 = 𝟏 ⇒ ( ) = , 𝒕 = −𝟏
𝟑 𝟐 𝟑
𝟑 𝒕
( ) =𝟏
𝟐
𝒕=𝟎

𝒕=𝟏
𝒙 = 𝟏 − 𝟐𝒕, 𝒙𝟏 = 𝟏 − 𝟐(−𝟏) ≠ 𝟑, 𝒙𝟐 = 𝟏 − 𝟐 ⋅ 𝟎 ≠ 𝟏
𝒙𝟑 = 𝟏 − 𝟐 ⋅ 𝟏 = −𝟏
𝟏 − 𝒙 > 0, 𝑥<1
Answer: −𝟏, 𝟏
Solution 2 by Toubal Fethi-Algeria
Let: 𝒂 = 𝟐𝒙 , 𝒃 = 𝟑𝒙 where: 𝒂, 𝒃 > 0, ∀𝑥 ∈ ℝ

𝟔
(𝒂 + 𝒃)√ = 𝟓, (𝒂 + 𝒃)√𝟔 = 𝟓√𝒂𝒃
𝒂𝒃

𝟔(𝒂𝟐 + 𝒃𝟐 + 𝟐𝒂𝒃) = 𝟐𝟓𝒂𝒃, 𝟔𝒂𝟐 + 𝟔𝒃𝟐 − 𝟏𝟑𝒂𝒃 = 𝟎


(𝟐𝒂 − 𝟑𝒃)(𝟑𝒂 − 𝟐𝒃) = 𝟎
𝟐𝒂 − 𝟑𝒃 = 𝟎 ⇒ 𝟐𝒙+𝟏 = 𝟑𝒙+𝟏 ⇒ 𝒙 = −𝟏

𝒙
𝟑 𝒙 𝟑 𝒙
𝟑𝒂 − 𝟐𝒃 = 𝟎 ⇒ 𝟑 ⋅ 𝟐 = 𝟐 ⋅ 𝟑 ⇒ ( ) = ⇒ 𝒙 = 𝟏
𝟐 𝟐
62 RMM-ABSTRACT ALGEBRA MARATHON 601-700
www.ssmrmh.ro
663. Solve for real numbers :
𝒙𝟔 + 𝒙𝟒 − 𝟒𝒙𝟑 + 𝟓𝒙𝟐 − 𝟓𝒙 + 𝟓 𝟒
𝟐𝒙
+ 𝒍𝒐𝒈𝟒 ( 𝒙 − 𝟒𝒙 + 𝟓 ) = 𝟐 𝒙𝟐 +𝟏
𝒙𝟐 + 𝟏
Proposed by Neculai Stanciu-Romania
Solution by Pham Duc Nam-Vietnam

𝒙 𝒙
𝒙𝟒 − 𝟒𝒙 + 𝟓 − + 𝒍𝒐𝒈𝟒 (𝒙𝟒
− 𝟒𝒙 + 𝟓) = 𝟒 𝒙𝟐 +𝟏
𝒙𝟐 + 𝟏
𝒙 𝒙
⇔ 𝒙𝟒 − 𝟒𝒙 + 𝟓 + 𝒍𝒐𝒈𝟒 (𝒙𝟒 − 𝟒𝒙 + 𝟓) = 𝟒𝒙𝟐 +𝟏 + 𝟐
𝒙 +𝟏
𝒙 𝒙
𝟒
⇔ 𝟒𝒍𝒐𝒈𝟒 (𝒙 −𝟒𝒙+𝟓) + 𝒍𝒐𝒈𝟒 (𝒙𝟒 − 𝟒𝒙 + 𝟓) = 𝟒𝒙𝟐+𝟏 + 𝟐
𝒙 +𝟏
∗ Consider: 𝒇(𝒕) = 𝟒𝒕 + 𝒕 ⇒ 𝒇′(𝒕) = 𝟒𝒕 𝒍𝒐𝒈(𝟒) + 𝟏 > 0∀𝒕 ∈ ℝ
⇒ 𝒇(𝒕) is always increasing ⇒
𝒙 𝒙
𝒇(𝒍𝒐𝒈𝟒 (𝒙𝟒 − 𝟒𝒙 + 𝟓)) = 𝒇 ( 𝟐 ) ⇔ 𝒍𝒐𝒈𝟒 (𝒙𝟒 − 𝟒𝒙 + 𝟓) = 𝟐
𝒙 +𝟏 𝒙 +𝟏
𝟒(𝒙𝟑 − 𝟏)
∗ Consider: 𝒈(𝒙) = 𝒍𝒐𝒈𝟒 (𝒙𝟒 − 𝟒𝒙 + 𝟓) ⇒ 𝒈′(𝒙) = ⇒ 𝒈′(𝒙) = 𝟎 ⇔ 𝒙 = 𝟏
(𝒙𝟒 − 𝟒𝒙 + 𝟓) 𝒍𝒐𝒈(𝟒)
⇒ 𝒈(𝒙) is decreasing in (−∞, 𝟏),and increasing in (𝟏, ∞)
𝟏 𝟏
At: 𝒙 = 𝟏 ⇒ 𝒈(𝟏) = ⇒ 𝒈(𝒙) ≥ (𝟏)
𝟐 𝟐
𝒙 𝟏 − 𝒙𝟐 𝟏
∗ Consider: 𝒉(𝒙) = 𝟐
⇒ 𝒉′ (𝒙) = 𝟐 𝟐
⇒ 𝒉′(𝒙) = 𝟎 ⇔ 𝒙 = ±𝟏 ⇒ 𝒉(±𝟏) = ±
𝒙 +𝟏 (𝒙 + 𝟏) 𝟐
⇒ 𝒉(𝒙) is decreasing in (−∞, −𝟏), (𝟏, ∞),and increasing in (−𝟏, 𝟏)
𝟏 𝟏
At: 𝒙 = ±𝟏 ⇒ 𝒉(±𝟏) = ± ⇒ 𝒉(𝒙) ≤ (𝟐)
𝟐 𝟐
𝒙
∗ 𝐅𝐫𝐨𝐦 ∶ (𝟏), (𝟐) ⇒ 𝒈(𝒙) = 𝒉(𝒙) ⇔ 𝒍𝒐𝒈𝟒 (𝒙𝟒 − 𝟒𝒙 + 𝟓) = ⇔𝒙=𝟏
𝒙𝟐 +𝟏
⇒ 𝒙 = 𝟏 is solution for the given equation
664. Solve for complex numbers:
(𝟏 − 𝒙𝒚 + 𝒚𝒛 − 𝒙𝒛)𝟐 + (𝒙 + 𝒚 + 𝒛 − 𝒙𝒚𝒛)𝟐 = 𝟎
Proposed by Daniel Sitaru – Romania
Solution by Sakthi Vel-India
(𝟏 − 𝒙𝒚 − 𝒚𝒛 − 𝒙𝒛)𝟐 + (𝒙 + 𝒚 + 𝒛 − 𝒙𝒚𝒛)𝟐 = 𝟎

63 RMM-ABSTRACT ALGEBRA MARATHON 601-700


www.ssmrmh.ro
𝒙 𝒚 + 𝟐𝒙 𝒚𝒛 + 𝒙 𝒛 + 𝟐𝒙𝒚𝟐 𝒛 + 𝟐𝒙𝒚𝒛𝟐 − 𝟐𝒙𝒚 − 𝟐𝒚𝒛 − 𝟐𝒙𝒛 +
𝟐 𝟐 𝟐 𝟐 𝟐

(+𝒚𝟐 𝒛𝟐 + 𝟏 + 𝒙𝟐 𝒚𝟐 𝒛𝟐 − 𝟐𝒙𝟐 𝒚𝒛 + 𝒙𝟐 − 𝟐𝒙𝒚𝟐 𝒛 − 𝟐𝒙𝒚𝒛𝟐 + 𝟐𝒙𝒚 +) = 𝟎


+𝟐𝒚𝒛 + 𝟐𝒙𝒛 + 𝒚𝟐 + 𝒛𝟐
(𝒙𝒚)𝟐 + (𝒚𝒛)𝟐 + (𝒙𝒛)𝟐 + 𝟐𝒙𝟐 𝒚𝒛 + 𝟐𝒙𝒚𝟐 𝒛 + 𝟐𝒙𝒚𝒛𝟐 − 𝟐𝒙𝒚 − 𝟐𝒚𝒛 − 𝟐𝒙𝒛 + 𝟏 +
+(𝒙𝒚𝒛)𝟐 − 𝟐𝒙𝟐 𝒚𝒛 − 𝟐𝒙𝒚𝟐 𝒛 − 𝟐𝒙𝒚𝒛𝟐 + 𝒙𝟐 + 𝒚𝟐 + 𝒛𝟐 + 𝟐𝒙𝒚 + 𝟐𝒚𝒛 + 𝟐𝒛𝒙 = 𝟎
(𝒙𝒚𝒛)𝟐 + (𝒙𝒚)𝟐 + (𝒚𝒛)𝟐 + (𝒙𝒛)𝟐 + 𝒙𝟐 + 𝒚𝟐 + 𝒛𝟐 + 𝟏 = 𝟎
Let 𝒙𝟐 = 𝒂; 𝒚𝟐 = 𝒃; 𝒛𝟐 = 𝒄
𝒂𝒃𝒄 + 𝒂𝒃 + 𝒃𝒄 + 𝒂𝒄 + 𝒂 + 𝒃 + 𝒄 + 𝟏 = 𝟎
𝒂(𝒃𝒄 + 𝒃 + 𝒄 + 𝟏) + (𝒃𝒄 + 𝒃𝒄 + 𝒄 + 𝟏) = 𝟎
(𝒃𝒄 + 𝒃 + 𝒄 + 𝟏)(𝒂 + 𝟏) = 𝟎
(𝒂 + 𝟏)(𝒃 + 𝟏)(𝒄 + 𝟏) = 𝟎, where 𝒂 = 𝒙𝟐 , 𝒃 = 𝒚𝟐 , 𝒄 = 𝒛𝟐
∴ The roots are 𝒙𝟐 = 𝒚𝟐 = 𝒛𝟐 = −𝟏

𝒙 = 𝒚 = 𝒛 = √−𝟏
∴𝒙=𝒚=𝒛=𝒊
𝟏 𝟎 𝟏 𝟏
665. 𝑨 = ( ),𝑩 = ( ). Solve for real numbers:
𝟏 𝟏 𝟎 𝟏
𝒙 𝟎 𝟐𝒆 − 𝟏
𝒆𝑨 ∙ ( ) + 𝒆𝑩 ∙ ( ) = ( ) , 𝒆𝑨 −exponential matrix
𝟎 𝒚 𝟐𝒆 − 𝟏
Proposed by Daniel Sitaru-Romania
Solution by Ravi Prakash-New Delhi-India
By mathematical induction:
𝟏 𝟎 𝟏 𝒏
𝑨𝒏 = ( ) , 𝑩𝒏 = ( )
𝒏 𝟏 𝟎 𝟏
∞ ∞
𝟏 𝟏 𝟏 𝟎 𝒆 𝟎
𝒆 = ∑ 𝑨𝒏 = ∑ (
𝑨
)=( ) = 𝒆𝑨
𝒏! 𝒏! 𝒏 𝟏 𝒆 𝒆
𝒏=𝟎 𝒏=𝟎
∞ ∞
𝟏 𝟏 𝟏 𝒏 𝒆 𝒆
𝒆 = ∑ 𝑩𝒏 = ∑ (
𝑩
)=( ) = 𝒆𝑩
𝒏! 𝒏! 𝟎 𝟏 𝟎 𝒆
𝒏=𝟎 𝒏=𝟎

𝒙 𝟎 𝟐𝒆 − 𝟏 𝒆 𝟎 𝒙 𝒆 𝒆 𝟎 𝟐𝒆 − 𝟏
𝒆𝑨 ∙ ( ) + 𝒆𝑩 ∙ ( ) = ( ), ( )∙( )+( )∙( )= ( )
𝟎 𝒚 𝟐𝒆 − 𝟏 𝒆 𝒆 𝟎 𝟎 𝒆 𝒚 𝟐𝒆 − 𝟏
𝒆𝒙 𝒆𝒚 𝟐𝒆 − 𝟏
( ) + (𝒆𝒚) = ( ) , 𝒆(𝒙 + 𝒚) = 𝟐𝒆 − 𝟏
𝒆𝒙 𝟐𝒆 − 𝟏

64 RMM-ABSTRACT ALGEBRA MARATHON 601-700


www.ssmrmh.ro
𝟏
𝒙+𝒚 =𝟐−
𝒆
𝒙=𝒂
Solution: {𝒚 = 𝟐 − 𝟏 − 𝒂 𝒂∈ℝ
𝒆

666. Solve for complex numbers:


𝒛 𝒊
𝒊 𝒊
|−𝒊 𝒛
𝒊 𝒊
𝒛 𝒊| = 𝟎
−𝒊 −𝒊
−𝒊 −𝒊 𝒛
−𝒊
Proposed by Daniel Sitaru-Romania
Solution 1 by George Florin Șerban-Romania

𝒛 𝒊 𝒊 𝒊
|−𝒊 𝒛 𝒊 𝒊
−𝒊 −𝒊 𝒛 𝒊| = 𝟎
−𝒊 −𝒊 −𝒊 𝒛

By Laplace theorem:
(−𝟏)𝟏+𝟐+𝟏+𝟐 ∙ | 𝒛 𝒊
|∙|
𝒛 𝒊
| + (−𝟏)𝟏+𝟐+𝟏+𝟑 ∙ |
𝒛 𝒊 −𝒊 𝒊
|∙| |+
−𝒊 𝒛 −𝒊 𝒛 −𝒊 𝒊 −𝒊 𝒛

𝒛 𝒊 −𝒊 𝒛 𝒊 𝒊 −𝒊 𝒊
+(−𝟏)𝟏+𝟐+𝟏+𝟒 ∙ | |∙| | + (−𝟏)𝟏+𝟐+𝟐+𝟑 ∙ | |∙| |+
−𝒊 𝒊 −𝒊 −𝒊 𝒛 𝒊 −𝒊 𝒛

𝒊 𝒊 −𝒊 𝒛 𝒊 𝒊 −𝒊 −𝒊
= +(−𝟏)𝟏+𝟐+𝟐+𝟒 ∙ | |∙| | + +(−𝟏)𝟏+𝟐+𝟑+𝟒 ∙ | |∙| |=𝟎
𝒛 𝒊 −𝒊 −𝒊 𝒊 𝒊 −𝒊 −𝒊

(𝒛𝟐 − 𝟏)𝟐 − (𝒊𝒛 − 𝟏)(−𝒊𝒛 − 𝟏) + (𝒊𝒛 − 𝟏)(−𝟏 + 𝒊𝒛) +

+(−𝟏 − 𝒊𝒛)(−𝒊𝒛 − 𝟏)— (𝟏 − 𝒊𝒛)(−𝟏 + 𝒊𝒛) = 𝟎

𝒛𝟒 − 𝟔𝒛𝟐 + 𝟏 = 𝟎

𝒛𝟏 = 𝟏 + √𝟐, 𝒛𝟐 = −𝟏 − √𝟐, 𝒛𝟑 = 𝟏 − √𝟐, 𝒛𝟒 = −𝟏 + √𝟐

Solution 2 by Qudrat Muhammadi-Afghanistan


𝒛 𝒊 𝒊 𝒊
|−𝒊 𝒛 𝒊 𝒊
−𝒊 −𝒊 𝒛 𝒊| = 𝟎
−𝒊 −𝒊 −𝒊 𝒛

65 RMM-ABSTRACT ALGEBRA MARATHON 601-700


www.ssmrmh.ro
𝒛 𝒊 𝒊 −𝒊 𝒊 𝒊 −𝒊 𝒛 𝒊 −𝒊 𝒛 𝒊
𝒛 |−𝒊 𝒛 𝒊 | − 𝒊 |−𝒊 𝒛 𝒊 | + 𝒊 | −𝒊 −𝒊 𝒊 | − 𝒊 | −𝒊 −𝒊 𝒛|=𝟎
−𝒊 −𝒊 𝒛 −𝒊 −𝒊 𝒛 −𝒊 −𝒊 𝒛 −𝒊 −𝒊 −𝒊
𝒛𝟒 − 𝟔𝒛𝟐 + 𝟏 = 𝟎

𝒛𝟏 = 𝟏 + √𝟐, 𝒛𝟐 = −𝟏 − √𝟐, 𝒛𝟑 = 𝟏 − √𝟐, 𝒛𝟒 = −𝟏 + √𝟐

Solution 3 by Amin Hajiyev-Azerbaijan


𝒛 𝒊 𝒊 𝒊
−𝒊 𝒛 𝒊 𝒊
| |=𝟎 𝚫=𝟎
−𝒊 −𝒊 𝒛 𝒊
−𝒊 −𝒊 −𝒊 𝒛
𝒂𝟏;𝟏 𝒂𝟏;𝟐 𝒂𝟏;𝟑
𝒂 𝒂𝟐;𝟑 𝒂𝟏;𝟐 𝒂𝟏;𝟑
Note: |𝑨| = |𝒂𝟐;𝟏 𝒂𝟐;𝟐 𝒂𝟐;𝟑 | = 𝒂𝟏;𝟏 | 𝟐;𝟐 | |
𝒂𝟑;𝟐 𝒂𝟑;𝟑 − 𝒂𝟐;𝟏 𝒂𝟑;𝟐 𝒂𝟑;𝟑 | +
𝒂𝟑;𝟏 𝒂𝟑;𝟐 𝒂𝟑;𝟑
𝒂𝟏;𝟐 𝒂𝟏;𝟑 𝒂 𝒃|
+𝒂𝟑;𝟏 |𝒂 𝒂𝟐;𝟑 | ⇔ | 𝒄 = 𝒂𝒅 − 𝒃𝒄
𝟐;𝟐 𝒅
𝒛 𝒊 𝒊 𝒊 𝒊 𝒊 𝒊 𝒊 𝒊 𝒊 𝒊 𝒊
𝚫 = 𝒛 |−𝒊 𝒛 𝒊 | + 𝒊 |−𝒊 𝒛 𝒊| − 𝒊 | 𝒛 𝒊 𝒊| + 𝒊 | 𝒛 𝒊 𝒊| =
−𝒊 −𝒊 𝒛 −𝒊 −𝒊 𝒛 −𝒊 −𝒊 𝒛 −𝒊 𝒛 𝒊
= 𝒛(𝒛𝟑 − 𝟑𝒛) − (𝒛 + 𝒊)𝟐 − (𝒛𝟐 + 𝟏) − (𝒛 − 𝒊)𝟐 =
= 𝒛𝟒 − 𝟑𝒛 − 𝒛𝟐 − 𝟐𝒊𝒛 + 𝟏 − 𝒛𝟐 − 𝟏 − 𝒛𝟐 + 𝟐𝒊𝒛 + 𝟏 =
= 𝒛𝟒 − 𝟔𝒛𝟐 + 𝟏
𝒕=𝒛𝟐 𝟔 ± 𝟒√𝟐
𝚫 = 𝟎 ⇔ 𝒛𝟒 − 𝟔𝒛𝟐 + 𝟏𝟎 ⇔ 𝒕𝟐 − 𝟔𝒕 + 𝟏 = 𝟎 𝒕 = = 𝟑 ± 𝟐√𝟐
𝟐
Answer: 𝒛𝟏 = 𝟏 − √𝟐; 𝒛𝟐 = 𝟏 − √𝟐; 𝒛𝟑 = √𝟐 − 𝟏; 𝒛𝟒 = 𝟏 + √𝟐
Solution 4 by Said Cerbah-Algeria
𝒛 𝒊 𝒊 𝒊
−𝒊 𝒛 𝒊 𝒊
We put 𝒇(𝒛) = | |; then 𝒇(𝒛) = 𝒇(−𝒛) because 𝐝𝐞𝐭(𝑨) = 𝐝𝐞𝐭( 𝒕𝑨)
−𝒊 −𝒊 𝒛 𝒊
−𝒊 −𝒊 −𝒊 𝒛
then if 𝒛 is a root then −𝒛 is a root also. We deduce that
𝒇(𝒛) = (𝒛𝟐 − 𝒛𝟐𝟎 )(𝒛𝟐 − 𝒛𝟏 ) where 𝒛𝟎 and 𝒛𝟏 are roots of 𝒇
𝒇(𝟎) = 𝒛𝟐𝟎 𝒛𝟐𝟏 . By calculating the determinant we find
𝒇(𝟎) = 𝟏

66 RMM-ABSTRACT ALGEBRA MARATHON 601-700


www.ssmrmh.ro
( ) 𝟐 𝟐
𝒇 𝒊 = (𝟏 + 𝒛𝟎 )(𝟏 + 𝒛𝟏 ) by calculating the determinant we find
𝒇(𝒊) = 𝟖 then we have (𝟏 + 𝒛𝟐𝟎 )(𝟏 + 𝒛𝟐𝟏 ) = 𝟖
⇒ 𝟏 + 𝒛𝟐𝟎 + 𝒛𝟐𝟏 + 𝒛𝟐𝟎 𝒛𝟐𝟏 = 𝟖 ⇒ 𝒛𝟐𝟏 + 𝒛𝟐𝟐 = 𝟔 then 𝒛𝟐𝟎 and 𝒛𝟐𝟏 are solution of 𝒙𝟐 − 𝟔𝒙 + 𝟏 = 𝟎
then we find: 𝒙 = 𝟑 + 𝟐√𝟐 or 𝒙 = 𝟑 − 𝟐√𝟐
𝟐 𝟐
𝟑 + 𝟐√𝟐 = (𝟏 + √𝟐) and 𝟑 − 𝟐√𝟐 = (𝟏 − √𝟐)
then the solutions of 𝒇(𝒛) = 𝟎 are:
{𝟏 + √𝟐, 𝟏 − √𝟐, −(𝟏 + √𝟐); −(𝟏 − √𝟐)}

Solution 5 by Ravi Prakash-New Delhi-India


𝒛 𝒊 𝒊 𝒊
−𝒊 𝒛 𝒊 𝒊
Let 𝚫 = | |
−𝒊 −𝒊 𝒛 𝒊
−𝒊 −𝒊 −𝒊 𝒛
Write
𝒛+𝒊−𝒊 𝒊 𝒊 𝒊
𝟎−𝒊 𝒛 𝒊 𝒊
𝚫=| | = (𝒛 + 𝒊)𝚫𝟏 − 𝒊𝚫𝟐
𝟎−𝒊 𝒛 𝒊 𝒊
𝟎−𝒊 −𝒊 −𝒊 𝒛
Where
𝟏 𝒊 𝒊 𝒊 𝟏 𝒊 𝒊 𝒊
𝟎 𝒛 𝒊 𝒊 𝟏 𝒛 𝒊 𝒊
𝚫𝟏 = | | and 𝚫𝟐 = | |
𝟎 −𝒊 𝒛 𝒊 𝟏 −𝒊 𝒛 𝒊
𝟎 −𝒊 −𝒊 𝒛 𝟏 −𝒊 −𝒊 𝒛
Now
𝒛 𝒊 𝒊
𝚫𝟏 = |−𝒊 𝒛 𝒊 | = 𝒛𝟑 − 𝒊𝟑 + 𝒊𝟑 − 𝟑𝒛 = 𝒛𝟑 − 𝟑𝒛
−𝒊 −𝒊 𝒛
𝟏 𝒊 𝒊 𝒊
𝒛−𝒊 𝟎 𝟎
𝟎 𝒛−𝒊 𝟎 𝟎
𝚫𝟐 = | | = | −𝟐𝒊 𝒛 − 𝒊 𝟎 | = (𝒛 − 𝒊 )𝟑 =
𝟎 −𝟐𝒊 𝒛 − 𝒊 𝟎
−𝟐𝒊 −𝟐𝒊 𝒛 − 𝒊
𝟎 −𝟐𝒊 −𝟐𝒊 𝒛 − 𝒊
= 𝒛𝟑 − 𝟑𝒊𝒛𝟐 − 𝟑𝒛 + 𝒊
Thus,
𝚫 = (𝒛 + 𝒊)(𝒛𝟑 − 𝟑𝒛) − 𝒊(𝒛𝟑 − 𝟑𝒊𝒛𝟐 − 𝟑𝒛 + 𝒊) = 𝒛𝟒 − 𝟔𝒛𝟐 + 𝟏

67 RMM-ABSTRACT ALGEBRA MARATHON 601-700


www.ssmrmh.ro
𝟏 𝟐
Now, 𝒛 − 𝟔𝒛 + 𝟏 = 𝟎 ⇒ 𝒛𝟐 = 𝟐 (𝟔 ± √𝟑𝟐) = 𝟑 ± 𝟐√𝟐 = (√𝟐 ± 𝟏)
𝟒 𝟐

⇒ 𝒛 = √𝟐 + 𝟏, −√𝟐 − 𝟏, √𝟐 − 𝟏, −√𝟐 + 𝟏
667. Solve for real numbers:
√𝟑(𝟐𝒙 + 𝟐𝒚 + 𝟏) = 𝟐√𝒙𝟐 + 𝒚𝟐 + 𝟏 + √𝟐𝒙𝟐 + 𝟏 + +𝟐√𝒙𝟐 + 𝟐𝒚𝟐

Proposed by Daniel Sitaru-Romania


Solution by Adrian Popa-Romania
𝑪𝑩𝑺
𝟐 𝟐
(𝟏 + 𝟏 + 𝟏)(𝒙 + 𝒚 + 𝟏) ≥ ⏞ (𝒙 + 𝒚 + 𝟏)𝟐 ⟹
( 𝒙 + 𝒚 + 𝟏) 𝟐
𝟐(𝒙 + 𝒚 + 𝟏)
⟹ 𝒙 𝟐 + 𝒚𝟐 + 𝟏 ≥ ⟹ 𝟐√𝒙𝟐 + 𝒚𝟐 + 𝟏 ≥
𝟑 √𝟑
𝑪𝑩𝑺
(𝟐𝒙 + 𝟏)𝟐
⏞ (𝒙 + 𝒙 + 𝟏)𝟐 ⟹ 𝟐𝒙𝟐 + 𝟏 ≥
(𝟏 + 𝟏 + 𝟏)(𝒙𝟐 + 𝒙𝟐 + 𝟏) ≥ ⟹
𝟑
𝟐𝒙 + 𝟏
⟹ √𝟐𝒙𝟐 + 𝟏 ≥
√𝟑
𝑪𝑩𝑺
(𝒙 + 𝟐𝒚)𝟐
⏞ (𝒙 + 𝒚 + 𝒚)𝟐 ⟹ 𝒙𝟐 + 𝟐𝒚𝟐 ≥
(𝟏 + 𝟏 + 𝟏)(𝒙𝟐 + 𝒚𝟐 + 𝒚𝟐 ) ≥ ⟹
𝟑
𝟐(𝒙 + 𝟐𝒚)
⟹ 𝟐√𝒙𝟐 + 𝟐𝒚𝟐 ≥
√𝟑

𝟐√𝒙𝟐 + 𝒚𝟐 + 𝟏 + √𝟐𝒙𝟐 + 𝟏 + 𝟐√𝒙𝟐 + 𝟐𝒚𝟐 ≥


𝟐(𝒙 + 𝒚 + 𝟏) 𝟐𝒙 + 𝟏 𝟐(𝒙 + 𝟐𝒚)
≥ + +
√𝟑 √𝟑 √𝟑

𝟔𝒙 + 𝟔𝒚 + 𝟑
√𝟑(𝟐𝒙 + 𝟐𝒚 + 𝟏) ≥
√𝟑
𝟔𝒙 + 𝟔𝒚 + 𝟑 ≥ 𝟔𝒙 + 𝟔𝒚 + 𝟑
Equality holds for:
𝒙 𝒚 𝟏
= = ⟹𝒙=𝒚=𝟏
𝟏 𝟏 𝟏
668.
𝝅 𝝅 𝒙
𝒇: [−𝟐, ∞) → [− , ) , 𝒇(𝒙) = 𝒔𝒊𝒏−𝟏 ( )
𝟐 𝟒 √𝟐(𝒙𝟐 + 𝟐𝒙 + 𝟐)
Solve for real numbers:
𝒇−𝟏 (𝒙) + 𝟏 = 𝟎
Proposed by Daniel Sitaru-Romania
68 RMM-ABSTRACT ALGEBRA MARATHON 601-700
www.ssmrmh.ro
Solution by Bedri Hajrizi-Mitrovica-Kosovo
𝟏
𝒇′ (𝒙) = (𝒙+𝟏)𝟐 +𝟏 > 0 ⟹ 𝑓 −increasing ⟹ 𝒇 −injectif
𝝅 𝝅
𝐥𝐢𝐦 𝒇(𝒙) = − , 𝐥𝐢𝐦 𝒇(𝒙) =
𝒙→−𝟐 𝟐 𝒙→∞ 𝟒
𝒙>−2
𝝅 𝝅
𝒇 −continuous, 𝒇 −has Darboux property⟹ 𝒇([−𝟐, ∞)) = [− 𝟐 , 𝟒 ) ⟹
𝝅 𝝅
Im𝒇 = [− 𝟐 , 𝟒 ) ⟹ 𝒇 −surjectif
𝒚
Denote: 𝒚 = 𝒇−𝟏 (𝒙) ⟹ 𝒙 = 𝒂𝒓𝒄𝒔𝒊𝒏 ⟹
√𝟐(𝒚𝟐 +𝟐𝒚+𝟐)

⟹ 𝒔𝒊𝒏𝟐 𝒙 ∙ (𝟐𝒚𝟐 + 𝟒𝒚 + 𝟒) = 𝒚𝟐

(𝟐𝒔𝒊𝒏𝟐 𝒙 − 𝟏)𝒚𝟐 + 𝟒𝒔𝒊𝒏𝟐 𝒙 ∙ 𝒚 + 𝟒𝒔𝒊𝒏𝟐 𝒙 = 𝟎


−𝟐𝒔𝒊𝒏𝟐 𝒙 + 𝟐√𝒔𝒊𝒏𝟐 𝒙 − 𝒔𝒊𝒏𝟒 𝒙
𝒚 = 𝒇−𝟏 (𝒙) =
𝟐𝒔𝒊𝒏𝟐 𝒙 − 𝟏
−𝟐𝒔𝒊𝒏𝟐 𝒙 + 𝟐√𝒔𝒊𝒏𝟐 𝒙 − 𝒔𝒊𝒏𝟒 𝒙
= −𝟏, 𝟐√𝒔𝒊𝒏𝟐 𝒙 − 𝒔𝒊𝒏𝟒 𝒙 = 𝟏
𝟐𝒔𝒊𝒏𝟐 𝒙 − 𝟏
𝟏 𝝅
𝟒(𝒔𝒊𝒏𝟐 𝒙 − 𝒔𝒊𝒏𝟒 𝒙) = 𝟏, (𝟐𝒔𝒊𝒏𝟐 𝒙 − 𝟏)𝟐 = 𝟎, 𝒔𝒊𝒏𝟐 𝒙 = ⟹ 𝒙 = −
𝟐 𝟒
669. Solve for real numbers:
𝟓𝒙 + 𝟏𝟑 = 𝟐𝒚 + 𝟑𝒛
{𝟐𝒙𝟐 + 𝒚𝟐 + 𝒛𝟐 = 𝟐𝟕
𝒙(𝒚 + 𝒛) = 𝟕
Proposed by Daniel Sitaru-Romania
Solution by Adrian Popa-Romania
𝟓𝒙 + 𝟏𝟑 = 𝟐𝒚 + 𝟑𝒛 (𝟏)
{𝟐𝒙𝟐 + 𝒚𝟐 + 𝒛𝟐 = 𝟐𝟕 (𝟐)
𝒙( 𝒚 + 𝒛) = 𝟕 (𝟑)

(𝟐) − 𝟐 ∙ (𝟑): 𝟐𝒙𝟐 + 𝒚𝟐 + 𝒛𝟐 − (𝟐𝒙𝒚 + 𝟐𝒙𝒛) = 𝟐𝟕 − 𝟏𝟒


(𝒙 − 𝒚)𝟐 + (𝒙 − 𝒛)𝟐 = 𝟏𝟑

(𝟏): 𝟓𝒙 + 𝟏𝟑 = 𝟐𝒚 + 𝟑𝒛 ⟺ 𝟐(𝒙 − 𝒚) + 𝟑(𝒙 − 𝒛) = −𝟏𝟑

𝒙−𝒚= 𝒂 𝟐 𝟐 𝒂 = −𝟐 𝒙 − 𝒚 = −𝟐
Denote: { ⟹ { 𝒂 + 𝒃 = 𝟏𝟑 ⟹ { ⟹{ ⟹
𝒙−𝒛 =𝒃 𝟐𝒂 + 𝟑𝒃 = −𝟏𝟑 𝒃 = −𝟑 𝒙 − 𝒛 = −𝟑
𝒚 =𝒙+𝟐
⟹{
𝒛= 𝒙+𝟑

𝒙(𝒚 + 𝒛) = 𝟕 ⟹ 𝒙(𝒙 + 𝟐 + 𝒙 + 𝟑) = 𝟕 ⟹ 𝟐𝒙𝟐 + 𝟓𝒙 − 𝟕 = 𝟎 ⟹

69 RMM-ABSTRACT ALGEBRA MARATHON 601-700


www.ssmrmh.ro
𝟕 𝟑 𝟏
𝒙 𝟏 = − 𝒚 𝟏 = − 𝒛 𝟏 = −
⟹{ 𝟐⟹{ 𝟐⟹{ 𝟐
𝒙𝟐 = 𝟏 𝒚𝟐 = 𝟑 𝒛𝟐 = 𝟒
670.
𝒂, 𝒃 > 0, 𝑎 ≠ 𝑏, 𝒇𝒎 (𝒙) = 𝒂 + (𝒙 − 𝒂)𝒎 , 𝒈𝒎 (𝒙) = 𝒃 + (𝒙 − 𝒃)𝒎
𝒇𝒎 : (𝒂, ∞) → (𝒂, ∞), 𝒈𝒎 : (𝒃, ∞) → (𝒃, ∞)
𝑮 = {𝒇𝒎 /𝒎 ∈ ℝ∗}, 𝑯 = {𝒈𝒎 /𝒎 ∈ ℝ∗ }.
Prove that: (𝑮,∘) ≅ (𝑯,∘).
Proposed by Daniel Sitaru-Romania
Solution by Ravi Prakash-New Delhi-India

Define : 𝑮 → 𝑯, 𝚽(𝒇𝒎 ) = 𝒈𝒎 , 𝒎 ∈ ℝ .
For 𝒎, 𝒏 ∈ ℝ, 𝒙 > 𝑎 ∶
(𝒇𝒎 ∘ 𝒇𝒏 )(𝒙) = 𝒇𝒎 (𝒇𝒏 (𝒙)) = 𝒇𝒎 (𝒂 + (𝒙 − 𝒂)𝒏 ) =
= 𝒂 + (𝒂 + (𝒙 − 𝒂)𝒏 − 𝒂)𝒎 = 𝒂 + (𝒙 − 𝒂)𝒎𝒏 = 𝒇𝒎𝒏 (𝒙)
𝚽 −morphism
𝚽(𝒇𝒎 ∘ 𝒇𝒏 ) = 𝚽(𝒇𝒎𝒏 ) = 𝒈𝒎𝒏 = 𝒈𝒎 ∘ 𝒈𝒏 = 𝚽(𝒇𝒎 ) ∘ 𝚽(𝒇𝒏 )
𝚽 −injectif
𝚽(𝒇𝒎 ) = 𝚽(𝒇𝒏 ) ⟹ 𝒈𝒎 = 𝒈𝒏 ⟹ 𝒈𝒎 (𝒙) = 𝒈𝒏 (𝒙), 𝒙 > 𝑏 ⟹
𝒃 + (𝒙 − 𝒃)𝒎 = 𝒃 + (𝒙 − 𝒃)𝒏 ⟹ 𝒎 = 𝒏
𝚽 −surjectif
For 𝒈𝒎 ∈ 𝑯, ∃𝒇𝒎 ∈ 𝑮 such that: 𝚽(𝒇𝒎 ) = 𝒈𝒎 , 𝒎 ∈ ℝ
𝚽 −isomorphism ⟹ (𝑮,∘) ≅ (𝑯,∘)

671. Find 𝜶 ∈ ℝ such that: 𝒙𝟏𝟔 𝟏𝟔 𝟏𝟔


𝟏 + 𝒙𝟐 + 𝒙𝟑 = 𝟗𝟎 where 𝒙𝟏 , 𝒙𝟐 , 𝒙𝟑 −are the
roots of the equation: 𝒙𝟑 + 𝜶𝒙 + 𝟏 = 𝟎.
Proposed by Daniel Sitaru-Romania
Solution by Adrian Popa-Romania
By Viete:
𝑺𝟏 = ∑ 𝒙𝟏 = 𝟎, 𝑺𝟐 = ∑ 𝒙𝟏 𝒙𝟐 = 𝜶, 𝑺𝟑 = 𝒙𝟏 𝒙𝟐 𝒙𝟑 = −𝟏
𝒄𝒚𝒄 𝒄𝒚𝒄

70 RMM-ABSTRACT ALGEBRA MARATHON 601-700


www.ssmrmh.ro
∑ 𝒙𝟐𝟏 = 𝑺𝟐𝟏 − 𝟐𝑺𝟐 = −𝟐𝜶, ∑ 𝒙𝟑𝟏 − 𝜶 ∑ 𝒙𝟏 + 𝟑 = 𝟎, ∑ 𝒙𝟑𝟏 = −𝟑
𝒄𝒚𝒄 𝒄𝒚𝒄 𝒄𝒚𝒄 𝒄𝒚𝒄

∑ 𝒙𝟒𝟏 + 𝜶 ∑ 𝒙𝟐𝟏 + ∑ 𝒙𝟏 = 𝟎 ⟹ ∑ 𝒙𝟒𝟏 = 𝟐𝜶𝟐 ⟹


𝒄𝒚𝒄 𝒄𝒚𝒄 𝒄𝒚𝒄 𝒄𝒚𝒄
𝟐

(∑ 𝒙𝟒𝟏 ) = 𝟒𝜶𝟒 ⟹ ∑ 𝒙𝟖𝟏 + 𝟐 ∑ 𝒙𝟒𝟏 𝒙𝟒𝟐 = 𝟒𝜶𝟒 ⟹


𝒄𝒚𝒄 𝒄𝒚𝒄 𝒄𝒚𝒄

∑ 𝒙𝟖𝟏 = −𝟐 ∑ 𝒙𝟒𝟏 𝒙𝟒𝟐 + 𝟒𝜶𝟒


𝒄𝒚𝒄 𝒄𝒚𝒄
𝟐

𝑺𝟐𝟐 = ∑ 𝒙𝟐𝟏 𝒙𝟐𝟐 + 𝟐𝑺𝟏 𝑺𝟑 ⟹ ∑ 𝒙𝟐𝟏 𝒙𝟐𝟐 = 𝜶𝟐 ⟹ (∑ 𝒙𝟐𝟏 𝒙𝟐𝟐 ) = (𝜶𝟐 )𝟐


𝒄𝒚𝒄 𝒄𝒚𝒄 𝒄𝒚𝒄

∑ 𝒙𝟒𝟏 𝒙𝟒𝟐 + 𝟐𝑺𝟐𝟑 ∑ 𝒙𝟐𝟏 = 𝜶𝟒 ⟹ ∑ 𝒙𝟒𝟏 𝒙𝟒𝟐 = 𝜶𝟒 + 𝟒𝜶


𝒄𝒚𝒄 𝒄𝒚𝒄 𝒄𝒚𝒄

∑ 𝒙𝟖𝟏 = −𝟐(𝜶𝟒 + 𝟒𝜶) + 𝟒𝜶𝟒 ⟹ ∑ 𝒙𝟖𝟏 = 𝟐𝜶𝟒 − 𝟖𝜶


𝒄𝒚𝒄 𝒄𝒚𝒄
𝟐

(∑ 𝒙𝟖𝟏 ) = (𝟐𝜶𝟒 − 𝟖𝜶)𝟐 ⟹ ∑ 𝒙𝟏𝟔 𝟖 𝟖 𝟖 𝟓


𝟏 + 𝟐 ∑ 𝒙𝟏 𝒙𝟐 = 𝟒𝜶 − 𝟑𝟐𝜶 + 𝟔𝟒𝜶
𝟐

𝒄𝒚𝒄 𝒄𝒚𝒄 𝒄𝒚𝒄

𝟐 ∑ 𝒙𝟖𝟏 𝒙𝟖𝟐 = 𝟒𝜶𝟖 − 𝟑𝟐𝜶𝟓 + 𝟔𝟒𝜶𝟐 − 𝟗𝟎


𝒄𝒚𝒄

∑ 𝒙𝟖𝟏 𝒙𝟖𝟐 = 𝟐𝜶𝟖 − 𝟏𝟔𝜶𝟓 + 𝟑𝟐𝜶𝟐 − 𝟒𝟓 (𝟏)


𝒄𝒚𝒄
𝟐

(∑ 𝒙𝟒𝟏 𝒙𝟒𝟐 ) = (𝜶𝟒 + 𝟒𝜶)𝟐


𝒄𝒚𝒄

∑ 𝒙𝟖𝟏 𝒙𝟖𝟐 + 𝟐𝑺𝟒𝟑 ∑ 𝒙𝟒𝟏 𝒙𝟒𝟐 = 𝜶𝟖 + 𝟖𝜶𝟓 + 𝟏𝟔𝜶𝟐


𝒄𝒚𝒄 𝒄𝒚𝒄

∑ 𝒙𝟖𝟏 𝒙𝟖𝟐 + 𝟐(𝜶𝟒 + 𝟒𝜶) = 𝜶𝟖 + 𝟖𝜶𝟓 + 𝟏𝟔𝜶𝟐 (𝟐)


𝒄𝒚𝒄
By (𝟏), (𝟐):
𝟐𝜶𝟖 − 𝟏𝟔𝜶𝟓 + 𝟑𝟐𝜶𝟐 − 𝟒𝟓 = 𝜶𝟖 + 𝟖𝜶𝟓 + 𝟏𝟔𝜶𝟐 − 𝟐(𝜶𝟒 + 𝟒𝜶)
𝜶𝟖 − 𝟐𝟒𝜶𝟓 + 𝟐𝜶𝟒 + 𝟏𝟔𝜶𝟐 + 𝟖𝜶 − 𝟒𝟓 = 𝟎 ⟹ 𝜶 = −𝟏

672. 𝑨, 𝑩, 𝑪 ∈ 𝑴𝟐 (ℝ), 𝒅𝒆𝒕𝑨 = 𝟏, 𝑨𝟐 = 𝑩𝑪, 𝑩𝟐 = 𝑪𝑨, 𝑪𝟐 = 𝑨𝑩.

71 RMM-ABSTRACT ALGEBRA MARATHON 601-700


www.ssmrmh.ro
Solve for complex numbers:
𝒙 + 𝟔(𝒙𝒅𝒆𝒕𝑩)𝟑 − 𝟏𝟒(𝒙𝒅𝒆𝒕𝑪) 𝟐 + 𝟔𝒙 + 𝟏 = 𝟎
𝟒

Proposed by Daniel Sitaru-Romania


Solution 1 by Florentin Vișescu-Romania
(𝒅𝒆𝒕𝑩)𝟑 = 𝒅𝒆𝒕𝑩𝟑 = 𝒅𝒆𝒕(𝑩𝟐 ∙ 𝑩) = 𝒅𝒆𝒕(𝑪𝑨𝑩) = 𝒅𝒆𝒕𝑪 ∙ 𝒅𝒆𝒕𝑨 ∙ 𝒅𝒆𝒕𝑩 =
= 𝒅𝒆𝒕𝑪 ∙ 𝟏 ∙ 𝒅𝒆𝒕𝑩 = 𝒅𝒆𝒕𝑩 ∙ 𝒅𝒆𝒕𝑪 = 𝒅𝒆𝒕(𝑩𝑪) = 𝒅𝒆𝒕𝑨𝟐 = (𝒅𝒆𝒕𝑨)𝟐 = 𝟏
(𝒅𝒆𝒕𝑪)𝟐 = 𝒅𝒆𝒕𝑪𝟐 = 𝒅𝒆𝒕(𝑨𝑩) = 𝒅𝒆𝒕𝑨 ∙ 𝒅𝒆𝒕𝑩 = 𝟏 ∙ 𝒅𝒆𝒕𝑩 = 𝒅𝒆𝒕𝑩
𝒅𝒆𝒕𝑨𝟐 = 𝒅𝒆𝒕(𝑩𝑪) ⟹ (𝒅𝒆𝒕𝑨)𝟐 = 𝒅𝒆𝒕𝑩 ∙ 𝒅𝒆𝒕𝑪 ⟹
⟹ 𝟏 = (𝒅𝒆𝒕𝑪)𝟐 ∙ 𝒅𝒆𝒕𝑪 ⟹ (𝒅𝒆𝒕𝑪)𝟑 = 𝟏 ⟹ 𝒅𝒆𝒕𝑪 = 𝟏
𝒙𝟒 + 𝟔(𝒙𝒅𝒆𝒕𝑩)𝟑 − 𝟏𝟒(𝒙𝒅𝒆𝒕𝑪)𝟐 + 𝟔𝒙 + 𝟏 = 𝟎
𝒙 + 𝟔𝒙𝟑 ∙ (𝒅𝒆𝒕𝑩)𝟑 − 𝟏𝟒𝒙𝟐 ∙ (𝒅𝒆𝒕𝑪)𝟐 + 𝟔𝒙 + 𝟏 = 𝟎
𝟒

𝒙𝟒 + 𝟔𝒙𝟑 − 𝟏𝟒𝒙𝟐 + 𝟔𝒙 + 𝟏 = 𝟎
𝟏 𝟏
𝒙𝟐 + 𝟐 + 𝟔 (𝒙 + ) − 𝟏𝟒 = 𝟎
𝒙 𝒙
𝟏 𝟏
𝒙 + = 𝒕, 𝒙𝟐 + 𝟐 = 𝒕𝟐 − 𝟐
𝒙 𝒙
𝒙𝟏 = 𝟏
𝒕 = −𝟖 𝒙𝟐 = 𝟏
𝒕𝟐 − 𝟐 + 𝟔𝒕 − 𝟏𝟒 = 𝟎, 𝒕𝟐 + 𝟔𝒕 − 𝟏𝟔 = 𝟎 ⟹ { 𝟏 ⟹
𝒕𝟐 = 𝟐 𝒙𝟑 = −𝟒 + √𝟏𝟓
{𝒙𝟒 = −𝟒 − √𝟏𝟓
Solution 2 by Adrian Popa-Romania
𝒅𝒆𝒕𝑨 = 𝟏, 𝑨𝟐 = 𝑩𝑪, 𝒅𝒆𝒕(𝑨𝟐 ) = 𝒅𝒆𝒕(𝑩𝑪), 𝟏 = 𝒙𝒚
𝒙 = 𝒅𝒆𝒕𝑩, 𝒚 = 𝒅𝒆𝒕𝑪
𝑩 = 𝑪𝑨 ⟹ 𝒅𝒆𝒕𝑩𝟐 = 𝒅𝒆𝒕𝑪𝑨 ⟹ 𝒙𝟐 = 𝒚 ⟹ 𝒙𝟑 = 𝒙𝒚 ⟹
𝟐

⟹ 𝒙𝟑 = 𝟏 ⟹ 𝒙 = 𝟏 ⟹ 𝒚 = 𝟏
𝒙 + 𝟔(𝒙𝒅𝒆𝒕𝑩)𝟑 − 𝟏𝟒(𝒙𝒅𝒆𝒕𝑪)𝟐 + 𝟔𝒙 + 𝟏 = 𝟎
𝟒

𝒙𝟒 + 𝟔𝒙𝟑 ∙ (𝒅𝒆𝒕𝑩)𝟑 − 𝟏𝟒𝒙𝟐 ∙ (𝒅𝒆𝒕𝑪)𝟐 + 𝟔𝒙 + 𝟏 = 𝟎


𝒙𝟒 + 𝟔𝒙𝟑 − 𝟏𝟒𝒙𝟐 + 𝟔𝒙 + 𝟏 = 𝟎
𝟏 𝟏
𝒙𝟐 + 𝟐 + 𝟔 (𝒙 + ) − 𝟏𝟒 = 𝟎
𝒙 𝒙
𝟏 𝟏
𝒙 + = 𝒕, 𝒙𝟐 + 𝟐 = 𝒕𝟐 − 𝟐
𝒙 𝒙
𝒙𝟏 = 𝟏
𝒕 = −𝟖 𝒙𝟐 = 𝟏
𝒕𝟐 − 𝟐 + 𝟔𝒕 − 𝟏𝟒 = 𝟎, 𝒕𝟐 + 𝟔𝒕 − 𝟏𝟔 = 𝟎 ⟹ { 𝟏 ⟹
𝒕𝟐 = 𝟐 𝒙𝟑 = −𝟒 + √𝟏𝟓
{𝒙𝟒 = −𝟒 − √𝟏𝟓
673. ⌊⋅⌋: floor function.Let: 𝒏 ≥ 𝟏,𝒂𝒏 = ⌊𝐥𝐧(𝒏)⌋.Prove that:
There exist infinitely many 𝒏 such that: 𝒂𝒏 divides 𝒏
Proposed by Toubal Fethi-Algeria

72 RMM-ABSTRACT ALGEBRA MARATHON 601-700


www.ssmrmh.ro
Solution by Michael Sterghiou-Greece
𝒏≥𝟏
Let , 𝒂 = [𝐥𝐧(𝒏)]
𝒏∈ℕ 𝒏
{[𝒙] integer part of 𝒙}. Prove that there are infinitely many 𝒏 such that 𝒂𝒏 divides 𝒏
Let 𝒎 ∈ ℕ: 𝒆𝒏 ≤ 𝒎 < 𝒆𝒏+𝟏 . Then 𝒏 ≤ 𝐥𝐧 𝒎 < 𝑛 + 1 and [𝒎] = 𝒏, ∀𝒎 ∈ [𝒆𝒏 , 𝒆𝒏+𝟏 ). But
>1
𝒆𝒏+𝟏 − 𝒆𝒏 = 𝒆𝒏 (𝒆 − 𝟏) > 𝒆𝒏 ≥ 𝒏 + 𝟏
By the pigeonhole principle (as in the interval [𝒆𝒏 , 𝒆𝒏+𝟏 ) exist at least 𝒏 + 𝟏 distinct
naturals) one of them will be divided by 𝒏. Therefore ∀𝒏 ∈ ℕ, 𝒏 ≥ 𝟏 ∃𝒂𝒏 = 𝒎 ∈
[𝒆𝒏 , 𝒆𝒏+𝟏 ):
𝒂𝒏 divides 𝒏. Infinitely many 𝒏 → infinitely many 𝒂𝒏 . Done!
674.
𝒙𝟏 𝒚𝟏 𝒙𝒏 𝒚𝒏
𝑨𝟏 (𝒙) = (−𝒚 𝒙𝟏 ) , 𝑨𝒏( )
𝒙 = ( −𝒚𝒏 𝒙𝒏 ) , 𝒏 ∈ ℕ − {𝟎},
𝟏

𝒙 𝒙
𝒙𝟏 = √ , 𝒚𝟏 = √ , 𝒙 ≥ 𝟎. Find:
𝟑(𝒙𝟐 +𝟏) 𝟒(𝒙𝟐 +𝟏)

𝛀 = 𝐥𝐢𝐦 (𝒙𝒏 + 𝒚𝒏 )
𝒏→∞

Proposed by Daniel Sitaru – Romania


Solution 1 by Ravi Prakash-New Delhi-India
𝒙 𝒙
Let 𝒙𝟏 = √𝟑(𝒙𝟐 +𝟏) = 𝒓 𝐜𝐨𝐬 𝜽 ; 𝒚𝟏 = √𝟒(𝒙𝟐 +𝟏) = 𝒓 𝐬𝐢𝐧 𝜽, where 𝒓 ≥ 𝟎

𝒙 𝟏 𝟏 𝟕 𝒙 𝟕
𝒙𝟐𝟏 + 𝒚𝟐𝟏 = 𝒓𝟐 = ( + ) = ⋅ ≤
𝒙𝟐 + 𝟏 𝟑 𝟒 𝟏𝟐 𝒙𝟐 + 𝟏 𝟐𝟒
𝒙 𝟏
as 𝒙𝟐 +𝟏 ≤ 𝟐

𝟕
∴𝟎≤𝒓≤√
𝟐𝟒
𝒓 𝐜𝐨𝐬 𝜽 𝒓 𝐬𝐢𝐧 𝜽 𝐜𝐨𝐬 𝜽 𝐬𝐢𝐧 𝜽
Now, 𝑨 = ( ) = 𝒓( )
−𝒓 𝐬𝐢𝐧 𝜽 𝒓 𝐜𝐨𝐬 𝜽 − 𝐬𝐢𝐧 𝜽 𝐜𝐨𝐬 𝜽
𝐜𝐨𝐬(𝒏𝜽) 𝐬𝐢𝐧(𝒏𝜽)
𝑨𝒏 = 𝒓𝒏 ( )
− 𝐬𝐢𝐧(𝒏𝜽) 𝐜𝐨𝐬(𝒏𝜽)
Thus, 𝒙𝒏 = 𝒓𝒏 𝐜𝐨𝐬(𝒏𝜽) ; 𝒚𝒏 = 𝒓𝒏 𝐬𝐢𝐧(𝒏𝜽) ⇒

73 RMM-ABSTRACT ALGEBRA MARATHON 601-700


www.ssmrmh.ro
⇒ 𝒙𝒏 + 𝒚𝒏 = 𝒓𝒏 [𝐜𝐨𝐬(𝒏𝜽) + 𝐬𝐢𝐧(𝒏𝜽)]
𝟏 𝟏
But − ≤ 𝐜𝐨𝐬(𝒏𝜽) + 𝐬𝐢𝐧(𝒏𝜽) ≤
√𝟐 √𝟐

𝟏 𝟏
∴− 𝒓 𝒏 ≤ 𝒙 𝒏 + 𝒚𝒏 ≤ 𝒓𝒏
√𝟐 √𝟐
𝟕
As 𝟎 ≤ 𝒓 ≤ √𝟐𝟒 < 1, we get

𝐥𝐢𝐦 𝒓𝒏 = 𝟎
𝒏→∞

Thus, 𝐥𝐢𝐦 (𝒙𝒏 + 𝒚𝒏 ) = 𝟎


𝒏→∞

Solution 2 by Adrian Popa-Romania

𝒙 𝒙
√ √ 𝟏 𝟏
𝟐
𝟑( 𝒙 + 𝟏 ) 𝟐
𝟒( 𝒙 + 𝟏) 𝒙 √𝟑 𝟐 𝒙
𝑨′ (𝒙) = =√ ⋅ =√ ⋅𝑩
𝒙𝟐 + 𝟏 𝟏 𝟏 𝒙𝟐 + 𝟏
𝒙 𝒙 −
−√ √ 𝟐 √𝟑)
𝟐
𝟒( 𝒙 + 𝟏 ) 𝟐
𝟑( 𝒙 + 𝟏) ⏟
(
( ) 𝑩
𝐜𝐨𝐬 𝜶 𝐬𝐢𝐧 𝜶
It is known that: the matrices of the form 𝑨 = 𝒓 ( ) have the form: 𝑨𝒏 =
− 𝐬𝐢𝐧 𝜶 𝐜𝐨𝐬 𝜶
𝐜𝐨𝐬 𝒏 𝜶 𝐬𝐢𝐧 𝒏 𝜶
𝒏( ) (it can be proved by mathematical induction)
− 𝐬𝐢𝐧 𝒏 𝜶 𝐜𝐨𝐬 𝒏𝜶
𝟏 𝟏
√𝟑 𝟐
𝟏 𝟏
√𝟕 √𝟕
𝟐 𝟏 𝟏 𝟕 𝟕 𝟏𝟐 𝟏𝟐
𝐝𝐞𝐭 𝑩 = || √𝟑 |= + = ⇒𝑩=√
𝟏 𝟏 | 𝟑 𝟒 𝟏𝟐 𝟏𝟐 𝟏 𝟏
− −𝟐
𝟐 √𝟑 √𝟑
√𝟕 √𝟕
( 𝟏𝟐 𝟏𝟐 )
𝟐
𝟏 𝟏𝟐 𝟏 𝟏𝟐 𝟏 𝟏𝟐 𝟏 𝟏𝟐 𝟏 𝟏
We notice that ( ⋅ √ 𝟕 ) + (𝟐 ⋅ √ 𝟕 ) = 𝟑 ⋅ +𝟒⋅ = 𝟏𝟐 (𝟐𝟏 + 𝟐𝟖) =
√𝟑 𝟕 𝟕

𝟏
𝟒+𝟑 𝟕 √𝟑
= 𝟏𝟐 ⋅ 𝟕⋅𝟏𝟐 = 𝟏𝟐 ⋅ 𝟕⋅𝟏𝟐 = 𝟏 ⇒ we can denote 𝟕
= 𝐜𝐨𝐬 𝜶 and

𝟏𝟐

74 RMM-ABSTRACT ALGEBRA MARATHON 601-700


www.ssmrmh.ro
𝟏
𝟐 = 𝐬𝐢𝐧 𝜶 ⇒ 𝑩 = √ 𝟕 ( 𝐜𝐨𝐬 𝜶 𝐬𝐢𝐧 𝜶
)⇒
𝟏𝟐 − 𝐬𝐢𝐧 𝜶 𝐜𝐨𝐬 𝜶
√𝟕
𝟏𝟐
𝒙 𝟕 𝐜𝐨𝐬 𝜶 𝐬𝐢𝐧 𝜶 𝟏 𝟏𝟐 𝟐
⇒ 𝑨 = √𝒙𝟐 +𝟏 ⋅ √𝟏𝟐 ( ), where 𝜶 = 𝐚𝐫𝐜𝐜𝐨𝐬 ⋅ √ 𝟕 = 𝐚𝐫𝐜𝐜𝐨𝐬
− 𝐬𝐢𝐧 𝜶 𝐜𝐨𝐬 𝜶 √𝟑 √𝟕
𝒏
𝟕𝒙
𝒏 𝒙𝒏 = (√ 𝟐 ) ⋅ 𝐜𝐨𝐬 𝒏𝜶
𝒏 (𝒙 + 𝟏) ⋅ 𝟏𝟐
𝒙 𝟕 𝐜𝐨𝐬 𝒏𝜶 𝐬𝐢𝐧 𝒏𝜶
𝑨𝒏 = √ 𝟐 ⋅√ ⋅( )⇒ 𝒏
𝒙 +𝟏 𝟏𝟐 − 𝐬𝐢𝐧 𝒏𝜶 𝐜𝐨𝐬 𝒏𝜶
𝟕𝒙
𝒚𝒏 = (√ 𝟐 ) ⋅ 𝐬𝐢𝐧 𝒏𝜶
(𝒙 + 𝟏) ⋅ 𝟏𝟐
{
𝟕𝒙 𝟕 𝟏
Now: √ ≤√ ⋅ √ = (because 𝒙𝟐 + 𝟏 ≥ 𝟐𝒙)
𝟏𝟐(𝒙𝟑 +𝟏) 𝟏𝟐 𝟐

𝒏 𝒏
√𝟕 √𝟕 √𝟕
= 𝟐√𝟔 < 1 ⇒ 0 < |𝒙𝒏 + 𝒚𝒏 | ≤ |𝒙𝒏 | + |𝒚𝒏 | ≤ (𝟐√𝟔) + (𝟐√𝟔) → 𝟎 when 𝒏 → ∞

So 𝛀 = 𝟎
675.
𝟏
𝑨𝒏 = (𝒂𝒊𝒋) , 𝒂𝒊𝒋 = , 𝒏 ∈ ℕ, 𝒏 ≥ 𝟐
𝟏≤𝒊,𝒋≤𝒏 𝒊+𝒋−𝟏
Find:
𝐝𝐞𝐭(𝑨𝒏+𝟏 )
𝛀 = 𝐥𝐢𝐦 ( )
𝒏→∞ (𝒏!)𝟑 ⋅ 𝐝𝐞𝐭(𝑨𝒏 )
Proposed by Daniel Sitaru – Romania
Solution 1 by Le Thu-Vietnam
Such matrices are known as Cauchy matrices. The special case where it satisfies
𝒙𝒊 − 𝒚𝒊 = 𝒊 + 𝒋 − 𝟏 is called a Hilbert matrix - 𝓗𝒏
𝐝𝐞𝐭(𝓗𝒏+𝟏) (𝒏!)𝟒
Note that = (𝟐𝒏)!(𝟐𝒏+𝟏)!
𝐝𝐞𝐭(𝓗𝒏 )

See: https://math.stackexchange.com/questions/4167272/
hilbert_matrices_determinant_recurrence_relation
𝒏!
Therefore, 𝛀 = 𝐥𝐢𝐦 (𝟐𝒏)!(𝟐𝒏+𝟏)! = 𝟎

75 RMM-ABSTRACT ALGEBRA MARATHON 601-700


www.ssmrmh.ro
Solution 2 by Hikmat Mammadov-Azerbaijan
𝟏
𝑨𝒏 = (𝒂𝒊𝒋 )𝟏≤𝒊,𝒋≤𝒏 → 𝒂𝒊𝒋 = 𝒊+𝒋−𝟏 → 𝒏 ∈ ℕ → 𝒏 ≥ 𝟐 → find:

𝐝𝐞𝐭(𝑨𝒏+𝟏 )
𝛀 = 𝐥𝐢𝐦 ( )
𝒏→∞ (𝒏!)𝟐 ⋅ 𝐝𝐞𝐭(𝑨𝒏 )
−𝟏
Consider Cauchy matrix with: 𝒂𝒊𝒋 = (𝒙𝒊 − 𝒚𝒋 )
Subtract the last row from 𝒏 − 𝟏 preceding rows.
𝟏 𝟏 𝒙 𝒏 − 𝒚𝒋 − 𝒙 𝒊 + 𝒚𝒋 𝒙𝒏 − 𝒙𝒊
− = =
𝒙𝒊 − 𝒚𝒊 𝒙𝒏 − 𝒚𝒋 (𝒙𝒊 − 𝒚𝒋 )(𝒙𝒏 − 𝒚𝒋 ) (𝒙𝒊 − 𝒚𝒋 )(𝒙𝒏 − 𝒚𝒋 )
𝑭𝒓𝒐𝒎 𝒓𝒐𝒘 𝒊 𝒕𝒂𝒌𝒆 𝒐𝒖𝒕: 𝒙𝒏 − 𝒙𝒋 → 𝒊 ∈ (𝟏, 𝟐, … , 𝒏 − 𝟏)
{ 𝟏 ⇒ 𝑨𝒏 now becomes →
𝑭𝒓𝒐𝒎 𝒄𝒐𝒍𝒖𝒎𝒏 𝒋 𝒕𝒂𝒌𝒆 𝒐𝒖𝒕: 𝒙 → 𝒋 ∈ (𝟏, 𝟐, … , 𝒏)
𝒏 −𝒚𝒋

𝟏
𝑨𝒏−𝟏
𝒙 𝟏 − 𝒚𝒏

→ 𝟏
𝒙𝒏−𝟏 − 𝒚𝒏
[ 𝟏 𝟏 ]
In the temaining determinant, subtract the last column for all the preceding columns.
𝟏 𝟏 𝒙 𝒊 − 𝒚𝒏 − 𝒙 𝒊 + 𝒚𝒊 𝒚𝒋 − 𝒚𝒏
− = =
𝒙𝒊 − 𝒚𝒋 𝒙𝒊 − 𝒚𝒏 (𝒙𝒊 − 𝒚𝒊 )(𝒙𝒊 − 𝒚𝒏 ) (𝒙𝒊 − 𝒚𝒋 )(𝒙𝒊 − 𝒚𝒏 )
𝟏
𝑭𝒓𝒐𝒎 𝒓𝒐𝒘 𝒊 𝒕𝒂𝒌𝒆 𝒐𝒖𝒕: → 𝒊 ∈ (𝟏, 𝟐, … , 𝒏 − 𝟏)
{ 𝒙𝒊 −𝒚𝒏 ⇒ 𝑨𝒏 becomes →
𝑭𝒓𝒐𝒎 𝒓𝒐𝒘 𝒋 𝒕𝒂𝒌𝒆 𝒐𝒖𝒕: 𝒚𝒋 − 𝒚𝒏 → 𝒋 ∈ (𝟏, 𝟐, … , 𝒏 − 𝟏)
𝑨𝒏−𝟏 𝟏
[ …]
𝟎𝟎 … 𝟎𝟏
𝟏 𝒙𝒏 −𝒙𝒌 𝒚 −𝒚
Thus → 𝐝𝐞𝐭{𝑨𝒏 } = 𝐝𝐞𝐭{𝑨𝒏−𝟏 } 𝒙 ∏𝒏−𝟏
𝒌=𝟏 ( ⋅ 𝒙𝒌−𝒚𝒏 )
𝒏 −𝒚𝒏 𝒙𝒏 −𝒚𝒌 𝒌 𝒏

If: 𝒙𝒊 = 𝒊 − 𝟏, 𝒚𝒋 = −𝒋 → 𝒙𝒏 − 𝒙𝒌 = 𝒏 − 𝒌
𝒙 𝒏 − 𝒚𝒌 = 𝒏 + 𝒌 − 𝟏
𝟏 𝟏
= → 𝒚𝒌 − 𝒚𝒏 = 𝒏 − 𝒌
𝒙𝒏 − 𝒚𝒏 𝟐𝒏 − 𝟏
𝒙 𝒌 − 𝒚𝒏 = 𝒏 + 𝒌 − 𝟏

76 RMM-ABSTRACT ALGEBRA MARATHON 601-700


www.ssmrmh.ro
𝒙 𝒏 − 𝒙 𝒌 𝒚𝒌 − 𝒚𝒏 (𝒏 − 𝒌 )𝟐
⇒ ⋅ =
𝒙 𝒏 − 𝒚𝒌 𝒙 𝒌 − 𝒚𝒏 ( 𝒏 + 𝒌 − 𝟏) 𝟐
𝒏−𝟏

∏ ( 𝒏 − 𝒌 ) = ( 𝒏 − 𝟏) !
𝒌−𝟏

and
𝒏−𝟏
(𝟐𝒏 − 𝟐)!
∏(𝒏 + 𝒌 − 𝟏) =
( 𝒏 − 𝟏) !
𝒌=𝟏
𝟒
((𝒏 − 𝟏)!)
𝐝𝐞𝐭{𝑨𝒏 } = 𝐝𝐞𝐭{𝑨𝒏−𝟏 }
(𝟐𝒏 − 𝟏)! (𝟐𝒏 − 𝟐)!
𝐝𝐞𝐭{𝑨𝒏+𝟏 } (𝒏!)𝟒 𝐝𝐞𝐭{𝑨𝒏+𝟏 } 𝒏!
⇒ = ⇒ 𝟑
=
𝐝𝐞𝐭{𝑨𝒏 } (𝟐𝒏 + 𝟏)! (𝟐𝒏)! (𝒏!) 𝐝𝐞𝐭{𝑨𝒏 } (𝟐𝒏 + 𝟏)! (𝟐𝒏)!
⇒𝛀=𝟎
676. 𝑨 ∈ 𝑴𝟓 (ℤ), 𝑨𝟒 = 𝑶𝟓 . Solve for complex numbers:
𝟐 +𝑰 )
𝒛 − 𝒊 𝟓 𝐝𝐞𝐭(𝑨 𝟓
( ) = −𝟏
𝒛+𝒊
Proposed by Daniel Sitaru – Romania
Solution 1 by George Florin Șerban-Romania
𝑨𝟒 = 𝑶𝟓 ⇒ 𝒎𝑨 |𝒙𝟒 T. Frobenius
𝒑𝒂 (𝒙) = (−𝟏)𝟓 𝐝𝐞𝐭(𝑨 − 𝒙𝑰𝟓 ) = 𝒙𝟓 , T. Casley – Hamilton
𝒑𝑨 (𝑨) = 𝑶𝟓 ⇒ 𝑨𝟓 = 𝑶𝟓 , 𝐝𝐞𝐭(𝑨 − 𝒙𝑰𝟓 ) = 𝒙𝟓
𝐝𝐞𝐭(𝑨𝟐 + 𝑰𝟓 ) = 𝐝𝐞𝐭(𝑨 + 𝒊𝑰𝟓 ) ⋅ 𝐝𝐞𝐭(𝑨 − 𝒊𝑰𝟓 ) =
= −(−𝒊)𝟓 ⋅ [−𝒊𝟓 ] = (−𝒊𝟐 )𝟓 = 𝟏𝟓 = 𝟏
𝒛−𝒊 𝟓
⇒( ) = −𝟏, −𝟏 = 𝐜𝐨𝐬 𝝅 + 𝒊 𝐬𝐢𝐧 𝝅
𝒛+𝒊
𝒛−𝒊 𝟓 𝒛−𝒊 𝝅 + 𝟐𝒌𝝅 𝝅 + 𝟐𝒌𝝅
= √𝐜𝐨𝐬 𝝅 + 𝒊 𝐬𝐢𝐧 𝝅 , = 𝐜𝐨𝐬 + 𝒊 𝐬𝐢𝐧 =𝜶⇒
𝒛+𝒊 𝒛+𝒊 𝟓 𝟓
⇒ 𝒛 − 𝒊 = 𝒛𝜶 + 𝒊𝜶, 𝒌 = 𝟎, 𝟒
𝒛 − 𝒛𝜶 = 𝒊 + 𝒊𝜶, 𝒛(𝟏 − 𝜶) = 𝒊(𝟏 + 𝜶)

77 RMM-ABSTRACT ALGEBRA MARATHON 601-700


www.ssmrmh.ro
𝝅 + 𝟐𝒌𝝅 𝝅 + 𝟐𝒌𝝅
𝒊(𝟏 + 𝜶) 𝒊 (𝟏 + 𝐜𝐨𝐬 𝟓 + 𝒊 𝐬𝐢𝐧 𝟓 )
𝒛= =
𝟏−𝜶 𝝅 + 𝟐𝒌𝝅 𝝅 + 𝟐𝒌𝝅
𝟏 − 𝐜𝐨𝐬 − 𝒊 𝐬𝐢𝐧
𝟓 𝟓
𝝅 + 𝟐𝒌𝝅 𝝅 + 𝟐𝒌𝝅 𝝅 + 𝟐𝒌𝝅
𝒊 (𝟐 𝐜𝐨𝐬 𝟐 𝟏𝟎 + 𝟐𝒊 𝐬𝐢𝐧 𝟏𝟎 ⋅ 𝐜𝐨𝐬 𝟏𝟎 )
𝒛=
𝝅 + 𝟐𝒌𝝅 𝝅 + 𝟐𝒌𝝅 𝝅 + 𝟐𝒌𝝅
𝟐 𝐬𝐢𝐧𝟐 − 𝟐𝒊 𝐬𝐢𝐧 𝟏𝟎 𝐜𝐨𝐬 𝟏𝟎
𝟏𝟎
𝝅 + 𝟐𝒌𝝅 𝝅 + 𝟐𝒌𝝅 𝝅 + 𝟐𝒌𝝅
𝒊 𝐜𝐨𝐬 𝟏𝟎 ⋅ (𝐜𝐨𝐬 𝟏𝟎 + 𝒊 𝐬𝐢𝐧 𝟏𝟎 )
𝒛=
𝝅 + 𝟐𝒌𝝅 𝝅 + 𝟐𝒌𝝅 𝝅 + 𝟐𝒌𝝅
−𝒊 𝐬𝐢𝐧 𝟏𝟎 ⋅ (𝐜𝐨𝐬 𝟏𝟎 + 𝒊 𝐬𝐢𝐧 𝟏𝟎 )
𝝅 + 𝟐𝒌𝝅
𝒛𝒌 = − 𝐜𝐨𝐭 , 𝒌 ∈ {𝟎, 𝟏, 𝟐, 𝟑, 𝟒}
𝟏𝟎
(𝒛 − 𝒊 )𝟓 = − (𝒛 + 𝒊 )𝟓
𝑪𝟎𝟓 𝒛𝟓 − 𝑪𝟏𝟓 𝒛𝟒 𝒊 + 𝑪𝟐𝟓 𝒛𝟑 𝒊𝟐 − 𝑪𝟑𝟓 𝒛𝟐 𝒊𝟑 + 𝑪𝟒𝟓 𝒛𝒊𝟒 − 𝑪𝟓𝟓 𝒊𝟓 =
= −𝑪𝟎𝟓 𝒛𝟓 − 𝑪𝟏𝟓 𝒛𝟒 𝒊 − 𝑪𝟐𝟓 𝒛𝟑 𝒊𝟐 − 𝑪𝟑𝟓 𝒛𝟐 𝒊𝟑 − 𝑪𝟒𝟓 𝒛𝒊𝟒 − 𝑪𝟓𝟓 𝒊𝟓
𝒛𝟓 − 𝟓𝒛𝟒 𝒊 + 𝟏𝟎𝒛𝟑 (−𝟏) + 𝟏𝟎𝒛𝟐 + 𝟓𝒛 − 𝒊 =
= −𝒛𝟓 − 𝟓𝒛𝟒 𝒊 + 𝟏𝟎𝒛𝟑 + 𝟏𝟎𝒛𝟐 − 𝟓𝒛 − 𝒊
⇒ 𝟐𝒛𝟓 − 𝟐𝟎𝒛𝟑 + 𝟏𝟎𝒛 = 𝟎
𝒛(𝒛𝟒 − 𝟏𝟎𝒛𝟐 + 𝟓) = 𝟎
𝒛𝟏 = 𝟎, 𝒛𝟒 − 𝟏𝟎𝒛𝟐 + 𝟓 = 𝟎, 𝒛𝟐 = 𝒕
𝒕𝟐 − 𝟏𝟎𝒕 + 𝟓 = 𝟎; 𝚫 = 𝟏𝟎𝟎 − 𝟐𝟎 = 𝟖𝟎
𝟏𝟎 ± 𝟒√𝟓
𝒕𝟏⁄ = = 𝟓 ± 𝟐√𝟓, 𝒛𝟐 = 𝟓 + 𝟐√𝟓
𝟐 𝟐

⇒ 𝒛𝟐 = √𝟓 + 𝟐√𝟓, 𝒛𝟑 = −√𝟓 + 𝟐√𝟓

𝒛𝟐 = 𝟓 − 𝟐√𝟓, 𝟓 − 𝟐√𝟓 = √𝟐𝟓 − √𝟐𝟎 > 0

𝒛𝟒 = √𝟓 − 𝟐√𝟓, 𝒛𝟓 = −√𝟓 − 𝟐√𝟓

𝑺 = {𝟎, √𝟓 + 𝟐√𝟓, −√𝟓 + 𝟐√𝟓, √𝟓 − 𝟐√𝟓, √𝟓 − 𝟐√𝟓}

Solution 2 by Hikmat Mammadov-Azerbaijan


𝑨 ∈ 𝑴𝟓 (ℤ), 𝑨𝟒 = 𝑶𝟓 ⇒ Eigenvalues = 𝟎

78 RMM-ABSTRACT ALGEBRA MARATHON 601-700


www.ssmrmh.ro
𝑬𝒊𝒈𝒆𝒏𝒗𝒂𝒍𝒖𝒆𝒔=𝟏
𝟓𝐝𝐞𝐭( )
⏟ 𝑨𝟐 +𝑰𝟓
𝒛−𝒊
Solve for complex numbers: (𝒛+𝒊) =𝟏 = −𝟏

⇒ (𝒛 − 𝒊)𝟓 = −(𝒛 + 𝒊)𝟓 ⇒ 𝒛𝟓 − 𝟏𝟎𝒛𝟑 + 𝟓𝒛 = 𝟎

⇒ 𝒛((𝒛𝟐 − 𝟓)𝟐 − 𝟐𝟎) = 𝟎 ⇒ 𝒛 = 𝟎; ±√𝟓 ± 𝟐√𝟓

677. Solve for natural numbers:


𝒏 𝒏
𝒏
𝟑𝟏 + ∑ (( ) ⋅ ∑ 𝒎𝒏−𝒌 ) = 𝟑𝟏𝟑𝟎
𝒌
𝒌=𝟏 𝒎=𝟏

Proposed by Daniel Sitaru – Romania


Solution by Bui Hong Suc – Vietnam
1)
𝒏 𝒏 𝒌 𝒏 𝒏
𝒏 𝒏 𝟏
𝟑𝟏 + ∑ (( ) ∑ 𝒎𝒏−𝒌 ) = 𝟑𝟏𝟑𝟎 ↔ ∑ 𝒎𝒏 ∑ ( ) ( ) = 𝟑𝟏𝟑𝟎 − 𝟑𝟏 ↔
𝒌 𝒌 𝒌
𝒌=𝟏 𝒎=𝟏 𝒎=𝟏 𝒌=𝟏

𝒏
𝟏 𝒏
↔ ∑ 𝒎 ((𝟏 + ) − 𝟏) = 𝟑𝟏𝟑𝟎 − 𝟑𝟏 ↔
𝒎
𝒎
𝒎=𝟏
𝒏 𝒏 𝒏 𝒏
𝟏 𝒏
↔ ∑ 𝒎 (𝟏 + ) − ∑ 𝒎𝒏 = 𝟑𝟏𝟑𝟎 − 𝟑𝟏 ↔ ∑ (𝒎 + 𝟏)𝒏 − ∑ 𝒎𝒏 = 𝟑𝟏𝟑𝟎 − 𝟑𝟏
𝒏
𝒎
𝒎=𝟏 𝒎=𝟏 𝒎=𝟏 𝒎=𝟏

↔ (𝒏 + 𝟏)𝒏 − 𝟏 = 𝟑𝟏𝟑𝟎 − 𝟑𝟏 ↔ (𝒏 + 𝟏)𝒏 = 𝟑𝟏𝟑𝟎 − 𝟑𝟎


2)
𝒏 𝒏 𝒌 𝒏 𝒏
𝒏 𝒏 𝟏
𝟏 + ∑ (( ) ∑ 𝒎 ) = 𝟑𝟏 ↔ ∑ 𝒎 ∑ ( ) ( ) = 𝟑𝟏𝟑𝟎 − 𝟏 ↔
𝒏−𝒌 𝟑𝟎 𝒏
𝒌 𝒌 𝒎
𝒌=𝟏 𝒎=𝟏 𝒎=𝟏 𝒌=𝟏

𝒏
𝟏 𝒏
↔ ∑ 𝒎𝒏 ((𝟏 + ) − 𝟏) = 𝟑𝟏𝟑𝟎 − 𝟏
𝒎
𝒎=𝟏
𝒏 𝒏 𝒏 𝒏
𝟏 𝒏
↔ ∑ 𝒎 (𝟏 + ) ∑ 𝒎𝒏 = 𝟑𝟏𝟑𝟎 − 𝟏 ↔ ∑ (𝒎 + 𝟏)𝒏 − ∑ 𝒎𝒏 = 𝟑𝟏𝟑𝟎 − 𝟏
𝒏
𝒎
𝒎=𝟏 𝒎=𝟏 𝒎=𝟏 𝒎=𝟏

↔ (𝒏 + 𝟏 )𝒏 − 𝟏 = 𝟑𝟏 𝟑𝟎
− 𝟏 ↔ (𝒏 + 𝟏 )𝒏 = 𝟑𝟏 𝟑𝟎
↔ 𝒏 = 𝟑𝟎
Hence: 𝒏 = 𝟑𝟎
79 RMM-ABSTRACT ALGEBRA MARATHON 601-700
www.ssmrmh.ro
678. Solve for real numbers:
𝟏 𝟏 𝟐
+ =
𝟏 + 𝐭𝐚𝐧𝟒 𝒙 𝟏𝟎 𝟏 + 𝟑 𝐭𝐚𝐧𝟐 𝒙
Proposed by Daniel Sitaru – Romania
Solution by Hikmat Mammadov-Azerbaijan

9

10
𝟏 𝟏 𝟐
𝒚= + −
𝟏 + 𝐭𝐚𝐧 𝒙 𝟏𝟎 𝟏 + 𝟑 𝐭𝐚𝐧𝟐 𝒙
𝟒

𝐭𝐚𝐧𝟐 𝒙 = 𝒕
𝟑𝒕𝟑 − 𝟏𝟗𝒕𝟐 + 𝟑𝟑𝒕 − 𝟗 = 𝟎, (𝒕 − 𝟑)(𝟑𝒕𝟐 − 𝟏𝟎𝒕 + 𝟑) = 𝟎
𝟏
(𝒕 − 𝟑)𝟐 (𝒕 − ) = 𝟎
𝟑
𝟏 𝟏 𝟐
+ =
𝟏 + 𝐭𝐚𝐧 𝒙 𝟏𝟎 𝟏 + 𝟑 𝐭𝐚𝐧𝟐 𝒙
𝟒

𝟏
𝐭𝐚𝐧𝟐 𝒙 = 𝟑 ⇒ 𝒙 = 𝝅 (𝒌 ± ) , 𝒌 ∈ ℤ
𝟑
𝟏 𝟏
𝐭𝐚𝐧𝟐 𝒙 = ⇒ 𝒙 = 𝝅 (𝒌 ± ) , 𝒌 ∈ ℤ
𝟑 𝟔
679. 𝐏𝐫𝐨𝐯𝐞 𝐭𝐡𝒂𝐭 ∶
𝐧+𝟏 𝐧 𝐦+𝟏
𝟏 𝐅𝐧+𝟏 𝐅𝐧+𝟐 𝐦 (
𝐤−𝟏
)
( ) ∑ ≥ 𝟏,
𝟐𝐧 𝟐𝐧 𝐅𝐤𝟐𝐦
𝐤=𝟏

𝐰𝐡𝐞𝐫𝐞 𝐅𝐤 𝐢𝐬 𝐤 − 𝐭𝐡 𝐅𝐢𝐛𝐨𝐧𝒂𝐜𝐜𝐢 𝐧𝐮𝐦𝐛𝐞𝐫, 𝐦 ≥ 𝟎


Proposed by D.M. Bătinețu-Giurgiu, Neculai Stanciu-Romania
Solution 1 by Tapas Das-India
𝒏

∑ 𝑭𝟐𝒌 = 𝑭𝒏 𝑭𝒏+𝟏
𝒌=𝟏

80 RMM-ABSTRACT ALGEBRA MARATHON 601-700


www.ssmrmh.ro
Let ∀𝒏 ∈ ℕ let 𝑷(𝒌) : ∑𝒏𝒌=𝟏 𝑭𝟐𝒌 = 𝑭𝒏 𝑭𝒏+𝟏
𝑷(𝟏) is true because 𝑭𝟐𝟏 = 𝟏 = 𝑭𝟑 − 𝟏 holds from definition
𝟏

∴ ∑ 𝑭𝟐𝒌 = 𝑭𝟐𝟏 = (𝒙) = 𝑭𝟏 × 𝑭𝟐


𝒌=𝟏

Let 𝑷(𝑲) is true, we need to show 𝑷(𝒌 + 𝟏) is true


𝒌

∑ 𝑭𝟐𝒋 = 𝑭𝒌 𝑭𝒌+𝟏
𝒌=𝟏
𝒌+𝟏 𝒌

∴ ∑ 𝑭𝟐𝒋 = ∑ 𝑭𝟐𝒋 + 𝑭𝟐𝒌+𝟏 = 𝑭𝒌 𝑭𝒌+𝟏 + 𝑭𝟐𝒌+𝟏 = (𝑭𝒌 + 𝑭𝒌+𝟏 ) ⋅ 𝑭𝒌+𝟏 = 𝑭𝒌+𝟐 𝑭𝒌+𝟏
𝒋=𝟏 𝒋=𝟏

∴ 𝑷(𝒌 + 𝟏) is true
𝒏+𝟏

∴ 𝑭𝒏+𝟏 𝑭𝒏+𝟐 = ∑ 𝑭𝟐𝒌


𝒌=𝟏

Note :
𝒏+𝟏

𝑭𝒏+𝟏 ⋅ 𝑭𝒏+𝟐 = ∑ 𝑭𝟐𝒌


𝒌=𝟏
𝒏+𝟏
𝒏
∑( ) = 𝒏𝑪𝟎 + 𝒏𝑪𝟏 + ⋯ + 𝒏𝑪𝒏
𝒌−𝟏
𝒌=𝟏

(𝟏 + 𝒙)𝒏 = 𝑪𝟎 + 𝑪𝟏 𝒙 + ⋯ + 𝑪𝒏 𝒙𝒏
We put 𝒙 = 𝟏, we get
𝟐𝒏 = 𝑪𝟎 + 𝑪𝟏 + ⋯ + 𝑪𝒏
Now
𝒏+𝟏 𝒏 𝒎+𝟏 𝟐 𝒎 [∑𝒏+𝟏 (
𝒏 𝒎+𝟏
𝟏 𝑭𝒏+𝟏 𝑭𝒏+𝟐 𝒎 ( ) 𝑹𝒂𝒅𝒐𝒏 𝟏 (∑𝒏+𝟏 𝑭 ) 𝒌=𝟏 ) ]
( ) ∑ 𝒌−𝟏 ≥
𝒌=𝟏 𝒌 𝒌−𝟏
𝒎
𝟐𝒏 𝟐𝒏 𝑭𝟐𝒎
𝒌 𝟐 𝒏 𝟐 𝒎𝒏
(∑𝒌=𝟏 𝑭𝟐𝒌 )
𝒏+𝟏
𝒌=𝟏
𝒏+𝟏 𝒎
𝟏 (𝟐𝒏 )𝒎+𝟏
= ⋅ × (∑ 𝑭𝟐𝒌 )
𝟐𝒎𝒏+𝒏 (∑𝒏+𝟏 𝟐 𝒎
𝒌=𝟏 𝑭𝒌 ) 𝒌=𝟏

81 RMM-ABSTRACT ALGEBRA MARATHON 601-700


www.ssmrmh.ro
𝒏+𝟏 𝒎
𝟏 𝟐𝒎𝒏+𝒏
= ⋅ × (∑ 𝑭𝟐𝒌 ) ≥ 𝟏
𝟐𝒎𝒏+𝒏 (∑𝒏+𝟏 𝟐 𝒎
𝒌=𝟏 𝑭𝒌 ) 𝒌=𝟏

Solution 2 by Soumava Chakraborty-Kolkata-India


𝐧 𝐦+𝟏 𝐧 𝐧 𝐧 𝐦+𝟏
𝐦 𝐧+𝟏 (
) 𝐦(( ) + ( ) + ⋯ + ( ))
𝟏 𝐅𝐧+𝟏 𝐅𝐧+𝟐 𝐤−𝟏 𝐑𝒂𝐝𝐨𝐧 𝟏 𝐅𝐧+𝟏 𝐅𝐧+𝟐 𝟎 𝟏 𝐧
( ) ∑ ≥ ( ) .
𝟐𝐧 𝟐𝐧 𝐅𝐤𝟐𝐦 𝟐𝐧 𝟐𝐧 𝟐 )𝐦
(∑𝐧𝐤=𝟏 𝐅𝐤𝟐 + 𝐅𝐧+𝟏
𝐤=𝟏

𝟏 𝐅𝐧+𝟏 𝐅𝐧+𝟐 𝐦 𝟐𝐦𝐧 . 𝟐𝐧 (𝐅𝐧+𝟏 𝐅𝐧+𝟐 )𝐦 (𝐅𝐧+𝟏 𝐅𝐧+𝟐 )𝐦


= 𝐧( ) . 𝐦 = 𝐦 = =𝟏
𝟐 𝟐𝐧 𝟐 )
(𝐅𝐧 𝐅𝐧+𝟏 + 𝐅𝐧+𝟏 (𝐅𝐧+𝟏 (𝐅𝐧 + 𝐅𝐧+𝟏 )) (𝐅𝐧+𝟏 𝐅𝐧+𝟐 )𝐦

𝐧+𝟏 𝐧 𝐦+𝟏
𝟏 𝐅𝐧+𝟏 𝐅𝐧+𝟐 𝐦 ( )
∴ 𝐧( ) ∑ 𝐤−𝟏 ≥ 𝟏 (𝐐𝐄𝐃)
𝟐 𝟐𝐧 𝐅𝐤𝟐𝐦
𝐤=𝟏

680.
𝐈𝐟 𝐅𝐧 𝒂𝐧𝐝 𝐋𝐧 𝒂𝐫𝐞 𝐭𝐡𝐞 𝐧𝐭𝐡 𝐅𝐢𝐛𝐨𝐧𝒂𝐜𝐜𝐢 𝐧𝐮𝐦𝐛𝐞𝐫 𝒂𝐧𝐝 𝐧𝐭𝐡 𝐋𝐮𝐜𝒂𝐬 𝐧𝐮𝐦𝐛𝐞𝐫 𝐫𝐞𝐬𝐩𝐞𝐜𝐭𝐢𝐯𝐞𝐥𝐲,
𝐭𝐡𝐞𝐧 𝐩𝐫𝐨𝐯𝐞 𝐭𝐡𝒂𝐭 ∶
𝐧 𝐧

𝐅𝐧. 𝐅𝐧+𝟏 + 𝟏 − ∏(𝐅𝐤𝟐 + 𝟏) ≤ 𝟎; 𝒂𝐧𝐝 𝐋𝐧. 𝐋𝐧+𝟏 − 𝟏 − ∏(𝐋𝟐𝐤 + 𝟏) ≤ 𝟎


𝐤=𝟏 𝐤=𝟏

Proposed by D.M. Batinetu-Giurgiu, Neculai Stanciu-Romania


Solution 1 by Tapas Das-India
∑𝒏𝒌=𝟏 𝑭𝟐𝒌 = 𝑭𝒏 𝑭𝒏+𝟏
Let ∀𝒏 ∈ ℕ, let 𝑷(𝒌): ∑𝒏𝒌=𝟏 𝑭𝟐𝒌 = 𝑭𝒏 𝑭𝒏+𝟏
𝑷(𝟏) is true because 𝑭𝟐𝟏 = 𝟏 = 𝑭𝟑 − 𝟏 holds from definition
𝒏

∴ ∑ 𝑭𝟐𝒌 = 𝑭𝟐𝟏 = (𝒙) = 𝑭𝟏 × 𝑭𝟐


𝒌=𝟏

Let 𝑷(𝒌) is true, we need to show 𝑷(𝒌 + 𝟏) is true


𝒏

∑ 𝑭𝟐𝒋 = 𝑭𝒌 𝑭𝒌+𝟏
𝒋=𝟏

𝒌+𝟏 𝒏

∴ ∑ 𝑭𝟐𝒋 = ∑ 𝑭𝟐𝒋 + 𝑭𝟐𝒌+𝟏 = 𝑭𝒌 𝑭𝒌+𝟏 + 𝑭𝟐𝒌+𝟏 = (𝑭𝒌 + 𝑭𝒌+𝟏 ) ⋅ 𝑭𝒌+𝟏 = 𝑭𝒌+𝟐 𝑭𝒌+𝟏
𝒋=𝟏 𝒋=𝟏

∴ 𝑷(𝒌 + 𝟏) is true

82 RMM-ABSTRACT ALGEBRA MARATHON 601-700


www.ssmrmh.ro
𝒏

𝑭𝒏 ⋅ 𝑭𝒏+𝟏 = ∑ 𝑭𝟐𝒌
𝒌=𝟏

Now
𝒏

∏(𝑭𝟐𝒌 + 𝟏) = 𝟏 + ∑ 𝑭𝟐𝒌 + ∑ 𝑭𝟐𝟏 ⋅ 𝑭𝟐𝟐 + ⋯ + ∏ 𝑭𝟏 𝑭𝟐 … 𝑭𝒏


𝒌=𝟏 𝒄𝒚𝒄
𝒏

∴ ∏(𝑭𝟐𝒌 + 𝟏) > 1 + ∑ 𝑭𝟐𝒌


𝒌=𝟏
𝒏 𝒏

∴ 𝑭𝒏 + 𝑭𝒏+𝟏 + 𝟏 − ∏(𝑭𝟐𝒌 + 𝟏) < ∑ 𝑭𝟐𝒌 + 𝟏 − 𝟏 − ∑ 𝑭𝟐𝒌 < 0


𝒌=𝟏 𝒌=𝟏

Similarly, ∑ 𝑳𝟐𝒌 + 𝟐 = 𝑳𝒏 𝑳𝒏+𝟏


𝒏

∴ ∏(𝑳𝟐𝒌 + 𝟏) > 1 + ∑ 𝑳𝟐𝒌


𝒌=𝟏

∴ 𝑳𝒏 𝑳𝒏+𝟏 − 𝟏 − ∏(𝑳𝟐𝒌 + 𝟏) < ∑ 𝑳𝟐𝒌 + 𝟏 − 𝟏 − 𝟏 − ∑ 𝑳𝟐𝒌 < 0

Solution 2 by Soumava Chakraborty-Kolkata-India


𝐧

𝐅𝐨𝐫 𝐧 = 𝟏, 𝐅𝐧 . 𝐅𝐧+𝟏 + 𝟏 − ∏(𝐅𝐤𝟐 + 𝟏) = 𝐅𝟏 . 𝐅𝟐 + 𝟏 − 𝐅𝟏𝟐 − 𝟏


𝐤=𝟏
𝐧

= 𝟏 ∗ 𝟏 − 𝟏 = 𝟎 ⇒ 𝐅𝐧 . 𝐅𝐧+𝟏 + 𝟏 − ∏(𝐅𝐤𝟐 + 𝟏) = 𝟎 𝐟𝐨𝐫 𝐧 = 𝟏


𝐤=𝟏
𝐧

𝐅𝐨𝐫 𝐧 = 𝟐, 𝐅𝐧 . 𝐅𝐧+𝟏 + 𝟏 − ∏(𝐅𝐤𝟐 + 𝟏) = 𝐅𝟐 . 𝐅𝟑 + 𝟏 − (𝐅𝟏𝟐 + 𝟏)(𝐅𝟐𝟐 + 𝟏)


𝐤=𝟏
𝐧

= 𝟏 ∗ 𝟐 + 𝟏 − 𝟐 ∗ 𝟐 = −𝟏 < 0 ⇒ 𝐅𝐧 . 𝐅𝐧+𝟏 + 𝟏 − ∏(𝐅𝐤𝟐 + 𝟏) < 0 𝐟𝐨𝐫 𝐧 = 𝟐


𝐤=𝟏
𝐧

𝐅𝐨𝐫 𝐧 ≥ 𝟑, ∏(𝐅𝐤𝟐 + 𝟏) = (𝐅𝟏𝟐 + 𝟏)(𝐅𝟐𝟐 + 𝟏) … (𝐅𝐧−𝟏


𝟐
+ 𝟏)(𝐅𝐧𝟐 + 𝟏)
𝐤=𝟏
≥ (𝐅𝟏𝟐 + 𝟏)(𝐅𝐧−𝟏
𝟐
+ 𝟏)(𝐅𝐧𝟐 + 𝟏) (∵ 𝐅𝐣𝟐 + 𝟏 > 1 ∀ 𝑗 = 𝟐, 𝐧 − 𝟐)
𝐂𝐁𝐒 ?
≥ (𝟏𝟐 + 𝟏)(𝐅𝐧−𝟏 𝐅𝐧 + 𝟏)𝟐 > 𝟏 + 𝐅𝐧 . 𝐅𝐧+𝟏
?
= 𝟏 + 𝐅𝐧 . (𝐅𝐧−𝟏 + 𝐅𝐧 ) ⇔ 𝟐(𝒙𝟐 𝐲 𝟐 + 𝟐𝒙𝐲 + 𝟏) > 𝟏 + 𝒙𝐲 + 𝐲 𝟐 (𝒙 = 𝐅𝐧−𝟏 , 𝐲 = 𝐅𝐧 )
?
⇔ 𝐲 𝟐 (𝟐𝒙𝟐 − 𝟏) + 𝟑𝒙𝐲 + 𝟏 > 𝟎 → 𝐭𝐫𝐮𝐞 ∵ 𝒙 ≥ 𝐅𝟐 = 𝟏 𝒂𝐧𝐝 𝐲 ≥ 𝐅𝟑 = 𝟐

83 RMM-ABSTRACT ALGEBRA MARATHON 601-700


www.ssmrmh.ro
𝐧

⇒ 𝐲 𝟐𝒙 − 𝟏) ≥ 𝟒 ∗ 𝟏 > 0 ∴ 𝐅𝐧 . 𝐅𝐧+𝟏 + 𝟏 − ∏(𝐅𝐤𝟐 + 𝟏) < 0 𝐟𝐨𝐫 𝐧 ≥ 𝟑


𝟐( 𝟐

𝐤=𝟏
𝐧

∴ 𝐜𝐨𝐦𝐛𝐢𝐧𝐢𝐧𝐠, 𝐅𝐧 . 𝐅𝐧+𝟏 + 𝟏 − ∏(𝐅𝐤𝟐 + 𝟏) ≤ 𝟎,′′ =′′ 𝐢𝐟𝐟 𝐧 = 𝟏


𝐤=𝟏
𝐧

𝐀𝐠𝒂𝐢𝐧, 𝐟𝐨𝐫 𝐧 = 𝟏, 𝐋𝐧 . 𝐋𝐧+𝟏 − 𝟏 − ∏(𝐋𝟐𝐤 + 𝟏) = 𝐋𝟏 . 𝐋𝟐 − 𝟏 − 𝐋𝟐𝟏 − 𝟏


𝐤=𝟏
𝐧

= 𝟏 ∗ 𝟑 − 𝟏 − 𝟐 = 𝟎 ⇒ 𝐋𝐧 . 𝐋𝐧+𝟏 − 𝟏 − ∏(𝐋𝟐𝐤 + 𝟏) = 𝟎 𝐟𝐨𝐫 𝐧 = 𝟏


𝟐

𝐤=𝟏
𝐧

𝐅𝐨𝐫 𝐧 = 𝟐, 𝐋𝐧 . 𝐋𝐧+𝟏 − 𝟏 − ∏(𝐋𝟐𝐤 + 𝟏) = 𝐋𝟐 . 𝐋𝟑 − 𝟏 − (𝐋𝟐𝟏 + 𝟏)(𝐋𝟐𝟐 + 𝟏)


𝐤=𝟏
𝐧

= 𝟑 ∗ 𝟒 − 𝟏 − 𝟐 ∗ 𝟏𝟎 = −𝟗 < 0 ⇒ 𝐋𝐧 . 𝐋𝐧+𝟏 − 𝟏 − ∏(𝐋𝟐𝐤 + 𝟏) < 0 𝐟𝐨𝐫 𝐧 = 𝟐


𝐤=𝟏
𝐧

𝐅𝐨𝐫 𝐧 ≥ 𝟑, ∏(𝐋𝟐𝐤 + 𝟏) = (𝐋𝟐𝟏 + 𝟏)(𝐋𝟐𝟐 + 𝟏) … (𝐋𝟐𝐧−𝟏 + 𝟏)(𝐋𝟐𝐧 + 𝟏)


𝐤=𝟏
≥ (𝐋𝟐𝟏 + 𝟏)(𝐋𝟐𝐧−𝟏 + 𝟏)(𝐋𝟐𝐧 + 𝟏) (∵ 𝐋𝟐𝐣 + 𝟏 > 1 ∀ 𝑗 = 𝟐, 𝐧 − 𝟐)
𝐂𝐁𝐒 ?
≥ (𝟏𝟐 + 𝟏)(𝐋𝐧−𝟏 𝐋𝐧 + 𝟏)𝟐 > 𝐋𝐧 . 𝐋𝐧+𝟏 − 𝟏
?
= 𝐋𝐧 . (𝐋𝐧−𝟏 + 𝐋𝐧 ) − 𝟏 ⇔ 𝟐(𝒂𝟐 𝐛𝟐 + 𝟐𝒂𝐛 + 𝟏) > 𝒂𝐛 + 𝐛𝟐 − 𝟏 (𝒂 = 𝐅𝐧−𝟏 , 𝐛 = 𝐅𝐧 )

?
⇔ 𝐛𝟐 (𝟐𝒂𝟐 − 𝟏) + 𝟑𝒂𝐛 + 𝟑 > 𝟎 → 𝐭𝐫𝐮𝐞 ∵ 𝒂 ≥ 𝐋𝟐 = 𝟑 𝒂𝐧𝐝 𝐛 ≥ 𝐋𝟑 = 𝟒
𝐧

⇒ 𝐛𝟐 (𝟐𝒂𝟐 − 𝟏) ≥ 𝟏𝟔 ∗ 𝟏𝟕 > 0 ∴ 𝐋𝐧 . 𝐋𝐧+𝟏 − 𝟏 − ∏(𝐋𝟐𝐤 + 𝟏) < 0 𝐟𝐨𝐫 𝐧 ≥ 𝟑


𝐤=𝟏
𝐧

∴ 𝐜𝐨𝐦𝐛𝐢𝐧𝐢𝐧𝐠, 𝐋𝐧 . 𝐋𝐧+𝟏 − 𝟏 − ∏(𝐋𝟐𝐤 + 𝟏) ≤ 𝟎,′′ =′′ 𝐢𝐟𝐟 𝐧 = 𝟏 (𝐐𝐄𝐃)


𝐤=𝟏

681. 𝐈𝐟 𝟎 ≤ 𝒂, 𝒃, 𝒄 ≤ 𝟐, 𝒂 + 𝒃 + 𝒄𝟐 = 𝟔 𝐭𝐡𝐞𝐧 𝐟𝐢𝐧𝐝 𝒎𝒊𝒏𝑷


𝟐 𝟐

𝑷 = √𝒂𝟐 + 𝒃𝟐 + √𝒃𝟐 + 𝒄𝟐 + √𝒄𝟐 + 𝒂𝟐

Proposed by Tran Quoc Thinh-Vietnam


Solution by Mohamed Amine Ben Ajiba-Tanger-Morocco
𝐖𝐋𝐎𝐆 𝐰𝐞 𝐦𝐚𝐲 𝐚𝐬𝐬𝐮𝐦𝐞 𝐭𝐡𝐚𝐭 𝒂 ≥ 𝒃 ≥ 𝒄.
𝒂𝟐 + 𝒃𝟐 + 𝒄𝟐
𝐖𝐞 𝐡𝐚𝐯𝐞 𝒂𝟐 ≥ = 𝟐, 𝐭𝐡𝐞𝐧 √𝟐 ≤ 𝒂 ≤ 𝟐.
𝟑
𝐍𝐨𝐰, 𝐰𝐞 𝐡𝐚𝐯𝐞

84 RMM-ABSTRACT ALGEBRA MARATHON 601-700


www.ssmrmh.ro
𝟐
(√𝒂𝟐 + 𝒃𝟐 + √𝒄𝟐 + 𝒂𝟐 ) = 𝟐𝒂𝟐 + 𝒃𝟐 + 𝒄𝟐 + 𝟐√𝒂𝟐 (𝒂𝟐 + 𝒃𝟐 + 𝒄𝟐 ) + 𝒃𝟐 𝒄𝟐
≥ 𝒂𝟐 + 𝟔 + 𝟐√𝟔𝒂𝟐 ,
𝐭𝐡𝐞𝐧 √𝒂𝟐 + 𝒃𝟐 + √𝒄𝟐 + 𝒂𝟐 ≥ 𝒂 + √𝟔, 𝐬𝐨 𝐢𝐭 𝐬𝐮𝐟𝐟𝐢𝐜𝐞𝐬 𝐭𝐨 𝐟𝐢𝐧𝐝 𝐭𝐡𝐞 𝐦𝐢𝐧𝐢𝐦𝐮𝐦 𝐯𝐚𝐥𝐮𝐞 𝐨𝐟
𝒇(𝒂) = 𝒂 + √𝒃𝟐 + 𝒄𝟐 = 𝒂 + √𝟔 − 𝒂𝟐 , 𝐰𝐡𝐞𝐫𝐞 𝒂 ∈ [√𝟐, 𝟐].
𝟔
𝐖𝐞 𝐡𝐚𝐯𝐞 𝒇′′ (𝒂) = − < 0, 𝐬𝐨 𝒇 𝐢𝐬 𝐜𝐨𝐧𝐜𝐚𝐯𝐞 𝐨𝐧 [√𝟐, 𝟐], 𝐭𝐡𝐞𝐧
√(𝟔 − 𝒂𝟐 )𝟑
𝐦𝐢𝐧 𝒇(𝒂) = 𝒎𝒊𝒏{𝒇(√𝟐), 𝒇(𝟐)} = 𝟐 + √𝟐.
𝒂∈[√𝟐,𝟐]
𝐓𝐡𝐞𝐫𝐞𝐟𝐨𝐫𝐞, 𝐭𝐡𝐞 𝐦𝐢𝐧𝐢𝐦𝐮𝐦 𝐯𝐚𝐥𝐮𝐞 𝐨𝐟 𝑷 𝒊𝒔 √𝟔 + 𝟐 + √𝟐 𝐫𝐞𝐚𝐜𝐡𝐞𝐝 𝐚𝐭
(𝒂, 𝒃, 𝒄) = (𝟐, √𝟐, 𝟎) 𝐚𝐧𝐝 𝐩𝐞𝐫𝐦𝐮𝐭𝐚𝐭𝐢𝐨𝐧𝐬.
682. Determine the number 𝒂𝒃𝒄 such that

𝒂𝒃 ⋅ 𝒃𝒂 = 𝒄𝒃𝒂𝒄
Proposed by Neculai Stanciu – Romania
Solution 1 by Adrian Popa
𝒂𝒃 ⋅
𝒃𝒂
−−
𝒎𝒏𝒄
𝒄𝒒𝒑
−−−
𝒄𝒃𝒂𝒄
We denote the last digit of 𝒂 ⋅ 𝒃 with 𝑼(𝒂 ⋅ 𝒃)
the first digit of 𝒂 ⋅ 𝒃 = 𝑷(𝒂 ⋅ 𝒃)
Because 𝑼(𝒂 ⋅ 𝒃) ≠ 𝑷(𝒂 ⋅ 𝒃) (if this thing happens ⇒ 𝒂 ⋅ 𝒃 ⋮ 𝟏𝟏 ⇒ one of 𝒂 or 𝒃 must be
𝟏𝟏 → false.) → 𝑼(𝒂 ⋅ 𝒃) = 𝑷(𝒂 ⋅ 𝒃) + 𝟏 ⇒
⇒ 𝒂 ⋅ 𝒃 = 𝟕 ⋅ 𝟖 = 𝟖 ⋅ 𝟕 = 𝟓𝟔
𝟕𝟖 ⋅
𝟖𝟕
−−
𝟓𝟒𝟔
𝟔𝟐𝟒
−−−
𝟔𝟕𝟖𝟔
𝒄𝒃𝒂𝒄
𝒄=𝟔
⇒ 𝒂 = 𝟖} ⇒ 𝒂𝒃𝒄 = 𝟖𝟕𝟔
𝒃=𝟕
85 RMM-ABSTRACT ALGEBRA MARATHON 601-700
www.ssmrmh.ro
Solution 2 by Bedri Hajrizi-Mitrovica-Kosovo
(𝟏𝟎𝒂 + 𝒃)(𝟏𝟎𝒃 + 𝒂) − 𝟏𝟎𝟎𝒃 − 𝟏𝟎𝒂 = 𝟏𝟎𝟎𝟏𝒄
(𝟏𝟎𝒂 + 𝒃)(𝟏𝟎𝒃 + 𝒂) − 𝟏𝟎(𝟏𝟎𝒃 + 𝒂) = 𝟏𝟎𝟎𝟏𝒄
(𝟏𝟎𝒂 + 𝒃 − 𝟏𝟎)(𝟏𝟎𝒃 + 𝒂) = 𝟏𝟎𝟎𝟏𝒄
(𝟏𝟎𝒂 + 𝒃 − 𝟏𝟎)(𝟏𝟎𝒃 + 𝒂) = 𝟕 ⋅ 𝟏𝟏 ⋅ 𝟏𝟑𝒄
(𝟏𝟎𝒂 + 𝒃 − 𝟏𝟎)(𝟏𝟎𝒃 + 𝒂)
𝒄=
𝟕 ⋅ 𝟏𝟏 ⋅ 𝟏𝟑
𝟏𝟎 + 𝒃 − 𝟏𝟎 = 𝟕𝟕
1) { ~ { 𝒂𝒃 = 𝟖𝟕
𝟏𝟎𝒃 + 𝒂 = 𝟏𝟑𝒄 𝒃𝒂 = 𝟏𝟑𝒄 = 𝟕𝟖
⇒ 𝒂 = 𝟖, 𝒃 = 𝟕, 𝒄 = 𝟔
𝟏𝟎𝒂 + 𝒃 − 𝟏𝟎 = 𝟏𝟏
2) { ~ { 𝒂𝒃 = 𝟐𝟏
𝟏𝟎𝒃 + 𝒂 = 𝟗𝟏𝒄 𝒃𝒂 = 𝟗𝟏𝒄
⇒ no solution
𝟏𝟎𝒂 + 𝒃 − 𝟏𝟎 = 𝟕𝟕𝒄 𝒂𝒃 = 𝟕𝟕𝒄 + 𝟏𝟏
3) { ~{
𝟏𝟎𝒃 + 𝒂 = 𝟏𝟑 𝒃𝒂 = 𝟏𝟑
⇒ no solution
𝟏𝟎𝒂 + 𝒃 − 𝟏𝟎 = 𝟏𝟏𝒄 𝒂𝒃 = 𝟏𝟏𝒄 + 𝟏𝟎
4) { ~{
𝟏𝟎𝒃 + 𝒂 = 𝟗𝟏 𝒃𝒂 = 𝟗𝟏
⇒ no solution
Finally 𝒂𝒃𝒄 = 𝟖𝟕𝟔
683. Solve for natural numbers:
𝒙 𝒙 𝒙 𝒙
( ) +𝟐( ) +𝟒( ) + ⋯ + 𝟐𝒙 ( ) = 𝟒𝒚
{ 𝟐𝒙 𝟐𝒙 − 𝟏 𝟐𝒙 − 𝟐 𝒙
𝒚
𝒙⋅𝟐 = 𝟖
Proposed by Daniel Sitaru – Romania
Solution 1 by Ravi Prakash-New Delhi-India
𝒙, 𝒚 ∈ ℕ, and (𝒙)(𝟐𝒚 ) = 𝟖 ⇒ 𝒙 ≠ 𝟎 ⇒ 𝒙 ≥ 𝟏
Thus, 𝟖 = (𝒙)(𝟐𝒚 ) ≥ 𝟐𝒚 ⇒ 𝟐𝒚 ≤ 𝟖 ⇒ 𝒚 = 𝟎, 𝟏, 𝟐, 𝟑
Case 1. When 𝒚 = 𝟎, 𝒙 = 𝟖
First equation becomes
𝟏𝟔 𝟏𝟓 𝟖
( ) + 𝟐 ( ) + ⋯ + 𝟐𝟖 ( ) = 𝟒𝟎 = 𝟏
𝟖 𝟖 𝟖
Not possible
Case 2. When 𝒚 = 𝟏, 𝒙 = 𝟒
In this case first equation becomes

86 RMM-ABSTRACT ALGEBRA MARATHON 601-700


www.ssmrmh.ro
𝟖 𝟕 𝟒
( ) + 𝟐 ( ) + ⋯ + 𝟐𝟒 ( ) = 𝟒
𝟒 𝟒 𝟒
Not possible
Case 3. When 𝒚 = 𝟐, 𝒙 = 𝟐.First equation becomes
𝟒 𝟑 𝟐
( ) + 𝟐 ( ) + 𝟒 ( ) = 𝟒𝟐 or 𝟔 + 𝟐 × 𝟑 + 𝟒 = 𝟏𝟔 which is true
𝟐 𝟐 𝟐
Case 4. When 𝒚 = 𝟑, 𝒙 = 𝟏
First equation becomes
𝟐 𝟐
( ) + 𝟐 ( ) = 𝟒𝟑 . Not possible
𝟏 𝟐
Thus, 𝒙 = 𝟐, 𝒚 = 𝟐
Solution 2 by Tapas Das-India
𝟐𝒙 𝟐𝒙 − 𝟏 𝟐𝒙 − 𝟐 𝒙
{( )+𝟐⋅( )+𝟒⋅( ) + ⋯ + 𝟐𝒏 ⋅ ( )} = 𝟒𝒚
𝒙 𝒙 𝒙 𝒙
∴ 𝟒𝒚 = ∑𝒙𝒓=𝟎 𝑪𝟐𝒙−𝒓
𝒙 ⋅ 𝟐𝒓 (1)
𝟖
Now, 𝒙 ⋅ 𝟐𝒚 = 𝟖 ∴ 𝒙 = 𝟐𝒚

𝒙 ∈ ℕ, clearly 𝟖 is divisible by 𝟐𝒚 . This will be possible when 𝒚 = 𝟏, 𝟐, 𝟑


[because 𝟖 is divisible by 𝟐, 𝟐𝟐 , 𝟐𝟑 ]. Corresponding values of 𝒙 are
𝟖
𝒚=𝟏∴𝒙= =𝟒
𝟐𝟏
𝒚=𝟐∴𝒙=𝟐
𝒚=𝟑∴𝒙=𝟏
∴ Possible solutions are (𝟒, 𝟏), (𝟐, 𝟐), (𝟏, 𝟑)
When 𝒙 = 𝟒, 𝒚 = 𝟏 from (1) we get
𝟒

∑ 𝑪𝟖−𝒓
𝟒 ⋅ 𝟐𝒓 = 𝟖𝑪𝟒 + 𝟕𝑪𝟒 ⋅ 𝟐𝟏 + ⋯ + 𝟒𝑪𝟒 ⋅ 𝟐𝟒 ≠ 𝟒𝟏
𝒓=𝟎

∴ 𝒙 = 𝟒, 𝒚 = 𝟏 not valid. When 𝒙 = 𝟐, 𝒚 = 𝟐, from (1)


𝟐

∑ 𝑪𝟒−𝒓
𝟐 ⋅ 𝟐𝒓 = 𝟒𝑪𝟐 ⋅ 𝟐𝟎 + 𝟑𝑪𝟐 ⋅ 𝟐𝟏 + 𝟐𝑪𝟐 ⋅ 𝟐𝟐 = 𝟔 + 𝟔 + 𝟒 = 𝟏𝟔 = 𝟒𝟐
𝒓

ValidȘ ∴ Solution 𝒙 = 𝟐, 𝒚 = 𝟐. Similarly 𝒙 = 𝟏, 𝒚 = 𝟑 not satisfied

87 RMM-ABSTRACT ALGEBRA MARATHON 601-700


www.ssmrmh.ro
𝟏

∴ ∑ 𝑪𝟐−𝒓
𝟏 ⋅ 𝟐𝒓 = 𝟐𝑪𝟏 + 𝟏𝑪𝟏 ⋅ 𝟐 ≠ 𝟒𝟑
𝒓=𝟎

Solution 3 by Hikmat Mammadov-Azerbaijan


𝟐𝒙 𝒌𝟐𝒙
𝒌 𝒌 𝟏
∑ ( ) 𝟐𝟐𝒙−𝒌 = 𝟒𝒚 = 𝟒𝒙 ∑ ( ) ( ) = 𝟒𝒙 →
𝒙 𝒙 𝟐
𝒌=𝒙 𝒌=𝒙

→ (let’s look below) ⇒ 𝒙 = 𝒚 ⇒ (𝒙)(𝟐𝒙 ) = 𝟖 ⇒ (𝒙, 𝒚) = (𝟐, 𝟐)


𝟐𝒙 𝒌
𝒌 𝟏
∀𝒙 ∈ ℤ≥𝟎 : ∑ ( ) ( ) = 𝟏
𝒙 𝟐
𝒌=𝒙
𝟐𝒙 𝟐𝒙
𝒌 (𝒌+𝟏)=( 𝒌 )+(𝒌
𝒙) 𝟏 𝒙 𝟏 𝒌
𝒌 𝟏 𝒙+𝟏 𝒙+𝟏 𝒌+𝟏 𝒌
∑( )( ) = ( ) + ∑ (( )−( )) ( )
𝒙 𝟐 𝟐 𝒙+𝟏 𝒙+𝟏 𝟐
𝒌=𝒙 𝒌=𝒙+𝟏
𝟐𝒙 𝟐𝒙
𝟏 𝒙 𝒌+𝟏 𝟏
𝒌
𝒎 𝟏 𝒎
=( ) + ∑ ( )( ) − ∑ ( )( )
𝟐 𝒙+𝟏 𝟐 𝒙+𝟏 𝟐
𝒌=𝒙+𝟏 ⏟
𝒎=𝒙+𝟏
𝑳𝒆𝒕:𝒎=𝒌+𝟏
𝟐𝒙 𝟐𝒙−𝟏
𝟏 𝒙 𝒌+𝟏 𝟏
𝒌
𝒌+𝟏 𝟏
𝒌+𝟏
=( ) + ∑ ( )( ) − ∑ ( )( )
𝟐 𝒙+𝟏 𝟐 𝒙+𝟏 𝟐
𝒌=𝒙+𝟏 𝒌=𝒙
𝟐𝒙−𝟏
𝟏 𝒙 𝟐𝒙 + 𝟏 𝟏
𝟐𝒙
𝟏 𝒙+𝟏 𝒌+𝟏 𝟏 𝒌 𝟏 𝒌+𝟏
= ( ) +( )( ) −( ) + ∑ ( ) (( ) − ( ) )
𝟐 𝒙+𝟏 𝟐 𝟐 𝒙+𝟏 𝟐 𝟐
𝒌=𝒙+𝟏
𝟐𝒙−𝟏
𝟏 𝒙+𝟏 𝟐𝒙 + 𝟏 𝟏
𝟐𝒙
𝒙+𝟏 𝟏
𝒌+𝟏
=( ) +( )( ) + ∑ ( )( )
𝟐 𝒙+𝟏 𝟐 𝒌×𝟏 𝟐
𝒌=𝒙+𝟏
𝟐𝒙−𝟏 𝒌+𝟏 𝟐𝒙
𝒌+𝟏 𝟏 𝟐𝒙 + 𝟏 𝟏
= ∑( )( ) +( )( )
⏟𝒙 + 𝟏 𝟐 𝒙+𝟏 𝟐
𝒌=𝒙
𝒌=𝒗+𝟏
𝟐𝒙
𝒗 𝟏 𝒗 𝟐𝒙 + 𝟏 𝟏
𝟐𝒙
= ∑ ( )( ) +( )( )
𝒙+𝟏 𝟐 𝒙+𝟏 𝟐
𝒗=𝒙+𝟏
𝟐𝒙
𝒗 𝟏 𝒗 𝟐𝒙 + 𝟏 𝟏
𝟐𝒙
𝟐𝒙 + 𝟏 𝟏
𝟐𝒙+𝟏
𝟐𝒙 + 𝟐 𝟏
𝟐𝒙+𝟐
= ∑ ( )( ) + ( )( ) − ( )( ) −( )( )
𝒙+𝟏 𝟐 𝒙+𝟏 𝟐 𝒙+𝟏 𝟐 𝒙+𝟏 𝟐
𝒗=𝒙+𝟏

88 RMM-ABSTRACT ALGEBRA MARATHON 601-700


www.ssmrmh.ro
𝟐𝒙+𝟐 𝒗 𝟐𝒙+𝟏
𝒗 𝟏 𝟐𝒙 + 𝟏 𝟏 (𝟐𝒙 + 𝟐)(𝟐𝒙 + 𝟏)! 𝟏 𝟏 𝟐𝒙+𝟏
= ∑ ( )( ) + ( )( ) − ( )( )
𝒙+𝟏 𝟐 ⏟𝒙 + 𝟏 𝟐 (𝒙 + 𝟏)𝒙! (𝒙 + 𝟏)! 𝟐 𝟐
𝒗=𝒙+𝟏
=𝟎
𝟐𝒙+𝟐
𝒗 𝟏 𝒗
= ∑ ( )( )
𝒙+𝟏 𝟐
𝒗=𝒙+𝟏

𝒌 𝟏 𝒌 𝒌 𝟏 𝒌
Thus, ∑𝟐𝒙 𝟐𝒙
𝒌=𝒙 ( ) (𝟐) is independent of 𝒙 ⇒ ∑𝒌=𝒙 ( ) (𝟐) = 𝟏, ∀𝒙 ∈ ℤ≥𝟎
𝒙 𝒙
684. Find all (𝒎, 𝒏, 𝒑) ∈ ℕ𝑿ℕ𝑿ℕ such that:
𝒎 + 𝒏 + 𝒑 + 𝒎𝒏 + 𝒏𝒑 + 𝒑𝒎 + 𝒎𝒏𝒑 = 𝟑𝟓
{
(𝒑 + 𝟐) ⋅ 𝒑𝒑 = (𝒑 + 𝟐)𝒑
Proposed by Daniel Sitaru – Romania
Solution 1 by Rajarshi Chakraborty-India
𝒎 + 𝒏 + 𝒑 + 𝒎𝒏 + 𝒏𝒑 + 𝒎𝒑 + 𝒎𝒏𝒑 = 𝟑𝟓 (i)
(𝒑 + 𝟐) ⋅ 𝒑𝒑 = (𝒑 + 𝟐)𝒑 (ii)
From (ii)
𝒑𝒑 = (𝒑 + 𝟐)𝒑−𝟏 ⇒ 𝒑|𝒑 + 𝟐 ⇒ 𝒑 = 𝟐 or 1
For 𝒑 = 𝟐, 𝟐𝟐 = (𝟐 + 𝟐)𝟐−𝟏 , and for 𝒑 = 𝟏, 𝟏𝟏 = (𝟏 + 𝟐)𝟏−𝟏
Case 1 (𝒑 = 𝟐): 𝒎 + 𝒏 + 𝟐 + 𝒎𝒏 + 𝟐𝒏 + 𝟐𝒎 + 𝟐𝒎𝒏 = 𝟑𝟓
⇒ 𝒎 + 𝒏 + 𝒎𝒏 = 𝟏𝟏
Possibilities
𝒏=𝟏 𝒎=𝟓
𝒏=𝟐 𝒎=𝟑
𝒏=𝟑 𝒎=𝟐
𝒏=𝟒 𝒏∉ℕ
𝒏=𝟓 𝒎=𝟏
Case 𝟐(𝒑 = 𝟏):
𝟐𝒎 + 𝟐𝒏 + 𝟐𝒎𝒏 = 𝟑𝟒 ⇒ 𝒎 + 𝒏 + 𝒎𝒏 = 𝟏𝟕
Possibilities
𝒏=𝟏 𝒎=𝟖

89 RMM-ABSTRACT ALGEBRA MARATHON 601-700


www.ssmrmh.ro
𝒏=𝟐 𝒎=𝟓
𝒏=𝟑 𝒎∉ℕ
𝒏=𝟒 𝒎∉ℕ
𝒏=𝟓 𝒎=𝟐
𝒏=𝟔 𝒎∉ℕ
𝒏=𝟕 𝒎∉ℕ
𝒏=𝟖 𝒎=𝟏
Thus (𝒎, 𝒏, 𝒑) ∈ {(𝟖, 𝟏, 𝟏), (𝟓, 𝟐, 𝟏), (𝟐, 𝟓, 𝟏), (𝟏, 𝟖, 𝟏), (𝟓, 𝟏, 𝟐), (𝟑, 𝟐, 𝟐), (𝟐, 𝟑, 𝟐), (𝟏, 𝟓, 𝟐)}
Solution 2 by Ertan Yildirim-Izmir-Turkiye
i) 𝒎 + 𝒑 + 𝒎𝒏 + 𝒏𝒑 + 𝒑𝒎 + 𝒎𝒏𝒑 = 𝟑𝟓
(𝟏 + 𝒎)(𝟏 + 𝒏)(𝟏 + 𝒑) = 𝟑𝟔
𝒑+𝟐 𝒑−𝟏 𝟐 𝒑
ii) 𝒑𝒑−𝟏 ⋅ 𝒑 = (𝒑 + 𝟐)𝒑−𝟏 ⇒ 𝒑 = ( ) = (𝟏 + 𝒑)
𝒑

∴ 𝒑|𝟐 then 𝒑 = 𝟏 or 𝒑 = 𝟐
𝑰𝒇 𝒑 = 𝟏, (𝟏 + 𝟐) ⋅ 𝟏𝟏 = (𝟏 + 𝟐)𝟏 ⇒ 𝟑 = 𝟑
} 𝒑 = 𝟏 and 𝒑 = 𝟐 satisfy the equation.
𝑰𝒇 𝒑 = 𝟐, (𝟐 + 𝟐) ⋅ 𝟐𝟐 = (𝟐 + 𝟐)𝟐 ⇒ 𝟏𝟔 = 𝟏𝟔
If 𝒑 = 𝟏 then (𝟏 + 𝒎)(𝟏 + 𝒏) = 𝟏𝟖, (𝒎, 𝒏) = (𝟎, 𝟏𝟕), (𝟏, 𝟖), (𝟐, 𝟓), (𝟏𝟕, 𝟎), (𝟖, 𝟏), (𝟓, 𝟐)
where 𝒑 = 𝟏
If 𝒑 = 𝟐 then (𝟏 + 𝒎)(𝟏 + 𝒏) = 𝟏𝟐, (𝒎, 𝒏) = (𝟎, 𝟏𝟏)(𝟏, 𝟓), (𝟐, 𝟑), (𝟏𝟏, 𝟎), (𝟓, 𝟏), (𝟑, 𝟐)
where 𝒑 = 𝟐
685. Solve for real numbers:
𝒙𝒙 = 𝟐𝒙+𝟒
𝟒𝒛
{ 𝒛𝟐 + + 𝐥𝐨𝐠 𝒚 𝒛 = 𝟗𝟏
𝒙
𝒚𝒚 = 𝟑𝒚+𝟗
Proposed by Daniel Sitaru – Romania
Solution by Ravi Prakash-New Delhi-India
𝒙𝒙 = 𝟐𝒙+𝟒 > 𝟐𝒙 ⇒ 𝒙 > 2
𝟒
Also, 𝒙 𝐥𝐨𝐠 𝟐 𝒙 = 𝒙 + 𝟒 ⇒ 𝐥𝐨𝐠 𝟐 𝒙 = 𝟏 + 𝒙

90 RMM-ABSTRACT ALGEBRA MARATHON 601-700


www.ssmrmh.ro
𝟒
𝒚 = 𝐥𝐨𝐠 𝟐 𝒙 is an increasing function and 𝒚 = 𝟏 + 𝒙 is a decreasing function for 𝒙 > 2
𝟒
∴ 𝒚 = 𝐥𝐨𝐠 𝟐 𝒙 and 𝒚 = 𝟏 + 𝒙

meet in exactly one point viz. 𝒙 = 𝟒

Thus, 𝒙𝒙 = 𝟐𝒙+𝟒 ⇒ 𝒙 = 𝟒. Similarly 𝒚𝒚 = 𝟑𝒚+𝟗 ⇒ 𝒚 = 𝟗


𝟒𝒛 𝟒𝒛
Now: 𝒛𝟐 + + 𝐥𝐨𝐠 𝒚 𝒛 = 𝟗𝟏 ⇒ 𝒛𝟐 + + 𝐥𝐨𝐠 𝟗 (𝒛) = 𝟗𝟏
𝒙 𝟒

⇒ 𝒛𝟐 + 𝒛 + 𝐥𝐨𝐠 𝟗 (𝒛) = 𝟗𝟏 (1)


If 𝟎 < 𝑧 < 9, then 𝑳𝑯𝑺 of (1) < 91
If 𝒙 > 9 then LHS of (1) > 91
For 𝒛 = 𝟗, then LHS of (1) = 𝟗𝟏
Thus, 𝒙 = 𝟒, 𝒚 = 𝟗, 𝒛 = 𝟗
686. If 𝒑𝟏 , 𝒑𝟐 , 𝒑𝟑 … 𝒑𝟐𝒌 are in G.P. Only given that
𝒑𝒌−𝟑𝟕 𝒎𝒏 𝒎
= and 𝒑𝒌+𝟏𝟗 = then fiind the value of
𝒑𝟐𝒌−𝟕𝟑 𝒏𝒎 𝒏
𝒑𝟐𝒌−𝟏𝟑
[∑ 𝒑𝒊 𝒑𝒋 ] in terms of 𝒌, 𝒎, 𝒏
Proposed by SK Sabiruddin-India
Solution by Bui Hong Suc-Vietnam
Because: 𝒑𝟏 , 𝒑𝟐 , … , 𝒑𝟐𝒌 are in G.P.
𝒑𝒋 = 𝒑𝟏 𝒓𝒋−𝟏 (𝒋 ≥ 𝟐)

91 RMM-ABSTRACT ALGEBRA MARATHON 601-700


www.ssmrmh.ro
𝒑𝒌−𝟑𝟕 𝒎𝒏 𝒑𝟏 𝒓𝒌−𝟑𝟕−𝟏 𝒎𝒏
𝒌−𝟑𝟔 𝒏𝒎
𝒏𝒎 𝒓= √
= 𝒏𝒎 𝟐𝒌−𝟕𝟑−𝟏 = 𝒏𝒎 𝒓𝒌−𝟑𝟔=𝒎𝒏 𝒎𝒏
**{𝒑𝟐𝒌−𝟕𝟑 𝒎
↔ { 𝒑𝟏 𝒓 ↔{ 𝒎↔ 𝒌+𝟏𝟖
𝒑𝒌+𝟏𝟗 = 𝒏 𝒑𝟏 𝒓𝒌+𝟏𝟗−𝟏 = 𝒏
𝒎 𝒑𝟏 𝒓𝒌+𝟏𝟖 = 𝒏 𝒎 𝒎𝒏 𝒌−𝟑𝟔
{𝒑𝟏 = 𝒏
⋅ ( 𝒎)
𝒏
𝒌+𝟏𝟖 𝒋−𝟏 𝒌+𝟏𝟗−𝒋
𝒎 𝒎𝒏 𝒌−𝟑𝟔 𝒏𝒎 𝒌−𝟑𝟔 𝒎 𝒎𝒏 𝒌−𝟑𝟔
𝒑𝒋 = 𝒑𝟏 𝒓𝒋−𝟏 = ⋅ ( 𝒎) ⋅ ( 𝒏) = ⋅ ( 𝒎)
𝒏 𝒏 𝒎 𝒏 𝒏
𝒌+𝟏𝟗−𝒊
𝒎 𝒎𝒏 𝒌−𝟑𝟔
𝒑𝒊 = 𝒑𝟏 𝒓𝒊−𝟏 = ⋅ ( 𝒎) ; 𝒑𝟐𝒌−𝟏𝟑 = 𝒑𝟏 𝒓𝟐𝒌−𝟏𝟑−𝟏 =
𝒏 𝒏
𝒌+𝟏𝟗−𝟐𝒌+𝟏𝟑 𝟑𝟐−𝒌
𝒎 𝒎𝒏 𝒌−𝟑𝟔 𝒎 𝒎𝒏 𝒌−𝟑𝟔
= ⋅ ( 𝒎) = ⋅ ( 𝒎)
𝒏 𝒏 𝒏 𝒏
𝒌+𝟏𝟗−𝒊 𝒌+𝟏𝟗−𝒋 𝒌+𝟏𝟗 −𝒊−𝒋
𝒎 𝒎𝒏 𝒌−𝟑𝟔 𝒎 𝒎𝒏 𝒌−𝟑𝟔 𝒎 𝟐 𝒎𝒏 𝟐𝒌−𝟑𝟔 𝒎𝒏 𝒌−𝟑𝟔
→ 𝒑𝒊 ⋅ 𝒑𝒋 = ⋅ ( 𝒎 ) ( ) = ( ) ⋅ ( 𝒎) ( 𝒎)
𝒏 𝒏 𝒏 𝒏𝒎 𝒏 𝒏 𝒏
𝒌+𝟏𝟗 −𝒊−𝒋 𝒑𝟐𝒌−𝟏𝟑
𝒑𝟐𝒌−𝟏𝟑 𝒎 𝟐 𝒎𝒏 𝟐𝒌−𝟑𝟔 𝒎𝒏 𝒌−𝟑𝟔
∗∗ 𝑺 = [∑ 𝒑𝒊 ⋅ 𝒑𝒋 ] = [( ) ⋅ ( 𝒎 ) ∑( ) ]
𝒏 𝒏 𝒏𝒎

𝒌+𝟏𝟗 𝟐𝒌 −𝒊 −𝒋 𝒑𝟐𝒌−𝟏𝟑
𝟐𝒌
𝒎 𝟐 𝒎𝒏 𝟐𝒌−𝟑𝟔 𝒎𝒏 𝒌−𝟑𝟔 𝒎𝒏 𝒌−𝟑𝟔
= [( ) ⋅ ( 𝒎 ) ∑ ( 𝒎) ∑ ( 𝒎) ]
𝒏 𝒏 𝒏 𝒏
𝒊=𝟏 𝒋=𝟏

−(𝟐𝒌+𝟏) 𝟐 𝒑𝟐𝒌−𝟏𝟑
𝒏
𝒌+𝟏𝟗 𝒎 𝒌−𝟑𝟔
𝒎 𝟐 𝒎𝒏 𝟐⋅𝒌−𝟑𝟔 𝟏 − ( 𝒏𝒎 )
= ( ) ⋅ ( 𝒎) −𝟏 + −𝟏 =
𝒏 𝒏
𝒎𝒏 𝒌−𝟑𝟔
𝟏 − ( 𝒏𝒎 )
[ ( ) ]
−𝟐𝒌 𝟐 𝒑𝟐𝒌−𝟏𝟑
𝟐𝒌+𝟑𝟔 𝒎𝒏 𝒌−𝟑𝟔
𝒎 𝟐 𝒎𝒏 𝒌−𝟑𝟔 𝟏− ( 𝒏𝒏 )
= ( ) ⋅ ( 𝒎) −𝟏
𝒏 𝒏
𝒎𝒏 𝒌−𝟑𝟔
𝟏 − ( 𝒏𝒎 )
[ ( ) ]
𝟑𝟐−𝒌
𝒎 𝒎𝒏 𝒌−𝟑𝟔
−𝟐𝒌 𝟐 𝒏 ⋅(𝒏𝒎 )
𝟐𝒌+𝟑𝟔 𝒎𝒏 𝒌−𝟑𝟔
𝒎 𝟐 𝒎𝒏 𝒌−𝟑𝟔 𝟏 − ( 𝒎)
𝒏
= ( ) ⋅ ( 𝒎) −𝟏
𝒏 𝒏
𝒎𝒏 𝒌−𝟑𝟔
𝟏 − ( 𝒏𝒎 )
[ ( ) ]

92 RMM-ABSTRACT ALGEBRA MARATHON 601-700


www.ssmrmh.ro
𝝅
𝒔𝒊𝒏(𝑨−𝟔 )
687. If 𝜴 = ∑ 𝒄𝒚𝒄 𝝅 𝝅 , in △ 𝑨𝑩𝑪, then solve for real numbers:
𝒄𝒐𝒔(𝑩− 𝟔 ) 𝒄𝒐𝒔(𝑪−𝟔 )
𝒙𝟒 − 𝟒𝜴𝒙𝟑 + 𝟔𝜴𝒙𝟐 − 𝟒𝜴𝒙 + 𝟏 = 𝟎
Proposed by Daniel Sitaru-Romania
Solution 1 by Pham Duc Nam-Vietnam, Solution 2 by Tapas Das-India

Solution 1 by Pham Duc Nam-Vietnam


𝝅 𝝅 𝝅 𝝅
Let: 𝒙 = 𝑨 − ,𝒚 = 𝑩 − ,𝒛 = 𝑪 − ⇒ 𝒙+ 𝒚+ 𝒛 =
𝟔 𝟔 𝟔 𝟐
𝝅
𝒔𝒊𝒏 (𝑨 − 𝟔 ) 𝒔𝒊𝒏(𝒙)
∑ 𝝅 𝝅 = ∑
𝒄𝒐𝒔(𝒚) 𝒄𝒐𝒔(𝒛)
𝒄𝒚𝒄 𝒄𝒐𝒔 (𝑩 − 𝟔 ) 𝒄𝒐𝒔 (𝑪 − 𝟔 ) 𝒄𝒚𝒄
𝒔𝒊𝒏(𝒙) 𝒔𝒊𝒏(𝒚) 𝒔𝒊𝒏(𝒛)
= + +
𝒄𝒐𝒔(𝒚) 𝒄𝒐𝒔(𝒛) 𝒄𝒐𝒔(𝒙) 𝒄𝒐𝒔(𝒛) 𝒄𝒐𝒔(𝒚) 𝒄𝒐𝒔(𝒙)
𝒔𝒊𝒏 𝒙 𝒄𝒐𝒔 𝒙 + 𝒔𝒊𝒏(𝒚) 𝒄𝒐𝒔(𝒚) + 𝒔𝒊𝒏(𝒛) 𝒄𝒐𝒔(𝒛) 𝟏 𝒔𝒊𝒏(𝟐𝒙) + 𝒔𝒊𝒏(𝟐𝒚) + 𝒔𝒊𝒏(𝟐𝒛)
( ) ( )
= = .
𝒄𝒐𝒔(𝒙) 𝒄𝒐𝒔(𝒚) 𝒄𝒐𝒔(𝒛) 𝟐 𝒄𝒐𝒔(𝒙) 𝒄𝒐𝒔(𝒚) 𝒄𝒐𝒔(𝒛)
𝟏 𝟐 𝒔𝒊𝒏(𝒙 + 𝒚) 𝒄𝒐𝒔(𝒙 − 𝒚) + 𝒔𝒊𝒏(𝟐𝒛) 𝟏 𝟐 𝒄𝒐𝒔(𝒛) 𝒄𝒐𝒔(𝒙 − 𝒚) + 𝒔𝒊𝒏(𝟐𝒛)
= . = .
𝟐 𝒄𝒐𝒔(𝒙) 𝒄𝒐𝒔(𝒚) 𝒄𝒐𝒔(𝒛) 𝟐 𝒄𝒐𝒔(𝒙) 𝒄𝒐𝒔(𝒚) 𝒄𝒐𝒔(𝒛)
𝟏 𝟐 𝒄𝒐𝒔(𝒛) (𝒄𝒐𝒔(𝒙 − 𝒚) + 𝒔𝒊𝒏(𝒛)) 𝒄𝒐𝒔(𝒛) (𝒄𝒐𝒔(𝒙 − 𝒚) + 𝒔𝒊𝒏(𝒛))
= . =
𝟐 𝒄𝒐𝒔(𝒙) 𝒄𝒐𝒔(𝒚) 𝒄𝒐𝒔(𝒛) 𝒄𝒐𝒔(𝒙) 𝒄𝒐𝒔(𝒚) 𝒄𝒐𝒔(𝒛)
𝒄𝒐𝒔(𝒛) (𝒄𝒐𝒔(𝒙 − 𝒚) + 𝒔𝒊𝒏(𝒛)) 𝒄𝒐𝒔(𝒛) (𝒄𝒐𝒔(𝒙 − 𝒚) + 𝒔𝒊𝒏(𝒛))
= 𝟐. = 𝟐. =𝟐
𝒄𝒐𝒔(𝒛) (𝒄𝒐𝒔(𝒙 + 𝒚) + 𝒄𝒐𝒔(𝒙 − 𝒚)) 𝒄𝒐𝒔(𝒛) (𝒄𝒐𝒔(𝒙 − 𝒚) + 𝒔𝒊𝒏(𝒛))
∗ 𝒙𝟒 − 𝟒𝜴𝒙𝟑 + 𝟔𝜴𝒙𝟐 − 𝟒𝜴𝒙 + 𝟏 = 𝟎
⇔ 𝒙𝟒 − 𝟖𝒙𝟑 + 𝟏𝟐𝒙𝟐 − 𝟖𝒙 + 𝟏 = 𝟎, since 𝒙 = 𝟎 is not solution, then divide both sides by 𝒙𝟐
𝟖 𝟏 𝟏 𝟐 𝟏
⇒ 𝒙𝟐 − 𝟖𝒙 + 𝟏𝟐 − + 𝟐 = 𝟎 ⇔ (𝒙 + ) − 𝟖 (𝒙 + ) + 𝟏𝟎 = 𝟎
𝒙 𝒙 𝒙 𝒙
𝟏
𝑥 + = 𝟒 + √𝟔 𝒙𝟐 − (𝟒 + √𝟔)𝒙 + 𝟏 = 𝟎
⇔[ 𝒙 ⇔[ 𝟐
𝟏 𝒙 − (𝟒 − √𝟔)𝒙 + 𝟏 = 𝟎(this equation has no real roots)
𝑥 + = 𝟒 − √𝟔
𝒙
𝟏 𝟏
𝑥= (√𝟔 + 𝟒) − √ (𝟒√𝟔 + 𝟗)
𝟐 𝟐

𝟏 𝟏
𝑥 = (√𝟔 + 𝟒) + √ (𝟒√𝟔 + 𝟗)
[ 𝟐 𝟐

Solution 2 by Tapas Das-India


Let
𝝅 𝝅 𝝅
𝒙 = 𝟐𝑨 − , 𝒚 = 𝟐𝑩 − , 𝒛 = 𝟐𝑪 −
𝟑 𝟑 𝟑

93 RMM-ABSTRACT ALGEBRA MARATHON 601-700


www.ssmrmh.ro
∴ 𝒙 + 𝒚 + 𝒛 = 𝟐 (∑ 𝑨) − 𝝅 = 𝟐𝝅 − 𝝅 = 𝝅

∴ 𝒙, 𝒚, 𝒛 are angles of a triangle


Let 𝑹, 𝒓 are the circumradius on in-radius of the new triangle
𝒔 = semi-perimeter
𝝅 𝒙
𝐬𝐢𝐧 (𝑨 − 𝟔 ) 𝐬𝐢𝐧 𝟐 ∑ 𝐬𝐢𝐧 𝒙
∴∑ 𝝅 𝝅 = ∑ 𝒚 𝒛 = 𝒙
𝐜𝐨𝐬 (𝑩 − 𝟔 ) 𝐜𝐨𝐬 (𝑪 − 𝟔 ) 𝐜𝐨𝐬 𝟐 𝐜𝐨𝐬 𝟐 𝟐𝝅 𝐜𝐨𝐬 𝟐
𝒔
𝒓 𝒔 𝟐𝑹
= 𝒔 = × =𝟐
𝟐 ⋅ 𝟒𝑹 𝑹 𝒔

∴𝛀=𝟐
𝒙𝟒 − 𝟒𝑹𝒙𝟑 + 𝟔𝑹𝒙𝟐 − 𝟒𝛀𝒙 + 𝟏 = 𝟎, 𝒙𝟒 − 𝟖𝒙𝟑 + 𝟏𝟐𝒙𝟐 − 𝟖𝒙 + 𝟏 = 𝟎
𝟏 𝟏
(𝒙𝟒 + 𝟏) − 𝟖(𝒙𝟑 + 𝒙) + 𝟏𝟐𝒙𝟐 = 𝟎, (𝒙𝟐 + 𝟐
) − 𝟖 (𝒙 + ) + 𝟏𝟐 = 𝟎
𝒙 𝒙
𝟏 𝟐 𝟏 𝟏
(𝒙 + ) − 𝟖 (𝒙 + ) + 𝟏𝟎 = 𝟎, 𝒑𝟐 − 𝟖𝒑 + 𝟏𝟎 = 𝟎 [𝒙 + = 𝒑]
𝒙 𝒙 𝒙
𝟖 ± √𝟔𝟒 − 𝟒𝟎
𝒑= = (𝟒 ± √𝟔)
𝟐
𝟏
∴ 𝒙 + = (𝟒 + √𝟔)
𝒙
∴ 𝒙𝟐 − 𝒙(𝟒 + √𝟔) + 𝟏 = 𝟎

𝟐
(𝟒 + √𝟔) ± √(𝟒 + √𝟔) − 𝟒 𝟏
∴𝒙= = [𝟒 + √𝟔 ± √𝟏𝟖 + 𝟖√𝟔]
𝟐 𝟐
𝟏
𝒙+ = 𝟒 − √𝟔
𝒙
𝟏
∴ 𝒙𝟐 − 𝒙(𝟒 − √𝟔) + 𝟏 = 𝟎, 𝒙= [𝟒 − √𝟔 ± √𝟏𝟖 − 𝟖√𝟔]
𝟐
𝟏
Four real roots 𝒙 = 𝟐 [𝟒 + √𝟔 ± √𝟏𝟖 + 𝟖√𝟔]

94 RMM-ABSTRACT ALGEBRA MARATHON 601-700


www.ssmrmh.ro
688. Prove that:
𝟑 𝑳𝟒𝒏 +𝑳𝟒𝒏+𝟏 𝑳𝟒𝒏+𝟏 +𝑳𝟒𝒏+𝟑 𝑳𝟒𝒏+𝟑 +𝑳𝟒𝒏
( + + )≥𝟏
𝟐𝑳𝟐𝒏+𝟒 𝑳𝒏 𝑳𝒏+𝟏 𝑳𝒏+𝟏 𝑳𝒏+𝟑 𝑳𝒏+𝟑 𝑳𝒏

where 𝑳𝒊 is i-th Lucas number.


Proposed by D.M. Bătinețu-Giurgiu, Neculai Stanciu – Romania
Solution by Tapas Das – India

𝟑 𝑳𝟒𝒏 + 𝑳𝟒𝒏+𝟏 𝑳𝟒𝒏+𝟏 + 𝑳𝟒𝒏+𝟑 𝑳𝟒𝒏+𝟑 + 𝑳𝟒𝒏


( + + )
𝟐𝑳𝟐𝒏+𝟒 𝑳𝒏 𝑳𝒏+𝟏 𝑳𝒏+𝟏 𝑳𝒏+𝟑 𝑳𝒏+𝟑 𝑳𝒏
𝟑 𝟏 𝑳𝟑𝒏 𝑳𝟑𝒏+𝟏 𝑳𝟑𝒏+𝟑 𝑳𝟑𝒏+𝟏 𝑳𝟑𝒏+𝟑 𝑳𝟑𝒏
= [( + + ) + ( + + )]
𝟐 𝑳𝟐𝒏+𝟒 𝑳𝒏+𝟏 𝑳𝒏+𝟑 𝑳𝒏 𝑳𝒏 𝑳𝒏+𝟏 𝑳𝒏+𝟑
𝑯𝒐𝒍𝒅𝒆𝒓 𝟑 𝟏 (𝑳𝒏 + 𝑳𝒏+𝟏 + 𝑳𝒏+𝟑 )𝟑 𝟏 (𝑳𝒏 + 𝑳𝒏+𝟑 + 𝑳𝒏+𝟏 )𝟑
≥ [ + ]
𝟐𝑳𝟐𝒏+𝟒 𝟑 𝑳𝒏+𝟏 + 𝑳𝒏+𝟑 + 𝑳𝒏 𝟑 (𝑳𝒏 + 𝑳𝒏+𝟏 + 𝑳𝒏+𝟑 )
𝟑 𝟏 (𝑳𝒏+𝟐 + 𝑳𝒏+𝟑 )𝟑 𝟏 (𝑳𝒏+𝟐 + 𝑳𝒏+𝟑 )𝟑
= [ + ]
𝟐𝑳𝟐𝒏+𝟒 𝟑 (𝑳𝒏+𝟐 + 𝑳𝒏+𝟑 ) 𝟑 𝑳𝒏+𝟐 + 𝑳𝒏+𝟑
𝟑 𝟏 𝑳𝟒𝒏+𝟒 𝟏 𝑳𝟑𝒏+𝟒 𝟑 𝟐 𝑳𝒏+𝟒
= 𝟐
[ + ]= 𝟐 ⋅ ⋅ =𝟏
𝟐𝑳𝒏+𝟒 𝟑 𝑳𝒏+𝟒 𝟑 𝑳𝒏+𝟒 𝟐𝑳𝒏+𝟒 𝟑 𝑳𝒏+𝟒
689. Prove that:

√𝟐(𝟏 + 𝑭𝟒𝒏 ) − 𝟐𝑭𝒏 𝑭𝒏+𝟏 + ∑𝒏−𝟏 𝟒 𝟒


𝒌=𝟏 √(𝑭𝒌 + 𝟏)(𝑭𝒌+𝟏 + 𝟏) > 0,

for any positive integer 𝒏 > 1, where 𝑭𝒏 in n-th Fibonacci number.


Proposed by D.M. Bătinețu-Giurgiu, Neculai Stanciu – Romania
Solution by Tapas Das – India
𝑪𝒂𝒖𝒄𝒉𝒚−𝑺𝒄𝒉𝒘𝒂𝒓𝒛
√(𝑭𝟒𝒌 + 𝟏)(𝑭𝟒𝒌+𝟏 + 𝟏) ≥ (𝑭𝟐𝒌 + 𝑭𝟐𝒌+𝟏 )

𝒏−𝟏 𝒏−𝟏 𝒏 𝒏−𝟏

∴∑ √(𝑭𝟒𝒌 + 𝟏)(𝑭𝟒𝒌+𝟏 + 𝟏) ≥ ∑(𝑭𝟐𝒌 + 𝑭𝟐𝒌+𝟏 ) = ∑ 𝑭𝟐𝒌 + ∑ 𝑭𝟐𝒌+𝟏 − 𝑭𝟐𝒏


𝒌=𝟏 𝒌=𝟏 𝒌=𝟏 𝒌=𝟏

= 𝑭𝒏 𝑭𝒏+𝟏 + (𝑭𝟐𝟏 + 𝑭𝟐𝟐 + ⋯ + 𝑭𝟐𝒏 ) − 𝑭𝟐𝒏 − 𝑭𝟐𝟏


= 𝟐𝑭𝒏 𝑭𝒏+𝟏 − 𝑭𝟐𝒏 − 𝟏 (1) ( ∵ 𝑭𝟏 = 𝟏 )

Now √𝟐(𝟏 + 𝑭𝟒𝒏 ) = √𝟐(𝟏𝟐 + 𝑭𝟐𝒏 ) > √(𝑭𝟐𝒏 + 𝟏)𝟐 = 𝑭𝟐𝒏 + 𝟏

95 RMM-ABSTRACT ALGEBRA MARATHON 601-700


www.ssmrmh.ro
𝒏−𝟏

∴ √𝟐(𝟏 + 𝑭𝟒𝒏 ) − 𝟐𝑭𝒏 𝑭𝒏+𝟏 + ∑(𝑭𝟒𝒌 + 𝟏)(𝑭𝟒𝒌+𝟏 + 𝟏)


𝒌=𝟏

> 𝑭𝟐𝒏+𝟏 − 𝟐𝑭𝒏 𝑭𝒏+𝟏 + 𝟐𝑭𝒏 𝑭𝒏+𝟏 − 𝑭𝟐𝒏 − 𝟏(using (1))= 𝟎


690.
𝐧 𝐧
𝟏 𝟐𝐦+𝟒
𝟏
𝐏𝐫𝐨𝐯𝐞 𝐭𝐡𝒂𝐭 ∶ (∑ 𝐅𝐤 ) (∑ ) ≥ 𝟏;
(𝐅𝐧𝐅𝐧+𝟏)𝟐 𝐅𝐤𝟐𝐦
𝐤=𝟏 𝐤=𝟏
𝐧 𝐧 𝐧 𝐧
𝟏 𝟏 𝟏 𝟏
(∑ 𝐋𝟐𝐦+𝟒
𝐤 ) (∑ 𝟐𝐦 ) ≥ 𝟏; (∑ 𝐅𝐤
𝐦+𝟐
) (∑ )≥𝟏
(𝐋𝐧𝐋𝐧+𝟏 − 𝟐)𝟐
𝐋 (𝐅𝐧+𝟐 − 𝟏 )𝟐 𝐅𝐤𝐦
𝐤=𝟏 𝐤=𝟏 𝐤 𝐤=𝟏 𝐤=𝟏

𝐰𝐡𝐞𝐫𝐞 𝐅𝐤 𝐢𝐬 𝐤 − 𝐭𝐡 𝐅𝐢𝐛𝐨𝐧𝒂𝐜𝐜𝐢 𝐧𝐮𝐦𝐛𝐞𝐫 𝒂𝐧𝐝 𝐋𝐤 𝐢𝐬 𝐤 − 𝐭𝐡 𝐋𝐮𝐜𝒂𝐬 𝐧𝐮𝐦𝐛𝐞𝐫


Proposed by D.M.Bătinețu-Giurgiu, Neculai Stanciu-Romania
Solution 1 by Tapas Das-India
𝒏

∑ 𝑭𝟐𝒌 = 𝑭𝒏 𝑭𝒏+𝟏
𝒌=𝟏

Let ∀𝒏 ∈ ℕ, let 𝑷(𝒌): ∑𝒏𝒌=𝟏 𝑭𝟐𝒌 = 𝑭𝒏 𝑭𝒏+𝟏 . 𝑷(𝟏) is true because 𝑭𝟐𝟏 = 𝟏 = 𝑭𝟑 − 𝟏
Holds from definition
𝒏

∴ ∑ 𝑭𝟐𝒌 = 𝑭𝟐𝟏 = (𝒙) = 𝑭𝟏 × 𝑭𝟐


𝒌=𝟏

Let 𝑷(𝒌) is true, we need to show 𝑷(𝒌 + 𝟏) is true


𝒌

∑ 𝑭𝟐𝒋 = 𝑭𝒌 𝑭𝒌+𝟏
𝒋=𝟏

𝒌+𝟏 𝒌

∴ ∑ 𝑭𝟐𝒋 = ∑ 𝑭𝟐𝒋 + 𝑭𝟐𝒌+𝟏 = 𝑭𝒌 𝑭𝒌+𝟏 + 𝑭𝟐𝒌+𝟏 = (𝑭𝒌 + 𝑭𝒌+𝟏 ) ⋅ 𝑭𝒌+𝟏 = 𝑭𝒌+𝟐 𝑭𝒌+𝟏
𝒋=𝟏 𝒋=𝟏

∴ 𝑷(𝒌 + 𝟏) is true
Part 1
𝒏 𝒏 𝒏 𝒏 𝒏 𝟐
𝟏 𝒎+𝟐 𝟐
𝟏 𝑪𝒂𝒖𝒄𝒉𝒚−𝑺𝒄𝒉𝒘𝒂𝒓𝒛 𝑭𝒎+𝟐
𝒌
(∑ 𝑭𝟐𝒎+𝟒
𝒌 ) (∑ ) = (∑(𝑭 𝒌 ) ) (∑ ) ≥ (∑ 𝒎 )
𝑭𝟐𝒎
𝒌
𝒎 𝟐
(𝑭 ) 𝑭 𝒌
𝒌=𝟏 𝒌=𝟏 𝒌=𝟏 𝒌=𝟏 𝒌 𝒌=𝟏

96 RMM-ABSTRACT ALGEBRA MARATHON 601-700


www.ssmrmh.ro
𝒏 𝟐

= (∑ 𝑭𝟐𝒌 ) = (𝑭𝒌 𝑭𝒌+𝟏 )𝟐


𝒌=𝟏
𝒏 𝒏 𝟐
𝟏 𝟐𝒎+𝟒
𝟏 𝟏
∴ ⋅ (∑ 𝑭𝒌 ) (∑ 𝟐𝒎 ) ≥ ( ) ⋅ (𝑭𝒏 ⋅ 𝑭𝒏+𝟏 )𝟐 = 𝟏
(𝑭𝒏 ⋅ 𝑭𝒏+𝟏 )𝟐 𝑭𝒌 𝑭𝒏 𝑭𝒏+𝟏
𝒌=𝟏 𝒌=𝟏

Part 2
𝒏 𝒏
𝟏 𝟏
(∑ 𝑳𝟐𝒎+𝟒
𝒌 ) ⋅ (∑ )
(𝑳𝒏 𝑳𝒏+𝟏 − 𝟏) 𝟐 𝑳𝟐𝒎
𝒌
𝒌=𝟏 𝒌=𝟏
𝒏 𝒏 𝟐 𝟐
𝟏 𝟐 𝟏 𝟏 𝑳𝒎+𝟐
𝒌
= (∑(𝑳𝒎+𝟐
𝒌 ) ⋅ (∑ ( 𝒎 ) )) ≥ (∑ )
(𝑳𝒏 𝑳𝒏+𝟏 − 𝟏) 𝟐 𝑳𝒌 (𝑳𝒏 𝑳𝒏+𝟏 − 𝟏)𝟐 𝑳𝒎𝒌
𝒌=𝟏 𝒌=𝟏

𝟏 𝟐 𝟏
= 𝟐
⋅ (∑ 𝑳𝟐𝒌 ) = ⋅ (𝑳𝒏 𝑳𝒏+𝟏 − 𝟏)𝟐 = 𝟏
(𝑳𝒏 𝑳𝒏+𝟏 − 𝟏) (𝑳𝒏 𝑳𝒏+𝟏 − 𝟏)𝟐
𝟐 𝟐
𝒏 𝒏 𝒏 𝒏
𝟏 𝑭𝒎+𝟐
(∑ 𝑭𝒌 ) (∑ 𝒎 ) ≥ (∑ √ 𝒌 𝒎 ) = (∑ 𝑭𝒌 ) = (𝑭𝒏+𝟐 − 𝟏)𝟐
𝒎+𝟐
𝑭𝒌 𝑭𝒌
𝒌=𝟏 𝒌=𝟏 𝒌=𝟏 𝒌=𝟏

𝒏 𝒏 𝟐
𝟏 𝒎+𝟐
𝟏 𝟏
∴ (∑ 𝑭 𝒌 ) (∑ ) ≥ ( ) (𝑭𝒏+𝟐 − 𝟏)𝟐 = 𝟏
(𝑭𝒏+𝟐 − 𝟏)𝟐 𝑭𝒎
𝒌 𝑭 𝒏+𝟐 − 𝟏
𝒌=𝟏 𝒌=𝟏

Note:
𝑭(𝒏) = 𝑭(𝒏 + 𝟐) − 𝑭(𝒏 + 𝟏)
𝑭(𝒏 − 𝟏) = 𝑭(𝒏 + 𝟏) − 𝑭(𝒏)
−−−−−−−−−−−−
−−−−−−−−−−−−
𝑭( 𝟏 ) = 𝑭( 𝟑 ) − 𝑭( 𝟐 )
Sum = 𝑭(𝒏 + 𝟐) − 𝑭(𝟐) … adding all
= 𝑭( 𝒏 + 𝟐 ) − 𝟏
Solution 2 by Soumava Chakraborty-Kolkata-India
𝐂𝐡𝐞𝐛𝐲𝐬𝐡𝐞𝐯
𝐧 𝐧 + 𝐧 𝐧
𝟏 𝐁𝐞𝐫𝐠𝐬𝐭𝐫𝐨𝐦 𝟏 𝟗
(∑ 𝐅𝐤𝟐𝐦+𝟒 ) (∑ 𝟐𝐦 ) ≥ (∑ 𝐅𝐤𝟐𝐦 ) (∑ 𝐅𝐤𝟒 ) . 𝐧
𝐅 𝟑 ∑𝐤=𝟏 𝐅𝐤𝟐𝐦
𝐤=𝟏 𝐤=𝟏 𝐤 𝐤=𝟏 𝐤=𝟏

97 RMM-ABSTRACT ALGEBRA MARATHON 601-700


www.ssmrmh.ro
𝐧 𝟐 𝐧 𝐧
𝟏 𝟏 𝟏
≥ 𝟑. (∑ 𝐅𝐤𝟐 ) = (𝐅𝐧 𝐅𝐧+𝟏 )𝟐 ⇒ 𝟐
(∑ 𝐅𝐤𝟐𝐦+𝟒 ) (∑ 𝟐𝐦 ) ≥ 𝟏
𝟑 (𝐅𝐧 𝐅𝐧+𝟏 ) 𝐅𝐤
𝐤=𝟏 𝐤=𝟏 𝐤=𝟏
𝐂𝐡𝐞𝐛𝐲𝐬𝐡𝐞𝐯
𝐧 𝐧 + 𝐧 𝐧
𝟏 𝐁𝐞𝐫𝐠𝐬𝐭𝐫𝐨𝐦 𝟏 𝟗
𝐀𝐥𝐬𝐨, (∑ 𝐋𝟐𝐦+𝟒
𝐤 ) (∑ 𝟐𝐦 ) ≥ (∑ 𝐋𝟐𝐦 𝟒
𝐤 ) (∑ 𝐋𝐤 ) . 𝐧
𝐋 𝟑 ∑𝐤=𝟏 𝐋𝟐𝐦
𝐤=𝟏 𝐤=𝟏 𝐤 𝐤=𝟏 𝐤=𝟏𝐤
𝐧 𝟐 𝐧 𝐧
𝟏 𝟏 𝟏
≥ 𝟑. (∑ 𝐋𝟐𝐤 ) = (𝐋𝐧 𝐋𝐧+𝟏 − 𝟐)𝟐 ⇒ 𝟐
(∑ 𝐋𝟐𝐦+𝟒
𝐤 ) (∑ 𝟐𝐦 ) ≥ 𝟏
𝟑 (𝐋𝐧 𝐋𝐧+𝟏 − 𝟐) 𝐋𝐤
𝐤=𝟏 𝐤=𝟏 𝐤=𝟏
𝐂𝐡𝐞𝐛𝐲𝐬𝐡𝐞𝐯
𝐧 𝐧 + 𝐧 𝐧
𝟏 𝐁𝐞𝐫𝐠𝐬𝐭𝐫𝐨𝐦 𝟏 𝟗
𝐀𝐠𝒂𝐢𝐧, (∑ 𝐅𝐤𝐦+𝟐 ) (∑ 𝐦 ) ≥ (∑ 𝐅𝐤𝐦 ) (∑ 𝐅𝐤𝟐 ) . 𝐧
𝐅 𝟑 ∑𝐤=𝟏 𝐅𝐤𝐦
𝐤=𝟏 𝐤=𝟏 𝐤 𝐤=𝟏 𝐤=𝟏
𝐧 𝟐 𝐧 𝐧
𝟏 𝟏 𝟏
≥ 𝟑. (∑ 𝐅𝐤 ) = (𝐅𝐧+𝟐 − 𝟏 )𝟐 ⇒ 𝟐
(∑ 𝐅𝐤𝐦+𝟐 ) (∑ 𝐦 ) ≥ 𝟏 (𝐐𝐄𝐃)
𝟑 (𝐅𝐧+𝟐 − 𝟏 ) 𝐅𝐤
𝐤=𝟏 𝐤=𝟏 𝐤=𝟏

691. Determine the nature of the triangle 𝑨𝑩𝑪 if the equality occurs:
𝟐 +𝒙
𝒍𝒐𝒈𝟑 (𝒙) + 𝟑𝒙 = 𝟏 + 𝟐𝟕𝒙+𝟏 , 𝒙 = 𝟐𝟒𝒄𝒐𝒔𝑨𝒄𝒐𝒔𝑩𝒄𝒐𝒔𝑪
Proposed by Radu Diaconu-Romania
Solution by Pham Duc Nam-Vietnam

𝟐 +𝒙 𝒙 𝟐
𝒍𝒐𝒈𝟑 (𝒙) + 𝟑𝒙 = 𝟏 + 𝟐𝟕𝒙+𝟏 ⇔ 𝒍𝒐𝒈𝟑 ( ) = 𝟑𝟑𝒙+𝟑 − 𝟑𝒙 +𝒙
𝟑
𝒙 𝟐
∗ 𝒙 > 0. Let: 𝒇(𝒙) = 𝒍𝒐𝒈𝟑 ( ) , 𝒈(𝒙) = 𝟑𝟑𝒙+𝟑 − 𝟑𝒙 +𝒙
𝟑
Clearly: 𝒇(𝒙) is monotone strictly increasing function
𝒙
∗ If: 𝟎 < 𝒙 < 3 ⇒ 𝒍𝒐𝒈𝟑 ( ) < 0, and 3𝒙 + 𝟑 − 𝒙𝟐 − 𝒙 = −𝒙𝟐 − 𝟐𝒙 + 𝟑,
𝟑
and − 𝒙𝟐 − 𝟐𝒙 + 𝟑 > 0 ⇔ 𝟎 < 𝑥 < 3
𝟐 𝒙 𝟐
⇒ 𝟑𝒙 + 𝟑 > 𝒙𝟐 + 𝒙 ⇒ 𝟑𝟑𝒙+𝟑 − 𝟑𝒙 +𝒙 > 0 ⇒ 𝒍𝒐𝒈𝟑 ( ) = 𝟑𝟑𝒙+𝟑 − 𝟑𝒙 +𝒙 has no solution where: 𝟎
𝟑
<𝒙<3
𝟐 𝟐
∗ If: 𝒙 ≥ 𝟑, 𝒈′(𝒙) = 𝟑𝒙 (𝟗𝟐+𝒙 − 𝟑𝒙 (𝟏 + 𝟐𝒙)) 𝒍𝒏(𝟑) , we have: 𝟗𝟐+𝒙 − 𝟑𝒙 (𝟏 + 𝟐𝒙)
𝟐 𝟐
= 𝟑𝟐𝒙+𝟒 − 𝟑𝒙 (𝟏 + 𝟐𝒙) < 𝟑𝟐𝒙+𝟒 − 𝟑𝒙 +𝟏 , since 1+2𝒙 ≥ 𝟕 > 3
Also: 𝟐𝒙 + 𝟒 ≤ 𝒙𝟐 + 𝟏∀𝒙 ≥ 𝟑 ⇒ 𝒈′(𝒙) < 0∀𝒙 ≥ 𝟑
⇒ 𝒈(𝒙) is monotone strictly decreasing function where 𝒙 ≥ 𝟑
And 𝒇(𝒙) is monotone strictly increasing function ⇒ If solution of 𝒇(𝒙)
= 𝒈(𝒙) exists then it is unique.
𝟐
𝒙 = 𝟑 is satisfied ⇒ 𝒍𝒐𝒈𝟑 (𝒙) + 𝟑𝒙 +𝒙 = 𝟏 + 𝟐𝟕𝒙+𝟏 ⇔ 𝒙 = 𝟑

98 RMM-ABSTRACT ALGEBRA MARATHON 601-700


www.ssmrmh.ro
𝟏
∗ 𝟐𝟒 𝒄𝒐𝒔 𝑨 𝒄𝒐𝒔 𝑩 𝒄𝒐𝒔 𝑪 = 𝒙 ⇔ 𝒄𝒐𝒔 𝑨 𝒄𝒐𝒔 𝑩 𝒄𝒐𝒔 𝑪 =
𝟖
𝟏
But: 𝒄𝒐𝒔 𝑨 𝒄𝒐𝒔 𝑩 𝒄𝒐𝒔 𝑪 ≤ , equality only holds if △ ABC is 𝐞𝐪𝐮𝐢𝐥𝐚𝐭𝐞𝐫𝐚𝐥 𝐭𝐫𝐢𝐚𝐧𝐠𝐥𝐞.
𝟖
⇒△ ABC is 𝐞𝐪𝐮𝐢𝐥𝐚𝐭𝐞𝐫𝐚𝐥 𝐭𝐫𝐢𝐚𝐧𝐠𝐥𝐞.
𝟏
∗ Note: in △ ABC, 𝒄𝒐𝒔 𝑨 𝒄𝒐𝒔 𝑩 𝒄𝒐𝒔 𝑪 ≤ .
𝟖
𝝅 𝟏
If one angle is greater or equal ⇒ 𝒄𝒐𝒔 𝑨 or 𝒄𝒐𝒔 𝑩 or 𝒄𝒐𝒔 𝑪 ≤ 𝟎 ⇒ 𝒄𝒐𝒔 𝑨 𝒄𝒐𝒔 𝑩 𝒄𝒐𝒔 𝑪 ≤ 𝟎 <
𝟐 𝟖
𝝅
If all angles is lesser than , by Jensen iequality with the fact that
𝟐
𝝅
function 𝒚 = 𝒄𝒐𝒔(𝒙) is concave function on (𝟎, )
𝟐
𝟏 𝑨+𝑩+𝑪 𝝅
⇒ (𝒄𝒐𝒔 𝑨 + 𝒄𝒐𝒔 𝑩 + 𝒄𝒐𝒔 𝑪) ≤ 𝒄𝒐𝒔 ( ) = 𝒄𝒐𝒔
𝟑 𝟑 𝟑
𝟏 𝟑
= , by AM-GM: 𝒄𝒐𝒔 𝑨 + 𝒄𝒐𝒔 𝑩 + 𝒄𝒐𝒔 𝑪 ≥ 𝟑 √𝒄𝒐𝒔 𝑨 𝒄𝒐𝒔 𝑩 𝒄𝒐𝒔 𝑪
𝟐
𝟑 𝟏
⇒ √𝒄𝒐𝒔 𝑨 𝒄𝒐𝒔 𝑩 𝒄𝒐𝒔 𝑪 ≤ (𝒄𝒐𝒔 𝑨 + 𝒄𝒐𝒔 𝑩 + 𝒄𝒐𝒔 𝑪)
𝟑
𝟑 𝟏 𝟏
⇒ √𝒄𝒐𝒔 𝑨 𝒄𝒐𝒔 𝑩 𝒄𝒐𝒔 𝑪 ≤ ⇒ 𝒄𝒐𝒔 𝑨 𝒄𝒐𝒔 𝑩 𝒄𝒐𝒔 𝑪 ≤ .
𝟐 𝟖
𝝅
692. Find 𝒙 ∈ (𝟎, ) such that:
𝟐

𝒕𝒂𝒏(𝟑𝒙) = 𝒕𝒂𝒏(𝒙) 𝒕𝒂𝒏(𝟏𝟑𝒙) 𝒕𝒂𝒏(𝟏𝟏𝒙)


Proposed by Carlos Paiva-Brazil
Solution by Pham Duc Nam-Vietnam

∗ 𝒄𝒐𝒔(𝒙) , 𝒄𝒐𝒔(𝟑𝒙) , 𝒄𝒐𝒔(𝟏𝟏𝒙) , 𝒄𝒐𝒔(𝟏𝟑𝒙) ≠ 𝟎

𝒔𝒊𝒏(𝟑𝒙) 𝒔𝒊𝒏(𝒙) 𝒔𝒊𝒏(𝟏𝟑𝒙) 𝒔𝒊𝒏(𝟏𝟏𝒙)


𝒕𝒂𝒏(𝟑𝒙) = 𝒕𝒂𝒏(𝒙) 𝒕𝒂𝒏(𝟏𝟑𝒙) 𝒕𝒂𝒏(𝟏𝟏𝒙) ⇔ =
𝒄𝒐𝒔(𝟑𝒙) 𝒄𝒐𝒔(𝒙) 𝒄𝒐𝒔(𝟏𝟑𝒙) 𝒄𝒐𝒔(𝟏𝟏𝒙)

⇔ 𝒔𝒊𝒏(𝟑𝒙) 𝒄𝒐𝒔(𝒙) 𝒄𝒐𝒔(𝟏𝟑𝒙) 𝒄𝒐𝒔(𝟏𝟏𝒙) − 𝒄𝒐𝒔(𝟑𝒙) 𝒔𝒊𝒏(𝒙) 𝒔𝒊𝒏(𝟏𝟑𝒙) 𝒔𝒊𝒏(𝟏𝟏𝒙) = 𝟎

𝟏 𝟏
⇔ (𝒔𝒊𝒏(𝟒𝒙) + 𝒔𝒊𝒏(𝟐𝒙))(𝒄𝒐𝒔(𝟐𝟒𝒙) + 𝒄𝒐𝒔(𝒙)) − (𝒔𝒊𝒏(𝟒𝒙) − 𝒔𝒊𝒏(𝟐𝒙))(𝒄𝒐𝒔(𝟐𝒙) − 𝒄𝒐𝒔(𝟐𝟒𝒙)) = 𝟎
𝟒 𝟒

⇔ (𝒔𝒊𝒏(𝟒𝒙) + 𝒔𝒊𝒏(𝟐𝒙))(𝒄𝒐𝒔(𝟐𝟒𝒙) + 𝒄𝒐𝒔(𝟐𝒙)) − (𝒔𝒊𝒏(𝟒𝒙) − 𝒔𝒊𝒏(𝟐𝒙))(𝒄𝒐𝒔(𝟐𝒙) − 𝒄𝒐𝒔(𝟐𝟒𝒙)) = 𝟎

⇔ 𝟐 𝒔𝒊𝒏(𝟒𝒙) 𝒄𝒐𝒔(𝟐𝟒𝒙) + 𝟐 𝒔𝒊𝒏(𝟐𝒙) 𝒄𝒐𝒔(𝟐𝒙) = 𝟎 ⇔ 𝟐 𝒔𝒊𝒏(𝟒𝒙) 𝒄𝒐𝒔(𝟐𝟒𝒙) + 𝒔𝒊𝒏(𝟒𝒙) = 𝟎

99 RMM-ABSTRACT ALGEBRA MARATHON 601-700


www.ssmrmh.ro

𝒔𝒊𝒏(𝟒𝒙) = 𝟎
⇔ 𝒔𝒊𝒏(𝟒𝒙) (𝟐 𝒄𝒐𝒔(𝟐𝟒𝒙) + 𝟏) = 𝟎 ⇔ [
2 𝒄𝒐𝒔(𝟐𝟒𝒙) + 𝟏 = 𝟎
𝒌𝝅 𝝅 𝝅
∗ 𝒔𝒊𝒏(𝟒𝒙) = 𝟎 ⇒ 𝟒𝒙 = 𝒌𝝅 ⇒ 𝒙 = , 𝒌 ∈ ℤ and 𝒙 ∈ (𝟎, ) ⇒ 𝒙 =
𝟒 𝟐 𝟒
𝟏 𝟐𝝅 𝝅 𝝅
∗ 𝟐 𝒄𝒐𝒔(𝟐𝟒𝒙) + 𝟏 = 𝟎 ⇔ 𝒄𝒐𝒔(𝟐𝟒𝒙) = − ⇒ 𝟐𝟒𝒙 = ± + 𝒌𝟐𝝅 ⇔ 𝒙 = ± +𝒌
𝟐 𝟑 𝟑𝟔 𝟏𝟐
𝝅 𝝅 𝝅 𝟕𝝅 𝟓𝝅 𝟏𝟑𝝅 𝟒𝝅 𝝅 𝟓𝝅 𝟐𝝅 𝟏𝟏𝝅 𝟕𝝅 𝟏𝟕𝝅
𝒙 ∈ (𝟎, ) ⇒ 𝒙 = , , , , , , , , , , ,
𝟐 𝟑𝟔 𝟗 𝟑𝟔 𝟏𝟖 𝟑𝟔 𝟗 𝟏𝟖 𝟑𝟔 𝟗 𝟑𝟔 𝟏𝟖 𝟑𝟔
⇒ There are 13 solutions for the given equation: 𝒙
𝝅 𝝅 𝟕𝝅 𝟓𝝅 𝟏𝟑𝝅 𝟒𝝅 𝝅 𝟓𝝅 𝟐𝝅 𝟏𝟏𝝅 𝟕𝝅 𝟏𝟕𝝅 𝝅
= , , , , , , , , , , , ,
𝟑𝟔 𝟗 𝟑𝟔 𝟏𝟖 𝟑𝟔 𝟗 𝟏𝟖 𝟑𝟔 𝟗 𝟑𝟔 𝟏𝟖 𝟑𝟔 𝟒

693. Let 𝒇(𝒙) = √𝒂𝟐 + 𝒙𝟐 , 𝒈(𝒙) = 𝟐𝒙 + 𝟒𝒂 − 𝟑 (𝒂 ∈ ℝ)


Find all values of a such that 𝒇(𝒙 + 𝟐) ≥ 𝒈(𝒙 − 𝟏), ∀𝒙 ∈ ℝ.
Proposed by Nguyen Van Canh-Vietnam
Solution by Bedri Hajrizi-Mitrovica-Kosovo

√𝒂𝟐 + (𝒙 + 𝟐)𝟐 ≥ 𝟐𝒙 + 𝟒𝒂 − 𝟓
𝟓−𝟒𝒂
(1) 𝒙 ∈ 𝟐

𝟓 − 𝟒𝒂
𝒙 ∈ (−∞, ]
𝟐
𝟓−𝟒𝒂
(2) 𝒙 > 𝟐

𝒙𝟐 + 𝟒𝒙 + 𝟒 + 𝒂𝟐 ≥ 𝟒𝒙𝟐 + 𝟏𝟔𝒂𝟐 + 𝟐𝟓 + 𝟏𝟔𝒂𝒙 − 𝟐𝟎𝒙 − 𝟒𝟎𝒂


𝟑𝒙𝟐 + (𝟏𝟔𝒂 − 𝟐𝟒)𝒙 + 𝟏𝟓𝒂𝟐 − 𝟒𝟎𝒂−≤ 𝟎
𝟏𝟐 − 𝟖𝒂 + √𝟏𝟗𝒂𝟐 − 𝟕𝟐𝒂 + 𝟖𝟏
𝒙≤
𝟑
𝟏𝟐−𝟖𝒂+√𝟏𝟗𝒂𝟐−𝟕𝟐𝒂+𝟖𝟏
(1) ∪ (2): 𝒙 ≤ , so inequality isn’t valid ∀𝒙 ∈ ℝ ⇒ such values of 𝒂 does
𝟑

not exist.
694.
𝑷(𝒙) = 𝒙𝟏𝟖 + 𝟗𝒙𝟏𝟔 + 𝟑𝟔𝒙𝟏𝟒 + 𝟖𝟏𝒙𝟏𝟐 + 𝟏𝟎𝟖𝒙𝟏𝟎 + 𝟖𝟏𝒙𝟖 + 𝟐𝟕𝒙𝟔 − 𝟐

100 RMM-ABSTRACT ALGEBRA MARATHON 601-700


www.ssmrmh.ro

𝟑 𝟑
𝑷(𝒙𝟏 ) = 𝑷(𝒙𝟐 ) = 𝟎, 𝒙𝟏 , 𝒙𝟐 ∈ ℝ, 𝒙𝟏 = −𝒙𝟐 = √ √ √𝒂 + 𝒃 − 𝒄

𝒂𝒃𝒄 =?
Proposed by Elsen Kerimov-Azerbaijan
Solution by Pham Duc Nam-Vietnam

𝒙𝟏𝟖 + 𝟗𝒙𝟏𝟔 + 𝟑𝟔𝒙𝟏𝟒 + 𝟖𝟏𝒙𝟏𝟐 + 𝟏𝟎𝟖𝒙𝟏𝟎 + 𝟖𝟏𝒙𝟖 + 𝟐𝟕𝒙𝟔 − 𝟐 = 𝟎


Let: 𝒕 = 𝒙𝟐 , 𝒕 ≥ 𝟎
⇔ 𝒕𝟗 + 𝟗𝒕𝟖 + 𝟑𝟔𝒕𝟕 + 𝟖𝟏𝒕𝟔 + 𝟏𝟎𝟖𝒕𝟓 + 𝟖𝟏𝒕𝟒 + 𝟐𝟕𝒕𝟑 − 𝟐 = 𝟎
Now let: 𝒖 = 𝒕 + 𝟏 ⇔ 𝒕 = 𝒖 − 𝟏 ⇔ 𝒖𝟗 − 𝟑𝒖𝟔 + 𝟑𝒖𝟑 − 𝟑 = 𝟎
𝟑
⇔ (𝒖𝟑 − 𝟏)𝟑 − 𝟐 = 𝟎 ⇔ (𝒖𝟑 − 𝟏) = 𝟐 ⇔ 𝒖𝟑 − 𝟏 = √𝟐

𝟑 𝟑 𝟑
⇔ 𝒖𝟑 = 𝟏 + √𝟐 ⇔ 𝒖 = √𝟏 + √𝟐

𝟑 𝟑
Sub: 𝒕 = 𝒖 − 𝟏 then 𝒕 = √𝟏 + √𝟐 − 𝟏 ⇒ 𝒙 = ±√ √𝟏 + √𝟐 − 𝟏
𝟑 𝟑

𝟑 𝟑
⇒ 𝒙𝟏 = √ √𝟏 + √𝟐 − 𝟏, 𝒙𝟐 = −√ √𝟏 + √𝟐 − 𝟏
𝟑 𝟑

𝟑
We have the form √ √ 𝟑√𝒂 + 𝒃 − 𝒄 then 𝒂 = 𝟐, 𝒃 = 𝟏, 𝒄 = 𝟏 ⇒ 𝒂𝒃𝒄 = 𝟐

695.

𝐈𝐟 𝒂, 𝐛, 𝐜 > 0, 𝑐 ≤ 3,2(𝒂𝐛 + 𝐛𝐜 + 𝐜𝒂) ≥ √(𝟗 + 𝟑𝒂𝐛𝐜)(√𝒂 + √𝐛 + √𝐜) 𝒂𝐧𝐝


𝒂𝟑 + 𝐛𝟑 + 𝐜 𝟑 = 𝟑(𝟑𝐜 𝟐 + 𝒂𝐛𝐜 − 𝟑𝒂𝐛 − 𝟗𝐜 + 𝟗), 𝐭𝐡𝐞𝐧 𝒂, 𝐛, 𝐜 = ?

Proposed by Pavlos Trifon-Greece


Solution by Soumava Chakraborty-Kolkata-India

𝒂𝟑 + 𝐛𝟑 + 𝐜 𝟑 = 𝟑(𝟑𝐜 𝟐 + 𝒂𝐛𝐜 − 𝟑𝒂𝐛 − 𝟗𝐜 + 𝟗)


⇒ 𝒂𝟑 + 𝐛𝟑 + (𝐜 𝟑 − 𝟗𝐜 𝟐 + 𝟐𝟕𝐜 − 𝟐𝟕) + 𝟑𝒂𝐛(𝟑 − 𝐜) = 𝟎
⇒ 𝒂𝟑 + 𝐛𝟑 + (𝐜 − 𝟑)𝟑 + 𝟑𝒂𝐛(𝟑 − 𝐜) = 𝟎
𝒂𝟐 + 𝐛𝟐 + (𝐜 − 𝟑)𝟐 −
⇒ 𝟑𝒂𝐛(𝐜 − 𝟑) + (𝒂 + 𝐛 + 𝐜 − 𝟑) ( ) + 𝟑𝒂𝐛(𝟑 − 𝐜) = 𝟎
𝒂𝐛 − 𝒂(𝐜 − 𝟑) − 𝐛(𝐜 − 𝟑)

101 RMM-ABSTRACT ALGEBRA MARATHON 601-700


www.ssmrmh.ro
𝟏 𝟐 𝟐 (∗)
⇒ (𝒂 + 𝐛 + 𝐜 − 𝟑) ((𝒂 − 𝐛)𝟐 + (𝒂 − (𝐜 − 𝟑)) + (𝐛 − (𝐜 − 𝟑)) ) = 𝟎
𝟐
𝟐 𝟐
𝐍𝐨𝐰, (𝒂 − 𝐛)𝟐 + (𝒂 − (𝐜 − 𝟑)) + (𝐛 − (𝐜 − 𝟑)) = 𝟎 ⇒ 𝒂 = 𝐛 = 𝐜 − 𝟑; 𝐛𝐮𝐭
𝐜 − 𝟑 ≤ 𝟎 ⇒ 𝒂, 𝐛 ≤ 𝟎 → 𝐜𝐨𝐧𝐭𝐫𝒂𝐝𝐢𝐜𝐭𝐢𝐨𝐧
𝟐 𝟐
∴ (𝒂 − 𝐛)𝟐 + (𝒂 − (𝐜 − 𝟑)) + (𝐛 − (𝐜 − 𝟑)) ≠ 𝟎 ∴ (∗) ⇒
𝟑
(∗∗) (∑𝐜𝐲𝐜 𝒂) 𝐆−𝐇
𝒂+𝐛+𝐜−𝟑 = 𝟎 ⇒ 𝟗 = ∴ (𝟗 + 𝟑𝒂𝐛𝐜)(√𝒂 + √𝐛 + √𝐜) ≥
𝟑
𝟑 𝟑
(∑𝐜𝐲𝐜 𝒂) 𝟐𝒂 (∑𝐜𝐲𝐜 𝒂) + 𝟗𝒂𝐛𝐜 𝟐𝒂𝟐 𝐁𝐞𝐫𝐠𝐬𝐭𝐫𝐨𝐦
( + 𝟑𝒂𝐛𝐜) (∑ )=( ) (∑ 𝟐 ) ≥
𝟑 𝒂+𝟏 𝟑 𝒂 +𝒂
𝐜𝐲𝐜 𝐜𝐲𝐜

𝟑 𝟐 𝟑 𝟐
(∑𝐜𝐲𝐜 𝒂) + 𝟗𝒂𝐛𝐜 𝟐(∑𝐜𝐲𝐜 𝒂) 𝒂+𝐛+𝐜 = 𝟑 (∑𝐜𝐲𝐜 𝒂) + 𝟗𝒂𝐛𝐜 𝟐(∑𝐜𝐲𝐜 𝒂)
( )( 𝟐
) = ( ) 𝟐
𝟑 ∑𝐜𝐲𝐜 𝒂 + ∑𝐜𝐲𝐜 𝒂 𝟑 (∑𝐜𝐲𝐜 𝒂)
∑𝐜𝐲𝐜 𝒂𝟐 + 𝟑
( )
?
≥ 𝟒(𝒂𝐛 + 𝐛𝐜 + 𝐜𝒂)𝟐
𝟑 𝟐 𝟐 𝟐
? 𝒂+𝐛+𝐜 = 𝟑
⇔ ((∑ 𝒂) + 𝟗𝒂𝐛𝐜) (∑ 𝒂) ≥ 𝟐 (∑ 𝒂𝐛) (𝟑 ∑ 𝒂𝟐 + (∑ 𝒂) ) ⇔
𝐜𝐲𝐜 𝐜𝐲𝐜 𝐜𝐲𝐜 𝐜𝐲𝐜 𝐜𝐲𝐜
𝟑 𝟐 𝟐 𝟐
∑𝐜𝐲𝐜 𝒂 ?
( ) ((∑ 𝒂) + 𝟗𝒂𝐛𝐜) (∑ 𝒂) ≥ 𝟐 (∑ 𝒂𝐛) (𝟑 ∑ 𝒂𝟐 + (∑ 𝒂) )
𝟑
𝐜𝐲𝐜 𝐜𝐲𝐜 𝐜𝐲𝐜 𝐜𝐲𝐜 𝐜𝐲𝐜

𝟑 𝟑 𝟐 𝟐
?
⏟ 𝟔 (∑ 𝒂𝐛) (𝟑 ∑ 𝒂𝟐 + (∑ 𝒂) )
⇔ ((∑ 𝒂) + 𝟗𝒂𝐛𝐜) (∑ 𝒂) ≥
𝐜𝐲𝐜 𝐜𝐲𝐜 (⦁) 𝐜𝐲𝐜 𝐜𝐲𝐜 𝐜𝐲𝐜

𝐀𝐬𝐬𝐢𝐠𝐧𝐢𝐧𝐠 𝐛 + 𝐜 = 𝒙, 𝐜 + 𝒂 = 𝐲, 𝒂 + 𝐛 = 𝐳 ⇒ 𝒙 + 𝐲 − 𝐳 = 𝟐𝐜 > 0, 𝑦 + 𝑧 − 𝒙 = 𝟐𝒂
> 0 𝒂𝐧𝐝 𝐳 + 𝒙 − 𝐲 = 𝟐𝐛 > 0 ⇒ 𝒙 + 𝐲 > 𝑧, 𝐲 + 𝐳 > 𝒙, 𝐳 + 𝒙 > 𝑦 ⇒ 𝒙, 𝐲, 𝐳 𝐟𝐨𝐫𝐦 𝐬𝐢𝐝𝐞𝐬
𝐨𝐟 𝒂 𝐭𝐫𝐢𝒂𝐧𝐠𝐥𝐞 𝐰𝐢𝐭𝐡 𝐬𝐞𝐦𝐢𝐩𝐞𝐫𝐢𝐦𝐞𝐭𝐞𝐫, 𝐜𝐢𝐫𝐜𝐮𝐦𝐫𝒂𝐝𝐢𝐮𝐬 𝒂𝐧𝐝 𝐢𝐧𝐫𝒂𝐝𝐢𝐮𝐬 = 𝐬, 𝐑, 𝐫 (𝐬𝒂𝐲)
𝐲𝐢𝐞𝐥𝐝𝐢𝐧𝐠 𝟐 ∑ 𝒂 = ∑ 𝒙 = 𝟐𝐬 ⇒ ∑ 𝒂 = 𝐬 → (𝟏) ⇒ 𝒂 = 𝐬 − 𝒙, 𝐛 = 𝐬 − 𝐲, 𝐜 = 𝐬 − 𝐳
𝐜𝐲𝐜 𝐜𝐲𝐜 𝐜𝐲𝐜
𝟐
∴ 𝒂𝐛𝐜 = 𝐫 𝐬 → (𝟐) 𝒂𝐧𝐝 𝐬𝐮𝐜𝐡 𝐬𝐮𝐛𝐬𝐭𝐢𝐭𝐮𝐭𝐢𝐨𝐧𝐬 ⇒ ∑ 𝒂𝐛 = ∑(𝐬 − 𝒙)(𝐬 − 𝐲)
𝐜𝐲𝐜 𝐜𝐲𝐜
𝟐
⇒ ∑ 𝒂𝐛 = 𝟒𝐑𝐫 + 𝐫 → (𝟑) 𝒂𝐧𝐝
𝐜𝐲𝐜
𝟐
𝐯𝐢𝒂 (𝟏) 𝒂𝐧𝐝 (𝟑)
∑ 𝒂𝟐 = (∑ 𝒂) − 𝟐 ∑ 𝒂𝐛 = 𝐬𝟐 − 𝟐(𝟒𝐑𝐫 + 𝐫 𝟐 )
𝐜𝐲𝐜 𝐜𝐲𝐜 𝐜𝐲𝐜

⇒ ∑ 𝒂 = 𝐬 − 𝟖𝐑𝐫 − 𝟐𝐫 𝟐 → (𝟒) ∴ 𝐯𝐢𝒂 (𝟏), (𝟐), (𝟑) 𝒂𝐧𝐝 (𝟒), (∗)


𝟐 𝟐

𝐜𝐲𝐜
⇔ (𝐬𝟑 + 𝟗𝐫 𝟐 𝐬)𝐬𝟑 ≥ 𝟔𝐫 𝟐 (𝟒𝐑 + 𝐫)𝟐 (𝟑(𝐬𝟐 − 𝟖𝐑𝐫 − 𝟐𝐫 𝟐 ) + 𝐬𝟐 )

102 RMM-ABSTRACT ALGEBRA MARATHON 601-700


www.ssmrmh.ro
(⦁⦁)
⇔ 𝐬𝟔 + 𝟗𝐫 𝟐 𝐬𝟒 − 𝐫 𝟐 𝐬𝟐 (𝟑𝟖𝟒𝐑𝟐 + 𝟏𝟗𝟐𝐑𝐫 + 𝟐𝟒𝐫𝟐 ) + 𝟑𝟔𝐫 𝟑 (𝟒𝐑 + 𝐫)𝟑 ≥ 𝟎 𝒂𝐧𝐝
𝟑 𝐆𝐞𝐫𝐫𝐞𝐭𝐬𝐞𝐧
∵ (𝐬𝟐 − 𝟏𝟔𝐑𝐫 + 𝟓𝐫 𝟐 ) ≥ 𝟎 ∴ 𝐢𝐧 𝐨𝐫𝐝𝐞𝐫 𝐭𝐨 𝐩𝐫𝐨𝐯𝐞 (⦁⦁), 𝐢𝐭 𝐬𝐮𝐟𝐟𝐢𝐜𝐞𝐬 𝐭𝐨 𝐩𝐫𝐨𝐯𝐞 ∶
𝟑
𝐋𝐇𝐒 𝐨𝐟 (⦁⦁) ≥ (𝐬𝟐 − 𝟏𝟔𝐑𝐫 + 𝟓𝐫 𝟐 )
⇔ (𝟒𝟖𝐑𝐫 − 𝟔𝐫 𝟐 )𝐬𝟒 − 𝐫 𝟐 𝐬𝟐 (𝟏𝟏𝟓𝟐𝐑𝟐 − 𝟐𝟖𝟖𝐑𝐫 + 𝟗𝟗𝐫 𝟐)
(⦁⦁⦁)
+𝐫 𝟑 (𝟔𝟒𝟎𝟎𝐑𝟑 − 𝟐𝟏𝟏𝟐𝐑𝟐 𝐫 + 𝟏𝟔𝟑𝟐𝐑𝐫𝟐 − 𝟖𝟗𝐫𝟑 ) ≥ 𝟎 𝒂𝐧𝐝
𝟐 𝐆𝐞𝐫𝐫𝐞𝐭𝐬𝐞𝐧
∵ (𝟒𝟖𝐑𝐫 − 𝟔𝐫 𝟐 )(𝐬𝟐 − 𝟏𝟔𝐑𝐫 + 𝟓𝐫 𝟐 ) ≥ 𝟎 ∴ 𝐢𝐧 𝐨𝐫𝐝𝐞𝐫 𝐭𝐨 𝐩𝐫𝐨𝐯𝐞 (⦁⦁⦁),
𝟐
𝐢𝐭 𝐬𝐮𝐟𝐟𝐢𝐜𝐞𝐬 𝐭𝐨 𝐩𝐫𝐨𝐯𝐞 ∶ 𝐋𝐇𝐒 𝐨𝐟 (⦁⦁⦁) ≥ (𝟒𝟖𝐑𝐫 − 𝟔𝐫 𝟐 )(𝐬𝟐 − 𝟏𝟔𝐑𝐫 + 𝟓𝐫 𝟐 )
(⦁⦁⦁⦁)
⇔ (𝟑𝟖𝟒𝐑𝟐 − 𝟑𝟖𝟒𝐑𝐫 − 𝟑𝟗𝐫 𝟐)𝐬𝟐 ≥ 𝐫(𝟓𝟖𝟖𝟖𝐑𝟑 − 𝟕𝟏𝟎𝟒𝐑𝟐 𝐫 + 𝟓𝟐𝟖𝐑𝐫 𝟐 − 𝟔𝟏𝐫 𝟑 )
𝐆𝐞𝐫𝐫𝐞𝐭𝐬𝐞𝐧
𝐍𝐨𝐰, (𝟑𝟖𝟒𝐑𝟐 − 𝟑𝟖𝟒𝐑𝐫 − 𝟑𝟗𝐫 𝟐)𝐬𝟐 ≥
?
(𝟑𝟖𝟒𝐑𝟐 − 𝟑𝟖𝟒𝐑𝐫 − 𝟑𝟗𝐫 𝟐 )(𝟏𝟔𝐑𝐫 − 𝟓𝐫 𝟐 ) ≥ 𝐫(𝟓𝟖𝟖𝟖𝐑𝟑 − 𝟕𝟏𝟎𝟒𝐑𝟐 𝐫 + 𝟓𝟐𝟖𝐑𝐫𝟐 − 𝟔𝟏𝐫𝟑 )
? 𝐑 ? 𝐄𝐮𝐥𝐞𝐫
⇔ 𝟒𝐭 𝟑 − 𝟏𝟓𝐭 𝟐 + 𝟏𝟐𝐭 + 𝟒 ≥ 𝟎 (𝐭 = ) ⇔ (𝟒𝐭 + 𝟏)(𝐭 − 𝟐)𝟐 ≥ 𝟎 → 𝐭𝐫𝐮𝐞 ∵ 𝐭 ≥ 𝟐
𝐫
⇒ (⦁⦁⦁⦁) ⇒ (⦁⦁⦁) ⇒ (⦁⦁) ⇒ (⦁) 𝐢𝐬 𝐭𝐫𝐮𝐞 ∴

√(𝟗 + 𝟑𝒂𝐛𝐜)(√𝒂 + √𝐛 + √𝐜) ≥ 𝟐(𝒂𝐛 + 𝐛𝐜 + 𝐜𝒂) 𝐰𝐢𝐭𝐡 ′′ = ′′ 𝐢𝐟𝐟 𝒂 = 𝐛 = 𝐜 → (𝟏)

𝐁𝐮𝐭, 𝟐(𝒂𝐛 + 𝐛𝐜 + 𝐜𝒂) ≥ √(𝟗 + 𝟑𝒂𝐛𝐜)(√𝒂 + √𝐛 + √𝐜)

∴ 𝟐(𝒂𝐛 + 𝐛𝐜 + 𝐜𝒂) = √(𝟗 + 𝟑𝒂𝐛𝐜)(√𝒂 + √𝐛 + √𝐜) 𝒂𝐧𝐝

(𝟏) ⇒ 𝒂 = 𝐛 = 𝐜 → (∗∗∗) ∴ (∗∗), (∗∗∗) ⇒ 𝒂 = 𝐛 = 𝐜 = 𝟏 (𝒂𝒏𝒔)

696. Let 𝒙𝟏 , 𝒙𝟐 , 𝒙𝟑 be the roots of the equation 𝒙𝟑 − 𝟑𝒙 − 𝟑 = 𝟎. Find:


𝛀 = (𝒙𝟏 𝒙𝟐 − 𝒙𝟐𝟑 )(𝒙𝟐 𝒙𝟑 − 𝒙𝟐𝟏 )(𝒙𝟏 𝒙𝟑 − 𝒙𝟐𝟐 )

Proposed by Rovsen Pirguliyev-Azerbaijan


Solution 1 by Daniel Sitaru-Romania
𝑺𝟏 = 𝒙𝟏 + 𝒙𝟐 + 𝒙𝟑 = 𝟎
{𝑺𝟐 = 𝒙𝟏 𝒙𝟐 + 𝒙𝟐 𝒙𝟑 + 𝒙𝟑 𝒙𝟏 = −𝟑
𝑺𝟑 = 𝒙𝟏 𝒙𝟐 𝒙𝟑 = 𝟑

𝒙𝟑𝟏 = 𝟑𝒙𝟏 + 𝟑, 𝒙𝟑𝟐 = 𝟑𝒙𝟐 + 𝟑, 𝒙𝟑𝟑 = 𝟑𝒙𝟑 + 𝟑

𝒙𝟑𝟏 + 𝒙𝟑𝟐 + 𝒙𝟑𝟑 = 𝟑𝒙𝟏 + 𝟑 + 𝟑𝒙𝟐 + 𝟑 + 𝟑𝒙𝟑 + 𝟑 = 𝟑𝑺𝟏 + 𝟗 = 𝟗

𝒙𝟑𝟏 𝒙𝟑𝟐 + 𝒙𝟑𝟐 𝒙𝟑𝟑 + 𝒙𝟑𝟏 𝒙𝟑𝟑 =

= (𝟑𝒙𝟏 + 𝟑)(𝟑𝒙𝟐 + 𝟑) + (𝟑𝒙𝟐 + 𝟑)(𝟑𝒙𝟑 + 𝟑) + (𝟑𝒙𝟑 + 𝟑)(𝟑𝒙𝟏 + 𝟑) =


103 RMM-ABSTRACT ALGEBRA MARATHON 601-700
www.ssmrmh.ro

= 𝟗𝑺𝟐 + 𝟏𝟖𝑺𝟏 + 𝟐𝟕 = 𝟎
𝛀 = (𝒙𝟏 𝒙𝟐 − 𝒙𝟐𝟑 )(𝒙𝟐 𝒙𝟑 − 𝒙𝟐𝟏 )(𝒙𝟏 𝒙𝟑 − 𝒙𝟐𝟐 ) =
= (𝒙𝟏 𝒙𝟐𝟐 𝒙𝟑 − 𝒙𝟑𝟏 𝒙𝟐 − 𝒙𝟐 𝒙𝟑𝟑 + 𝒙𝟐𝟏 𝒙𝟐𝟑 )(𝒙𝟏 𝒙𝟑 − 𝒙𝟐𝟐 ) =
= 𝒙𝟐𝟏 𝒙𝟐𝟐 𝒙𝟐𝟑 − 𝒙𝟏 𝒙𝟒𝟐 𝒙𝟑 − 𝒙𝟑𝟏 𝒙𝟐 𝒙𝟑 + 𝒙𝟑𝟏 𝒙𝟑𝟐 − 𝒙𝟏 𝒙𝟐 𝒙𝟒𝟑 + 𝒙𝟑𝟐 𝒙𝟑𝟑 − 𝒙𝟑𝟏 𝒙𝟑𝟑 − 𝒙𝟐𝟏 𝒙𝟐𝟐 𝒙𝟐𝟑 =

= −𝑺𝟑 (𝒙𝟑𝟏 + 𝒙𝟑𝟐 + 𝒙𝟑𝟑 ) + (𝒙𝟑𝟏 𝒙𝟑𝟐 + 𝒙𝟑𝟐 𝒙𝟑𝟑 + 𝒙𝟑𝟏 𝒙𝟑𝟑 ) = −𝟐𝟕

Solution 2 by Ravi Prakash-New Delhi-India

𝒙𝟏 𝒙𝟐 𝒙𝟑 = 𝟑 ⟹ 𝒙𝟏 𝒙𝟐 𝒙𝟑 𝛀 = (𝒙𝟏 𝒙𝟐 𝒙𝟑 − 𝒙𝟑𝟑 )(𝒙𝟏 𝒙𝟐 𝒙𝟑 − 𝒙𝟑𝟏 )(𝒙𝟏 𝒙𝟐 𝒙𝟑 − 𝒙𝟑𝟐 )

𝒙𝟏 𝒙𝟐 𝒙𝟑 𝛀 = (𝟑 − 𝒙𝟑𝟑 )(𝟑 − 𝒙𝟑𝟏 )(𝟑 − 𝒙𝟑𝟐 ) ⟹ 𝟑𝛀 = (−𝟑𝒙𝟏 )(−𝟑𝒙𝟐 )(−𝟑𝒙𝟑 )

𝟑𝛀 = −𝟐𝟕𝒙𝟏 𝒙𝟐 𝒙𝟑 ⟹ 𝟑𝛀 = −𝟖𝟏 ⟹ 𝛀 = −𝟐𝟕

697. 𝐋𝐞𝐭 𝐧 𝐚𝐧𝐝 𝐦 𝐛𝐞 𝐭𝐰𝐨 𝐩𝐨𝐬𝐢𝐭𝐢𝐯𝐞 𝐢𝐧𝐭𝐞𝐠𝐞𝐫𝐬 𝐬𝐮𝐜𝐡 𝐭𝐡𝐚𝐭 𝐦 ≤ 𝐧. 𝐏𝐫𝐨𝐯𝐞 𝐭𝐡𝐚𝐭
𝟐𝒌𝒎 − 𝒏 𝟐𝒌𝒎 − 𝒏
𝒏 − 𝒎 + 𝟐 ∑𝒏𝒌=𝟏 (⌊ ⌋−⌊ ⌋)
𝒏 𝟐𝒏
𝟏 + 𝒍𝒄𝒎(𝒏, 𝒎)
𝐢𝐬 𝐢𝐧𝐭𝐞𝐠𝐞𝐫, 𝐰𝐡𝐞𝐫𝐞 𝒍𝒄𝒎 𝐢𝐬 𝐭𝐡𝐞 𝐥𝐞𝐚𝐬𝐭 𝐜𝐨𝐦𝐦𝐨𝐧 𝐦𝐮𝐥𝐭𝐢𝐩𝐥𝐞 𝐚𝐧𝐝
⌊. ⌋ 𝐝𝐞𝐧𝐨𝐭𝐞 𝐭𝐡𝐞 𝐟𝐥𝐨𝐨𝐫 𝐟𝐮𝐧𝐜𝐭𝐢𝐨𝐧.

Proposed by Toubal Fethi-Algeria


Solution by Mohamed Amine Ben Ajiba-Tanger-Morocco

𝟏
𝐁𝐲 𝐇𝐞𝐫𝐦𝐢𝐭𝐞′ 𝐬 𝐢𝐝𝐞𝐧𝐭𝐢𝐭𝐲, 𝐰𝐞 𝐡𝐚𝐯𝐞 ⌊𝟐𝒕⌋ = ⌊𝒕⌋ + ⌊𝒕 + ⌋, ∀𝒕 ∈ ℝ, 𝐭𝐡𝐞𝐧
𝟐
𝟐𝒌𝒎 − 𝒏 𝟐𝒌𝒎 − 𝒏 𝒌𝒎 𝒌𝒎 𝟏 𝒌𝒎
⌊ ⌋−⌊ ⌋ = (⌊𝟐. ⌋ − 𝟏) − (⌊ + ⌋ − 𝟏) = ⌊ ⌋.
𝒏 𝟐𝒏 𝒏 𝒏 𝟐 𝒏
𝐍𝐨𝐰, 𝐰𝐞 𝐰𝐢𝐥𝐥 𝐩𝐫𝐨𝐯𝐞 𝐭𝐡𝐞 𝐟𝐨𝐥𝐥𝐨𝐰𝐢𝐧𝐠 𝐢𝐝𝐞𝐧𝐭𝐢𝐭𝐲 𝐭𝐡𝐚𝐭
𝒏−𝟏
𝒌𝒎 (𝒎 − 𝟏)(𝒏 − 𝟏) + 𝒈𝒄𝒅(𝒏, 𝒎) − 𝟏
𝑺 = ∑⌊ ⌋= .
𝒏 𝟐
𝒌=𝟏
𝐖𝐞 𝐡𝐚𝐯𝐞
𝒏−𝟏 𝒏−𝟏
𝒌𝒎 (𝒏 − 𝒌 )𝒎 𝒌𝒎 𝒌𝒎
𝟐𝑺 = ∑ (⌊ ⌋+⌊ ⌋) = ∑ (⌊ ⌋ + ⌊− ⌋) + (𝒏 − 𝟏)𝒎.
𝒏 𝒏 𝒏 𝒏
𝒌=𝟏 𝒌=𝟏
𝐖𝐞 𝐤𝐧𝐨𝐰 𝐭𝐡𝐚𝐭 𝐢𝐟 𝒙 ∈ ℤ, 𝐰𝐞 𝐡𝐚𝐯𝐞 ⌊𝒙⌋ + ⌊−𝒙⌋ = 𝟎, 𝐨𝐭𝐡𝐞𝐫𝐰𝐢𝐬𝐞 ⌊𝒙⌋ + ⌊−𝒙⌋ = −𝟏.

104 RMM-ABSTRACT ALGEBRA MARATHON 601-700


www.ssmrmh.ro
𝒌𝒎
𝐋𝐞𝐭 𝒎 = 𝒅𝒂, 𝒏 = 𝒅𝒃 𝐰𝐡𝐞𝐫𝐞 𝒅 = 𝒈𝒄𝒅(𝒏, 𝒎) 𝐚𝐧𝐝 𝒂, 𝒃 ∈ ℕ. 𝐓𝐡𝐞 𝐧𝐮𝐦𝐛𝐞𝐫
𝒏
𝒌𝒂
=
𝐢𝐬 𝐢𝐧𝐭𝐞𝐠𝐞𝐫 𝒅 − 𝟏
𝒃
𝐭𝐢𝐦𝐞𝐬 𝐟𝐨𝐫 𝒌 = 𝒃, 𝟐𝒃, … , (𝒅 − 𝟏)𝒃, 𝐭𝐡𝐞𝐧
𝟐𝑺 = −(𝒏 − 𝒅) + (𝒏 − 𝟏)𝒎 = (𝒎 − 𝟏)(𝒏 − 𝟏) + 𝒈𝒄𝒅(𝒏, 𝒎) − 𝟏,
𝐰𝐡𝐢𝐜𝐡 𝐜𝐨𝐦𝐩𝐥𝐞𝐭𝐞𝐬 𝐭𝐡𝐞 𝐩𝐫𝐨𝐨𝐟 𝐨𝐟 𝐭𝐡𝐞 𝐢𝐝𝐞𝐧𝐭𝐢𝐭𝐲. 𝐔𝐬𝐢𝐧𝐠 𝐭𝐡𝐢𝐬 𝐢𝐝𝐞𝐧𝐭𝐢𝐭𝐲 𝐰𝐞 𝐡𝐚𝐯𝐞
𝒏 𝒏
𝟐𝒌𝒎 − 𝒏 𝟐𝒌𝒎 − 𝒏 𝒌𝒎
𝒏 − 𝒎 + 𝟐 ∑ (⌊ ⌋−⌊ ⌋) = 𝒏 − 𝒎 + 𝟐 ∑ ⌊ ⌋ = 𝒏 − 𝒎 + 𝟐(𝑺 + 𝒎)
𝒏 𝟐𝒏 𝒏
𝒌=𝟏 𝒌=𝟏
= 𝒏 + 𝒎 + (𝒎 − 𝟏)(𝒏 − 𝟏) + 𝒈𝒄𝒅(𝒏, 𝒎) − 𝟏 = 𝒎𝒏 + 𝒈𝒄𝒅(𝒏, 𝒎)
= (𝒍𝒄𝒎(𝒏, 𝒎) + 𝟏). 𝒈𝒄𝒅(𝒏, 𝒎).
𝟐𝒌𝒎 − 𝒏 𝟐𝒌𝒎 − 𝒏
𝒏 − 𝒎 + 𝟐 ∑𝒏𝒌=𝟏 (⌊ 𝒏
⌋−⌊
𝟐𝒏
⌋)
⇒ = 𝒈𝒄𝒅(𝒏, 𝒎) 𝐢𝐬 𝐢𝐧𝐭𝐞𝐠𝐞𝐫.
𝟏 + 𝒍𝒄𝒎(𝒏, 𝒎)

698. 𝐏𝐫𝐨𝐯𝐞 𝐭𝐡𝐚𝐭 𝐭𝐡𝐞𝐫𝐞 𝐢𝐬 𝐧𝐨 𝐩𝐨𝐬𝐢𝐭𝐢𝐯𝐞 𝐢𝐧𝐭𝐞𝐠𝐞𝐫𝐬 𝒏 𝐚𝐧𝐝 𝒙 𝐬𝐮𝐜𝐡 𝐭𝐡𝐚𝐭


𝒙𝒏
𝟐𝒙𝒏 − 𝒌 𝟐𝒙𝒏 − 𝟐 − 𝒌 𝟐𝒙𝒏 − 𝒌 𝟐𝒙𝒏 − 𝟐 − 𝒌
∑ 𝒌 (⌊ ⌋+⌊ ⌋−⌊ ⌋−⌊ ⌋) = 𝒙𝒏 + 𝟏,
𝒌 𝟐𝒌 𝟐𝒌 𝒌
𝒌=𝟏
𝐰𝐡𝐞𝐫𝐞 𝒙 𝐢𝐬 𝐚 𝐞𝐯𝐞𝐧 𝐧𝐮𝐦𝐛𝐞𝐫, 𝒙 > 𝟐 𝐚𝐧𝐝 ⌊. ⌋ 𝐝𝐞𝐧𝐨𝐭𝐞 𝐭𝐡𝐞 𝐟𝐥𝐨𝐨𝐫 𝐟𝐮𝐧𝐜𝐭𝐢𝐨𝐧
Proposed by Toubal Fethi-Algeria
Solution by Mohamed Amine Ben Ajiba-Tanger-Morocco

𝟐𝒙𝒏 − 𝒌 𝟐𝒙𝒏 − 𝟐 − 𝒌 𝟐𝒙𝒏 − 𝒌 𝟐𝒙𝒏 − 𝟐 − 𝒌


𝐋𝐞𝐭 𝒂𝒌 = ⌊ ⌋+⌊ ⌋−⌊ ⌋−⌊ ⌋ , 𝒌 ≥ 𝟏.
𝒌 𝟐𝒌 𝟐𝒌 𝒌
𝟏
𝐁𝐲 𝐇𝐞𝐫𝐦𝐢𝐭𝐞′ 𝐬 𝐢𝐝𝐞𝐧𝐭𝐢𝐭𝐲, 𝐰𝐞 𝐡𝐚𝐯𝐞 ⌊𝟐𝒕⌋ = ⌊𝒕⌋ + ⌊𝒕 + ⌋, ∀𝒕 ∈ ℝ.
𝟐
𝐁𝐲 𝐮𝐬𝐢𝐧𝐠 𝐭𝐡𝐢𝐬 𝐢𝐝𝐞𝐧𝐭𝐢𝐭𝐲, 𝐰𝐞 𝐡𝐚𝐯𝐞
𝒙𝒏 𝒙𝒏 − 𝟏 𝟏 𝒙𝒏 𝟏 𝒙𝒏 − 𝟏
𝒂𝒌 = (⌊𝟐. ⌋ − 𝟏) + (⌊ + ⌋ − 𝟏) − (⌊ + ⌋ − 𝟏) − (⌊𝟐. ⌋ − 𝟏)
𝒌 𝒌 𝟐 𝒌 𝟐 𝒌

𝒙𝒏 𝒙𝒏 𝟏 𝒙𝒏 − 𝟏 𝟏 𝒙𝒏 𝟏 𝒙𝒏 − 𝟏 𝒙𝒏 − 𝟏 𝟏
= (⌊ ⌋ + ⌊ + ⌋) + ⌊ + ⌋ − ⌊ + ⌋ − (⌊ ⌋+⌊ + ⌋) =
𝒌 𝒌 𝟐 𝒌 𝟐 𝒌 𝟐 𝒌 𝒌 𝟐

𝒙𝒏 𝒙𝒏 − 𝟏
=⌊ ⌋−⌊ ⌋
𝒌 𝒌

𝒙𝒏 𝒙𝒏 𝟏
𝐭𝐡𝐞𝐧 𝒂𝟏 = 𝒙 − (𝒙𝒏 − 𝟏) = 𝟏, 𝒂𝟐 =
𝒏
− ( + ⌊− ⌋) = 𝟏, 𝒂𝒙𝒏 = 𝟏 − 𝟎 = 𝟏,
𝟐 𝟐 𝟐
𝐚𝐧𝐝 𝒂𝒌 ≥ 𝟎, ∀𝒌 ∈ ⟦𝟑, 𝒙𝒏 − 𝟏⟧, 𝐭𝐡𝐞𝐫𝐞𝐟𝐨𝐫𝐞

105 RMM-ABSTRACT ALGEBRA MARATHON 601-700


www.ssmrmh.ro
𝒙𝒏

∑ 𝒌𝒂𝒌 ≥ 𝒂𝟏 + 𝟐𝒂𝟐 + 𝒙𝒏 𝒂𝒙𝒏 > 𝒙𝒏 + 𝟏,


𝒌=𝟏
𝐒𝐨 𝐭𝐡𝐞𝐫𝐞 𝐢𝐬 𝐧𝐨 𝐩𝐨𝐬𝐢𝐭𝐢𝐯𝐞 𝐢𝐧𝐭𝐞𝐠𝐞𝐫𝐬 𝐧 𝐚𝐧𝐝 𝐱 𝐰𝐡𝐢𝐜𝐡 𝐯𝐞𝐫𝐢𝐟𝐲 𝐭𝐡𝐞 𝐠𝐢𝐯𝐞𝐧 𝐞𝐪𝐮𝐚𝐭𝐢𝐨𝐧.

699. 𝐒𝐨𝐥𝐯𝐞 𝐢𝐧 ℝ+ 𝐭𝐡𝐞 𝐬𝐲𝐬𝐭𝐞𝐦:


𝟑
𝒙𝟐 + 𝒙𝒚 + 𝒚𝟐 = (𝟐𝒙 + 𝒚) √𝒙𝒛𝟐
{ 𝒚𝟐 + 𝒚𝒛 + 𝒛𝟐 = (𝟐𝒚 + 𝒛) 𝟑√𝒚𝒙𝟐
𝟑
𝒛𝟐 + 𝒛𝒙 + 𝒙𝟐 = (𝟐𝒛 + 𝒙) √𝒛𝒚𝟐

Proposed by Mihaly Bencze, Neculai Stanciu-Romania


Solution 1 by Mohamed Amine Ben Ajiba-Morocco
𝐁𝐲 𝐀𝐌 − 𝐆𝐌 𝐢𝐧𝐞𝐪𝐮𝐚𝐥𝐢𝐭𝐲, 𝐰𝐞 𝐡𝐚𝐯𝐞
𝟑
𝟑(𝒙𝟐 + 𝒙𝒚 + 𝒚𝟐 ) = (𝟐𝒙 + 𝒚). 𝟑 √𝒙𝒛𝟐 ≤ (𝟐𝒙 + 𝒚)(𝒙 + 𝟐𝒛).

𝐒𝐢𝐦𝐢𝐥𝐚𝐫𝐥𝐲, 𝐰𝐞 𝐠𝐞𝐭

𝟑(𝒚𝟐 + 𝒚𝒛 + 𝒛𝟐 ) ≤ (𝟐𝒚 + 𝒛)(𝒚 + 𝟐𝒙) 𝐚𝐧𝐝 𝟑(𝒛𝟐 + 𝒛𝒙 + 𝒙𝟐 ) ≤ (𝟐𝒛 + 𝒙)(𝒛 + 𝟐𝒚).

𝐀𝐝𝐝𝐢𝐧𝐠 𝐭𝐡𝐞𝐬𝐞 𝐢𝐧𝐞𝐪𝐮𝐚𝐥𝐢𝐭𝐲, 𝐰𝐞 𝐡𝐚𝐯𝐞

𝟔(𝒙𝟐 + 𝒚𝟐 + 𝒛𝟐 ) + 𝟑(𝒙𝒚 + 𝒚𝒛 + 𝒛𝒙) ≤ 𝟐(𝒙𝟐 + 𝒚𝟐 + 𝒛𝟐 ) + 𝟕(𝒙𝒚 + 𝒚𝒛 + 𝒛𝒙)

⇔ 𝟐(𝒙 − 𝒚)𝟐 + 𝟐(𝒚 − 𝒛)𝟐 + 𝟐(𝒛 − 𝒙)𝟐 ≤ 𝟎 ⇒ 𝒙 − 𝒚 = 𝒚 − 𝒛 = 𝒛 − 𝒙 = 𝟎.

𝐒𝐨, 𝐭𝐡𝐞 𝐬𝐨𝐥𝐮𝐭𝐢𝐨𝐧 𝐢𝐬 𝒙 = 𝒚 = 𝒛.

Solution 2 by Mohamed Amine Ben Ajiba-Morocco


𝐌𝐮𝐥𝐭𝐢𝐩𝐥𝐲𝐢𝐧𝐠 𝐭𝐡𝐞 𝐭𝐡𝐫𝐞𝐞 𝐞𝐪𝐮𝐚𝐭𝐢𝐨𝐧𝐬, 𝐰𝐞 𝐡𝐚𝐯𝐞

(𝒙𝟐 + 𝒙𝒚 + 𝒚𝟐 )(𝒚𝟐 + 𝒚𝒛 + 𝒛𝟐 )(𝒛𝟐 + 𝒛𝒙 + 𝒙𝟐 ) = 𝒙𝒚𝒛(𝟐𝒙 + 𝒚)(𝟐𝒚 + 𝒛)(𝟐𝒛 + 𝒙).

𝐁𝐮𝐭 𝐛𝐲 𝐀𝐌 − 𝐆𝐌 𝐢𝐧𝐞𝐪𝐮𝐚𝐥𝐢𝐭𝐲, 𝐰𝐞 𝐡𝐚𝐯𝐞

𝒙 + 𝟐𝒚 (𝒙 − 𝒚)𝟐 𝟑
𝒙𝟐 + 𝒙𝒚 + 𝒚𝟐 = (𝟐𝒙 + 𝒚). + ≥ (𝟐𝒙 + 𝒚) √𝒙𝒚𝟐 ,
𝟑 𝟑

𝐰𝐢𝐭𝐡 𝐞𝐪𝐮𝐚𝐥𝐢𝐭𝐲 𝐟𝐨𝐫 𝒙 = 𝒚. 𝐒𝐢𝐦𝐢𝐥𝐚𝐫𝐥𝐲, 𝐰𝐞 𝐡𝐚𝐯𝐞


𝟑 𝟑
𝒚𝟐 + 𝒚𝒛 + 𝒛𝟐 ≥ (𝟐𝒚 + 𝒛) √𝒚𝒛𝟐 𝐚𝐧𝐝 𝒛𝟐 + 𝒛𝒙 + 𝒙𝟐 ≥ (𝟐𝒛 + 𝒙) √𝒛𝒙𝟐 .

106 RMM-ABSTRACT ALGEBRA MARATHON 601-700


www.ssmrmh.ro
𝐌𝐮𝐥𝐭𝐢𝐩𝐥𝐲𝐢𝐧𝐠 𝐭𝐡𝐞𝐬𝐞 𝐢𝐧𝐞𝐪𝐮𝐚𝐥𝐢𝐭𝐢𝐞𝐬, 𝐰𝐞 𝐡𝐚𝐯𝐞

(𝒙𝟐 + 𝒙𝒚 + 𝒚𝟐 )(𝒚𝟐 + 𝒚𝒛 + 𝒛𝟐 )(𝒛𝟐 + 𝒛𝒙 + 𝒙𝟐 ) ≥ 𝒙𝒚𝒛(𝟐𝒙 + 𝒚)(𝟐𝒚 + 𝒛)(𝟐𝒛 + 𝒙),

𝐰𝐢𝐭𝐡 𝐞𝐪𝐮𝐚𝐥𝐢𝐭𝐲 𝐟𝐨𝐫 𝒙 = 𝒚 = 𝒛. 𝐒𝐨, 𝐭𝐡𝐞 𝐨𝐧𝐥𝐲 𝐬𝐨𝐥𝐮𝐭𝐢𝐨𝐧 𝐨𝐟 𝐭𝐡𝐞 𝐬𝐲𝐬𝐭𝐞𝐦 𝐢𝐬 𝒙 = 𝒚 = 𝒛.

700. Compare:

√𝟐𝟎𝟐𝟒 + √𝟐𝟎𝟐𝟒 − √𝟐𝟎𝟐𝟑 + √𝟐𝟎𝟐𝟐 and 𝝅√𝒆

Proposed by Nguyen Van Canh-Vietnam


Solution by Eric Cismaru-Romania
Since 𝟒 > 𝝅 and 𝟒 > 𝒆, we have 𝟒𝟓 > 𝟏𝟔 > 𝝅√𝒆 , so it is sufficient to prove that the radical
expression is greater than 𝟒𝟓.

If we denote 𝒙 = √𝟐𝟎𝟐𝟒 − √𝟐𝟎𝟐𝟑 + √𝟐𝟎𝟐𝟐, then we have

√𝟐𝟎𝟐𝟒 + 𝒙 > 𝟒𝟓 ⇔ 𝟐𝟎𝟐𝟒 + 𝒙 > 𝟐𝟎𝟐𝟓 ⇔ 𝒙 > 𝟏 ⇔ 𝟐𝟎𝟐𝟒 − √𝟐𝟎𝟐𝟑 + √𝟐𝟎𝟐𝟐 > 𝟏 ⇔

⇔ 𝟐𝟎𝟐𝟑 > √𝟐𝟎𝟐𝟑 + √𝟐𝟎𝟐𝟐 ⇔ 𝟐𝟎𝟐𝟑 ⋅ 𝟐𝟎𝟐𝟐 > √𝟐𝟎𝟐𝟐 ⇔ 𝟐𝟎𝟐𝟑 ⋅ √𝟐𝟎𝟐𝟐 > 𝟎

In conclusion, we have √𝟐𝟎𝟐𝟒 + √𝟐𝟎𝟐𝟒 − √𝟐𝟎𝟐𝟑 + √𝟐𝟎𝟐𝟐 > 𝝅√𝒆

107 RMM-ABSTRACT ALGEBRA MARATHON 601-700


www.ssmrmh.ro

It’s nice to be important but more important it’s to be nice.


At this paper works a TEAM.
This is RMM TEAM.
To be continued!
Daniel Sitaru

108 RMM-ABSTRACT ALGEBRA MARATHON 601-700

You might also like